Anda di halaman 1dari 413

SOAL SELEKSI

OLIMPIADE SAINS TINGKAT KABUPATEN/KOTA 2005

CALON TIM OLIMPIADE KIMIA INDONESIA 2006

Bidang Kimia

KEMENTERIAN PENDIDIKAN DAN KEBUDAYAAN


DIREKTORAT JENDERAL PENDIDIKAN MENENGAH
DIREKTORAT PEMBINAAN SEKOLAH MENENGAH ATAS
TAHUN 2005
SELEKSI TINGKAT KABUPATEN/KOTA
TAHUN 2005
Bagian I

1. Struktur penta eritriol yang merupakan salah satu bahan untuk membuat cat dengan rumus
bangun sebagai berikut :

Yang dapat dinyatakan mengenai pentaeritriol adalah :


A. Dapat bereaksi dengan pereaksi Benedict
B. Mempunyai pusat stereogenik
C. Dapat mengalami dehidratasi dengan H2SO4 pekat membentuk alkena
D. Dapat larut dalam air
E. Pernyataan A, B, C, dan D tersebut betul semua

2. Suatu senyawa organik X (Mr = 56 ) ternyata bersifat dapat menghilangkan warna Br 2/CCl4.
Tentukan senyawa X tersebut.

A. D.

B. E.

C.

3. Berikut ini adalah suatu struktur polimer :

Yang manakah yang merupakan monomer dari polimer tersebut ?

A. B.

C. D.
E.
4. Berikut ini, manakah yang merupakan rumus molekul suatu asam organik.
A. CHO C. CH2O e. C2H6O2
B. C2HO2 D. CH2O2
5. Dari reaksi berikut ini, tentukan persamaan reaksi yang menunjukkan hasil pembakaran
metana (CH4) dengan udara berlebih.
A. CH4 (g) + O2 (g)  C (s) + H2O (g)
B. CH4 (g) + O2 (g)  CO2 (g) + H2 (g)
C. CH4 (g) + 2 O2 (g)  CO2 (g) + 2 H2O (g)
D. CH4 (g) + O2 (g)  CO (g) + 4 H2 (g)
e. Semua reaksi mungkin terjadi

6. Hasil ang diperoleh jika etanol direaksikan dengan KMnO4 dan K2Cr2O7 dalam suasana
asam adalah :
A. etena C. Etuna E. eter
B. etana D. Asam etanoat

7. Etanol dapat dibuat dengan cara mereaksikan etena dengan uap air dalam kondisi tertentu.
Tentukan tipe reaksi yangterjadi :
A. subtitusi C. Dehidrasi E. adisi
B. hidrolisa D. Hidrolisa

8. Berikut ini, reaksi yang tidak memproduksi karbon dioksida adalah :


A. pembakaran metana
B. fermentasi gula
C. oksidasi etanol menjadi asam etanoat
D. reaski asam etanoat dengan
kalsium karbonat
E. pembakaran alkohol

9. Apa produk yang akan terbentuk bila pada suhu kamar, sikloheksana direaksikan dengan
bromine ?

A. C. E.

B.D.

10. Pernyataan yang tidak tepat untuk kedua alkena berikut ini :
dan
adalah :
A. keduanya mempunyai isomer geometri
B. keduanya dapat berpolimerisasi
C. keduanya bereaksi dengan bromin membentuk 1,4 – dibromobutana
D. keduanya bereaksi dengan hidrogen membentuk butana
E. keduanya bereaksi dengan KMnO4 memberntuk diol
11. Sebanyak 20 cm3 gas hidrokarbon terbakar sempurna dalam oksigen berlebihan dan
menghasilkan 60 cm3 karbon dioksida serta 40 cm 3 uap air (STP). Bagaimanakah rumus
molekul dari hidrokarbon tersebut.
A. C2H6 C. C3H6 E. C3H4
B. C3H8 D. C6H6
12. Senyawa 1-propanoat dapat dibedakan dari 2-propanol adalah dengan reagen :
A. larutan KMnO4 dalam suasana asam
B. larutan I2 dalam NaOH
C. larutan Br2dalam CCl4
D. larutan K2Cr2O7 dalam suasana asam
E.jawaban A,B,C dan D betul semua

13. Satu mol senyawa X bila direaksikan dengan PCl5 berlebihan akan menghasilkan 2 mol
HCl. Tentukan senyawa X tersebut.
A. CH3CH = CH2
B. C6H5 – CH2Cl
C. p-hidroksifenol
D. p-hidroksi asam benzoat
E. 2 – hidroksi asam asetat

14. Tipe reaksi yang terjadi antara etena dan HBr adalah
A. dehalogenasi D. netralisasi
B. hidrogenasi E. hidratasi
C. hidrohalogenasi

15. Proses apakah yang terjadi bila etanol diubah menjadi asam etanoat ?
A. dehidratasi D. netralisasi
B. oksidasi E. hidrogenasi
C. reduksi

16. Berikut ini, senyawa asam amino yang tidak aktif optik adalah :
A. HOCH2 – CH2 – CH (NH2) – COOH
B. CH3 - CHNH2 – COOH
C. NH2 – CH2 – COOH
D. NH2 – CHOH – CO2H

E.

17. Ada berapa banyak isomer dari senyawa


dinitrobenzena,C6H4(NO2)2? A. 1 B. 2 C. 3 D. 4 E. 5

18. Berikut ini, manakah yang merupakan isomer dari asam propanoat ?
A. HO – CH2 – CH2 – CHO
B. CH2 = CH – O – CO – CH3
C. CH3 – CHOH – CH3
D. CH3 – O – CH2 – CH2 – OH
E. CH3 – CH2 – CO2CH3

19. Senyawa yang dapat digunakan sebagai pewangi buatan pada sirop adalah :
A. keton C. Aldehida E. Ester
B. alkohol D. asam karboksilat

20. Suatu senyawa sulfat (aq) dapat diidentifikasi dengan cara :


A. panaskan padatan sulfat tersebut dan tentukan apakah ada gas SO2 yang dihasilkan
B. tambahkan larutan perak nitrat kedalam larutan senyawa sulfat tersebut, Tentukan
apakah ada endapan yang terjadi, dan larut dalam asam nitrat
C. tambahkan larutan barium nitrat dan terjadi endapan putih yang tidak larut dalam asam
nitrat
D. tambahkan larutan kalsium nitrat kedalam larutan sulfat dan kemudian bila terbentuk
endapan putih tambahkan asam sulfat encer
E. Jawaban A, B, C dan D semua betul

21. Bila ke dalam air sungai ditambahkan asam klorida kemudian ditambahkan larutan barium
nitrat dan hasilnya terbentuk endapan putih. Kesimpulannya dalam air sungai tersebut
terdapat ion
A. kalsium C. Klorida E. Sulfat
B. magnesium D. Karbonat

22. Ke dalam senyawa X ditambahkan larutan ence asam klorida dan ternyata terbentuk gas
dan dihasilkan larutan yang keruh. Setelah reaksi berhenti , kemudian ditambahkan larutan
amonium hidroksida berlebih, dan ternyata dihasilkan larutan yang berwarna biru. Tentukan
senyawa X tersebut.
A. CuSO4 C. CuCO3 E. CuBr2
B. Cu(NO3)2 D. CuCl2
23. Bila suatu senyawa yang keadaan antar partikel-partikel molekulnya masih berhubungan
dan dapat bergerak bebas secaraterbatas dan kemudian berubah menjadi keadaan dimana
partikel-partikel molekunya dapat bergerak bebas dalam ruangan yang tidak terbatas, maka
proses ini disebut :
A. kondensasi D. pembekuan
B. difusi E. sublimasi
C. penguapan

24. Perhatikan gambar di bawah ini :

Kurva I di dapat bila 1 gram logam Zn granular direaksikan dengan HCl berlebihan pada
suhu 30 0C. Tentukan perubahan kondisi dalam reaksi yang menghasilkan kurva II
A. Bila digunakan bubuk Zn
B. Panaskan asam HCl sampai 40 0C
C. Pergunakan hanya ½ gram Zn
D. Encerkan asam HCl dengan air
E. Jawaban A, B, C, dan D semua betul.

25. Energi kisi RbF, CsF, dan RbCl berturut-turut adalah – 760 kJ/mol, - 730 kJ/mol, dan – 650
kJ/mol, Berapakah energi kisi CsCl ? (No. Atom Rb = 37 ; Cs = 55 )
A. – 620 kJ/mol D. – 900 kJ/mol
B. – 720 kJ/mol E. – 1020 kJ/mol
C. – 800 kJ/mol
26. Dengan kran penghubung tertutup, suatu bejana yang volumenya 10 cm3 berisi gas X
dengan tekanan 50 kPa dihubungkan dengan bejana kedua yang volumenya 30 cm 3 dan
berisi gas Y dengan tekanan 100 kPa. Pada temperatur yang sama, maka setelah kran
dibuka dan tidak terjadi reaksi kimia, tentukanlah tekanan dalam masing-masing bejana
setelah kran penghubung kedua tabung tersebut dibuka.
A. 75 kPa C. 100 kPa E. 150 kPa
B. 87,5 kPa D. 125 kPa
2 –
27. bila larutan yang mengandung ion etanadioat, C2O4 dioksidasi dengan larutan KMnO4
dalam suasana asam, terjadi reaksi sebagai berikut :
– 2– + 2+
2 MnO4 + 5 C2O4 + 16 H  2 Mn + 10 CO2 + 8 H2O
Berapa volume larutan KMnO4 0,02 mol/L yang dibutuhkan untuk mengoksidasi sempurna
0,001 mol garam KHC2O4 – H2C2O4 ?
A. 20 cm3 C. 50 cm3 E. 250 cm3
B. 40 cm3 D. 125 cm3
28. Berikut ini berturutan adalah delapan energi ionisasi suatu unsur ( kJ/mol ).
703 ; 1610 ; 2460 ; 4350 ; 5400 ; 8500 ; 10.300 ; 12300
Nyatakanlah termasuk golongan berapakah unsur tersebut ?
A. III A B. IV A C. V A D. VI A E. VII A
29. Unsur radioaktif G dan H mempunyai waktu paruh berturut-turut 5 dan 15 menit. Bila jumlah
G adalah 4 kali banyaknya H, maka bila keduanya meluruh bersamaan, berapa lama waktu
yang dibutuhkan agar supaya jumlah atom unsur G yang tersisa sama banyaknya dengan
unsur H ?
A. 5 menit C. 15 meit E. 30 menit
B. 10 menit D. 20 menit
30. Berikut ini, tentukanlah oksida dari golongan IV mana yang tidak dapat dibuat dengan
mereaksikan langsung unsurnya dengan oksigen.
A. SiO2 C. GaO2 E. PbO2
B. CO2 D. SnO2
31. Bagaimanakah struktur elektronik yang paling luar atom unsur X, dimana oksidanya
membentuk XO2 dan XO3 ; serta membentuk H2XO4 sebagai asam kuat dan XF6 sebagai
fluoridanya.
A. s2 p2 C. s2 p4 E. s2 p6
B. s2 p3 D. s2 p5

32. Titanium mempunyai struktur elektronik sebagai berikut :


1s2, 2s2, 2p6, 3s2, 3p6 ,3d2, 4s2
senyawa yang tidak mungkin terjadi adalah :
A. K2TiO4 C. TiCl3 E. TiO2
B. K3TiF6 D. TiO
33. Unsur logam transisi adalah unsur yang elektron valensinya berasal dari :
A. hanya dari subkulit d
B. subkulit yang berbeda pada bilangan kuantum yang berbeda
C. subkulit yang berbeda dari bilangan kuantum yang sama
D. hanya dari subkulit d dan f
E. hanya dari subkulit s

34. Perhatikan reaksi yang belum setara ini :


MnO4 – + H+ + Fe 2+  Mn 2+ + Fe 3+ + H2O
Pernyataan yang benar mengenai reaksi tersebut adalah :
A. MnO4 – merupakan pereduksi
B. Mangan mengalami oksidasi
C. Fe 2+ mengambil elektron
D. Besi mengalami reduksi
E. Fe 2+ merupakan reduktor

35. Reaksi : A + 2 B  C + 3 D berlangsung pada suhu 300 K. Bila laju awal pembentukan C
adalah [C]/t = 1 M/jam, maka pernyataan di bawah ini yang sesuai dengan persamaan
laju awal tersebut adalah
A. – [B]/t = 2 M/jam D. [B]/t = 1 M/jam
B. [A]/t = 1/3 M/jam E. Salah semua
C. [A]/t = 1 M/jam

36. Untuk titrasi H3PO4 dengan larutan NaOH dipakai indikator fenolftalein (HPh). HPh tidak
berwarna sedangkan Ph – berwarna merah. Bila terus menerus ditambahkan OH –, maka
indikator akan berubah warna :
A. sebelum H2PO4 – terbentuk
B. Setelah H3PO4 berubah menjadi H2PO4–
C. Setelah H2PO4– berubah menjadi HPO4 2–
D. Setelah HPO42– berubah menjadi PO43–
E. setelah 3 mol NaOH ditambahkan ke dalam 1 mol H3PO4

37. Berikut ini, senyawa manakah yang dapat bertindak sebagai asam Lewis ?
A. Zn 2+
B. BF3
C. H3C – CO – CH3
D. Zn 2+ dan BF3
E. Zn 2+ , BF3, dan H3C – CO – CH3

38. Nilai pH untuk larutan asam lemah HX adalah 1,340. Berapakah tetapan kesetimbangan
asam, Ka HX ?
A. 2,09 x 10 – 2 D. 1,80 x 10 –5
–2 –5
B. 3,85 x 10 E. 3,60 x 10
C. 4,57 x 10 – 2

39. Jumlah koefisien untuk reaksi pembakaran sempurna propana adalah :


A. 4 B. 8 C. 12 D. 13 E. 22

40. Senyawa berikut ini, manakah yang merupakan basa paling kuat ?
A. CH3CH2CHO D. CH3CH2CO2H
B. CH3CH2CH2OH E. CH3C  CH
C. CH3CH2CH2NH2
BAGIAN II : ( 56 Poin)

Soal I ( 10 poin )

Diberikan data entalpi pembentukan standar (HF0) senyawa berikut ini :

HF0 Massa
Senyawa
(kJ/mol) Molar
Iso-oktana, C8H18 (l) – 255,1 114
CO2 (g) – 393,5 44
CO (g) – 110,5 28
H2O (g) – 241,8 18
H2O (l) – 285,8 18
O2 (g) 0 32

Sebuah mobil menggunakan bahan bakar 100 % iso-oktana yang densitasnya 0,79 g/mL. Mobil
tersebut menempuh jarak 8 km/liter iso-oktana. Bila mobil tersebut menghasilkan gas CO 2 dan
uap H2O, maka :
a. Tuliskan reaksi pembakaran sempurna iso-oktana dalam mesin mobil tersebut.
b. i. Tentukan entalpi pembakaran 1 mol iso-okana dalam keadaan standar.
ii. Tentukanlah berapa kJ entalpi pembakaran untuk 1 liter oktana
c. Berapa volume gas CO2 yang dibebaskan ke udara bila mobil tersebut telah menempuh jarak
100 km.
(pada kondisi tersebut volumemolar gas = 22,5 L/mol)

Soal 2 ( 9 poin)

Suatu senyawa tak dikenal yang mengandung unsur C, H dan Cr dengan formula C xHyCrz larut
dalam pelarut kloroform (CHCl3). Komposisi unsurnya dianalisis dengan membakar sejumlah
senyawa tersebut. Gas CO2 dan uap H2O hasil pembakaran ditampung dan ditentukan
kandungan C dan H. Untuk menentukan massa molarnya, digunakan metode pengukuran
tekanan osmosa larutan zat tersebut dalam pelarut kloroform.
Diperoleh data sebagai berikut :
- senyawa tersebut presentase unurnya mengandung 73,94% C dan 8,27% H ; sisanya
kromium
- pada temperatur 25 0C, tekanan osmosa larutan yang mengandung 5,00 mg zattak dikenal
tersebut dalam 100 mL kloroform adalah 3,17 mmHg.

Diketahui : tetapan Gas universal, R = 0,082507 L.atm.K – 1 mol – 1 = 8,314510 J.K – 1 .mol – 1 ;
1 atm = 760 mmHg = 760 torr ; 0 K = - 273,15 0C. Massa molar Cr = 52,0 g/mol ; C = 12,0 g/mol
; H = 1 g/mol.

Tentukanlah :
a. Rumus empiris senyawa
b. Massa empiris senyawa
c. Massa molar senyawa
d. Rumus molekul senyawa

Soal 3 (17 poin)

Gas nitrogenmonoksida, NO adalah gas pencemar yang dihasilkan oleh pembakaran bahan
bakar fosil, yang dapat mengakibatkan terjadinya kabut fotokimia di udara. Di udara gas
tersebut dapat bereaksi dengan oksigen dan membentuk gas nitrogen dioksida, NO 2. bila terjadi
hujan, gas NO2 tersebut akan larut dalam air dan menimbulkan hujan asam karena terbentuk
asam nitrit (HNO2) dan asam nitrat (HNO3).
a. i. Tuliskan persamaan reaksi antara gas NO dan O2 dan kemudia gas hasil
reaksi tersebut dilarutkan dalam air.
ii. Apa jenis reaksi yangterjadi bila gas NO2 dilarutkan dalam air, mengapa ?
iii. Bagaimana perbandingan mol HNO2 dan HNO3 bila gas NO direaksikan
dengan O2 dan dilarutkan dalam air.

Sebanyak 450 mL gas NO2 dialirkan ke dalam 100 mL air murni.. Di dalam
air, asam nitrit , HNO 2 adalah asam lemah dengan nilai tetapan
kesetimbangan asam, Ka = 4,6 x 10 – 4 . diketahui bahwa pada kondisi
tersebut volume molar gas adalah 22,5 liter/mol, dan volume larutan yang
diperoleh tidak berubah.

Setelah dialirkan gas NO2, maka hitunglah :

b. i. Konsentrasi larutan asam yang terbentuk dalam 100 mL


air tersebut di atas. ii. Berapakah pH larutan yang telah
dialiri gas NO2 tersebut.

Larutan yang telah dialirkan gas NO2 tersebut, ditambahkan dengan sejumlah
NaOH (40 g/mol) padat sehingga semua HNO3 yang terlart dinetralkan.

c. i. Berapa gram NaOH yang dibutuhkan untuk menetralkan HNO 3 yang


terdapat dalam larutan tersebut.
ii. Berapa pH larutan setelah semua HNO3 dalam larutan dinetralkan.

Soal 4 (20 poin)

Berikut ini adalah reaksi antara tembaga (Cu) dengan


asam nitrat (HNO3) : 8 HNO3 + 3 Cu  2NO + 3
Cu(NO3)2 + 4 H2O

Pertanyaan :
a. Apakah jenis reaksi yang terjadi dan tulislah reaksi di atas dalam bentuk
setengah reaksi dan reaksi totalnya dalam bentuk ion.
b. i. Hitunglah berapa gram berat Cu yang dibutuhkan untuk menghasilkan
gas NO sebanyak 0,05 mol.
ii. Berapakah volume gas NO yang dibebaskan bila pada keadaan
tersebut, bila 1 g gas N 2 (28 g/mol) volumenya 1 L, dan massa molar
Cu = 63,5 g/mol.
c. Mengenai gas NO yang terbentuk :
i. Apa jenis ikatan kimia pada molekul NO
ii. Tulis struktur Lewis dari gas NO
iii. Bagaimana sifat magnit gas NO, mengapa demikian.

Berikut ini mengenai logam Cu dan ion Cu 2+ dalam larutan.

d. Tuliskan konfigurasi elektron ion Cu 2+ dan bagaimana sifat magnitnya ( nomor


atom Cu = 29)
e. Larutan Cu 2+ bila ditambahkan larutan amoniak (NH3) berlebih akan
membentuk larutan yang berwarna biru terang.
i. Tuliskan reaksi yang terjadi
ii. Apa jenis ikatan yang terbentuk antara ion Cu 2+ dengan molekul NH3
pada senyawa berwarna biru tersebut.
iii. Bagaimanakah struktur senyawa biru tersebut, mengapa ?

SOAL SELEKSI
OLIMPIADE SAINS TINGKAT KABUPATEN/KOTA 2006

CALON TIM OLIMPIADE KIMIA INDONESIA 2007

Bidang Kimia

KEMENTERIAN PENDIDIKAN DAN KEBUDAYAAN


DIREKTORAT JENDERAL PENDIDIKAN MENENGAH
DIREKTORAT PEMBINAAN SEKOLAH MENENGAH ATAS
TAHUN 2006
SELEKSI TINGKAT KABUPATEN/KOTA
TAHUN 2006

Bagian I

1. Berapa banyak ozon, O3 yang dapat dibentuk dari 48 gram oksigen, O2 ?


A. 1,00 mol C. 1,50 mol E. 2,5 mol
B. 1,30 mol D. 2,00 mol
10
2.Massa atom relatif boron 10,80 yang terdiri dari isotop : B5 dan 11B5 , persentase antom 11B5
dalam campuran isitop ini :
A. 0,8% C. 20% E. 92%
B. 8,0% D. 80%

3. Keracunan merkuri dapat merusak syaraf karena di dalam tubuh merkuri membentuk
senyawa kompleks koordinasi 1 : 1 dengan gugus lipoil, yang berperanan penting dalam
metabolisme glukosa. Bila konsentrasi rata-rata gugus lipoil dalam cairan tubuh adalah 1,0 x
10 – 8 mol.kg – 1 , berapa massa merkuri yang dapat membentuk kompleks dengan gugus
lipoil, bila dalam tubuh manusia mengandung sebanyak 5,0 kg cairan tubuh.
(massa atom relatif Hg = 200)
A. 2,5 x 10 – 9 g D. 1,0 x 10 – 5 g
–8
B. 4,0 x 10 g E. 1,0 x 10 – 4 g
C. 1,0 x 10 – 7 g

4. Dalam laboratorium patologi, hasil analisis sampel urin, yang mengandung 0,12 gram urea,
NH2CONH2 (Mr = 60) direaksikan dengan asam nitrit berlebih. Urea bereaksi sesuai
persamaan reaksi berikut :
NH2CONH2 + 2 HNO2  CO2 + 2 N2 + 3 H2O
Gas yang dihasilkan dilewatkan dalam larutan akua natrium hidroksida dan volume akhir
gas sesudah melewati larutan tersebut diukur. Berapakah volume gas yang diukur pada
temperatur dan tekanan ruang? (volum molar gas pada rtp adalah 24.000 cm3.mol – 1 )
A. 9,6 cm3 C. 48,0 cm3 E. 144,0 cm3
3 3
B. 14,4 cm D. 96,0 cm

5. Bila 20 cm3 gas hidrokarbon dibakar sempurna dengan oksigen berlebih, terbentuk sebanyak
60 cm3 karbon dioksida dan 40 cm3 uap air. Semua volume diukur pda suhu dan tekanan
yang sama. Maka formula (rumus molekul) hidrokarbon tersebut :
A. C2H6 C. C3H6 E. C6H8
B. C3H4 D. C3H8
6. Gas CO2 yang terbentuk pada reaksi 200,2 g CaCO3 dengan HCl pada temperatur kamar 25
0
C dan tekanan 1 atm , akan mempunyai volume (dalam liter) sebanyak :
A. 22,4 C. 41,0 E. 49,8
B. 44,8 D. 48,9

7.Padatan berikut terdiri dari atom-atom atau molekul-molekul yang berikatan/berinteraksi antar
sesamanya hanya oleh gaya van der Waals adalah :
A. CO2 C. H2O E.SiO3
B. Cu D. MgO

8.Senyawa manakah yang momen dipolnya adalah nol ?


A. C6H6 C. CO E. H2S
B. NO D. NH3
9. Senyawa X dan Y yang terdapat dalam larutan dapat dipisahkan melalui teknik kristalisasi
fraksional. Hal ini dapat dilakukan bila X dan Y mempunyai perbedaan dalam hal :
A. titik didih D. warna kristal
B. titik leleh E. kelarutan
C. berat jenis

10. Dari pernyataan berikut ini, fenomena manakah yang dapat diterangkan/dijelaskan oleh
ikatan hidrogen antar molekular?
A. Titik leleh golongan I hidroksida menaik dengan bertambahnya massa molekular relatif
(Mr).
B. Titik didih alkana menaik dengan bertambahnya massa molekular relatif
C. CH3OCH3 (Mr = 46) mempunai titik didih lebih tinggi daripada CH3 CH2CH3 (Mr = 44 )
D. Hidrogen klorida membentuk suatu larutan asam ketika melarut dalam air
E. Es mempunyai densitas lebih rendah daripada air pada 0 0C.

11. X, Y dan Z adalah unsur-unsur dalam periode pendek yang sama dari sistem periodik
unsur. Oksida dari X amfoter, oksida Y bersifat basa dan oksida Z bersifat asam.
Bagaimana order (urutan) kenaikan nomor atom (proton) dari unsur-unsur ini?
A. XYZ C. YXZ E. ZXY
B. XZY D. YZX

12. Dari sifat – sifat unsur-unsur golongan II (magnesium – barium) berikut ini, manakah yang
meningkat sifat senyawanya dengan bertambahnya nomor atom (proton) ?
A. kecenderungannya membentuk ion kompleks
B. pH dari larutan garam kloridanya
C. kelarutan dalam air dari garam sulfatnya
D. kestabilan garam karbonatnya terhadap panas
E. besaran perubahan entalpi hidrasi dari ion logamnya

13. Dalam atom tunggal (yang tidak bergabung dengan sesama unsur-unsur), manakah unsur-
unsur berikut ini yang tidak mempunyai elektron p tunggal ?
Unsur nomor atom (proton)
A. Karbon 6
B. Oksigen 8
C. neon 10
D. magnesium 12
E. silikon 14

14. Dari oksida golongan IV berikut ini, manakah yang tidak dapat dibentuk dari kombinasi
langsung unsurnya dan oksigen ?
A. CO2 C. GeO2 E. PbO2
B. SiO2 D. SnO2
15. Di dalam pelarut ammonia cair, mengapa asam etanoat bersifat asam yang lebih kuat
dibandingkan dalam air ?
A. ammonia adalah basa yang lebih kuat daripada air
B. molekul-molekul asam etanoat membentuk ikatan hidrogen dengan air
C. asam etanoat lebih larut daam cairan ammonia daripada dalam air
D. asam etanoat mempunyai perubahan entalpi hidrasi yang lebih tinggi
E. ion etanoat membentuk ikatan hidrogen dengan air
16. Sn(IV)iodida dapat dibuat dengan mendidihkan 0,04 mol Sn (refluks) dengan 0,03 mol I2
yang dilarutkan dalam 50 mL tetraklorometan (titik didih 77 0C)
Reaksinya : Sn + 2 I2  SnI4
Hasilnya adalah kristal kuning yang diperoleh dengan menyaring saat larutan campuran
reaksi masih panas, dan kemudian filtratnya didinginkan. Berikut ini, manakah yang dapat
menunjukkan bahwa reaksi telah sempurna ?
A. titik didih campuran adalah 77 0C.
B. Tidak terdapat Sn tersisa dalam wadah reaksi
C. Kristal mulai dideposisi/mengendap dari pelarut yang mendidih
D. Tidak nampak uap berwarna purple lebih lanjut
E. Tidak ada sisa cairan dalam wadah reaksi

17. Jari-jari (r) dan muatan tiap-tiap enam ion ditunjukkan dalam tabel berikut :
Ion J+ L+ M 2+ X Y Z2–
r 0,14 0,18 0,15 0,14 0,18 0,15
Padatan ionik JK, LY dan MZ mempunyai tipe/jenis kisi yang sama. Manakah urutan energi
kisi yang benar (dari yang nilai tertinggi ke rendah).
A. JX > LY > MZ
B. JX > MZ > LY
C. LY > MZ > JX
D. MZ > JX > LY
E. MZ > LY > JX

18. Potensial elektroda standar untuk logam X dan Y masing-masing adalah :


X 2+(aq) / X (s) ; E0 = + 0,30 volt
Y 2+(aq) / Y (s) ; E0 = - 0,40 volt
Ke dalam masing-masing larutannya dicelupkan elektroda masing-masing logam, X dan Y.
Manakah mengenai deskripsi berikut yang benar ?

Elektroda positif dimana ion- E0/ V


ion masuk ke dalam larutan
A X 0,10
B X 0,70
C X 0,75
D Y 0,10
E Y 0,70

19. Bila sejumlah arus dilewatkan ke dalam larutan akua Cu(II)sulfat diasamkan, terjadi
pembebasan serentak di katoda, yaitu x mol Cu dan y dm3 hidrogen (diukur pada stp).
Berapa mol elektron yang dilewatkan ke dalam larutan itu ?
A. x + y/22,4 D. 2x + y/11,2
B. x + y/11,2 E. 2x + y/22,4
C. x + y/5,6

20. Reaksi kesetimbangan berikut (Proses Haber) :


N2 (g) + 3 H2 (g)  2 NH3 (g) H0= - x kJ/mol
Perubahan berikut ini, manakah yang dapat mempengaruhi harga tetapan kesetimbangan
Kp dan juga proporsi ammonia yang terdapat pada kesetimbangan ?
A. penambahan katalis
B. menurunkan suhu
C. menambahkan massa nitrogen
D. menaikkan tekanan
E. menambahkan massa hidrogen
21. Efek apakah terhadap adanya katalis pada tetapan laju reaksi, k1 untuk reaksi ke kanan dan
k – 1 untuk reaksi sebaliknya, dan pada tetapan kesetimbangan , K, untuk reaksi reversibel ?
k1 k –1 K
A. naik turun tidak ada efek
B. naik turun naik
C. naik naik tidak ada efek
D. naik naik naik
E. tidak ada efek tidak ada efek naik

22. Yang manakah berikut ini yang reaksinya adalah reaksi redoks ?
A. AlH3 (g) + H – 1 (g)  AlH4 –
B. Al 3+(aq) + 3 OH – (aq)  Al(OH)3 (s)
C. 2 Al (s) + 3 Cl2 (g)  2 AlCl3 (s)
D. AlO2 – (aq) + H+(aq) + H2O (l)  Al(OH)3 (s)
E. Al2Cl6 (g)  2 AlCl3 (s)

23. Pada pembuatan larutan KOH 10% dari KOH padat yang mengandung 90% KOH murni,
terlanjut ditambahkan air terlalu banyak sehingga kadar KOH menjadi 8%. Berapa gram
KOH padat harus ditambahkan pada 250 gram larutan ini, agar diperoleh larutan KOH 10%.
A. 5,75 C. 6,25 E. 6,75
B. 6,00 D. 6,50

24. Tekanan uap eter, C2H5OC2H5 murni pada 10 0C adalah 291,8 torr. Jika 4,46 g asam
salisilat dilarutkan dalam 80,7 g eter, tekanan uapnya turun menjadi 8,5 torr, maka massa
molekul asam salisilat menurut percobaan ini adalah :
A. 140,5 C. 136,5 E. 132,5
B. 138,5 D. 134,5

25. Tiga energi ionisasi pertama kedua dan ketiga suatu unsur X berturut-turut adalah 735 ;
1445 dan 7730 kJ/mol. Formula ion X yang paling cocok adalah :
A. X + B. X 2+ C. X 3+ D. X – E. X 2 –
26. Dari hidroksida berikut ini :
I. Al(OH)3 III. NaOH
II. Ca(OH)2 IV. Zn(OH)2
Tentukan mana yang bersifat amfoter :
A. I C. I dan III E. I dan II
B. III D. I dan IV

27. Bila 0,1 M larutan dari HClO4, NH4Br, KOH dan KCN disusun berdasarkan kenaikan [H+] ,
maka urutan yang benar adalah :
A. KOH < KCN < NH4Br < HClO4
B. KCN < KOH < HClO4 < NH4Br
C. HClO4 < NH4Br < KCN < KOH
D. NH4Br < HClO4 < KOH < KCN
E. Salah semua

28. Tentukan reaksi yang akan menghasilkan endapan, bila masing-masing 0,1 mol larutuan di
bawah ini dicampurkan :
A. NaOH + H2S D. CuSO4 + NH4Cl
B. Ca(OH) 2 + K2 CO 3 E. BaCl2 + KNO3
C. Al(NO3)3 + K2SO4

29. Manakah asam amino berikut yang mengandung dua atom karbon chiral ?
A. H2NCH(CH2OH)2
B. H2NCH(CO2H)(CH2CH2OH)
H2NCH(CO2H)(CH(CH3)2)
H2NCH(CO2H)( CH2 – C2H5)
H2NCH(CO2H) (CH(CH3)CH2 – CH3)

1 Mengenai dua senyawa alkena berikut ini : CH 3CH2CH = CH2 dan CH3CH = CHCH3,
manakah pernyataan yang benar ?
A. keduanya tidak menunjukkan isomer cis-trans
B. keduanya tidak dapat dipolimerisasi
C. keduanya tidak bereaksi dengan brom membentuk 1,4 – dibromobutana
D. keduanya tidak bereaksi dengan hidrogen membentuk butana
E. keduanya tidak dapat dioksidasi oleh alkali kalium manganat (VII) membentuk diol.

2 Dalam pembuatan etena, ke dalam etanol ditambahkan setetes reagen Y sambil


dipanaskan. Etena yang tidak murni dicuci dengan cara di bubble melalui larutan Z dan
kemudian dikumpulkan. Apakah reagen Y dan larutan Z yang mungkin digunakan ?
Reagen Y Larutan Z
A K2Cr2O7 diasamkan Larutan NaOH
B H2SO4 pekat H2SO4 encer
C H2SO4 pekat NaOH encer
D NaOH etanolat H2SO4 encer
E NaOH etanolat NaOH encer

2. Asam karboksilat yang sama dapat diperoleh melalui hidrolisis senyawa nitril P atau melalui
oksidasi senyawa alkohol Q. Yang manakah pasangan berikut ini sebagai P dan Q ?
P Q
A CH3CH2CN CH3CH2OH
B (CH3) 2CHCN (CH3) 3COH
C C6H5CH(CH3)CN C6H5CH2CH(OH)CH3
D C6H5CH2CN C6H5CH2CH2OH
E C6H5CN C6H5OH

33. Senyawa yang merupakan alkena adalah :


A. C2H2 C. C4H6 E. C4H8
B. C3H8 D. C6H6
34. Tentukan senyawa yang tidak mengandung gugus karbonil :
A. asam karboksilat D. keton
B. amida E. aldehid
C. ether

5. Berapa banyak rumus bangun yang dapat ditulis dari senyawa dengan rumus molekul
C5H12?
A. 2 B. 3 C. 4 D. 5 E. 6

6. Suatu ester terbentuk bila terjadi reaksi antara :


2 mol asam
2 mol alkohol
2 mol aldehid
1 mol alkohol dan 1 mol asam karboksilat
semua betul

7. Tentukan produk utama yang akan diperoleh bila etanol dipanaskan dengan H2SO4 :
A. Etana C. Etuna E. Salah semua
B. Etena D. etilsulfat
7. Tentukan senyawa dengan rumus bangun :

2 – etil – 4,4 – dimetilpentana


2 – metil – 4 – t –butilbutana
2,2,4 – trimetilheksana
1 – t – butil – 2 – metilbutana
salah semua

8. Kedua senyawa berikut ini, CH3CH2OH dan CH3OCH3 mempunyai massa relatif sama
dengan titik didih masing-masing 78 0C dan – 24 0C. Perbedaan titik didih ini disebabkan
oleh adanya perbedaan :
rumus molekul
berat jenis
panas spesifik
ikatan antar molekulnya
panas pembakaran

9. Suatu gas hidrokarbon dengan rumus empiris : CH 2 mempunyai densitas 1,88 g/L pada 0
0
C dan tekanan 1 atm. Maka rumus molekul senyawa tersebut adalah :
A. CH2 C. C3H6 E. C5H10
B. C2H4 D. C4H8

BAGIAN 2 (ESSAY)

Soal 1 (13 poin)

Pada masa lalu, ke dalam bahan bakar bensin mesin kendaraan bermotor ditambahkan /
dicampurkan sejumlah Pb tetraetil, Pb(Et)4 sebagai aditif untuk menaikkan bilangan oktannya.
1 (i) apa jenis ikatan di dalam molekul Pb tetraetil ?
A. Gambarkan struktur dot Lewisnya
B. menurut teori hibridisasi, bagaimana hibridisasi dan bentuk ruangnya
2 Pada temperatur kerja mesin kendaraan bermotor, pembakaan molekul Pb tetraetil
menghasilkan dua komponen utama. Jelaskan dua kemungkinan komponen senyawa
tersebut.
3 Tuliskan persamaan reaksi pembakaran sempurna Pb Tetraetil
4 (i) Menurut anda, produk manakah dari hasil pembakaran (c) yang dapat merusak
mesin.
A. untuk mencegah kerusakan, sejumlah 1,2 – dibromoetana ditambahkan ke dalam
bahan bakar yang mengandung Pb(Et)4 . Apa kesimpulan anda bila senyawa yang
mengandung brom dibebaskan melalui pipa knalpot (exhaust fumes)
B. Jelaskan mengapa emisi gas buang ke udara bebas tidak diinginkan/tidak
diperbolehkan.
5 Pada tahun 1972, sebelum adanya peraturan mengenai bahan bakar nir-Pb (tanpa Pb),
sebanyak 1,2 x 10 12 L(dm3) bahan bakar dipakai kendaraan pertahunnya. Hitunglah
(dalam Kg), massa Pb yang diemisikan dari pipa gas buang kendaraan bermotor dalam
1 tahun.

Soal 2 (15 poin)

Suatu stasiun pembangkit listrik tenaga batubara dihubungkan dengan peralatan untuk
menghilangkan gas sulfur oksida /FGD (flue gas desulphurization). Peralatan tersebut berfungsi
untuk menangkap gas sulfur dioksida dari limbah gas buang hasil pembakaran batubara. Di
dalam unit FGD, limbah gas direaksikan dengan bubuk batu kapur, CaCO 3 dan menghasilkan
kalsium sulfit, CaSO3. Hasil reaksi ini dioksidasi oleh udara membentuk CaSO4 padat. Diagram
berikut menunjukkan jumlah senyawa yang dipakai,dan hasilnya, hasil kegiatan stasiun tenaga
listrik bahan bakar batubara dengan unit FGD dalam 1 tahun.

9. (i) Proses apakah yang memberikan energi yang dipakai dalam stasiun tenaga ?
A gas manakah yang tidak terdapat dalam diagram, sebagai komponen utama dari flue
gas?
B jelaskan mengapa oksida nitrogen (NOx) terdapat dalam gas buang
10. Tuliskan persamaan reaksi setimbang di masing-maing masalah untuk menunjukkan
bagaimana :
batu kapur bereaksi dengan sulfur dioksida
udara mengoksidasi kalsium sulfit.

A. (i) gunakan persamaan (b)(i) untuk menentukan massa maksimum sulfur dioksida yang
dapat ditangkap oleh 3 x 105 ton batu kapur dalam unit FGD
Gunakan persamaan (b) untuk menentukan massa maksimum kalsium sulfat yang
dapat dihasilkan dari 3 x 105 ton batu kapur
B. unit FGD memisahkan 90% sulfur dioksida dari gas limbah. Gunakan jawaban anda (c)(i),
hitunglah massa sulfur dioksida yang dilepaskan ke dalam atmosfer tiap-tiap tahun oleh
stasiun tenaga bila 5 x 106 ton batubara dibakar.
C. Senyawa lain apakah yang terbentuk, selain kalsium sulfat, yang merupakan hasil reaksi
batu kapur dengan komponen gas buang
D. Berikan dua kemungkinan kerugian dari hasil operasi unit FGD

Soal 3 (12 poin)

Ammonium karbamat (NH2COONH4) adalah garam asam karbamat yang terdapat dalam darah
dan urin mamalia. Pada temperatur 250 0C terjadi kesetimbangan sebagai berikut :
NH2COONH4 (s)  2 NH3 (g) + CO2 (g)
Dengan nilai Kc = 1,58 x 10 – 8 .
Bila 7,8 g NH2COONH4 (78 g/mol) dimasukkan ke dalam wadah yang volumenya 0,5 liter, maka
:
12. Tuliskan pernyataan Kc untuk reaksi tersebut
13. Pada temperatur 250 0C, tentukan berapa konsentrasi NH3 dan CO2 bila telah tercapai
kesetimbangan
14. Tentukan berapa tekanan di dalam wadah bila telah tercapai kesetimbangan
15. Tentukan nilai Kp reaksi tersebut pada temperatur 250 0C.

Soal 4 (16 poin)

Suatu larutan mengandung asam nikotinat (Hnik) yang konsentrasinya 0,001 M Hnik. Bila Ka
asam nikotinat, Hnik, adalah 1,4 x 10 – 5 , maka :
13. berapa pH-nya ?
14. Berapa derajat ionisasinya ?

Ke dalam 500 mL larutan tersebut ditambahkan 22 mg NaOH (40 g/mol)

15. Bila volume larutan tidak berubah setelah penambahan NaOH :


Tuliskanlah reaksi yang terjadi
Hitunglah berapa pH larutan setelah penambahan NaOH tersebut.

Soal 5 (13 poin)

Larutan maltosa 4% (massa) dalam air membeku pada temperatur – 0,229 0C dan nilai
penurunan titik beku molal air ( Kf = 1,86 0C/m)

1 Tentukan berapa massa molar maltosa (tidak terionisasi dalam air)

2 Pada temperatu 25 0C, densitas larutan maltosa tersebut adalah 1,014 g/mL, maka
tentukanlah tekanan osmosa larutan maltosa tersebut

3 Bila larutuan maltosa airnya hendak dimurnikan dengan cara osmosa terbalik, berapa
tekanan minimum yang harus diberikan ke permukaan larutan maltosa tersebut. (R =
0,082057 L.atm/K.mol – 1 )

4 Berapa konsentrasi larutan NaCl yang isotonik dengan larutan maltosa tersebut.

Soal 6 (22 poin)

16. tentukan senyawa A, B, C dan D serta kondisi reaksinya berdasarkan langkah reaksi berikut
ini :
A –Cl2/uv  H3C-CH2Cl –B  H2C=CH2 –C
 H3C – CH3 –D  CO2 + H2O

17. Suatu hidrokarbon gas CxHy dibakar dengan oksigen berlebihan, dan menghasilkan
sebanyak 344 mg CO2 dan 56,2 mg H2O.
18. Tentukan rumus empiris hirokarbon (CxHy) tersebut
Bila Mr CxHy = 64, tuliskan rumus bangun yang mungkin (2 buah) dari senyawa tersebut.
OLIMPIADE S AINS NASI ONAL 2010

SELEKSI K ABUPATEN / KOTA

J AWABAN

(DOKU MEN NEG RA)

UjianTeori

W ktu 2 Jam

D epartemen Pendidikan Nasional

Direktorat Jenderal

Manage men Pendi dikan Dasa r dan Menengah

Direktor at Pembinaan Sekolah Menengah Atas

2010

Olimpiad e Kimia Kab upaten Kot a 2010 Page 1


Petunjuk :

1. Isilah Biodata anda dengan lengkap (di lembar Jawaban)


Tulis dengan huruf cetak dan jangan disingkat !

2. Soal Teori ini terdiri dari dua bagian:

30 soal pilihan Ganda = 60 poin

5 Nomor soal essay = 82poin

TOTAL Poin = 142

3. Waktu yang disediakan: 2 jam.

4. Semua jawaban harus ditulis di lembar jawaban yang tersedia

5. Diperkenankan menggunakan kalkulator.

6. Diberikan Tabel periodik Unsur.


7. Anda dapat mulai bekerja bila sudah ada tanda mulai dari pengawas.
8. Anda harus segera berhenti bekerja bila ada tanda berhenti dari Pengawas.
9. Letakkan jawaban anda di meja sebelah kanan dan segera meninggalkan ruangan.
10. Anda dapat membawa pulang soal ujian !!

Olimpiade Kimia Kabupaten Kota 2010 Page 2


LEMBAR JAWABAN

Bagian A

Beri Tanda Silang (X) pada Jawaban Yang Anda Pilih

No Jawaban No Jawaban

1 A B C D E 16 A B C D E

2 A B C D E 17 A B C D E

3 A B C D E 18 A B C D E

4 A B C D E 19 A B C D E

5 A B C D E 20 A B C D E

6 A B C D E 21 A B C D E

7 A B C D E 22 A B C D E

8 A B C D E 23 A B C D E

9 A B C D E 24 A B C D E

10 A B C D E 25 A B C D E

11 A B C D E 26 A B C D E

12 A B C D E 27 A B C D E

13 A B C D E 28 A B C D E

14 A B C D E 29 A B C D E

15 A B C D E 30 A B C D E

Olimpiade Kimia Kabupaten Kota 2010 Page 3


Olimpiade Kimia Kabupaten Kota 2010 Page 4
Tetapan dan rumus berguna
23 –1
Tetapan (bilangan) Avogadro NA = 6.022·10 partikel.mol
-1 -1 7 -1 -1
R= 8,314 J.K .mol = 8,314 x10 erg. Mol .K
-1 -1 -1 -1
Tetapan gas universal, R =1,987 cal.mol .K = 0,082054 L.atm.mol .K
Tekanan gas 1 atmosfir =760 mmHg =760 torr

Persamaan gas Ideal PV= nRT


o
Hubungan antara tetapan G = -RTlnK
kesetimbangan dan energi Gibbs
Energi Gibbs pada temperatur konstan G H TS
Isotherm reaksi kimia G = G + RT·ln Q
Tekanan Osmosa pada larutan p =c RT
-19
Muatan elektron 1,6022 x 10 C
Tetapan Kesetimbangan air (Kw) pada
oC
25 Kw= 1,0x10

Olimpiade Kimia Kabupaten Kota 2010 Page 5


A. Pilih jawaban yang paling tepat

1. Fosfor merah merupakan salah satu alotropi fosfor yang dibuat dari pemanasan fosfor
o
putih pada temperature lebih tinggi dari 240 C. Berapa jumlah proton, elektron dan
netron yang dimiliki oleh fosfor merah tersebut?

A. 16;16;15
B. 15,15,1
C. 15;15;16
D. 16;16;31
E. 16;16;1

Jawab (C)
2 Dalam atom tunggal (yang tidak bergabung dengan sesama unsur-unsur) berikut
ini, manakah unsur-unsur yang tidak mempunyai 1 elektron p tunggal?
Unsur: nomor atom (proton):
A. Nitrogen 7
B. Fluor 9
C. Natrium 11
D. Aluminium 13
E. Fosfor 15
Jawab (B)

3. Gas nitrogen (N2 ) yang terdapat di atmosfir bersifat kurang reaktif karena:
A. Atom N tidak terdapat orbital 2d di kulit terluarnya
B. Kuatnya ikatan kovalen dalam molekul N2
C. Energi ionisasi yang tinggi dari nitrogen dibandingkan dengan oksigen
D. Kelarutan nitrogen dalam air sangat rendah
E. Gas nitrogen memenuhi
Jawab (B)

4. Diantara pasangan senyawa berikut ini, manakah senyawa yang moment dipolenya adalah
nol?
NO dan SO2
CO dan CH4
CH3-CH2OH
NH3 dan BCl3
H2S dan H2O

Jawab: B

Olimpiade Kimia Kabupaten Kota 2010 Page 6


5. Dalam molekul berikut ini: nyatakanlah jenis hibridisasi atom dalam pembentukan ikatan

untuk atom yang yang bertanda panah:

Dalam molekul berikut ini dalam pembentukan ikatannya, nyatakanlah jenis hibridisasi atom
atom yang diberi bertanda panah: (N dan O):
3
1 N sp ; O sp
3
2 N sp; O sp
3 2
3 N sp ; O sp
2
4 N sp ; O sp
3 3
5 N sp ; O sp

Jawab::B

6. Jari-jari (r) dan muatan tiap-tiap enam ion ditunjukkan dalam table berikut :

ion : J+ L+ M2+ X- Y- Z2-


r : 0,14 0,18 0,15 0,14 0,18 0,15

Padatan ionik JX, LY dan MZ mempunyai tipe/jenis kisi yang sama. Manakah urutan energi
kisi yang benar (dari yang nilai terendah ke tinggi).
1. JX < LY <MZ
2. JX > MZ > LY
3. LY < MZ < JX
4. MZ < LY < JX
5. MZ <JX < LY
Jawab: E

7. Bila Na dibakar diudara dengan kelembaban yang tinggi maka reaksi yang paling
utama terjadi adalah reaksi :
Na + O2  Na2O
Na + H2O NaOH + H2
4 Na + O2 + 2 H2O  4NaOH
2Na + O2 + CO2 2Na2CO3
2Na + H2O + CO2  2NaHCO3 + H2

Jawab: C

Olimpiade Kimia Kabupaten Kota 2010 Page 7


8. Yang manakah berikut ini mempunyai pengaruh terkecil/ terendah untuk kelarutan
padatan dalam pelarut cair?
1. Temperatur
2. Tekanan
C.Sifat alami dari zat terlarut
Sifat alami dari pelarut
Tidak dapat diperkirakan
Jawab: B

9 Larutan akua garam garam berikut ini, manakah yang mempunyai tekanan uap paling rendah
pada 25°C?
A. 0,01 m CaCl2
B. 0,01 m NaCl
C. 0,02 m NaCl
D. 0,015 m CaCl2
E. 0,01m Na2SO4
Jawab: D

10. Diberikan data termodinamik berikut ini pada 25° C:


2 HCl(g) + F2(g)  2 HF(l) + Cl2(g) ∆H° = -988 kJ/mol
H2(g) + F2(g)  2 HF(l) ∆H° = -1200 kJ/mol
2 H2(g) + O2(g)  2 H2O(l) ∆H° = -572 kJ/mol
Hitunglah ∆H° dari reaksi berikut ini: 4 HCl(g) + O2(g)  2 H2O(l) + 2 Cl2(g)
1. -148 kJ/mol
2. 996 kJ/mol
3. 1184 kJ/mol
4. -3748 kJ/mol
5. -2760 kJ/mol

Jawab: A

11. Dalam system tertutup dari persamaan reaksi kesetimbangan berikut: ZnCO 3(s)
 ZnO(s) + CO2(g). Pernyataan berikut manakah yang benar?
1. Kc = [CO2][ZnO]/[ZnCO3]
2. Sistem tidak akan mencapai kesetimbangan.
3. Tekanan parsial CO2 diperbesar menyebabkan ZnCO3(s) lebih banyak.
4. Penambahan volume wadah akan menyebabkan bertambahnya ZnCO3(s).
5. Penambahan ZnO(s) menyebabkan bertambahnya ZnCO3(s).
Jawab: C

Olimpiade Kimia Kabupaten Kota 2010 Page 8


L. Didalam pelarut air, zat yang terlarut dapat bersifat sebagai elekrolit kuat, lemah dan non-
elektorlit. Berikut ini adalah larutan larutan senyawa dalam air yang konsentrasinya masing
masing 0,1M
Larutan gula (non-elektrolit)
Larutan NaCl
Larutan Na2SO4
-5
Larutan Asam asetat (Ka= 1,8 x 10 )
Urut-urutan daya hantar listrik ke 4 larutan tersebut, mulai dari yang terbesar adalah:

a >b > c > d


b > c >d > a
c > d> b > a
c > b > d >a
c > d >a > d.
Jawab: D
-3
13. Larutan p-nitrophenol yang konsentrasinya 1,00x10 M dititrasi dengan larutan NaOH yang
-3 -8
konsntrasinya 1,00x10 M. Bila nilai Ka dari p-nitrophenol adalah 7,08x10 pada 25º C.,
berapa pH pada titik ekivalen ?:

A. 8,92
B. 8,62
C. 7,15
D. 5,10
E. 10,20
Jawab: C

14. Berikut ini, manakah pernyataan yang tidak benar:


A
+ - -14
1. [H3O ][OH ] = 1.0x10 di dalam semua larutan akua pada 25° C.
+ -
2. [H3O ] = [OH ] di dalam semua larutan akua netral.
3. pH = 7 hanya di dalam air murni pada 25° C.
+ -7
4. [H3O ] >1.0x10 di dalam larutan akua asam.
+ - -7
5. [H3O ] = [OH ] = 1.0x10 di dalam air murni pada 25° C.

Jawab: C
-9
15 Berapa pH larutan yang mengandung 3,98 x 10 M ion hidroksida dalam air?

A. 8,400
B. 5,600
C. 9,000
D. 3,980
E. 7,000

Jawab: B

Olimpiade Kimia Kabupaten Kota 2010 Page 9


17. Menurut teori Asam-Basa Bronsted-Lowry, assam didefenisikan sebagai:
+
A. meningkatkan [H ] bila dimasukkan dalam H2O
+
B. menurunkan [H ] bila dimasukkan dalam H2O
-
C. meningkatkan [OH ] bila dimasukkan dalam H2O
D. bertindak sebagai aseptor proton dalam setiap system
E. bertindak sebagai donor proton dalam setiap system

Jawab:E

17. Berikut ini yang manakah asam Lewis tetapi bukan asam Brønsted-Lowry?
A. NH3
B. HF
C. BF3
D. HNO3
E. NaOH

Jawab: C

18. Berikut ini, manakah yang BUKAN merupakan reaksi asam-basa Lewis?
2+ +2
Cu (aq) + 4NH3 (aq)  [Cu(NH3 )] (aq)

HCl(g) + NH3 (g)  NH4Cl(s)


+ -
H (aq) + OH (aq)  H2O(l)
2Na(s) + Cl2 (g) 2NaCl(s)
+ +
H (aq) + H2O(l)  H3O (aq)
Jawab: D

19 Bila diketahui pKa CH3OH = 16.5 dan pKa CH3CO2H = 5, Kemanakah arah kecenderungan
dari reaksi kesetimbangan berikut ini?
CH3OH(aq) + KO2CCH3(aq)   KOCH3 (aq) + CH3CO2H(aq)

A Kearah kiri kesetimbangan


B Ke arah kanan kesetimbangan
C Tergantung entalpi reaksinya
D Bila tekanan dinaikkan
E Tidak ada kecenderungan reaksi
Jawab: A

20. Berikut ini, pasangan manakah yang tidak dapat membentuk campuran buffer?
NH3 , NH4Cl
NaCH3O2 , HCl
RbOH, HBr
KOH, HF
H3PO4 , KH2PO4
Jawab: C

Olimpiade Kimia Kabupaten Kota 2010 Page 10


-9
21. Setiap garam berikut ini mempunyai nilai Ksp 1,00 x 10 . Berikut ini garam manakah
yang paling mudah larut dalam air.
XY
XY2
X3Y
XY3
X2Y3

Jawab: E
-2 +
22. Dalam setengah reaksi oksidasi berikut ini: 4H2O + H2S  SO4 + 10H
Berapa banyak elektron yang dilepaskan dalam reaksi
2
4
6
8
10
Jawab: D
–1 –1
W. Untuk reaksi: A(g) + B(g) Produk; laju awal reaksinya adalah 0,056 mole.L s .
Kemudian reaksi tersebut didiamkan beberapa lama, dan setelah 10 menit diukur kembali
–1 –1
lajunya. Ternyata laju reaksinya juga 0.056 mole.L s . Berapa order reaksi tersebut?
Order Nol
Order kesatu
Order kedua
Order negative
Tak dapat ditentukan
Jawab: A

24. Berikut ini, manakah yang merupakan reaksi redoks?


- -
AlH3(g) + H (g)  AlH4 (g)
3+ -
Al (aq) + 3 OH (aq)  Al(OH)3(s)
2 Al(s) + 3 Cl2(g)  2 AlCl3(s)
- +
AlO2 (aq) + H (aq) + H2O(l)  Al(OH)3(s)
Al2Cl6(g)  2 AlCl3(g)

Jawab: C

Olimpiade Kimia Kabupaten Kota 2010 Page 11


25. Bila diketahui:
2+ - o 2+ - o
Cd + 2e  Cd E red = -0.40 V; Zn + 2e  Zn E red = -0.76 V
2+ - o 2+ - o
Ni + 2e  Ni E red = -0.25 V ; Cu + 2e  Cu E red = +0.34 V
Berdasarkan data tersebut, manakah dari spesi berikut ini yang merupakan oksidator
terbaik?
1. Cd
2+
2. Zn
3. Ni
4. Cu
2+
5. Cu

Jawab: E

26. Berapa banyak isomer yang terdapat dalam senyawa dengan rumus molekul C5H12 ?
2
3
4
5
6
Jawab: B

27. Tentukan produk utama yang akan diperoleh bila etanol dipanaskan dengan H2SO4 :
etana
etena
etuna
etilsulfat
salah semua
Jawab: C

28. Mengenai dua senyawa alkena berikut ini : CH3CH2CH=CH2 dan CH3CH=CHCH3,
manakah pernyataan yang benar ?

Keduanya tidak menunjukkan isomer cis-trans


Keduanya tidak dapat dipolimerisasi
Keduanya tidak bereaksi dengan brom membentuk 1,4-dibromobutana
Keduanya tidak bereaksi dengan hidrogen membentuk butana
Keduanya tidak dapat dioksidasi oleh alkali kalium manganat (VII) membentuk
diol.
Jawab: C

Olimpiade Kimia Kabupaten Kota 2010 Page 12


30. Dalam pembuatan etena, ke dalam etanol ditambahkan setetes reagen Y sambil
dipanaskan. Etena yang tidak murni dicuci dengan cara di bubble melalui larutan Z dan
kemudian dikumpulkan. Apakah reagen Y dan larutan Z yang mungkin digunakan ?:

Reagen Y larutan Z
A. K2Cr2O7 diasamkan larutan NaOH
B. H2SO4 pekat H2SO4 encer
C. H2SO4 pekat NaOH encer
D. NaOH etanolat H2SO4 encer
E. NaOH etanolat NaOH encer
Jawab: C

31. Asam karboksilat yang sama dapat diperoleh melalui hidrolisis senyawa nitril P atau melalui
oksidasi senyawa alkohol Q. Yang manakah pasangan berikut ini sebagai P dan Q ? :

P Q
A. CH3CH2CN CH3CH2OH
B. (CH3)2CHCN (CH3)3COH
C. C6H5CH(CH3)CN C6H5CH2CH(OH)CH3
D. C6H5CH2CN C6H5CH2CH2OH
E. C6H5CN C6H5OH

Jawab: D

Olimpiade Kimia Kabupaten Kota 2010 Page 13


2. Selesaikanlah Soal berikut ini:

Soal 1. Entalpi reaksi (7 poin).

Dengan menggunakan data berikut ini:


o
H2(g) 2H(g) H = + 436,0 kJ
o
Br2(g)2Br (g) H = +193,9 kJ
o
HBr(g) H(g) + Br(g) H = +365,7 kJ
o
Hitunglah berapa entalpi standard dari reaksi ( Hr ):
H2(g) + Br2(g)  2HBr (g)

Solusi:

Persamaan reaksi terakhir dibalik dan x 2:

o
2H(g) + 2Br(g) 2HBr(g) H = - 2x 365.7 kJ (2 poin)

Kemudian dijumlahkan dengan dua persamaan reaksi yang pertama, (3 poin)


diperoleh
o
H2(g)  2H(g) H = + 436,0 kJ
o
Br2(g)  2Br (g) H = +193,9 kJ
o
2H(g) + 2Br(g)  2HBr(g) H = - 2x 365.7 kJ
____________________________________________ +
o
H2(g) + Br2(g) 2HBr (g) Hr =
o
Hr = + 436,0 +193,9 – 2x365,7 = -101.5 kJ (2 poin)

Soal 2. Reaksi pengendapan. (10 poin)


Sebanyak 5,0 g contoh CaCl2 yang bercampur dengan zat lainnya, dilarutkan dalam air
bersama dengan kalium sulfite (K 2SO3) . Setelah didiamkan, ternyata menghasilkan
endapan garam tak-larut kalsium sulfit. Sesudah endapan dipisahkan dan dikeringkan,
ternyata diperoleh berat endapan sebanyak 2,1 g. Bila diketahui massa molar kalsium
sulfit adalah 120,14 g/mol dan CaCl2 adalah 110,99 g/mol, maka:
(a) Tuliskan persamaan reaksi yang terjadi, dan juga persamaan ion untuk reaksi tersebut
(2 poin)
CaCl2 (aq) + K2SO3 (aq)  CaSO3 (s) + 2KCl (aq)
2. Tuliskan persamaan ion untuk reaksi tersebut (2 poin)
2+ 2-
Ca + SO3  CaSO3 (s)
3. Berapa persentase berat CaCl2 dalam contoh semula?. (6 poin)
Mol CaSO3 (s) = 2,1 g / 120,14 (g/mol) (1 poin)
mol CaCl2: mol CaSO3 = 1 : 1 (1 poin)

Olimpiade Kimia Kabupaten Kota 2010 Page 14


mol CaCl2 = (mol CaSO3) x 110,99 g/mol = 1,94 g CaCl2 (2 poin) %

CaCl2 dalam contoh = 1,94/5,0 X 100% = 38,8 % ≈39% (2 poin)

Soal 3. Kesetimbangan Heterogen (13 poin)

Amonium Karbamat (NH2COONH4) adalah garam asam karbamat yang terdapat dalam darah
o
dan urin mamalia. Pada temperatur 250 C terjadi kesetimbangan sbb:
-8
NH2COONH4(s)  2 NH3 (g) + CO2 (g), dengan nilai Kc = 1,58 x 10 .
Bila 7,8 g NH2COONH4 (78 g/mol) dimasukkan kedalam wadah yang volumenya 0,5 liter maka:
a. Tuliskan pernyataan Kc untuk reaksi kesetimbangan tersebut. (1 poin)
b. Apakah reaksi tersebut akan mencapai kesetimbangan (3 poin)
c. Pada temperatur 250 oC , tentukan berapa konsentrasi NH3 dan CO2 bila telah
tercapai kesetimbangan. (5 poin)
d. Tentukan berapa tekanan didalam wadah bila telah tercapai kesetimbangan.
(2 poin)
o
e. Tentukan nilai Kp reaksi tersebut pada temperatur 250 C (2 poin)
Diketahui : tetapan gas universal, R= 0,082057L.atm/K.mol

Jawab:
2
a. Kc = [NH3] [CO2] (1 poin)
,
b. 7,8 g NH2COONH4 = / 0,05 (1 poin)

reaksi: NH2COONH4 (s)  2 NH3 (g) + CO2 (g),


2x x

2 -4
Seandainya terurai sempurna: Q= (0,05) (0,05)=1,25 x 10 > Kc
Q>Kc maka: kesetimbangan bergeser kearah reaktan (ke kiri), berarti dengan jumlah 3,9 g
NH2-COONH4 dalam ruang tertutup reaksi akan mencapai kesetimbangan. (2 poin)

-8
c. reaksi: NH2COONH4 (s)  2 NH3 (g) + CO2 Kc= 1,58x10 2x
x
-8 2
1,58 x 10 .= (2x) .(x)
-8 3
1,58 x 10 = 4x
Olimpiade Kimia Kabupaten Kota 2010 Page 15
1/ 3
8
1,58 -3 -3
x 10 =1,5808 x 10 1,581 x 10 (3 poin)
4

-3 -3
[NH3] = 2 x1,581 x10 = 3,162 x 10 M (1 poin)
-3 -3
[CO2] = 1 x 1,581 x 10 =1,581 x 10 M (1 poin)

d. PV =nRT

p n RT MRT
V
-3 -3
p = (3,162 x 10 + 1,581 x 10 )mol/L .( 0,082057L.atm/K.mol) (273 +

250)K p= 0,2035 atm

Tekanan dalam wadah, p = 0,2035 atm (2 poin)

n -8 3
e. Kp = Kc(RT) = 1,58 x 10 .( 0,082057 x 523)

-3 -3
= 1,249 x10 1,25 x 10 (2 poin)

Soal 4. Senyawa kompleks koordinasi Cisplatin (19 poin)


Cisplatin adalah senyawa kompleks koordinasi yang memiliki rumus kimia [Pt(NH3)2Cl2]
dan berperan sebagai zat anti tumor. Zat ini dibuat dari reaksi kimia antara kalium
tetrakloro platinate dengan gas amonia. Jika awalnya ada 10 g kalium tetrakloroplatinate
dan 10 g amonia, serta diketahui Massa atom relatif K = 39,1; Pt = 195,1; N = 14 dan H =1,
maka
1 Tuliskan rumus kimia kalium tetrakloro platinate (3 poin)
2 Tuliskan persamaan reaksi pembentukan cisplatin (2 poin)
3 Berapa jumlah mol kalium tetrakloroplatinate?
4 Berapa pula jumlah mol amonia?
5 Zat mana yang jumlahnya berlebihan dan berapa kali lebih besar jumlahnya?
6 Jika reaksi berlangsung secara stoikiometri, berapa g sisa zat yang berlebih?
7 Berapa g cisplatin yang terbentuk?

Jawab:

Olimpiade Kimia Kabupaten Kota 2010 Page 16


a. Rumus kimia: kalium tetrakloro platinate: K2PtCl4 (3 poin)
b. K2PtC4 + 2NH3 Pt(NH3)2Cl2 + 2KCl (2 poin)

c. jumlah mol K2PtCl4 = / 0,0241


,

jumlah mol K2PtCl4 = 0,0241 mol (2 poin)

d. jumlah mol NH3 = 0,588 (2 poin)

jumlah mol NH3 =0,588 mol

5. Yang berlebihan adalah Amonia dan jumlahnya 24,4 kali lebih besar dari
K2PtC4 (2 poin)
6 Sisa NH3 = 0,588mol - (2x0,0241)mol = 0,5398 mol
= 0,5398 mol x 17 g/mol = 9,18 g 9,2 g (3 poin)
7. Jumlah cisplatin yang terbentuk= 0,0241 mol:
=0,0241mol x 300,1 g/mol = 7,23 g (3 poin)

Soal 5. Gas beracun fosgen


Fosgen adalah gas beracun yang digunakan ketika perang dunia I. Senyawa ini dibuat dari
campuran gas karbonmonoksida dan gas klorin dengan bantuan sinar matahari. Fosgen
mengandung karbon sebanyak 12.14% , oksigen 16.17% , dan klor 71.69% massa. Massa
molarnya 98.9 g/mol.
(a) Dari data tersebut tentukan rumus kimia fosgen (5 poin)
(b) Tuliskan persamaan reaksi pembentukan fosgen (2 poin)
(c) Gambarkan tiga struktur Lewis dari fosgen yang mengikuti aturan Oktet (6 poin)
(d) Nyatakan muatan formal masing-masing atom pada ketiga struktur tersebut (6 poin)
(e) Dengan dasar muatan formal, Tuliskan persamaan reaksi pembentukan fosgen dengan
menggambarkan struktur Lewis semua pereaksi dan produknya

Jawab:

Olimpiade Kimia Kabupaten Kota 2010 Page 17


Massa molarnya 12.01 + 16.00 + 2(35.45) = 98.91 g/mol

Maka Rumus kimianya COCl2 (5 poin)

(b) Persamaan reaksi: CO(g) + Cl2(g)  COCl2(g) (2 poin)

(c)

Masing masing struktur 2 poin

(d)

Masing masing 2 poin

(e) Reaksi:

(3 poin)

Olimpiade Kimia Kabupaten Kota 2010 Page 18


Soal 6. eaksi Klorinasi Alkana (12 po n)

Tuliskan rumus dari semua produk substitus i monoklori nasi yang dihasilkan dari
reaksi substitusi senyawa b erikut dengan Cl2:
(a) heksana ; (3 poin)
(b) 3-metilpenta a; (4 poin)
(c) M etilsikloheksana (5 poin)

JAWAB:
(a) ClC H2CH2CH2CH2CH2CH3 ; CH3CH(Cl)CH2CH2C H2CH3 ;
CH3 CH2CH(Cl)CH2CH2CH3 (masing masing 1 poin)
(b) ClCH2CH2CH(CH3 )CH2CH3 ; CH3CH(Cl) H(CH3)CH2 CH3 ;
CH3CH2C(Cl)(CH3) CH2CH3 ; C H3CH2CH(CH2Cl)CH2C 3

(masing masing 1 oin)

(c). (mas ing masin 1 poin)

***RSL***

Olimpiad e Kimia Kab upaten Kot a 2010 Page 1


OLIMPIADE SAINS NASIONAL 2012
SELEKSI KABUPATEN / KOTA

JAWABAN
(DOKUMEN NEGARA)

UjianTeori
Waktu: 100 menit

Kementerian Pendidikan Nasional Dan Kebudayaan


Direktorat Jenderal
Managemen Pendidikan Dasar dan Menengah
Direktorat Pembinaan Sekolah Menengah Atas

2012
Petunjuk
1. Isilah Biodata anda dengan lengkap (di lembar Jawaban)
Tulis dengan huruf cetak dan jangan disingkat !

2. Soal Teori ini terdiri dari dua bagian:

20 soal pilihan Ganda = 40 poin

4 Nomor soal essay = 69 poin

TOTAL = 109 poin

3 Waktu yang disediakan: 100 menit.

4 Semua jawaban harus ditulis di lembar jawaban yang tersedia

5 Diperkenankan menggunakan kalkulator.

6 Diberikan Tabel periodik Unsur.


7 Anda dapat mulai bekerja bila sudah ada tanda mulai dari pengawas.
8 Anda harus segera berhenti bekerja bila ada tanda berhenti dari Pengawas.
9 Letakkan jawaban anda di meja sebelah kanan dan segera meninggalkan ruangan.
10 Anda dapat membawa pulang soal ujian !!

OSK2012 ii
LEMBAR JAWABAN

Bagian A

Beri Tanda Silang (X) pada Jawaban Yang Anda Pilih

No Jawaban No Jawaban

1 A B C D E 11 A B C D E

2 A B C D E 12 A B C D E

3 A B C D E 13 A B C D E

4 A B C D E 14 A B C D E

5 A B C D E 15 A B C D E

6 A B C D E 16 A B C D E

7 A B C D E 17 A B C D E

8 A B C D E 18 A B C D E

9 A B C D E 19 A B C D E

10 A B C D E 20 A B C D E

OSK2012 iii
OSK2012 iv
Tetapan dan rumus berguna

Tetapan (bilangan) Avogadro NA = 6.022∙1023 partikel.mol–1

Temperatur K = OC + 273,15

R= 8,314 J.K-1.mol-1
= 8,314 x107 erg. Mol-1.K-1
Tetapan gas universal, R =1,987 cal.mol-1.K-1
= 0,082054 L.atm.mol-1.K-1
Tekanan gas 1 atmosfir =760 mmHg =760 torr
Persamaan gas Ideal PV= nRT
Tekanan Osmosa pada larutan p =c RT
Muatan elektron 1,6022 x 10-19 C
Tetapan Kesetimbangan air (Kw) pada
25oC Kw= 1,0x10

OSK2012 v
BAGIAN A

Pilih Jawaban yang palin tepat (40 poin)

1. Reaksi fasa gas berlangsung dalam syringe pada temperature dan tekanan
konstan. Bila volume awal adalah 40 cm3 dan volume akhir adalah 60 cm3, reaksi
manakah yang berlangsung?
A. A(g) + B(g)  AB(g)
B. 2A(g) + B(g)  A2B(g)
C. 2AB2(g)  A2(g) + 2B2(g)
D. 2AB(g)  A2(g) + B2(g)
E. 2A2(g) + 4B(g)  4AB(g)
2. Yang manakah berikut ini mengandung dua mol partikel zat terlarut?
1,0 dm3 dari 0,50 mol.dm-3 Na2SO4(aq)
1,0 dm3 dari 0,20 mol.dm-3 Al2(SO4)3(aq)
4,0 dm3 dari 0,25 mol.dm-3 CH3CO2Na(aq)
8,0 dm3 dari 0,125 mol.dm-3 CH3CO2H(aq)
1,0 dm3 dari 0,33 mol.dm-3 Cu(NH3)4SO4(aq)

3. Sebanyak 20 cm3 gas hidrokarbon terbakar sempurna dalam oksigen berlebihan


dan menghasilkan 60 cm3 karbon dioksida serta 40 cm3 uap air ( STP ).
Bagaimanakah rumus molekul dari hidrokarbon tersebut.
A. C2H6
B. . C3H6
C. C3H4
D. C3H8
E. C6H6

D. Perak (nomor atomnya 47) suatu target yang atraktif untuk sintesis nanokristal,
mempunyai dua isotop alami: perak-107 dan perak-109. Kelimpahan relatif dari
dua isotop ini adalah 51,84% atom untuk perak-107 dan 48,16% atom untuk perak-
109.
Jumlah proton, neutron dan elektron untuk kation 109Ag2+ yaitu:

Proton neutron elektron

A. 47 62 45
B. 47 61 46
C. 47 60 45
D. 47 62 47
E. 47 61 45

OSK2012 1
5. Berikut ini berturut-turut adalah delapan energi ionisasi ( kJ / mol ) unsur X:
703; 1610; 2460; 4350; 5400; 8500; 10.300; 12300
Berdasarkan energi ionisasi teresebut dapat diperkirakan bahwa unsur X adalah
unsur Utama Golongan

II
III
IV
V
VI

F. Suatu atom unsur X, oksidanya dapat berupa XO2 dan XO3. Bila oksida tersebut
dilarutkan dalam air akan terbentuk asam lemah H2XO3 dan asam kuat H2XO4 .
Bagaimanakah konfigurasi elektronik yang paling luar dari unsur X tersebut?

s2 p2
s2 p3
s2 p4
s2 p5
s2 p6

G. Berikut ini, manakah diagram orbital yang benar untuk atom 7N dalam keadaan
dasar:

Jawab: D

OSK2012 2
8. Berikut ini, manakah yang tepat mengenai deskripsi dot-elektron untuk sulfur
dioksida, SO2?

Jawab: B

9. Tentukanlah spesi dibawah ini YANG PALING MUDAH LARUT dalam propana
cair, CH3CH2CH3(l):
CH3 CH2 CH2 CH3( l )
CH3 CH2 CH2 CH2 Cl( l )
CH3 CH2 CH2 CH2 OH( l )
CH3 CH2 CH2 COOH( l )
H2 O( l )

10. Berdasarkan reaksi berikut ini :


KIO3(aq) + 5KI(aq) + 6H+(aq)  3I2(aq) + 6K+(aq) + 3H2O(l):
Berapa mol iodine yang dapat diperoleh dari 1/6 mol Kalium iodat(V) ?
1. 1 mol
2. 0,75 mol
3. 0,50 mol
4. 0,25 mol
5. 0,20 mol

11. Dari data berikut ini pada 25° C,


2 NOCl(g) → 2 NO(g) +Cl2 (g) H0 = +75.56 kJ
2 NO(g) + O2(g) → 2 NO2(g) ∆H0 = –113.05 kJ
2 NO2(g) → N2O4(g) ∆H0 = –58.03 kJ

OSK2012 3
Berapa nilai ∆H0 pada 25°C untuk reaksi berikut ini:
N2O4(g) + Cl2(g) → 2 NOCl(g) + O2(g)

1. +95,52 kJ
2. +299 kJ
3. –186. 8 kJ
4. -85,52 kJ
E. –246.65 kJ

12. Uap hidrogen dan iodine berada dalam kesetimbangan dengan hidrogen iodida di
dalam ‘gas syringe’pada temperatur konstan: H2 + I 2  2HI,

Yang manakah berikut ini akan bertambah besar bila tekanan dinaikkan pada
temperatur konstan? (asumsikan campuran ini bersifat ideal)

Energy aktivasi
perubahan entapi
nilai Kp
tekanan parsial dari HI
konstanta laju untuk reaksi ke kanan.

13 Kolam renang umum sering diklorinasi untuk membunuh bakteri. Sebagai suatu
alternatif klorinasi, ion perak dapat dipakai dalam konsentrasi tidak lebih dari 10 -6 mol
dm-3 dan tidak kurang dari 10-7 mol dm-3 dari ion perak. Yang manakah berikut
ini senyawa yang dalam larutan jenuh memberikan konsentrasi ion perak yang
dibutuhkan?
Hasil kali kelarutan(Ksp) Senyawa
5 x 10-13 mol2 dm-6 A. AgBr
2 x 10-10 mol2 dm-6 B. AgCl
2 x 10-8 mol2 dm-6 5 C. AgIO3
x 10-12 mol3 dm-9 2 x D. Ag2CO3
10-5 mol3 dm-9 E. Ag2SO4

14. Perhatikan Gambar dibawah


ini:

Kurva I diperoleh dengan dara mereaksikan 1 gram logam Zn granular dengan HCl
berlebihan pada tem[peratur 30oC.

OSK2012 4
Tentukanlah apa perubahan kondisi dalam reaksi yang menghasilkan kurva II
6. Bila digunakan bubuk Zn
7. Pergunakan hanya ½ gram Zn
8. Panaskan asam HCl sampai 40oC
9. Encerkan asam HCl dengan air.
10. Jawaban A, B, C dan D semua betul

15. Perhatikan reaksi yang belum setara berikut ini :


MnO4- (aq) + H+(aq) + Fe2+ (aq)  Mn2+(aq) + Fe3+(aq) + H2O (l)
Pernyataan yang benar mengenai reaksi tersebut adalah:
MnO4- adalah reduktor dan Fe2+ adalah oksidator
Bilangan oksidasi Mangan berubah dari +2 menjadi +7
Fe2+ mengambil electron dari MnO4-
Besi mengalami reduksi dan MnO4- mengalami oksidasi
Fe2+ merupakan reduktor

16. Bila kedalam larutan alkali dialirkan gas klorin (Cl2), dalam larutan tersebut Cl2
berubah menjadi ion-ion Cl- dan ClO3-. Perbandingan mol ion Cl- dan ion ClO3-yang
terdapat dalam larutan adalah:

1:1
2:1
3:1
5:1
7:1
17. Molekul atau ion dibawah ini, manakah yang tidak dapat bertindak sebagai basa
Lewis?
Cl–
CN–
NH3
H2O
BF3

18. Dalam suatu eksperimen, larutan 50 cm3 0,1 mol dm-3 dari larutan garam logam
tertentu secara tepat bereaksi dengan 25 cm 3 0,1 mol dm-3 larutan akua natrium
sulfit. Persamaan setengah reaksi untuk oksidasi ion sulfit adalah:
SO32-(aq) + H2O(l)  SO42-(aq) + 2H+(aq) + 2e-.
Jika bilangan oksidasi awal logam dalam garam ini adalah +3 , berapa bilangan
oksidasi yang baru dari logam ini?
1. 0
2. +1
3. +2
4. +3
5. +4

OSK2012 5
19. Pernyataan yang tidak tepat untuk kedua senyawa alkena berikut ini:
CH3 – CH2 – CH = CH2 dan CH3 – CH = CH CH3 adalah

1 keduanya mempunyai isomeri geometri


2 keduanya dapat berpolimerisasi
C.keduanya bereaksi dengan bromin membentuk 1, 4 dibromobutana
4. keduanya bereaksi dengan hidrogen membentuk butane
5. keduanya bereaksi dengan KMnO4 dalam suasana alkali membentuk diol

20. Suatu senyawa hidrokarbon X ( Mr = 56 ) ternyata bersifat dapat menghilangkan


warna larutan Br2/CCl4. Senyawa X tersebut adalah.

H H H H H

A. H C C C H B. H C C H

H H H H C C H

H H

H H H H H H H
Jawab: D
D.C C C C H E. H C C C C

H
H
H H H H
OSK2012 6
BAGIAN B

Soal-1 (20 poin)


Oksigen dapat dihasilkan dari pemanasan zat kimia yang kaya oksigen, seperti zat padat
kalium klorat dengan katalis mangan dioksida.
a) Tuliskan rumus kimia kalium klorat dan Mangan dioksida (2 poin)
b) Tuliskan reaksi pemanasan garam tersebut yang menghasilkan oksigen (2 poin)
o
c) Dari pemanasan 2 g kalium klorat, berapa L oksigen diperoleh jika diukur pada 25 C
dan 1 atm dan reaksi dianggap berlangsung sempurna (4 poin)

Oksigen juga dapat dibuat dari reaksi natrium peroksida dengan air
d) Tuliskan reaksi natrium peroksida dengan air (2 poin)
o
5. Untuk mendapatkan 1 L oksigen pada 25 C 1 atm, berapa banyak natrium peroksida
yang diperlukan jika dianggap reaksi penguraian berlangsung sempurna (4 poin)
f) Berapa bilangan oksidasi oksigen pada hidrogen peroksida? (1 poin)

Senyawa yang mengandung oksigen disebut oksida


g) Tuliskan reaksi belerang dengan oksigen, oksida apa yang dihasilkan? (2 poin)
h) Tuliskan reaksi kalsium dengan oksigen, oksida apa yang terbentuk? (2 poin)
i) Oksida mana yang bersifat asam? Buktikan (1 poin)

Jawab:
F. KClO3 dan MnO2
G. 2KClO3  2KCl + 3O2
H. O2 = 3/2*2/122,5*22,4 L = 0,55L
I. 2Na2O2 + 2H2O  4NaOH + O2
J. Na2O2 = 2*78/22,4 = 6,96g
K. 2- untuk O2
L. S + O2  SO2 oksida asam
M. Ca + O2  CaO oksida basa
N. SO2 + H2O  H2SO3  HSO3- + H+

OSK2012 7
Soal-2 (20 poin)

Furan adalah suatu senyawa organik yang berwujud cair pada keadaan standar yang hanya
mengandung karbon, hidrogen, dan oksigen. Senyawa ini banyak digunakan sebagai pelarut
dan bahan baku nilon. Furan mengandung 70,6% karbon dan 5,8% hidrogen.
1. Tentukan rumus empiris furan! (3)
2. Menurut percobaan, massa molekul relatif furan adalah sekitar 70. Tentukan
rumus molekul furan! (2)
3. Tuliskan persamaan reaksi, termasuk fasanya, yang menunjukkan reaksi
pembentukan furan dari unsur-unsurnya pada keadaan standar!(2)
4. Gunakan persamaan pada (c) dan data perubahan entalpi standar berikut:

C(s) →C(g) H0 = +717 kJ/mol;


½H2(g) → H(g) H0 = +218 kJ/mol;
½O2(g) → O(g) H0 = +248 kJ/mol
Hitunglah berapa perubahan entalpi reaksi penguraian cairan furan menjadi
unsur-unsurnya pada fasa gas! Tuliskan persamaan reaksinya! (4)
5. Furan adalah senyawa organik lingkar lima yang memiliki gugus eter (C-O-C)
dan memiliki dua ikatan rangkap. Gambarkan struktur furan!(2)
6. Berdasarkan struktur furan dan data energi ikatan berikut: C-C : 348 kJ/mol;
C=C: 612 kJ/mol; C-H: 412 kJ/mol; C-O: 360 kJ/mol; perkirakan perubahan
entalpi untuk penguraian gas furan menjadi atom-atomnya pada fasa gas!
Tuliskan persamaan reaksinya! (4)
7. Dengan menggunakan jawaban soal (d) dan (f), tentukan perubahan entalpi
untuk proses perubahan wujud dari cairan furan menjadi gas furan! Tuliskan
persamaan reaksinya! (3)

Jawab:
a. % berat O = 100(-70,6 +5,8) = 23,6 %
C : H : O= 70,6/12 : 5,8/1 : 23,6/16 = 5,9 : 5,8 : 1,475 = 4 : 4 : 1
Rumus empiris furan = C4H4O
2. berat molekul = 4(12) +4(1)+ 16 = 68
Rumus molekul furan =(C4H4O)n
n =68/70 = ~1
Rumus molekul furan = C4H4O

3. 4C(s) + 2H2(g) + ½ O2(g) → C4H4O(l)

OSK2012 8
4 Reaksi: C4H4O(l) → 4C(g) + 4H(g) + O(g), H0 = +4050 kJ/mol
5 Struktur furan:
O

6. Reaksi: C4H4O(g) → 4C(g) + 4H(g) + O(g), H0 = +3940 kJ/mol


7. Reaksi: C4H4O(l) → C4H4O(g), H0 = +111 kJ/mol

Soal 3 (9 poin)

Nilai pH darah dalam tubuh manusia dijaga dalam rentang yang sempit yaitu dari 7,35
hingga 7,45 oleh sistem buffer alami yang terdiri dari asam karbonat, H 2CO3(aq) dan ion
hidrogenkarbonat, HCO3-(aq)
1. Larutan buffer yang konsentrasi H2CO3(aq) dan HCO3-(aq) sama, mempunyai pH
= 6,10. Hitunglah konstanta dissosiasi, Ka, untuk H2CO3(aq). (2 poin)
2. Hitunglah rasio (perbandingan) konsentrasi HCO3-(aq) dan H2CO3(aq) dalam
darah pada pH = 7,40 (3 poin)
c. Apakah pH darah seseorang naik ataukah turun selama kerja fisik? Jelaskan
jawaban anda! (2 poin)
d. Jelaskan dengan singkat mengapa dalam sistem buffer H2CO3(aq)/HCO3-(aq)
dapat menjaga pH darah. (2 poin)

Jawab:

1. Ka = 7,94 x 10-7 mol dm-3.


2. 20
3. Naik, karena kerja menghasilkan CO2, kadar H2CO3 turun kadar basa relatif tetap
sehingga pH naik.
4. Perubahan baik oleh pengenceran, penambahan sedikit asam atau basa dalam
persamaan fungsi logaritma pengaruhnya kecil, sehingga perubahan pH yang
relatif kecil dapat diabaikan.

Soal-4 (Total: 20 poin)

Ketika natrium etanoat (CH3CO2Na) dilarutkan dalam air, terjadi reaksi

kesetimbangan sebagai berikut: ⇌


CH3CO2-(aq) + H2O(l) CH3CO2H(aq) + OH-(aq)

Larutan natrium etanoat 0,01 M memiliki pH = 8,87.


a. Hitung konsentrasi H+ dalam larutan! (3 poin)

OSK2012 9
2 Hitung pula konsentrasi OH- dalam larutan! (2 poin)
3 Hitung konsentrasi asam etanoat dalam larutan! (2 poin)
4 Tentukan tentukan tetapan disosiasi asam, Ka, untuk asam etanoat! (4 poin)
5 Hitung massa (dalam gram) natrium etanoat yang harus ditambahkan ke
dalam 1 L larutan asam etanoat 0,10 M (dengan nilai K a yang diperoleh dari
jawaban soal (d). Jika tidak diperoleh jawaban pada (d), gunakan nilai K a
asam etanoat= 2x10-5) untuk menghasilkan larutan bufer (penyangga)
dengan pH 4,7! (4 poin)
6 Gambarkan struktur molekul asam etanoat! (2 poin)
7 Jika asam etanoat direaksikan dengan isopropil alkohol (isopropanol),
gambarkan struktur produk yang terbentuk dan tuliskan nama IUPAC-nya! (3
poin)

Jawab:
+ 8,87 9
1. [H ] = 10 = 1,35 x 10 M
+ 14 9 6
2. [OH ] = Kw/[H ] = 10 /1,35 x 10 M = 7,41 x 10 M
6
3. [CH3CO2H] = [OH ] = 7,41 x 10 M
4. Dari persamaan kesetimbangan di atas diperoleh:
62 9
Kb ≈[CH3CO2H][OH ]/[ CH3CO2 ] = (7,41 x 10 ) /(0,01) = 5,49 x 10
14 9 6
Ka = Kw/Kb = 10 /5,49 x 10 = 1,82 x 10
6
5. Jika menggunakan nilai Ka = 1,82 x 10 , maka:
pH = pKa + log[ CH3CO2 ]/ [CH3CO2H]
4,7 = 5,74 + log[ CH3CO2 ]/ (0,1) -1,04 + log(0,1) = log[ CH 3CO2 ] log[
CH3CO2 ] = -2,04 [ CH3CO2 ] = 0,0091 M.
Dalam 1 L larutan maka mol CH3CO2 = mol CH3CO2Na = 0,0091 mol
Maka massa natrium etanoat = 0,0091 mol x 82 g/mol = 0,7462 g
5
Dengan cara yang sama, jika digunakan nilai Ka asam etanoat= 2x10 , maka
[ CH3CO2 ] = 0,1 M, sehingga dalam 1 L larutan terdapat mol CH3CO2 = mol
CH3CO2Na = 0,1 mol. Maka massa natrium etanoat = 0,1 mol x 82 g/mol = 8,2 g

OSK2012 10
O

C
H3 C OH
6. asam etanoat

O CH3

C CH
H3 C O CH3
g. (1-metil)etil etanoat atau isopropil etanoat

SEMOGA BERHASIL
OSK2012
11
OLIMPIADE SAINS NASIONAL 2012
SELEKSI KABUPATEN / KOTA

JAWABAN
(DOKUMEN NEGARA)

UjianTeori
Waktu: 100 menit

Kementerian Pendidikan Nasional Dan Kebudayaan


Direktorat Jenderal
Managemen Pendidikan Dasar dan Menengah
Direktorat Pembinaan Sekolah Menengah Atas

2012
Petunjuk
1. Isilah Biodata anda dengan lengkap (di lembar Jawaban)
Tulis dengan huruf cetak dan jangan disingkat !

2. Soal Teori ini terdiri dari dua bagian:

A. 20 soal pilihan Ganda = 40 poin

B. 4 Nomor soal essay = 58 poin

TOTAL = 98 poin

3. Waktu yang disediakan: 100 menit.

4. Semua jawaban harus ditulis di lembar jawaban yang tersedia

5. Diperkenankan menggunakan kalkulator.

6. Diberikan Tabel periodik Unsur.


7. Anda dapat mulai bekerja bila sudah ada tanda mulai dari pengawas.
8. Anda harus segera berhenti bekerja bila ada tanda berhenti dari Pengawas.
9. Letakkan jawaban anda di meja sebelah kanan dan segera meninggalkan ruangan.
10. Anda dapat membawa pulang soal ujian !!

OSK2012 ii
LEMBAR JAWABAN

Bagian A

Beri Tanda Silang (X) pada Jawaban Yang Anda Pilih

No Jawaban No Jawaban

1 A B C D E 11 A B C D E

2 A B C D E 12 A B C D E

3 A B C D E 13 A B C D E

4 A B C D E 14 A B C D E

5 A B C D E 15 A B C D E

6 A B C D E 16 A B C D E

7 A B C D E 17 A B C D E

8 A B C D E 18 A B C D E

9 A B C D E 19 A B C D E

10 A B C D E 20 A B C D E

OSK2012 iii
OSK2012 iv
Tetapan dan rumus berguna

Tetapan (bilangan) Avogadro NA = 6.022∙1023 partikel.mol–1

Temperatur K = OC + 273,15

R= 8,314 J.K-1.mol-1
= 8,314 x107 erg. Mol-1.K-1
Tetapan gas universal, R =1,987 cal.mol-1.K-1
= 0,082054 L.atm.mol-1.K-1
Tekanan gas 1 atmosfir =760 mmHg =760 torr
Persamaan gas Ideal PV= nRT
Tekanan Osmosa pada larutan p =c RT
Muatan elektron 1,6022 x 10-19 C
Tetapan Kesetimbangan air (Kw) pada -14
oC
25 Kw= 1,0x10

OSK2012 v
BAGIAN A
Pilih Jawaban Yang paling Tepat (40 poin)

1. Berapa densitas gas Xe (MW=131,3 g/mol) dalam wadah 1,0 L pada 100 oC dan tekanan
2,0 atm?
0.065 g/L
0.65 g/L
8.6 g/L
15.4 g/L
131 g/L

2. Berapa banyak ozon, O3 , yang dapat dibentuk dari 48 g oksigen, O2 ?


1,00 mol
2,00 mol
1,25 mol
2,5 mol
1,50 mol

3. Reaksi pembentukan urea dari gas CO2 dan NH3 :


CO2 (g) + 2 NH3 (g) CO(NH2)2+ 2 H2O

Hitunglah berapa total volume CO2 dan NH3 pada 200,0 atm dan 450°C yang
dibutuhkan untuk menghasilkan 2,50x103 g urea. Masa molar urea adalah 60,06 g/mol.
54,2 liter
40,8 Liter
37,1 Liter
33,6 Liter
25,6 Liter

4 Konfigurasi elektron dari ion S2- adalah (nomor atom S = 16):


A. 1s22s22p63s23p5
B. 1s22s22p63d10
C. 1s22s22p6
D. 1s22s22p63s23p63d10
E. 1s22s22p63s23p6

5 Mengenai atom B (Z=5) berikut ini, manakah pernyataan yang benar


A. Elektron velensi =3; elektron tunggal=1
B. Elektron valensi=3; elektron tunggal=3
C. Elektron valensi=1; elektron tunggal=1
D. Elektron valensi=2; elektron tunggal=1
E. Elektron valensi=5; elektron tunggal=3

6 Berikut ini mengenai radius ion, berturut turut: F -, N3- , P 3-, S= . Urut urutan yang benar
mengenai radius ion tersebut adalah:
A. Radius F- > N3- > P 3- > S=
B. Radius S= > P3- >N3- >F-

OSK2012 1
C. Radius P3- > N3- > S= > F-
D. Radius P3- > S= >N3- > F-
E. Radius N3- >S2-> F- > P3-

7. Konfigurasi elektronik ion G2+ adalah 1s22s22p6 dan unsur L adalah1s22s22p6 3s2 3p5.
Formula senyawa yang paling mungkin terbentuk antara unsur G dan L adalah:
A. GL
B. GL2
C. G2L
D. G2L3
E. G2L5

8. Dari kelompok senyawa berikut ini, manakah yang tidak mematuhi aturan oktet:
A. NH3, PH3, SO3
B. NO2-, SO32-, SCN-
C. H2O, CH2Cl2, CO2
D. HOCl, Cl2CO, N2H4
E. BF3, SF4, ICl3-

9. Reaksi dibawah ini, yang entalpi pembentukan senyawa ( Hf) dan juga entalpi reaksi
( Hr) adalah:
A. 2Al (s) + 3Cl2 (g) 2AlCl3 (g)
B. N2 (g) + 3H2 (g) 2NH3 (g)
C. 2 C (s) + O2 (g)2CO (g)
D. C (s) + O2 (g)CO2 (g)
2S (s) + 3O2 (g)2SO3 (g)

10. Berikut ini, manakah diagram orbital yang benar untuk atom 7N dalam keadaan dasar:

Jawab: D

OSK2012 2
11. Berikut ini, manakah yang tepat mengenai deskripsi dot-elektron untuk sulfur dioksida,
SO2?

Jawab: B
12. Dari spesi molekul dibawah ini, manakah yang diramalkan titik didihnya paling tinggi?
A. CH2Cl2 (l)
B. CH4 (l)
C. Br2 (l)
D. HF(l)
E. HCl (l)

M. Tentukanlah spesi dibawah ini YANG PALING MUDAH LARUT dalam propana cair,
CH3CH2CH3(l):
CH3 CH2 CH2 CH3 ( l )
CH3 CH2 CH2 CH2 Cl( l )
CH3 CH2 CH2 CH2OH(l )
CH3 CH2 CH2 COOH(l )
H2 O( l )
N. Berikut ini, manakah reaksi oksidasi-reduksi. ?
9. PCl3(l ) + Cl2( g )  PCl5( l )
35. Cu(s) + 2 AgNO3(aq) - Cu(NO3)2(aq) + 2 Ag(s)
61. H2CO3(aq) + 2 LiOH(aq)  Li2CO3(aq) + 2 H2O(l)
IV. FeCl2(aq) + 2 NaOH(aq) -Fe(OH)2(s) + 2 NaCl(aq)
A. III
B. I dan II
C. I dan V
D. I, II, dan III
E. I, II, III, dan IV

OSK2012 3
15. Kedalam larutan garam X ditambahkan larutan encer asam klorida dan ternyata garam
tersebut larut dan terbentuk gas dan bila gas tersebut dialirkan kedalam larutan kalsium
hidroksida dihasilkan larutan yang keruh. Setelah reaksi berhenti, kemudian kedalam
larutan ditambahkan ditambahkan larutan amonium sullfat, dan ternyata dihasilkan
endapan berwarna putih.Garam X tersebut adalah:

A. BaSO4
B. BaCO3
C. Cu(NO3)2
D. CaCl2
E. KBr

16. Magnesium oksida (MgO) adalah oksida tahan panas dan dipakai untuk pembuatan
tungku peleburan logam. Magnesium oksida tersebut dapat diperoleh dari air laut
(mengandung MgCl2) dengan cara menambahkan sejumlah larutan kalsium hidroksida
dan terbentuk endapan sesuai reaksi:
Ca(OH)2(aq) + MgCl2(aq)  Mg(OH)2(s) + CaCl2(aq)
Magnesium hidroksida kemudian disaring , kemudian dipanggang membentuk
magnesium oksida. Berikut ini, manakah perbedaan antara kalsium dan magnesium
yang dapat menjelaskan mengapa terbentuk magnesium hidroksida?

A. Magnesium kurang elektropositif daripada kalsium


B. Magnesium lebih rendah reaktivitasnya daripada kalsium dalam golongannya
C. Perubahan entalpi hidrasi Mg2+ kurang eksotermik daripada Ca2+
D. Hasil kali kelarutan (Ksp) Mg(OH)2 lebih rendah daripada Ca(OH)2
E. Besaran energi kisi Mg(OH)2 lebih kecil daripada Ca(OH)2.

17. Anda diberikan tiga larutan berbeda, asam format (Ka = 1,7 x 10-4), fenol (Ka=1,3 x 10-
10
), dan asam asetat (Ka = 1,8 x 10-5), yang masing masing konsentrasi molarnya sama.
Pilihlah urut urutan berikut ini berdasarkan persen disosiasi pada kesetimbangan.
Asam format < Fenol < Asam asetat
Asam format < Asam asetat < Fenol
Asam asetat < Asam format < Fenol
Fenol < Asam asetat < Asam format
Tidak ada jawaban yang benar

18. Dalam reaksi berikut ini:


XeF2 + BrO3- + H2OXe + 2 HF + BrO4-
Manakah pernyataan yang benar?
2. xenon dioksidasi, oksgen direduksi.
3. xenon direduksi, bromine dioksidasi.
4. fluorine direduksi, bromine dioksidasi.
5. xenon direduksi, fluorine dioksidasi
6. bromine direduksi, xenon dioksidasi.

OSK2012 4
19 Tentukan senyawa yang merupakan isomer dari aseton.
A. Propil alkohol
B. Propanon
C. Propion aldehid
D. Asetaldehid
E. Metil etil eter.

20 Untuk mengdentifikasi senyawa alkena sering dilakukan dengan menggunakan pereaksi:

A. HCl
B. HBr
C. H2SO4
D. Br2/CCl4
E. Cl2/CCl4

OSK2012 5
BAGIAN B

Soal 1: (9 poin)

Suatu senyawa tak dikenal yang mengandung unsur C, H dan Cr mempunyai formula
CxHyCrz dan larut dalam pelarut kloroform (CHCl 3). Komposisi unsurnya dianalisis dengan
membakar sejumlah senyawa tersebut. Gas CO2 dan uap H2O hasil pembakaran ditampung
dan ditentukan kandungan C dan H. Massa molar senyawa tersebut ditentukan dengan
menggunakan metoda pengukuran tekanan osmosa larutan zat tersebut dalam pelarut
kloroform. Diperoleh data sebagai berikut:
-Senyawa tersebut, persentase berat unsurnya mengandung 73,94 % C dan 8,27 % H;
sisanya adalah kromium.
-Pada temperatur 25 oC, tekanan osmosa larutan yang mengandung 5,00 mg zat tak
dikenal tersebut dalam 100 mL kloroform menghasilkan tekanan 3,17 mm Hg.

Bila Diketahui: tetapan Gas universal, R = 0,082507 L.atm.K-1.mol-1 = 8,314510 J.K-


1
; 1atm = 760 mm Hg = 760 torr; 0 K = -273,15 oC.
Massa molar: Cr = 52,0 g/mol; C =12,0 g/mol; H=1,0 g/mol

Tentukanlah:
a. Rumus empiris senyawa (3 poin)
b. Massa empiris senyawa (1 poin)
c. Massa molar senyawa (3 poin)
d. Rumus molekul senyawa (2 poin)

JAWAB SOAL 1. (9 poin)

a. % Cr = (100-73,95- 8,27) = 17,78 %


73,98 8,27 17,78
Perbandingan mol C : H : Cr = : : 52 6,165 : 8,27 : 0,342
12 1,0
= 18,02 : 24,18 : 1 = 18 : 24 : 1

Formula empiris adalah : C18H24Cr (3 poin)

b. Massa empiris = (18x12) + (24x1) + ( 1x52)


= 216 + 24 + 52 = 292 (1 poin)

c. Massa molar (M): = cRT


OSK2012 6
3
3,17 atm 5 10 g M 0,082507.L.atm.K 1.mol 1 (273,15 25)K
760 100
1000

3
760 5 10 1000 0,082507 298,15 g
M= 294,88 g mol 295
100 3,17 mol mol

Massa molar = 295 g/mol (4 poin)

d. Formula = (C18H24Cr)n
292 x n =295 n 1
Formula molekul = C18H24Cr (2 poin)

Soal-2 (Total: 20 poin)

Amonia adalah gas yang mudah larut dalam air menghasilkan basa lemah, tetapan basa
Kb = 1,8 x10-5
a) Tuliskan rumus molekul amonia (1 poin)
2. Tuliskan reaksi amonia dalam air yang menghasilkan basa (1 poin)
3. Jika diketahui larutan amonia 10% massa dengan densitas ~1 berapa konsentrasi
amonia(Mol/Liter) yang terkandung dalam 10 mL larutan tersebut? (4 poin)
4. Berapakah pH larutan amonia ini? (2 poin)

Jika 5 mL amonia 10% tersebut dimasukkan dalam labu takar 100 mL dan diencerkan
dengan air sampai tepat 100 mL
e) Berapa konsentrasi amonia yang dinyatakan dalam M? (2 poin)
6. Berapa pH larutan encer tersebut? (2 poin)
7. Jika 25 mL larutan basa encer ini dicampur dengan 5 mL HCl 0,1M, campuran
larutan ini disebut apa? berapakah pH larutan campuran tersebut? (2 poin)
8. Jika 5 mL larutan basa encer ini dicampur dengan 25 mL HCl 0,1M berapakah pH
larutan campuran tersebut? (3 poin)
9. Berapa mL HCl yang diperlukan agar tepat bereaksi dengan 10 mL amonia encer
tsb? (3 poin)

Jawab
6. NH3
7. NH3 + H2O NH4+ + OH-
8. [NH3] = 100/17M = 5,88M
9. pH = 12
OSK2012 7
5 0,294M
6 pH = 11,4
7 Buffer pH =3,6
8 Kelebihan asam pH =- log 1,03/30 = 1,5
9 29,4 mL

Soal 3 (9 poin)

Nilai pH darah dalam tubuh manusia dijaga dalam rentang yang sempit yaitu dari 7,35
hingga 7,45 oleh sistem buffer alami yang terdiri dari asam karbonat, H 2CO3(aq) dan ion
hidrogenkarbonat, HCO3-(aq)
6. Larutan buffer yang konsentrasi H2CO3(aq) dan HCO3-(aq) sama, mempunyai pH
= 6,10. Hitunglah konstanta dissosiasi, Ka, untuk H2CO3(aq). (2 poin)
7. Hitunglah rasio (perbandingan) konsentrasi HCO3-(aq) dan H2CO3(aq) dalam
darah pada pH = 7,40 (3 poin)
c. Apakah pH darah seseorang naik ataukah turun selama kerja fisik? Jelaskan
jawaban anda! (2 poin)
d. Jelaskan dengan singkat mengapa dalam sistem buffer H2CO3(aq)/HCO3-(aq)
dapat menjaga pH darah. (2 poin)

Jawab:
6. Ka = 7,94 x 10-7 mol dm-3.
7. 20
8. Naik, karena kerja menghasilkan CO2, kadar H2CO3 turun kadar basa relatif tetap
sehingga pH naik.
9. Perubahan baik oleh pengenceran, penambahan sedikit asam atau basa dalam
persamaan fungsi logaritma pengaruhnya kecil, sehingga perubahan pH yang
relatif kecil dapat diabaikan.

Soal-4 (Total: 20 poin)

Garam natrium etanoat (CH3CO2Na) bila dilarutkan dalam air akan terjadi reaksi

kesetimbangan sebagai⇌ berikut:


CH3CO2-(aq) + H2O(l) CH3CO2H(aq) + OH-(aq)
Bila larutan natrium etanoat 0,01 M tersebut memiliki pH = 8,87, maka:
O. Hitung konsentrasi H+ dalam larutan! (3 poin)
P. Hitung pula konsentrasi OH- dalam larutan! (2 poin)
c. Hitung konsentrasi asam etanoat dalam larutan! (2 poin)
d. Tentukan tentukan tetapan disosiasi asam, Ka, untuk asam etanoat! (4 poin)
5. Hitung massa (dalam gram) natrium etanoat yang harus ditambahkan ke dalam 1 L
larutan asam etanoat 0,10 M (dengan nilai Ka yang diperoleh dari jawaban soal (d).
Jika tidak diperoleh jawaban pada (d), gunakan nilai Ka asam etanoat= 2x10 5) untuk
menghasilkan larutan bufer (penyangga) dengan pH 4,7! (4 poin)

OSK2012 8
6. Gambarkan struktur molekul asam etanoat! (2 poin)
7. Jika asam etanoat direaksikan dengan isopropil alkohol (isopropanol), gambarkan
struktur produk yang terbentuk dan tuliskan nama IUPAC-nya! (3 poin)

Jawab Soal 3:
6. [H+] = 10-8,87 = 1,35 x 10-9 M
7. [OH-] = Kw/[H+] = 10-14/1,35 x 10-9 M = 7,41 x 10-6 M
8. [CH3CO2H] = [OH-] = 7,41 x 10-6M
9. Dari persamaan kesetimbangan di atas diperoleh:
Kb ≈[CH3CO2H][OH-]/[ CH3CO2- ] = (7,41 x 10-6)2/(0,01) = 5,49 x
10-9 Ka = Kw/Kb = 10-14/5,49 x 10-9 = 1,82 x 10-6
10. Jika menggunakan nilai Ka = 1,82 x 10-6,
maka: pH = pKa + log [CH3CO2- ]/[CH3CO2H]
4,7 = 5,74 + log [CH3CO2-]/(0,1) -1,04 + log(0,1) = log[CH 3CO2-]
log[ CH3CO2-] = -2,04 [CH3CO2-] = 0,0091 M.
Dalam 1 L larutan maka mol CH 3CO2-= mol CH3CO2Na = 0,0091 mol
Maka massa natrium etanoat = 0,0091 mol x 82 g/mol = 0,7462 g
Dengan cara yang sama, jika digunakan nilai Ka asam etanoat= 2x10-5, maka
[CH3CO2-] = 0,1 M, sehingga dalam 1 L larutan terdapat mol CH 3CO2- = mol
CH3CO2Na = 0,1 mol. Maka massa natrium etanoat = 0,1 mol x 82 g/mol = 8,2 g

C
H3 C OH
6 asam etanoat
O CH3

C CH
H3 C O CH3
g. (1-metil)etil etanoat atau isopropil etanoat

SEMOGA BERHASIL
Pembahasan Soal Multiplechoice OSK Kimia Tahun 2014
Oleh Urip Kalteng @ http://urip.wordpress.com

Dengan senang hati jika ada yang mau mengoreksi pembahasan ini

A. Pilih jawaban yang paling tepat ( 25 soal @ 2 poin)


1. Analisis unsur suatu asam menunjukkan bahwa persen massa unsur penyusun molekul asam tersebut
adalah 5,89% H, 70,6% C, dan 23,5% O. Bila berat molekul asam tersebut adalah 136 g/mol,
bagaimana formula molekulnya?
A. C6H16O3
B. C7H4O3
C. C8H8O2
D. C9H12O
E. C9H16O
Pembahasan:
Perbandingan mol setiap unsur H : C : O = 5,89/1 : 70,6/12 : 23,5/16
= 5,89 : 5,88 : 1,47 (bagilah dengan angka terkecil)
= 5,89/1,47 : 5,88/1,47 : 1,47/1,47
=4 : 4 :1
Jadi rumus empiris senyawa tersebut adalah C4H4O, sampai di sini kita sudah bisa tebak jawabannya
adalah C, karena C8H8O2 adalah 2 x (C4H4O)
Berat molekul senyawa = (C4H4O) × n  136 = (12×4 + 1×4 + 16×1) x n = (48+4+16) x n = 68×n  n = 2
So... (C4H4O) × n = C8H8O2

2. Untuk mendapatkan gas nitrogen dioksida (NO2), anda mereaksikan dua gas tak dikenal (gas A dan
B). Dengan menggunakan data yang anda peroleh, anda dapat menuliskan
persamaan reaksi setara sebagai berikut:

2A(g) + B(g)  2NO2(g)
Bila anda mulai dengan 8 L gas A dan 3 L gas B, Berapa liter gas NO 2 yang akan anda peroleh
? (anggaplah semuanya dalam keadaan s.t.p.)
A. 2L
B. 3L
C. 4L
D. 6L
E. 8L
Pembahasan:
2A(g) + B(g)  2NO2(g)
Awal : 8L 3L –
Berekasi: –(2x3)L –(1x3)L (2x3)L
Akhir: 2L – 6L

3. 10 g parafin, C20H42, suatu zat terlarut yang tak mudah menguap, dilarutkan dalam 50 g benzena,
C6H6. Pada suhu 53°C, tekanan uap murni benzena adalah 300 torr. Tekanan uap Iarutan pada suhu
tersebut adalah
A. 298 torr
B. 292 torr .
C. 284 torr
D. 275 torr
E. 267 torr
Pembahasan Soal Multiplechoice OSK Kimia Tahun 2014 Oleh Urip Kalteng @ http://urip.wordpress.com Halaman 1
Pembahasan:
Penurunan tekanan uap larutan, jika terdapat zat terlarut maka tekanan uapnya akan mengalami
penurunan berdasarkan fraksi mol–nya.
10 g parafin (C20H42) = 10/282 = 0,035 mol
50 g benzena (C6H6) = 50/78 = 0,641 mol
Mol parafin + mol benzena = 0,035 + 0,641 = 0,676 mol
Fraksi mol Parafin = 0,035/0,676 = 0,052 ; fraksi mol benzena (pelarut) = 1–0,052 = 0,948
Penurunan tekanan uap larutan = 0,052 x 300 torr = 15,6. Jadi tekanan uap larutan = 300 – 15,6 = 284,3
~ 2,84.

4. Larutan pekat asam klorida, HCI, adalah Iarutan 36% w/w HCI dalam air. Larutan ini memiliki rapat
3
massa 1,18 g/cm . Konsentrasi Iarutan tersebut adalah
A. 1,2 M .
B. 12 M
C. 24 M
D. 30 M
E. 7 M
Pembahasan:
Ini adalah soal terkait konversi konsentrasi suatu larutan.
Anggaplah terdapat 100 mL (0,1 L) larutan maka massa HCl = 36% x 100 x 1,18 g/mL = 42,48
g Massa molar HCl = 35,5 + 1 = 36,5 g/mol
42,48 g = 42,48 g :36,5 g/mol = 1,16 mol
Jadi dalam larutan tersebut konsentrasinya = 1,16 mol : 0,1 L = 11,6 M ~12 M

5. Molalitas senyawa para-diklorobenzena (C 6H4Cl2) dalam suatu Iarutan yang dibuat dengan cara
melarutkan 2,65 g C6H4CI2 dalam 50 mL benzena (kerapatan = 0,879 g/mL) adalah:

A. 0,018 m
B. 0,041 m
C. 0,180 m
D. 0,410 m
E. 1,810 m
Pembahasan:
Molalitas = jumlah mol zat terlarut tiap 1 kg pelarut = mol (C 6H4Cl2) : 1
kg 2,65 g C6H4Cl2 (zat terlarut) = 2,65 g : 147 g/mol = 0,018 mol
Massa benzena (pelarut)= 50 mL x 0,879 g/mL = 43,95 g = 0,04395 kg
Jadi molalitasnya = 0,018 mol : 0,04395 kg = 0,409556 ~ 0,41 molal

6. Dengan menggunakan data berikut ini:


N2(g) + 3O2(g) + H2(g)  2HNO3(aq) H = –414,8 kJ H =
N2O5(g) + H2O(g)  2HNO3(aq) 218,4 kJ H = 483,6
2H2O(g)  2H2(g) + O2(g) kJ
Tentukan H untuk reaksi:
2N2O5(g)  N2(g) + 5O2(g)
1. 149,6 kJ
2. 90,8 kJ
3. –876,4 kJ
4. 782,8 kJ
5. 1750 kJ

Pembahasan Soal Multiplechoice OSK Kimia Tahun 2014 Oleh Urip Kalteng @ http://urip.wordpress.com Halaman 2
Pembahasan:
Molalitas = jumlah mol zat terlarut tiap 1 kg pelarut = mol (C 6H4Cl2) : 1 kg
4HNO3(aq)  2N2(g) + 4O2(g) + 2H2(g) H = 2 x 414,8 kJ
2N2O5(g) + 2H2O(g)  4HNO3(aq) H = 2 x 218,4 kJ
2H2(g) + O2(g)  2H2O(g) H = –483,6 kJ
2N2O5(g)  2N2(g) + 5O2(g) H = 782,8 kJ

23
7. Suatu isotop atom Mg mempunyai muatan +2. Jumlah proton, neutron, dan elektron yang
dimilikinya adalah:

Jumlah Proton Jumlah neutron Jumlah elektron


A. 12 13 10
B. 12 11 11
C. 12 11 10
D. 10 13 11
E. 10 11 12
Pembahasan:
Mg nomor atom 12 dan nomor massa 23, dengan muatan +2, maka: jumlah
proton = nomor atom = 12 proton
jumlah neutron = nomor massa –nomor atom = 23 – 12 = 11 neutron
jumlah elektron = nomor atom – muatan = 12 – 2 = 10 elektron

8. Berikut ini pereaksi manakah bila dicampur dan dipanaskan dengan amonium sulfat
membebaskan amonia?
Akua bromin
Asam hidroklorida encer
Air kapur
Asam nitrit
Kalium dikromat (VI) diasamkan
Pembahasan:
Air kapur = Ca(OH)2 ...... maka (NH4)2SO4 + Ca(OH)2  CaSO4 + H2O + NH3

+3
9. Mengenai ion Fe , manakah konfigurasi elektron boks dari sub–kulit 3d yang benar?

A. ↑↓ ↑ ↑ ↑ ↑
B. ↑↓ ↑ ↑ ↑
C. ↑ ↓ ↑ ↓ ↑
D. ↑ ↑ ↑ ↑ ↑
E. ↓↓ ↓ ↓ ↓
Pembahasan:
Fe nomor atom–nya 26, karena bermuatan +3 maka elektron yang tersisa adalah 26–
2 2 6 2 6 5 0
3=23 Sehingga konfigurasi elektron 1s 2s 2p 3s 3p 3d 4s

Pembahasan Soal Multiplechoice OSK Kimia Tahun 2014 Oleh Urip Kalteng @ http://urip.wordpress.com Halaman 3
10 Atom–atom berikut ini, manakah yang mempunyai energi ionisasi kedua yang paling besar?
A. Al
B. Mg
C. Ca
D. Ti
E. Na
Pembahasan:
Energi ionisasi kedua adalah energi yang dibutuhkan ketika melepaskan elektron yang kedua setelah
yang pertama. Ini terkait pula dengan jari–jari dan kestabilan atom/ion.
Konfigurasi elektron Al –13 = 2 8 3  stabil jika melepas 3 ev
Konfigurasi elektron Mg – 12 = 2 8 2  stabil jika melepas 2 ev
Konfigurasi elektron Ca – 20 = 2 8 8 2  stabil jika melepas 2 ev
Konfigurasi elektron Ti – 22 = 2 8 10 2  stabil jika melepas 2 ev
Konfigurasi elektron Na –11 = 2 8 1  stabil jika melepas 1 ev

Na punya elektron valensi 1, dan akan stabil jika melepaskan elektron valensi itu (tidak diperlukan energi
+
yang terlalu besar) untuk menjadi ion Na , namun energi ionisasi kedua yang diperlukan akan sangat
+
besar dibanding atom–atom yang lain, karena kecilnya jari–jari ion Na dan ion ini juga sudah stabil.

11. Jumlah elektron valensi yang terdapat dalam ion oksalat, C2O42– adalah:
A. 2
B. 10
C. 32
D. 34
E. 44
Pembahasan:
Jumlah elektron valensi (ev) pada suatu senyawa atau ion adalah jumlah elektron valensi dari setiap
atom yang ada pada senyawa atau ion itu.
2 atom C = 2 x 4 ev = 8 ev ; 4 atom O = 4 x 6 ev = 24 ev ; karena mendapat tambahan muatan 2 elektron
maka ini dianggap sebagai eelektron valensi. Jadi total ev ion oksalat tersebut = 8 + 24 + 2 = 34 ev

12. Dari persamaan berikut ini, manakah yang merupakan reaksi oksidasi–reduksi?
A. 2HCl(aq) + Mg(s) → MgCl2(aq) + H2(g)

B. Na2O(s) + H2O(l) → 2NaOH(aq)


C. CO2(g) + H2O(l) → H2CO3(aq)

D. CaO(s) + SO3(g) → CaSO4(s)

E. NH3(g) + HCl(g) → NH4Cl(s)

Pembahasan:
Reaksi oksidasi reduksi dapat ditandai dengan ada tidaknya perubahan bilangan
oksidasi. Pada reaksi HCl + Mg → MgCl2 + H2
Bilangan oksidasi H pada HCl = +1, pada H2 = 0 ; Bilangan oksidasi Mg pada Mg = 0, pada MgCl 2 = +2
Jadi reaksi HCl + Mg → MgCl2 + H2 adalah reaksi redoks.

Pembahasan Soal Multiplechoice OSK Kimia Tahun 2014 Oleh Urip Kalteng @ http://urip.wordpress.com Halaman 4
M. Dari pernyataan berikut ini, manakah pernyataan yang benar untuk menjelaskan sifat–sifat yang
dikaitkan dengan ikatan ionik dan kovalen?
Senyawa kovalen tidak dapat membentuk suatu elektrolit.
Senyawa kovalen dengan titik leleh tinggi hanya terjadi bila terdapat ikatan
hidrogen.
Beberapa senyawa kovalen yang mengandung oksigen dan hidrogen dalam
molekulnya dapat membentuk ikatan hidrogen.
Ikatan ionik dan ikatan kovalen keduanya tidak dapat terjadi dalam suatu
senyawa yang sama.
Senyawa ionik berbeda dari logam di mana senyawa ionik tidak
menghantarkan listrik dalam keadaan padat.

Pembahasan:
Alternatif C dan E keduanya merupakan pernyataan yang benar.

14. Suatu reaksi kimia eksotermik berlangsung dalam dua tahap reaksi,
ℎ 1 ℎ 2

Pereaksi → Intermediet → Produk


–1
Energi aktivasi tahap 1 adalah 50 kJ.mol , sedangkan perubahan entalpi reaksi keseluruhan adalah –
–1
100 kJ.mol . Berdasarkan data tersebut, manakah diagram yang menggambarkan diagram tingkat
energi untuk reaksi ini?

Pembahasan:
Alternatif D yang benar, pada tahap 1 energi aktivasinya adalah 50 kJ/mol (puncak 1) sementara secara
keseluruhan ia akan justru melepaskan kalor sebesar 100 kJ/mol (eksoterm) grafik akhir (produk pada posisi
–100 kJ/mol.

Pembahasan Soal Multiplechoice OSK Kimia Tahun 2014 Oleh Urip Kalteng @ http://urip.wordpress.com Halaman 5
15. Arus listrik 0,0965 A dialirkan ke dalam 50 mL larutan NaCl 0,1 M selama 1000 detik, maka

konsentrasi OH dalam larutan adalah
0,0005 M
0,0010 M
0,0020 M
0,0100 M
0,0200 M
Pembahasan:
Reaksi redoks pada elektrolisis larutan NaCl yang mengalami reaksi reduksi adalah air (pelarutnya)

sedangkan ion Cl sangat mudah teroksidasi menjadi gas klor sehingga reaksinya adalah:
– –
Anoda (reaksi oksidasi): 2Cl  Cl2 + 2e Katoda
(reaksi reduksi): 2H2O + 2e–  2OH– + H2
Jumlah mol elektron = (i x t)/96.500 = (0,0965 x 1000)/96.500 = 0,001 mol
Berdasarkan perbandingan koefisien elektron dan OH– maka jumlah mol OH– = jumlah elektron = 0,001 mol,
dan karena volum larutannya 50 mL (= 0,05 L) maka konsentrasi OH– = 0,001 mol : 0,05 L = 0,02 M

P. Elektrolisis larutan CuSO4 dengan elektroda platina menggunakan arus 1 A selama 965 detik.
Berat Cu yang mengendap pada katoda adalah
3,175 g
0,03175 g
0,0635 g
0,3175 g
31,75 g
Pembahasan:
2+ –
Reaksi reduksi pada katoda: Cu + 2e  Cu
Massa endapan Cu pada katoda = (e × i × t)/96.500 = (63,5/2 × 1 × 965)/96.500 = 0,3175 g

17. Suatu gas senyawa organik, X, dibakar dengan oksigen berlebih. Sebanyak 0,112 dm 3 sampel X
diukur pada s.t.p., menghasilkan 0,88 g karbon dioksida. Berapa banyak atom karbon yang
terdapat dalam satu molekul senyawa X ini?
1
2
3
4
8
Pembahasan:
X(g) + O2 (g)  CO2 (g)
Pada keadaan STP 1 mol gas = 22,4 L  sampel gas X sebanyak 0,112 L = 0,112 L/22,4 L/mol = 0,005
mol 0,88 g CO2 = 0,88 g : 44 g/mol = 0,02 mol
Perbandingan mol gas X dengan CO2 = 0,005 : 0,02 = 1 : 4
X(g) + O2 (g)  4 CO2 (g) maka dapat disimpulkan bahwa jumlah atom C pada senyawa gas X
tersebut sebanyak 4 atom karbon.

Pembahasan Soal Multiplechoice OSK Kimia Tahun 2014 Oleh Urip Kalteng @ http://urip.wordpress.com Halaman 6
18. Senyawa berikut ini, manakah yang mempunyai titik didih paling tinggi?
CH3CH2CH2CH2CH2CH3
CH3CH2OH
CH3CH2CH(OH)CH3
CH3CH2(CH2)2CHO
CH3(CH2)5OH
Pembahasan:
Secara sederhana kita bisa bandingkan titik didih antar zat berdasarkan kepolarannya dan juga panjang
pendeknya rantai karbon (semakin panjang rantai karbon titik didihnya semakin tinggi), bercabang atau
lurus (dengan jumlah atom C yang sama rantai lurus titik didihnya semakin tinggi). Kepolaran alkohol
> aldehide > alkana demikian juga titik didihnya, semakin polar maka titik didihnya akan semakin
tinggi, karena untuk memutus gaya tarik menarik kutub positif dan negatif antarmolekulnya.
6. CH3CH2CH2CH2CH2CH3 (alkana suku ke–6);
7. CH3CH2OH (alkohol suku ke–2);
8. CH3CH2CH(OH)CH3 (alkohol skunder suku ke–4);
4. (CH3CH2(CH2)2CHO)Aldehida suku ke–5;
5. (CH3(CH2)5OH) alkohol primer suku ke–6 Jadi
alternatif E memiliki titik didih paling tinggi.

S. Dari kelima senyawa organik tersebut di bawah ini, manakah yang paling benar tata
namanya?
2–kloro–5–metil–5–heksena
2–metil–1–pentena–4–ol
3–etil–2–metil–2–pentena
2–metil–2–heksena–4–ol
2,3–dimetilsikloheksena
Pembahasan:
Untuk mengetahui yang paling benar sebaiknya digambarkan struktur molekulnya terlebih dahulu.
1 Seharusnya nama yang tepat adalah 5–kloro–2–metil–2–heksena

2 Seharusnya nama yang tepat adalah 4–metil–4–pentena–2–ol

3 Benar namanya adalah 3–etil–2–metil–2–pentena

Pembahasan Soal Multiplechoice OSK Kimia Tahun 2014 Oleh Urip Kalteng @ http://urip.wordpress.com Halaman 7
4. Seharusnya nama yang tepat adalah 5–metil–4–heksena–3–ol

5. Seharusnya nama yang tepat adalah 1,6–dimetilsikloheksena

T. Senyawa radikal bebas di bawah ini manakah yang paling stabil?


CH3 ∙
(CH3)2CH ∙
(CH3)3C ∙
C2H5 ∙
(CH3)2CHCH2 ∙
Pembahasan:
Radikal bebas yang paling stabil jika memiliki halangan sterik yang meruah (memiliki cabang yang
banyak) dalam hal ini posisi C radikalnya merupakan atom C tersier atom C yang mengikat 3 atom C yang
lain. Urutan stabilitas radikal bebas: C tersier > C skunder > C primer.
Jadi jawaban yang tepat adalah C.

21. Tentukan urutan kereaktifan dari senyawa turunan asam karboksilat berikut:
Asetamida
Asetil klorida
Etil asetat
Anhidrida asam asetat
1>2>3>4
2>1>3>4
3>4>2>1
2>4>3>1
4>3>2>1
Pembahasan:
Urutan kereaktifan senyawa turunan asam karboksilat :
asil halida > asam anhidrida > alkil tioester > aril esters > alkil esters > aril amida > alkil amida

22 Semua senyawa berikut adalah merupakan isomer dari butanon, kecuali?


A. t–butileter
B. Tetrahidrofuran
C. 2,3–epoksibutan
D. Etil vinyl eter
E. 1,2–butenol

Pembahasan Soal Multiplechoice OSK Kimia Tahun 2014 Oleh Urip Kalteng @ http://urip.wordpress.com Halaman 8
Pembahasan:
Isomer suatu senyawa: memiliki rumus molekul sama tetapi rumus strukturnya berbeda, jumlah atom–atom
penyusun–nya sama. Di antara alternatif yang tersedia yang bukan merupakan isomer dari butanon
(C4H8O) adalah t–butileter.

23. Manakah dari senyawa berikut yang tidak menunjukkan isomer cis–trans?

Pembahasan:
Alternatif B tidak mungkin menunjukkan isomer cis–trans karena pada sisi kiri atom C mengikat
gugus yang mana (gugus metil)

24. Dengan menggunakan pereaksi Grignard, di bawah ini senyawa mana yang dapat
menghasilkan CH3CH2CH(OH)CH2CH3?
A Metil etilketon dan metil Grignard
B Propanon dan metil Grignard
C Butil Grignard dan asetaldehida
D Etil Grignard dan propionaldehida
E Krotonaldehida dan etil Grignard
Pembahasan:
CH3CH2CHO + CH3CH2MgBr + H2O  CH3CH2CH(OH)CH2CH3 + HOMgBr
Propionaldehida etil Grignard

Pembahasan Soal Multiplechoice OSK Kimia Tahun 2014 Oleh Urip Kalteng @ http://urip.wordpress.com Halaman 9
25. Reaksi esterifikasi asam karboksilat dengan alkil halida dalam suasana basa akan
menghasilkan produk akhir adalah:

A. B.

C. D.

E.

Pembahasan:

Pada reaksi esterifikasi selalu menghasilkan ester, namanya juga esterifikasi Jadi jawaban yang tepat adalah
ester, alternatif E.
RCOOH + NaOH  RCOONa + H2O
RCOONa + R’X  RCOOR’ + NaX
–––––––0–––––––
OLIMPIADE SAINS NASIONAL 2009
SELEKSI Tingkat PROVINSI

UjianTeori
Waktu 2,5 Jam

Departemen Pendidikan Nasional

Direktorat Jenderal Managemen Pendidikan Dasar dan Menengah

Direktorat Pembinaan Sekolah Menengah Atas

2009

Seleksi Olimpiade Sains Provinsi 2009 1


Seleksi Olimpiade Sains Provinsi 2009 2
TETAPAN DAN RUMUS BERGUNA

Tetapan Avogadro NA = 6.022∙1023 mol–1

R= 8,314 J.K-1.mol-1= 8,314 x107 erg. Mol-1.K-1


Tetapan gas universal, R =1,987 cal.mol-1.K-1 = 0,082054 L.atm.mol-1.K-1

Tekanan gas 1 atmosfir =760 mmHg =760 torr

Persamaan gas Ideal PV= nRT

1 mol gas (STP) 22,4 liter


 E 
k  A exp   A
 atau, k = A.e-Ea/RT

Persamaan Arrhenius  RT 

Energi Gibbs untuk fasa terkondensasi


G = pV + konstanta
pada tekanan p

Hubungan antara tetapan Go = -RT ln K
kesetimbangan dan energi Gibbs
Energi Gibbs pada temperature G H  T
S
konstan
Isotherm reaksi kimia G = G + RT∙ln Q
Tekanan Osmosa larutan p =c RT

o
Persamaan Nerst pada 298K, EE  0,0592 logQ
n

Seleksi Olimpiade Sains Provinsi 2009 3


DEPARTEMEN PENDIDIKAN NASIONAL
DIREKTORAT JENDERAL MANAJEMEN PENDIDIKAN DASAR DAN MENENGAH
DIREKTORAT PEMBINAAN SEKOLAH MENENGAH ATAS

Petunjuk :
1. Isilah Biodata anda dengan lengkap (di lembar Jawaban)
Tulis dengan huruf cetak dan jangan disingkat !

2. Soal Teori ini terdiri dari 10 nomor soal essay.

TOTAL Poin = 131 poin

3. Waktu yang disediakan: 150 menit.

4. Semua jawaban harus ditulis di lembar jawab yang tersedia

5. Diperkenankan menggunakan kalkulator.

6. Diberikan Tabel Periodik Unsur, dan data yang diperlukan


7. Anda dapat mulai bekerja bila sudah ada tanda mulai dari pengawas.
8. Anda harus segera berhenti bekerja bila ada tanda berhenti dari Pengawas.
9. Letakkan jawaban anda di meja sebelah kanan dan segera meninggalkan
ruangan.
10. Anda dapat membawa pulang soal ujian !!

Seleksi Olimpiade Sains Provinsi 2009 4


Soal 1. Gas Nitrogen dan Senyawanya (12 poin)

Lengkapi isian berikut dengan jawaban yang paling tepat.

Nitrogen adalah unsur yang paling banyak terdapat di udara. Pada tekanan udara 1 atm
dan tempertatur 25oC, nitrogen berbentuk gas dengan rumus kimia (a)………...
Di atmosfer, kadar gas nitrogen terdapat kurang lebih sebanyak (b)………%
v/v. Struktur Lewis gas nitrogen digambarkan sebagai (c)……………..

Gas nitrogen bereaksi dengan gas hidrogen pada temperatur dan tekanan tinggi
membentuk senyawa berupa gas berbau menusuk, dengan rumus kimia (d)………..
Produksi senyawa gas bau menusuk tersebut disebut proses (e)……….. dimana unsur
(f)………digunakan sebagai katalis. Senyawa gas (d) tersebut mudah larut dalam air
menghasilkan larutan dengan (g) pH………….. Senyawa (d) tersebut juga bereaksi dengan
asam nitrat membentuk padatan putih dengan rumus kimia (h)……………. yang merupakan
bahan peledak. Senyawa (d) tersebut juga bereaksi dengan asam sulfat membentuk padatan
putih dengan rumus kimia (i)………………yang digunakan sebagai pupuk.

Gas nitrogen bereaksi dengan oksigen pada temperatur tinggi dalam mesin-motor/mobil
membentuk senyawa dengan rumus kimia (j)……….. yang berbahaya bagi lingkungan.
Dalam rangka mengurangi polusi gas buang kendaraan bermotor, diperlukan katalis
(k)…………..untuk mengubah gas-gas berbahaya seperti (l)…….. dan (m)………….
menjadi gas tidak berbahaya yaitu (n)………….dan (o)…………………..

Soal 2. Orbital atom. (14 poin)

Orbital adalah suatu ruang yang menggambarkan kebolehjadian menemukan elektron.


Orbital diperoleh dari bilangan kuantum ke dua (azimuth). Pada gambar berikut,
ditampilkan bentuk orbital atom yang berada di perioda dua pada sistim Periodik modern.

Seleksi Olimpiade Sains Provinsi 2009 5


a. Tuliskan nama orbital apa yang digambarkan pada A, B, C , D dan E tersebut?
(5 poin)
b. Atom Berilium terletak pada golongan 2 dan periode-2 Sistim Periodik Modern.
Orbital apa saja yang tidak terisi elektron pada atom berilium tersebut? (nyatakan
dengan A,B,C,D,E) (3 poin)
c. Pada suatu unsur X, orbital A dan C terisi elektron secara penuh, tetapi orbital B dan
E terisi setengah penuh, sedangkan orbital D tidak terisi elektron. Tentukan jumlah
proton pada atom X dan perkirakan massa unsur X (4 poin)
d. Berapa bilangan oksidasi ion karbon yang memiliki elektron pada orbital A dan C
secara penuh? (2 poin)

Soal 3. Mineral Erevanite (15 poin)

Erevanite adalah mineral alkali karbonat silikat yang formula umumnya adalah
M2CO3.xSiO2. Mineral ini dapat digunakan sebagai bahan dasar pembuatan gelas
transparan. Bila sejumlah erevanite dengan berat yang sama dipanaskan, ternyata akan
menghasilkan jenis dan jumlah gas yang sama bila erevanite direaksikan dengan larutan
asam (HCl dan HF). Dari pengamatan, diperoleh data sbb:
Perlakuan terhadap Berat endapan Berat Volume gas
erevanit (a gram) yang diperoleh Residu (STP)
Penguraian dengan
pemanasan (900oC) - 7,32 g 1,35 L
Penambahan larutan HCl 3,61 g - 1,35 L
Penambahan larutan HF 10,6 g - 1,35 L

Bila mineral erevanite hanya mengandung satu jenis logam alkali, maka:
a. Tentukanlah jenis dan jumlah gas yang dibebaskan pada proses perlakuan
diatas (3 poin)
b. Tentukanlah logam M dan x dalam komposisi erevanite (dengan
perhitungan). (6 poin)
c. Tuliskanlah persamaan reaksi dengan proses perlakuan di atas (lengkap
dengan fasanya). (6 poin)

Soal 4. Bahan Pemutih (14 poin)

Natrium hipoklorit, NaOCl banyak digunakan sebagai bahan pemutih dan zat antiseptik.
Suatu larutan antiseptik dibuat dengan melarutkan 0,745 g NaOCl di dalam air sampai
volume tepat 100 mL.
Tetapan ionisasi (Ka) asam hipoklorit, HOCl = 10 -8.
a. Tuliskan reaksi pengionan NaOCl dan reaksi hidrolisis yang terjadi. (2 poin)
b. Tuliskan pasangan asam–basa konjugasi yang terlibat (2 poin)

Seleksi Olimpiade Sains Provinsi 2009 6


c. Hitung konsentrasi Na+, OCl-, HOCl, dan OH- di dalam larutan tersebut
setelah tercapai kesetimbangan, nyatakan dengan Mol/L. (4 poin)

Larutan antiseptik dengan pH tertentu dapat dihasilkan dengan mencampurkan larutan


HOCl dengan larutan NaOH.
d. Hitung pH larutan antiseptik yang dihasilkan dari pencampuran 100 mL larutan
HOCl 0,1 M dengan 50 mL larutan NaOH 0,1 M. (3 poin)
e. Larutan d ditambahkan 1 mmol HCl dengan meneteskan beberapa tetes larutan
HCl pekat (volume larutan HCl dapat diabaikan). Hitung pH larutan setelah
penambahan HCl. (3 poin)

Soal 5. Reaksi Redoks (15 poin)

Senyawa K3[Mn(C2O4)3]·xH2O dapat berfungsi sebagai oksidator. Sebanyak 200 mg


sampel K3[Mn(C2O4)3]·xH2O dilarutkan dengan 25 mL air dalam Erlenmeyer 250 mL,
kemudian di tambahkan 1 g KI dan 10 mL HCl 4M, dan iodium yang dihasilkan bereaksi
sempurna dengan larutan natrium tiosulfat, (Na2S2O3) 0,025 M sebanyak 16,32 mL.

a. Tuliskan bilangan oksidasi ion logam dalam senyawa K3[Mn(C2O4)3]·xH2O


(2 poin)

b. Ion mana pada K3[Mn(C2O4)3]·xH2O yang bertindak sebagai oksidator?, tuliskan


setengah reaksi reduksinya. (3 poin)
c. Tuliskan setengah reaksi oksidasi KI menjadi iodium (2 poin)
d. Tuliskan persamaan reaksi iodium dengan ion tiosulfat. (3 poin)
e. Berapa mili-ekivalen ion tiosulfat yang bereaksi dengan iodium? (2 poin)
f. Hitung nilai x (3 poin)

Soal 6. Gas pencemar dan beracun CO (9 poin)

Gas karbon monoksida CO adalah gas beracun yang sangat berbahaya karena dapat
bereaksi dengan hemoglobin (Hb) menggantikan O2. Laju reaksi antara hemoglobin (Hb)
dan karbon monoksida (CO) dipelajari pada 20°C. Dari hasil observasi, diperoleh data
berikut ini:

Ekperimen [Hb]0 [CO]0 Laju awal


(mol/L) (mol/L) (mol/L·s)

1 2,21 1,00 0,619

2 4,42 1,00 1,24

3 4,42 3,00 3,71

dengan 2,21 x 10-6 mol/L.

Seleksi Olimpiade Sains Provinsi 2009


7
a. Tentukan order reaksi terhadap Hb dan CO. (3 poin)
b. Tentukanlah hukum laju reaksinya (1 poin)
c. Hitunglah nilai konstanta laju reaksinya. (2 poin)
d. Berapa laju awal untuk eksperimen dengan [Hb]0 = 3,36 mol/L dan [CO]0 = 2,40
mol/L? (3 poin)

Dalam gas buang dari knalpot kendaraan bermotor, gas beracun karbon monoksida dapat
terbentuk melalui reaksi antara karbon (jelaga) dengan karbon dioksida:


C(s) +CO2(g) 2CO(g)
Bila tekanan total pada kesetimbangan tersebut adalah 1,0 atm dan
tetapan kesetimbangan Kp untuk reaksi di atas pada 900C adalah 1,0.

e. Hitunglah tekanan parsial CO pada kesetimbangan

Soal 7. Kelarutan garam perak asetat. (14 poin)

Perak asetat, Ag(O2CCH3), adalah garam yang sedikit larut dalam air. Pada temperatur 20 C,
100 g air dapat melarutkan 1,04 gram kristal perak asetat, dan densitas larutan yang diperoleh
adalah 1,01 g/cm3. Diketahui Ka CH3CO2H = 1,75x10-5 ; 1 mol CH3CO2H = 60 g ;
1 mol Ag(O2CCH3) =167 g

a. Hitunglah konsentrasi molar larutan jenuh perak asetat pada 20 C, jika densitas
larutan adalah 1,01 g/cm3. (3 poin)
b. Hitunglah tetapan hasil kali kelarutan untuk perak asetat. (2 poin)
c. Bila dibandingkan dengan kelarutannya dalam air murni, bagaimana kelarutan perak
asetat dalam:
i. larutan yang mengandung asam asetat? (2 poin)
ii. larutan asam nitrat 0,1 M ? (2 poin)
Berikan alasannya
d. Ion Ag+ dalam air mengalami hidrolisis dan tetapan hidrolisis, Kh = 2,0 x 10-12
i. Tuliskan reaksi hidrolisis ion Ag+ (2 poin)
ii. Bagaimana pH larutan jenuh Ag(O2CCH3), asam atau basa? Jelaskan (3 poin)

Soal 8. Proses pelapisan logam (elektroplating) ( 12 poin)

Proses elektroplating (pelapisan logam) adalah salah satu cara melindungi besi dari korosi
(pembentukan karat). Benda dari besi permukaannya dilapisi dengan logam yang relatif
kurang reaktif dibandingkan besi, misalnya zinc (Zn). Selembar lempeng besi hendak
dilapisi dengan zinc dengan cara elektrolisis (proses electroplating). Sebagai sumber listrik
yang digunakan untuk elektrolisis adalah baterei.
a. Identifikasi (tunjukkan) elektroda mana yang dihubungkan dengan kutub negatif dan
mana dihubungkan dengan kutub positif baterei, dan jelaskan. (3 poin)
b. i. Identifikasi apa yang akan bertindak sebagai elektroda dalam proses tersebut
(2 poin)
ii. setengah reaksi pada setiap elektroda, serta reaksi keseluruhan (3 poin)

Seleksi Olimpiade Sains Provinsi 2009


8
c. i. Elektrolit yang digunakan untuk proses elektrolisis (2 poin)
ii. konsentrasi elektrolit sesudah proses elektrolisis (2 poin)

Soal 9. Reaksi Gas Isobutilena (12 poin)

Gas isobutilena dilarutkan dalam H2SO4 63% menghasilkan suatu padatan berwarna putih
A. Ketika dipanaskan dalam air, padatan A berubah menjadi zat cair B dengan titik didih
83oC.
a. Gambarkan struktur A dan B. (6 poin)
b. Tuliskan skema reaksi yang terjadi dan nama senyawa A dan B. (6 poin)

Soal 10. Pemanis buatan (14 poin)

Sakarin adalah pemanis buatan yang banyak dipakai dalam minuman ringan (soft-drink). Di
industri, sakarin dibuat dengan menggunakan bahan dasar metilbenzena melalui tahapan
reaksi seperti berikut:

a. i. Reaksi tahap I , II dan III termasuk jenis reaksi apa ? (6 poin)

ii. Reagen apa yang dipakai pada tahap reaksi II dan III ? (4 poin)

Jika metilbenzen dinitrasi dengan campuran asam nitrat dan asam sulfat pekat, produknya
merupakan isomer dengan rumus molekul C7H7NO2.
b. Tentukan struktur isomer dari C7H7NO2. (4 poin)

Semoga Berhasil
O LIMPIADE SAINS NASIONAL 2010

SELEKSI TINGKAT PROVINSI

JAWABAN

(DOKUMEN NEGARA)

UjianTeori

Waktu 150 menit

K ementerian Pendidikan Nasional

Direktorat Jenderal

Manaje men Pendidikan Dasar dan Menengah

Direkto rat Pembinaan Sekolah Menengah Atas

2010
Petunjuk :

Petunjuk :

1 Isilah Biodata anda dengan lengkap (di lembar Jawaban)


Tulis dengan huruf cetak dan jangan disingkat !

2 Soal Teori ini terdiri dari dua bagian:

A. 30 soal pilihan Ganda = 60 poin

jawaban benar = 2 poin


jawaban salah = -1 poin
tidak menjawab = 0 poin

B. 6 Nomor soal essay = 114 poin

TOTAL Poin = 174 poin

3. Waktu yang disediakan: 150 menit

4. Semua jawaban harus ditulis di lembar jawaban yang tersedia

5. Diperkenankan menggunakan kalkulator.

6. Diberikan Tabel periodik Unsur.


7. Anda dapat mulai bekerja bila sudah ada tanda mulai dari pengawas.
8. Anda harus segera berhenti bekerja bila ada tanda berhenti dari Pengawas.
9. Letakkan jawaban anda di meja sebelah kanan dan segera meninggalkan ruangan.
10. Anda dapat membawa pulang soal ujian !!
OSP2010 Olimpiada Kimia Indonesia
Page 2
LEMBAR JAWABAN

Bagian A

Beri Tanda Silang (X) pada Jawaban Yang Anda Pilih

No Jawaban No Jawaban

1 A B C D E 16 A B C D E

2 A B C D E 17 A B C D E

3 A B C D E 18 A B C D E

4 A B C D E 19 A B C D E

5 A B C D E 20 A B C D E

6 A B C D E 21 A B C D E

7 A B C D E 22 A B C D E

8 A B C D E 23 A B C D E

9 A B C D E 24 A B C D E

10 A B C D E 25 A B C D E

11 A B C D E 26 A B C D E

12 A B C D E 27 A B C D E

13 A B C D E 28 A B C D E

14 A B C D E 29 A B C D E

15 A B C D E 30 A B C D E
OSP2010 Olimpiada Kimia Indonesia
Page 3
OSP2010 Olimpiada Kimia Indonesia Page 4
Tetapan dan rumus berguna

Tetapan (bilangan) Avogadro NA = 6.022∙1023 partikel.mol–1

R= 8,314 J.K-1.mol-1 = 8,314 x107 erg. Mol-1.K-1


Tetapan gas universal, R =1,987 cal.mol-1.K-1 = 0,082054 L.atm.mol-1.K-1

Tekanan gas 1 atmosfir =760 mmHg =760 torr

Persamaan gas Ideal PV= nRT


Hubungan antara tetapan Go= -RTlnK
kesetimbangan dan energi Gibbs
Energi Gibbs pada temperatur konstan GH T S
Isotherm reaksi kimia G = G + RT∙ln Q
Tekanan Osmosa pada larutan p =c RT
Muatan elektron 1,6022 x 10-19 C
Tetapan Kesetimbangan air (Kw) pada
25oC Kw= 1,0x10
A. Pilih Jawaban yang paling tepat.

1. Ion berikut ini, manakah yang mempunyai elektron lebih banyak daripada proton dan
lebih banyak proton daripada neutron? [H = 11H; D = 21H; He 42He; O 168O]
A. D- D. OD
+
B. D3O E. OH-
C. He+

2. Energi ionisasi berturutan (dalam kJ.mol-1) dari unsur X berikut ini adalah :
870 1800 3000 3600 5800 7000 13200
Apakah unsur X ini:
A 33As D. 52Te
B. 53I E. 40Zr.
C. 8O

3. Nilai perubahan entalpi ( H) untuk proses perubahan yang digambarkan oleh


persamaan:
Na(s)  Na+(g) + e-,
Adalah sama dengan :
energi ionisasi pertama natrium.
Perubahan entalpi penguapan natrium
Jumlah energi ionisasi pertama dan afinitas elektron natrium.
Jumlah perubahan entalpi atomisasi dan energi ionisasi pertama natrium.
Jumlah perubahan entalpi atomisasi dan afinitas elektron natrium.

4. Tabel berikut ini menunjukkan tingkat oksidasi yang mungkin dari lima unsur blok-d
dalam tabel periodik unsur (unsur diwakili oleh huruf yang bukan simbolnya).

Unsur bilangan oksidasi yang mungkin

P - - 3 - - - -
Q - 2 3 4 - - -
R 1 2 3 4 5 - -
S - - 3 4 5 6 -
T - 2 - 4 5 6 7

Berdasarkan data tersebut, manakah ion berikut ini yang paling mungkin keberadaanya?

A. PO2+ D. SO4-
B. QO3 - E. TO22+
C. RO4 2-
OSP2010 Olimpiada Kimia Indonesia Page
6
5. Kalor yang dibebaskan dalam reaksi netralisasi di bawah ini adalah -114 kJ
2NaOH(aq) + H2SO4(aq)  Na2SO4(aq) + 2H2O(l)
Dengan menggunakan informasi ini, berapakah nilai yang paling mendekati untuk kalor
yang dibebaskan dalam netralisasi berikut ini?
Ba(OH)2(aq) + 2HCl(aq)  BaCl2(aq) + 2H2O(l)
A. -57 kJ D. -171 kJ
B. -76 kJ E. -228 kJ
-114 kJ

F. Perhatikan diagram energi potensial berikut ini:

Mengenai ilustrasi diatas, manakah pernyataan yang BENAR:


A. Reaksi A adalah eksoterm sedangkan reaksi B adalah endoterm
B. Reaksi A adalah endoterm sedangkan reaksai B adalah eksoterm.
3. Reaksi A adalah yang mungkin paling spontan sedangkan reaksi B adalah yang
mungkin tidak spontan.
4. Reaksi A membebaskan panas sedangkan reaksi B menyerap panas.
5. Tidak ada pernyataan yang benar.

7. Manakah gaya antar-molekul yang paling kuat didalam molekul


CH2Cl2? A.Gaya dispersi London
Gaya Dipole-dipole
Ikatan Hydrogen
Gaya Van der Waals
Bukan salah satu diatas

8. Senyawa berikut ini, manakah yang paling sukar larut dalam CCl4 cair?

NH3
CH3-CH2-CH2-CH2-CH2-CH2-CH2-CH3
CO2
I2
CH3-CH2-CH3
OSP2010 Olimpiada Kimia Indonesia Page
7
9. Kurva manakah yang menunjukkan grafik yang benar dari pV terhadap p untuk sejumlah
massa tertentu suatu gas ideal pada temperatur konstan?

A B C D E

pV pV pV pV
pV

0 p 0 p 0 p 0 p 0 p

Jawaban : C

10. Diagram dibawah ini adalah representasi model 2-dimensi contoh air pada temperatur
tertentu .

Berikut ini, diagram manakah yang merupakan diagram paling tepat (terbaik) untuk
merepresentasikan sistem yang sama sesudah penambahan sukrosa (C 12H22O11)?

Jawab:E
OSP2010 Olimpiada Kimia Indonesia Page
8
11. Jika 75,0 mL larutan 0,250 M HNO3 dan 70.0 mL larutan 0,250 M KOH
dicampurkan, berapakah molaritas dari garam ini di dalam larutan yang dihasilkan?

0,250 M
0,500 M
0,333 M
0,125 M
0,167 M

12. Zat cair berikut ini mempunyai massa molar yang mirip. Berdasarkan data tekanan uap
pada 20 oC yang diberikan, ramalkan mana zat cair yang attraksi antar-molekul paling
lemah.

Asam Asetat (tekanan uap = 14 mm Hg)


ethyl chlorida (tekanan uap = 1050 mm Hg)
ethyl methyl ether (tekanan uap = 1260 mm Hg)
butana (tekanan uap = 1550 mm Hg)
isopropyl alkcohol (tekanan uap = 35 mm Hg)

13. Perhatikan kesetimbangan berikut :


2SO2 + O2  2SO3 .
dimana semua komponen gas berada pada 2000oC:
Dimisalkan bahwa 10,0 mol SO2 dan 8,0 mol O2 dimasukkan ke dalam wadah 1 L
dan didiamkan hingga tercapai kesetimbangan. Sesudah cukup waktu untuk
mencapai kondisi kesetimbangan,ternyata diperoleh 4,0 mol SO3. Berapa konsentrasi
dari SO2 dan O2 dalam kesetimbangan?

Keduanya, SO2 dan O2 = 4,0 M


SO2 = 6,0 M dan O2 = 4,0 M
Keduanya, SO2 dan O2 = 6,0 M
SO2 = 14,0 M dan O2 = 10,0 M
SO2 = 4,0 M dan O2 = 14,0 M

14. Untuk reaksi berikut ini: A(g) + B(g)  C(g) + D(g) , dalam wadah 1 L konsentrasi gas-gas
ini dalam campuran kesetimbangan adalah: [A] = 0,5M , [B] = 4,0 M , [C] = 6,0 M , dan
D. = 8,0 M. Berapa mol D harus ditambahkan ke dalam campuran ini untuk mencapai
konsentrasi A = 1,0 M?
12,1 mol
0,5 mol
0,034 mol
8,034 mol
10,56 mol
OSP2010 Olimpiada Kimia Indonesia Page
9
15. Didalam pelarut air pada temperatur 25 oC terjadi kesetimbangan: NH3(aq)
+ H2O(l)  NH4+(aq) + OH-(aq) , nilai pKb adalah 4,8.
Berapakah nila pKa untuk reaksi:
NH4+(aq) + H2O(l)  NH3(aq) + H3O+(aq) pada temperatur ini?

A. -4,8 D. 9,2
B. 2,2 E. 11,8
C. 4,8

16. Berikut ini adalah reaksi asam hipoklorit (HOCl) dengan air :
HOCl + H2O  H3O+ + OCl-
Apa pengaruhnya bila kedalam larutan asam hipoklorit ditambahkan sejumlah natrium
hipoklorit (NaOCl) terhadap reaksi kesetimbangan tersebut?

Konsentrasi HOCl dan H3O+ akan bertambah.


Konsentrasi HOCl dan H3O+ akan berkurang
Konsentrasi HOCl akan bertambah dan konsentrasi H3O+ akan berkurang.
Konsentrasi HOCl akan berkurang dan konsntrasi H3O+ akan bertambah.
Tidak akan ada perubahan karena natrium hipoklorit adalah garam tanpa
sifat asam atau basa.

17. Kesadahan dalam air kran dapat ditentukan dengan titrasi sampel terhadap pereaksi
yang membentuk ion kompleks dengan ion logam yang larut. Indikator untuk metoda
titrasi ini memerlukan penjagaan pH larutan sekitar 10. Berikut ini, manakah larutan akua
yang dapat digunakan untuk melakukan ini?

A. ammonia dan ammonium klorida


B. ammonium klorida dan asam hidroklorida.
C. Natrium etanoat dan asam etanoat.
D. Natrium hidroksida dan natrium etanoat.
E. Hanya natrium saja.

18. Suatu eksperimen dibuat untuk mengukur laju hidrolisis metil etanoat
CH3CO2CH3 + H2O  CH3CO2H + CH3OH

Hidrolisis ditemukan berjalan lambat dalam larutan akua netral tetapi reaksi ini
mencapai laju yang terukur bila larutan ini diasamkan dengan asam hidroklorida.
Apakah fungsi asam hidroklorida dalam reaksi campuran ini?
(a)menaikkan laju reaksi dengan aksi katalitik.
(b)Untuk meyakinkan bahwa reaksi ini mencapai kesetimbangan.
(c) Untuk menjaga pH konstan selama reaksi ini.
(d)Untuk menekan ionisasi dari terbentuknya asam etanoat.
(e)Untuk melarutkan metil etanoat.

OSP2010 Olimpiada Kimia Indonesia Page 10


19. Bila kedalam larutan jenuh perak chlorida (AgCl) ditambahkan sejumlah padatan AgCl
(Ksp = 1,8 x 10-10) akan menyebabkan:

Konsentrasi ion Cl- menjadi lebih besar dibandingkan dalam larutan jenuhnya.
Tidak ada perubahan konsntrasi ion Cl-
Konsentrasi ion Cl- dan Ag+ lebih besar dibandingkan dengan larutan jenuhnya.
Konsentrasi ion Cl- dan Ag+ lebih kecil dibandingkan dengan larutan jenuhnya.
Konsentrasi ion Cl- menjadi lebih besar dibandingkan dalam larutan jenuhnya

20. Untuk reaksi berikut ini, 2 NO + Br2  2 NOBr2, laju reaksi ini dapat dinyatakan
sebagai:
ି .[ ۰ ]
A.

]
B. ି .[ ۰

]
C. ା .[ ۰

]
D. ା .[ ۰

E. ା .[ ۰ ]


Jawab: C
21. Berapa konstanta laju untuk reaksi order pertama A  B , diberikan data berikut ini:
Waktu (s) [A]
0 1,76
6 0,88
12 0,44
18 0,22

d. 0,23
e. 1,0
f. 0,17
g. 0,12
tidak dapat ditentukan

v. Propelan padat yang dipakai dalam pendorong roket untuk‘space shuttle’ adalah suatu
campuran aluminium dan senyawa X. Senyawa X mengandung klor dengan tingkat
oksidasi +7. Berikut ini, manakah yang merupakan senyawa X?

A. NCl3 D. NH4ClO4
B. NH4Cl E. N2H5Cl
C. NH4ClO3
OSP2010 Olimpiada Kimia Indonesia Page
11
xxiii. Ion etandioat, C2O42-, dioksidasi dalam suasana asam oleh kalium
manganat(VII) menurut persamaan berikut:
2MnO4-(aq) + 5C2O42-(aq)  2Mn2+(aq) + 10CO2(g) + 8H2O(l)

Berapa volume larutan 0,020 mol/L kalium manganat(VII) yang dibutuhkan untuk
mengoksidasi sempurna 1,0x10-3 mol garam KHC2O4.H2C2O4?
a 20 mL
b 40 mL
c 50 mL
d 125 mL
e 250 mL

x. Dalam larutan asam, ion MnO4- mengoksidasi ion Cl- menjadi Cl2. Nilai E° untuk reaksi
adalah +0,16 V. Pernyataan atau persamaan manakah berikut ini yang benar?

Bilangan oksidasi klor berubah dari -1 menjadai +2.


Bilangan oksidasi mangan beribah dari +7 menjadai +4.
2MnO4- + 2Cl-  2MnO42- + Cl2.
2MnO4- + 8H+ + 6Cl-  2MnO2 + 4H2O + 3Cl2.
2MnO4- + 16H- + 10Cl-  2Mn2+ + 8H2O + 5Cl2.

y. Berikut ini, ditunjukkan empat isomer C6H12 :

Pasangan berikut ini, manakah yang merupakan pasangan isomer cis-trans?


i 1 dan 2
ii. 1 dan 3
iii. 1 dan 4
iv. 2 dan 4
v. 3 dan 4

z. Suatu senyawa ester metil asetat jika direduksi akan menghasilkan etanol, pereduksi
yang dapat dipakai adalah:
A. NaBH4 D. LiAlH4
B. H2, Ni E. SnCl2.
C. HCl, Fe
OSP2010 Olimpiada Kimia Indonesia Page
12
27. Suatu senyawa dikatakan aromatis apabila dapat memenuhi aturan Hϋckel (4n + 2 ).
Dibawah ini, manakah senyawa yang merupakan senyawa anti-aromatis.

Jawab: (C).

28. Produk yang dihasilkan dari reaksi adisi 1,3-butadiena dengan Cl2 adalah:

Jawab : (B)
OSP2010 Olimpiada Kimia Indonesia
Page 13
29. Susun urutan berdasarkan kenaikan reaktivitas reaksi substitusi elektrofilik dari senyawa
berikut adalah:

A. Nitrobenzena > toluena phenol > benzena


B. Nitrobenzena > phenol > toluena > benzena
C. Phenol > Toluena > benzena> > nitrobenzena
D. Toluena > phenol > nitrobenzena > benzena
E. Toluena > benzena > phenol > nitrobenzena

30. Produk reaksi yang terbentuk jika siklopentil metil keton direaksikan dengan
etilmagnesiumbromida , lalu dihidrolisa dalam suasana basa ,seperti terlihat
dalam reaksi di bawah ini adalah:

O
1. pel. eter
+
C + CH3CH2MgBr 2. H3O
CH3

Jawab: ( C ).
B. Selesaikanlah soal berikut ini

Soal 1. Reaksi gas metana dan produk reaksi (10 poin)

Gas metana (CH4) dapat bereaksi dengan belerang akan menghasilkan senyawa A dan
B. Senyawa B memiliki bau seperti telur busuk. Senyawa A akan bereaksi dengan gas C
yang berwarna hijau pucat dan menghasilkan senyawa D dan belerang. Senyawa D juga
dapat langsung dihasilkan dengan mereaksikan metana dan gas C. Tentukan senyawa
A, B, C, dan D, serta tuliskan reaksi setara untuk masing-masing langkah reaksi di atas.

JAWAB:

A= CS2 B = H2S C= Cl2 D= CCl4 (4 poin, masing masing


jawaban 1 poin)

CH4 + S CS2 + H2S (2 poin)

CS2 + Cl2 CCl4 + S (2 poin)

CH4 + Cl2 CCl4 (2 poin)

Soal 2. Sifat senyawa melalui analisis kualitatif. (22 poin)


Hasil pencampuran dua gas tak-dikenal, diperoleh suatu senyawa padat putih.
Berdasarkan analisis kualitatifnya, senyawa tersebut ternyata mengandung nitrogen,
boron, hydrogen dan fluor. Kedua gas tak-dikenal tersebut larut dalam air. Larutan gas
yang pertama mempunyai pH > 7 dan sedangkan gas yang lain larutannya memberikan
pH < 7.
Tentukan :

a. Tentukan rumus senyawa yang diperoleh ini, dalam perhitungan komposisi unsur
sebagai berikut: nitrogen 16,51%, boron 12,74%, hydrogen 3,54% dan fluor
67,21%. (3 poin)
b. Apakah kedua gas yang tak dikenal tersebut, dan tuliskan masing masing struktur
Lewis kedua gas dan senyawa yang diperoleh. (4 poin)
c. Berikan keadaan hibridisasi nitrogen dan boron dalam masing masing molekul gas
awal. (2 poin)
d. Berikan geometri molekul dari kedua gas tersebut. (4 poin)
e. Tentukan bilangan oksidasi dari unsur-unsur dalam gas-gas ini dan dalam
senyawa yang diperoleh. (2 poin)
f. Berapa jumlah dan jenis ikatan kimia yang ada antar atom dalam masing masing
senyawa yang diperoleh? (3 poin)

Kedua gas tersebut larut dalam air.


g. Tuliskan persamaan setara untuk interaksi masing masing gas tersebut dengan air.
(petunjuk: dengan air, salah satu gas menghasilkan asam okso yang sangat lemah
dan asam kompleks yang sangat kuat. (4 poin)

OSP2010 Olimpiada Kimia Indonesia Page 15


Solusi no 2.
a. (16,51/14) : (12,74/10,8) : (3,511/1) : (67,21/19) = 1 : 1 : 3 : 3; NB H 3F3. (3 poin)
b. NH3 dan BF3. (4 poin)
3 2
c. N –sp , susunan tetrahe dral, B – sp , dan susunan trigonal plan ar. (2 poin)
d. trigonal piramida, triangle (trigonal planar, segitiga datar) (4 poin )

e. N-(-3), B-(+3), didalam senyawa diperoleh adalah sama. (2 poin)


f. enam kovalen polar ikat an- dan satu ikatan kovalen polar koordinatif. (3 poin)

+ -
g. NH3 + H2O  NH4 + O H ;
4BF3 + 3H2O  H3BO3 + 3H[BF4] (4 poin)

Soal 3. Struktur dan sifat molekul. (18 poin)

Anda mempunyai senya wa SNF3 dan SeF4 (atom pusatnya ditandai d engan
huruf tebal).
a. Apa orbital hibrida masing masing atom pusat pada SNF3 dan Se F4, dan
gambarkan sketsa b entuk geometri molekul masing masing (6 poin)
b. Manakah dari kedua senyawa tersebut yang bersifat polar? Jelas kan. (4 poin)
c. Gaya antar molekul apa yang dominan pada padatan SNF3? (2 poin)
d. Seandainya atom pusat pada molekul SNF3 adalah S (NSF3):
OSP2010 Olimpiada Kimia Indonesia Page 16
i. tentukan orbital hibrida atom S, dan sketsakan bentuk geometrinya.
(3 poin)

ii. Struktur molekul manakah yang stabil antara SNF3 dan NSF . (3poin)
3

Jawab:

a. Sketsakan bentuk molekulnya SNF3 (Tetrahedral) SeF4 (see saw-jungkat


jungkit)

b. Manakah dari kedua senyawa tersebut yang bersifat polar? Jelaskan.


Keduanya, SNF3 (atomnya berbeda S dan F) SeF4 (ada pasangan electron
bebas)

c. Gaya antar molekul apa yang dominan pada padatan SNF3? Dipol-dipol

3 3
d. Jenis orbital hibrida: N- sp ; Se-dsp

Soal 4. Termolisis dan larutan garam ammonium chlorida.

Telah diketahui dengan baik bahwa termolisis dari garam ammonium halida akan
menghasilkan hidrogen halida dan ammonia (proses disosiasi). Reaksi sebaliknya akan
terjadi bila uap campuran hasil disosiasi tersebut didinginkan mengalami sublimasi dan
diperoleh kembali padatan garam ini. Sejumlah sampel ammonium chlorida padat
dipanaskan dalam wadah hampa (terevakuasi) yang tertutup rapat. Untuk dissosiasi
termal padatan ammonium klorida pada temperatur 427 C tekanan uap dalam wadah
adalah 608 kPa, dan tekanan ini hampir sama dengan jumlah tekanan parsial dari
produk dissosiasi. Bila temperatur naik menjadi 459 C, tekanannya naik menjadi 1115
kPa. Anggaplah pada ranah temperatur ini nilai r konstan, maka:
OSP2010 Olimpiada Kimia Indonesia Page 17
a. Tuliskan reaksi termolisis garam amonium chlorida.
b. Hitunglah nilai konstanta kesetimbangan termodinamika dari reaksi tersebut pada
temperatur 427 oC dan 458 oC.
c. Tentukan Entalpi reaksi standar ( r ), dan tentukan apakah reaksi pembentukan
padatan dari gas gas (sublimasi) tersebut eksoterm atau endoterm.

Bila anda melarutkan garam ammmonium chlorida dalam air, maka:


d. Bagaimana sifat pH larutannya, asam atau basa, jelaskan dan tuliskan reaksinya.
e. Dapatkah anda membuat larutan buffer dari larutan ammoniumchlorida?
Berikan alasan jawaban anda, dan bagaimana pH buffer yang diperoleh

JAWAB

a. NH4Cl(s)  NH3(g) + HCl(g),

b. NH4Cl(s)  NH3(g) + HCl(g),


Kp1= (p(NH3)/p )x(p(HI)/p )
Pada 427 C : p(NH3) + p(HCl) = p

p(NH3) = p(HCl) = p/2 = 608/2 = 304 kPa = 3,04x105

Pa. K1 = (3,042x1010)/(105x105) = 9,24

Pada 459 C: K2 = ((1115/2)2x106)/(105x105) =


31,08 p(NH3) = p(HCl) = p/2 = 1115/2

K2 = ((1115/2)2x106)/(105x105) = 31,08

c. ln(K2/K1) = -( Hr /R)x(1/T2 – 1/T1)

Hr = (ln(K1/K2)xR)/(1/T2 – 1/T1)

= (ln(31,08/9,24)x8,314)/(1/732 – 1/700)

= 1,617x105 K.mol-1.

Hr = 161,7 kJ.mol-1

Kp2= (p(NH3)/p )x(p(HI)/p )

d. NH4Cl (aq)  NH4+(aq) + Cl-(aq) NH4+


(aq) + H2O(l)  NH3 (aq) + H3O+,

terhjadi hidrolisis sehingga larutan bersifat asam (pH<7)

e. Dapat membentuk buffer dengan menambahkan larutan NH 3, dan


larutan membnetuk buffer basa (pH>7).
OSP2010 Olimpiada Kimia Indonesia Page
18
Soal 5. Senyawa oksida timbal (plumbum)

Timbal (Pb) yang berbahaya bila masuk kedalam tubuh manusia, adalah unsur
golongan 14 (4A) dengan konfigurasi elektron [Xe]4f14 5d106s26p2..
a. Tuliskan senyawa oksida timbal yang mungkin dan tentukan bilangan oksidasinya
Senyawa Pb2O3 dan minium, Pb3O4, keduanya mewakili campuran oksida timbal,
seperti halnya pada Fe3O4 yang merupakan campuran FeO dan Fe 2O3. Campuran
oksida timbal Pb3O4 dikenal juga sebagai “timbal merah (red lead)”, dan digunakan
terutama sebagai pigment dan lapisan permukaan pencegah korosi pada besi. Oksida
ini juga digunakan untuk produksi pelumas temperatur tinggi.
b. Jelaskan mengapa oksida Pb2O3 dan Pb3O4 ini disebut oksida campuran.
Tunjukkan bilangan oksidasi timbal dan besi dalam rumus oksida campuran
tersebut, dan bagaimanana komposisi masing masing oksida dalam
campuran oksida tersebut.
c. Bila minium, Pb3O4, dipandang sebagai suatu garam, yaitu garam yang terbentuk
dari kation timbal(II) dengan anion orthoplumbat(IV), maka tuliskan rumus
empirisnya
d. Bila garam timbal(II) orthoplumbat(IV) adalah garam yang sukar larut , dan
dilarutkan dalam air, tuliskan persamaan reaksi kesetimbangannya dalam air.
Hitunglah kelarutan molar dari minium dalam air murni anggap = 100%. Hasil
kali kelarutan garam ini Ksp = 5,3x10-51.
e. Hitunglah massa timbal (g/L) dalam air yang dimasukkan dalam tangki ini, dimana
permukaan dalam tangki air tersebut dilapisi oleh minium untuk mencegah korosi.
Anggap bahwa keadaan telah mencapai kesetimbangan.

Minium ditempatkan dalam larutan asam nitrat akua dengan pH = 3.

f. Hitunglah kelarutan molar minium dalam larutan ini tanpa mengingat hidrolisis
garam.

JAWAB:
II IV
a. Pb O dan Pb O2

b. Oksida ini disebut oksida campuran, karena bilangan oksidasi timbal (atau besi)
dalam rumus oksida ini berbeda:
II IV II IV
Pb2O3: Pb O.Pb O2 (campuran dengan perbandingan mol Pb O:Pb O2 =1 : 1)
II IV II IV
Pb3O4: 2Pb O.Pb O2 (campuran dengan perbandingan mol Pb O:Pb O2 = 2 : 1)
II III II III
Fe O.Fe2 O3. (campuran dengan perbandingan mol Fe O: Fe2 O3 = 1 : 1
II IV
c. Minium sebagai garam: Pb2PbO4 (Pb2 Pb O4 – timbal(II) orthoplumbat(IV))

2+ 4-
d. Pb2PbO4(s)  2Pb (aq) + PbO4
2+ 2 4-
(aq) Ksp = [Pb ] x[PbO4 ]
OSP2010 Olimpiada Kimia Indonesia Page 19
1/2 -51 1/2 -17
S = (Ksp/4) = (5,3x10 /4) = 1,1x10
2+ -17
mol/L e. [Pb ] = 2s = 2x1,1x10 mol/L
4- -17
[PbO4 ] = s = 1,1x10 mol/L
2+ 4- 3 -17 -17 -13
n(Pb) = Vx([Pb ] + [PbO4 ]) = 5x10 x(2,2x10 + 1,1x10 ) = 1,65x10
-13 -11
mol/L m(Pb) = n(Pb)xM(Pb) = 1,65x10 x 207 g= 3,42x10 g/L
f. Pb2PbO4 + 4HNO3  2Pb(NO3)2 + PbO2(s) +
+ -pH -3
2H2O [HNO3] = [H ] = 10 = 10 mol/L
2+ -4
[Pb ] = 1/2x[HNO3] = 5x10 mol/L
2+
(Pb dari minium diabaikan)
4- 2+ 2 -51 -4 2 -44
s = [PbO4 ] = Ksp / [Pb ] = 5,3x10 / (5x10 ) = 2,1x10 mol/L

Soal 6. Additif bahan bakar minyak

Senyawa 2,2,4-trimetilpentana merupakan salah satu komponen penting yang


menentukan kualitas bahan bakar minyak untuk kendaraan bermotor. Adanya senyawa
tersebut menyebabkan ‘bilangan oktan’ dalam bahan bakar minyak menjadi tinggi.
Senyawa tersebut memiliki ‘bilangan oktan’ 100. Soal-soal berikut berkaitan dengan
senyawa yang memiliki struktur seperti ditampilkan di bawah ini:

a. Jelaskan manakah yang titik didihnya lebih tinggi: senyawa 2,2,4-trimetilpentana


atau n-oktana?
Jawab:

Yang titik didihnya lebih tinggi adalah n-oktana karena interaksi antarmolekulnya
lebih kuat daripada 2,2,4-trimetilpentana. Hal ini disebabkan oleh struktur ruang
akibat adanya interaksi antarmolekul pada n-oktana menjadi lebih
rapat/kompak di samping distribusi kerapatan elektronnya menjadi lebih merata
sehingga interaksi antarmolekulnya lebih kuat , maka energi luar berupa kalor
yang dibutuhkan untuk memutuskan interaksi tersebut menjadi lebih besar yang
berbanding lurus dengan suhu yang lebih tinggi.

b. Senyawa 2,2,4-trimetiloktana ketika direaksikan dengan Br 2 di bawah sinar UV


menghasilkan banyak senyawa turunan alkil bromidanya, di antaranya adalah
senyawa 1-bromo-2,4,4-trimetilpentana (A) dan 2-bromo-2,4,4-trimetilpentana
(B). Gambarkan struktur kedua senyawa isomer tersebut (senyawa A dan B)
dan jelaskan mana produk yang paling banyak
OSP2010 Olimpiada Kimia Indonesia Page 20
Produk yang paling banyak adalah senyawa B, karena terbentuk dari intermediet
o
(zat antara) radikal yang paling stabil (radikal tersier (3 )).

c. Jika senyawa A dari soal (b) direaksikan dalam larutan yang mengandung ion
OH- akan menghasilkan senyawa alkohol (C). Gambarkan struktur senyawa C
dan jelaskan bagaimana kelarutan senyawa C dan senyawa 2,2,4-trimetilpentana
dalam air.

Jawab

Kelarutan senyawa C dalam air lebih besar daripada senyawa


2,2,4-trimetilpentana karena memiliki potensi untuk membentuk
ikatan hydrogen dengan molekul air.
d. Senyawa C dari soal (c) ketika direaksikan dengan asam sulfat pekat akan
mengalami dehidrasi dan menghasilkan senyawa alkena D, E dan F beserta
isomernya. Gambarkan struktur alkena D, E dan F beserta isomernya!

JAWAB

e. Senyawa C dari soal (c) ketika direaksikan dengan KCrO 3 pada awalnya akan
menghasilkan senyawa aldehid G dan kemudian akan teroksidasi lebih lanjut
OSP2010 Olimpiada Kimia Indonesia
Page 21
menghasilkan senyawa karboksilat H. Gambarkan struktur senyawa G dan H
serta tuliskan nama kedua senyawa tersebut.

JAWAB

f. Ketika senyawa H dari soal (e) direaksikan dengan senyawa C dari soal (c)
dalam suasana asam dan dipanaskan akan terbentuk senyawa ester I.
Gambarkan senyawa I dan tuliskan reaksi yang terjadi!

JAWAB

g. Senyawa aldehid G dapat mengalami reaksi kondensasi Aldol dalam suasana


basa. Gambarkan struktur hasil kondensasi aldehid G dengan penambahan
larutan KOH!

JAWAB:
OSP2010 Olimpiada Kimia Indonesia Page 22
SOAL UJIAN
SELEKSI CALON PESERTA OLIMPIADE SAINS NASIONAL 2011
SELEKSI TINGKAT PROVINSI
CALON TIM OLIMPIADE KIMIA INDONESIA 2012

Kimia

7 – 8 Juni 2011

Waktu: 150 Menit

Kementerian Pendidikan Nasional


Direktorat Jenderal Manajemen Pendidikan Dasar dan Menengah
Direktorat Pembinaan Sekolah Menengah Atas

2011
Petunjuk Pengerjaan Soal
1. Isilah biodata anda dengan sebenar – benarnya dan tulis biodata anda di lembar
jawaban dengan huruf cetak tanpa disingkat.

2. Soal teori ini terdiri dari soal pilihan ganda dan soal essay dengan poin:

Soal Pilihan Ganda = 60 Poin


Benar =+2 Poin
Salah =-1 Poin
Kosong =0 Poin
Soal Essay = 104Poin

Total Poin = 164 Poin

3. Waktu tersedia: 150 Menit

4. Semua jawaban harus ditulis di lembar jawaban yang tersedia.

5. Diperkenankan menggunakan kalkulator.

6. Diberikan tabel periodik unsur di halaman 3 dan daftar tetapan di halaman 4.

7. Anda dapat mulai bekerja bila sudah ada tanda mulai dari pengawas. Soal ini bersifat
final sehingga tidak akan ada ralat sama sekali.

8. Anda harus berhenti bekerja bila ada tanda berhenti dari pengawas.

9. Soal boleh dibawa pulang

Seleksi Olimpiade Sains Nasional Kimia Tingkat Provinsi 2011 Halaman 2


Seleksi Olimpiade Sains Nasional Kimia Tingkat Provinsi 2011 Halaman 3
Daftar Tetapan
-1 -1 1 -1
Tetapan Gas Universal (R) = 0,082 L atm mol K = 8,314 J mol K
-34
Tetapan Planck (h) = 6,626 x 10 Js
7 -1
Tetapan Rydberg (R) = 1,097 x 10 m
23 -1
Bilangan Avogadro (NA) = 6,02 x 10 Partikel mol

-23 -1
Tetapan Boltzmann (k) = 1,38 x 10 JK
-1
Kecepatan Cahaya Pada Ruang Vakum (c) = 299 792 458 m s
-1
Tetapan Faraday (F) = 96 450 C mol
o -14
Tetapan Kesetimbangan Ionisasi Air (Kw) 25 C = 1,0 x 10
o o -1
Konstanta Penurunan Titik Beku Air (KfAir) 25 C = 1,86 C m
o o -1
Konstanta Kenaikan Titik Didih Air (KbAir) 25 C = 0,52 C m

o
Standard Temperature and Pressure (Fluida Ideal) = 0 C dan 1 atm
o
Standard Temperature and Pressure (Termokimia dan Elektrokimia) = 25 C, 1 atm, dan 1 M

Seleksi Olimpiade Sains Nasional Kimia Tingkat Provinsi 2011 Halaman 4


Soal Pilihan Ganda (60 Poin)

A. Berikut ini, ion manakah yang mempunyai jumlah elektron lebih banyak dari
proton dan jumlah proton lebih banyak dari jumlah neutron? [D = 1H-2]
- +
A. D D. D3O
+ -
B. He E. OH
-
C. OD
2-
2. Ion etanadioat (C2O4 ) dioksidasi oleh larutan kalium manganat(VII) dalam
suasana asam sesuai persamaan reaksi berikut:

- 2- +  2+
2MnO4 (aq) + 5C2O4 (aq) + 16H (aq) 2Mn (aq) + 10CO2(g) + 8H2O(l)
-3
Berapa volume dari larutan 0,020 mol dm kalium manganat(VII) yang dibutuhkan
-3
untuk mengoksidasi sempurna 1,0 x 10 mol garam KHC2O4.H2C2O4?
3 3
A. 20 cm D. 125 cm
3 3
B. 40 cm E. 250 cm
3
C. 50 cm
-1
C. Dibawah ini adalah energi ionisasi berurutan (dalam kJ mol ) dari unsur X:

870 1800 3000 3600 5800 7000 13200

Apakah unsur X ini?

A. As D. Te
B. I E. Zr
C. O

4. Bila air diadil dengan glukosa, mula – mula akan terbentuk ikatan hidrogen yang
kuat antara molekul glukosa dan air, tetapi jika penambahan air dilanjutkan, ikatan
hidrogen akan terputus. Berikut ini, grafik manakah yang terbaik untuk
menggambarkan perubahan suhu yang teramati?

A.T B.T C. T

0 Volume Air Ditambahkan 0 Volume Air Ditambahkan 0 Volume Air Ditambahkan

Seleksi Olimpiade Sains Nasional Kimia Tingkat Provinsi 2011 Halaman 5


D. T E. T

0
0
Volume Air Ditambahkan Volume Air Ditambahkan

5. Nilai perubahan entalpi untuk proses yang digambarkan oleh persamaan ini:

 + -
Na(s) Na (g) +e
Adalah sama dengan. . .

Energi ionisasi pertama dari natrium


Perubahan entalpi penguapan natrium
Jumlah energi ionisasi dan afinitas elektron dari natrium
Jumlah perubahan entalpi atomisasi dan energi ionisasi pertama dari natrium
Jumlah perubahan entalpi atomisasi dan afinitas elektron dari natrium

6. Kalor yang dibebaskan dalam reaksi netralisasi berikut adalah -114 kJ.


2NaOH(aq) + H2SO4(aq) Na2SO4(aq) + 2H2O(l)
Dengan menggunakan informasi tersebut, berapa nilai yang cocok untuk kalor
yang dibebaskan dalam netralisasi berikut?


Ba(OH)2(aq) + 2HCl(aq) BaCl2(aq) + 2H2O(l)
A. – 57 kJ D. – 171 kJ
B. – 76 kJ E. – 226 kJ
C. – 114 kJ

7. Kesadahan dalam air kran dapat ditentukan dengan titrasi larutan sampel
terhadap pereaksi yang membentuk ion kompleks dengan ion logam terlarut.
Indikator untuk reaksi ini membutuhkan pH larutan yang dijaga sekitar 10. Berikut
ini, manakah larutan akua yang dapat digunakan untuk pekerjaan ini?

A. Amoniak dan ammonium klorida


B. Ammonium klorida dan asam hidroklorida
C. Natrium etanoat dan asam etanoat
D. Natrium hidroksida dan natrium etanoat
E. Hanya natrium hidroksida

8. Mengapa silikon tetraklorida bila dilarutkan dalam air akan segera terhidrolisis,
sedangkan karbon tetraklorida tidak?

A. Ikatan Si – Cl lebih polar daripada ikatan C – Cl

Seleksi Olimpiade Sains Nasional Kimia Tingkat Provinsi 2011 Halaman 6


2. Ikatan Si – Cl lebih lemah daripada ikatan C – Cl
3. Silikon dapat memakai orbital d, sedangkan karbon tidak.
4. Orbital d dari karbon terisi, sedangkan yang silikon tidak
5. Molekul karbon tertraklorida dan molekul silikon tetraklorida mempunyai bentuk
yang berbeda
2 2 6 2 6 2 2
9 Titanium mempunyai struktur elektronik 1s 2s 2p 3s 3p 4s 3d . Berikut ini,
manakah senyawa yang keberadaannya tidak sesuai dengasn sifat unsur
titanium?

A. K2TiO4 D. TiO
B. K3TiO6 E. TiO2
C. TiCl3

10. Berturut – turut, bagaimana masing – masing hibridasi dari atom nomor 1 hingga
nomor 4 berikut ini:

O
H H H
2 H 2
N C C N C C N

1 2 3 4
3 2 3 3
sp sp sp sp
3 2 3
sp sp sp sp
3 2 2
sp sp sp sp
2 2 3
sp sp sp sp
2 2 3
sp sp sp sp

11. Berikut ini, manakah senyawa yang kelarutannya dalam air paling kecil?

CH3CH(NH2)CO2H
CH3CH(OH)CH3
CH3CH2CH2NH2
C6H5CO2Na
C6H5NH2

12. Pasangan berikut ini, manakah molekul yang mempunyai bentuk geometri sama?

AlCl3 dan BCl3


AlCl3 dan PCl3
BF3 dan NH3
BeCl2 dan H2O
CO2 dan SO2
+ 2+
13. Spesies Ar, K , dan Ca adalah isoelektronik (Mempunyai jumlah elektron sama).
Bagaimana urutan kenaikan jari – jarinya dari yang terkecil hingga terbesar?

2+ +
Ar ; Ca ;K
Seleksi Olimpiade Sains Nasional Kimia Tingkat Provinsi 2011 Halaman 7
+ 2+
Ar ; K ; Ca
2+ +
Ca ; Ar ; K
2+ +
Ca ; K ; Ar
+ 2+
K ; Ar ; Ca

14 Molekul gas SO2 bersifat polar (µ = 1,62 D), sedngkan gas CO 2 molekulnya
bersifat non polar (µ = 0). Perbedaan ini disebabkan oleh fakta bahwa. . .

A. C dan O kira – kira mempunyai elektronegativitas yang hampir sama


sedangkan S dan O mempunyai elektronegativitas berbeda.
B. CO2 mempunyai jumlah ikatan ganda dua yang genap sedangkan SO 2
mempunyai jumlah ikatan rangkap dua ganjil.
C. C dan O beda golongan sedangkan S dan O sama golongannya.
D. Ikatan C=O adalah non polar sedangkan ikatan S=O adalah polar.
E. Geometri molekul CO2 adala linear sedangkan SO2 tidak linear.

15 Perhatikan senyawa Q hingga T berikut ini:

CH2CH2CH2OH CH2CH2COOH
Q R

CH3CH3CH3CH2OH CH3CH2CH2CH3
S T

Berikut ini, manakah urutan titik didih yang benar (tinggi ke rendah) dan senyawa
tersebut?

A Q>R>S>T
B R>S>Q>T
C R>T>S>Q
D R>Q>S>T
E S>T>Q>R

16. Larutan 20 mg Insulin dalam 5,0 mL air pada 300 K memberikan tekanan osmosa
sebesar 12,5 mmHg. Berat molekul Insulin adalah. . .
-1
16700 g mol
-1
12360 g mol
-1
8680 g mol
-1
5990 g mol
-1
3480 g mol

17. Anda mempunyai dua sampel senyawa X dan masing – masing ditambahkan
larutan natrium hidroksida dan asam sulfat pekat. Bila masing – masing campuran
senyawa X tersebut dipanaskan, keduanya akan menghasilkan gas. Bila gas hasil
pemanasan tersebut direaksikan, maka akan terbentuk kembali senyawa semula.
Yang manakah senyawa X ini?

CH3CO2C2H5
NH2CH2CO2CH3
NH4Cl

Seleksi Olimpiade Sains Nasional Kimia Tingkat Provinsi 2011 Halaman 8


NH4I
(NH4)2SO4
o
18. Suatu zat padat X meleleh secara tajam di atas suhu 100 C. Padatan atau lelehan
senyawa X tidak menghantar listrik, tetapi X larut dalam pelarut hidrokarbon.
Apakah struktur paling cocok untuk padat X ini?

A. Suatu kristal atomik


B. Suatu kristal ionik
C. Suatu kristal molekular raksasa
D. Suatu kristal molekular
E. Logam

19. Perhatikan reaksi dalam larutan akua berikut ini:

- -  - -
I (aq) + OCl (aq) IO (aq) + Cl (aq)
Dan berikut ini adalah data konsentrasi awal dan laju awal untuk reaksi ini:

- - -1
[I ], M [OCl ], M Laju Awal, M s
-4
0,1000 0,0500 3,05 x 10
-4
0,2000 0,0500 6,10 x 10
-4
0,3000 0,0100 1,83 x 10
-4
0,3000 0,0200 3,66 x 10

Manakah pernyataan hukum laju untuk reaksi tersebut?


-
A. Laju = r = k [I ]
-
B. Laju = r = k [OCl ]
-2
C. Laju = r = k [I ]
- -
D. Laju = r = k [I ] [OCl ]
-2 -
E. Laju = r = k [I ] [OCl ]

20. Sebanyak 1 mol sampel asam etanoat diencerkan pada suhu konstan hingga
volume V. Manakah diagram yang menunjukkan perubahan pH terhadap V ?

A B C

pH pH pH

7 7 7

0 V 0 V 0 V

Seleksi Olimpiade Sains Nasional Kimia Tingkat Provinsi 2011 Halaman 9


D E

pH pH

7 7

0 V 0 V

21 Kesetimbangan berikut terdapat dalam suatu campuran asam nitrat pekat dan
asam sulfat pekat:
+ - +
HNO3(aq) + 2 H2SO4(aq) NO2 (aq) + 2 HSO4 (aq) + H3O (aq)

Pernyataan yang manakah tentang kesetimbangan ini yang benar ?


+
A. Pernambahan H2O akan menurunkan konsentrasi NO2
+
B. HNO3 dan NO2 adalah pasangan asam – basa konjugasi
C. Asam nitrat bertindak sebagai oksidator
D. Asam sulfat bertindak sebagai dehydrator
E. Asam sulfat bertindak sebagai basa

22. Jus buah dan minuman ringan berkarbonat seperti lemonade sering dijual dalam
wadah kaleng aluminium. Apa alsan yang paling utama, mengapa logam
aluminium cocok untuk hal ini ?

A. Aluminium dapat didaur ulang


B. Aluminium mempunyai densitas yang sangat rendah
C. Aluminium adalah logam yang kelimpahannya di kulit bumi terbesar
D. Aluminium lahan korosi oleh asam
E. Aluminium tahan korosi oleh air

23. Dalam Standard Hydrogen Electrode , kawat platina digunakan sebagai elektroda.
Perhatikan potensial reduksi standar berikut ini:

ө
Setengah Reaksi E red , Volt

Zn 2+
(aq) + 2e -
Zn(s) - 0,76
+
2H O
3 (aq)
+ 2e - 
2H O
2 (l)
+ H 2(g) 0,00

Pt2+(aq) + 2e- Pt(s) 1,20

Pernyataan berikut ini, manakah yang paling tepat untuk menjelaskan apa yang
terjadi bila kawat platina dalam Standard Hydrogen Electrode diganti dengan
kawat zink (Anggap bahwa perubahan hanya terjadi pada setengah sel)?

5. Ion Zink dalam larutan akan direduksi, massa elektroda Zink akan bertambah
6. pH larutan akan turun
7. Elektroda Zink akan dioksidasi, massa elektroda Zink akan berkurang

Seleksi Olimpiade Sains Nasional Kimia Tingkat Provinsi 2011 Halaman 10


D. Perubahan ini tidak berpengaruh, Standard Hydrogen Electrode berfungsi
tanpa ada perubahan
E. Tidak mungkin dijelaskan

24. Berikut ini, pereaksi manakah yang dapat digunakan untuk membuat
Siklopropana jika bahan baku awalnya adalah 1,3 – Dibromopropana ?

A Zn, Natrium Iodida


o
B Dipanaskan pada temperatur 125 C
C Zn, NaOH
D Dehidrohalogenasi
E Reduksi dengan Ni, H2

25. Reaksi klorinasi senyawa 4 – Metilsikloheksena dengan menggunakan Cl 2, UV


akan menghasilkan produk utama. . .

A 3 – Kloro – 3 – Metilsikloheksena
B 3 – Kloro – 6 – Metilsikloheksena
C 3,5 – Diklorosikloheksena
D 1,2 – Diklorosikloheksana
E 1 – Klorosikloheksena

26. Aroma dari buah Almond berasal dari senyawa Amygdalin. Hidrolisis dari
Amygdalin menghasilkan Z, kemudian bila Z dihidrolisis maka akan menghasilkan
Produk lain. Tentukan Z dan Produk lain tersebut.

C6H5 O Gula
C → Z→ Produk Lain
H CN
Amygdalin

Z adalah C6H5CH(OH)COOH dan Produk Lain adalah C6H5CH(OH)CH2OH


Z adalah C6H5CH(OH)CN dan Produk Lain adalah C6H5CH(OH)COOH
Z adalah C6H5COOH dan Produk Lain adalah HCHO
Z adalah C6H5OH dan Produk Lain adalah CH3CN
Z adalah C6H5CN dan Produk Lain adalah HCHO

27. 4 – Metil – 2 – Pentena jika direaksikan dengan HCl akan terjadi penataan ulang
dari karbokation, sehingga hasil yang paling besar dari reaksi adisi Markovnikov
ini adalah. . .

2 – Kloro – 3 – Metilpentana
2 – Kloro – 2 – Metilpentana
3 – Kloro – 2 – Metilpentana
4 – Kloro – 2 – Metilpentana
2 – Kloro – 4 – Metilpentana

28. Senyawa yang mempunyai rumus molekul seperti berikut di bawah ini yang
mempunyai 4 derajat kekurangan atom hidrogen adalah. . .

Seleksi Olimpiade Sains Nasional Kimia Tingkat Provinsi 2011 Halaman 11


C6H10
C10H16
C5H5N
C10H10O2
C6H6Cl6

CC. Jika Benzena berturut – turut yang direaksikan dengan:


Asilasi Friedel Craft
Dinitrasi dengan HNO3, H2SO4
Reduksi Sn, HCl
Pereaksi Wolff Kishner (H2NNH2, KOH)
Maka senyawa yang dihasilkan adalah . . .

p – Nitroasetofenon
m – Etilanilin
3 – Metilanilin
o – Nitroetilbenzena
p – Nitroanilin

DD. Molekul – molekul dibawah semuanya adalah non-aromatik, kecuali?

A. D.

B. E.

C.

Seleksi Olimpiade Sains Nasional Kimia Tingkat Provinsi 2011 Halaman 12


Soal Essay (104 Poin)

Soal 1 – Analisis Kemurnian Benzil Klorida (14 Poin)

Benzil klorida (C6H5CH2Cl) secara komersial dibuat melalui proses klorinasi dari
toluena (C6H5CH3). Untuk menentukan kemurnian dari bahan komersil benzil klorida,
ditimbang sebanyak 0,255 gram contoh untuk dianalisis. Contoh tersebut dicampur
dengan 25 mL larutan NaOH 4 M dalam suatu labu bulat 100 mL, dan kemudian direfluks
dan dipanaskan selama 1 jam. Setelah didinginkan dalam temperatur kamar, pertama
kali campuran tersebut ditambahkan 50 mL larutan HNO 3 20% dan diikuti dengan
penambahan 25 mL larutan AgNO 3 0,100 M dan ternyata terbentuk endapan putih.
Kelebihan perak nitrat kemudian dititrasi dengan larutan NH 4SCN 0,100 M dengan
menggunakan larutan Besi(III) amonium sulfat sebagai indikator. Dalam titrasi ini
terbentuk endapan putih, dan titik akhir titrasi ditentukan dengan timbulnya warna merah
2+
(Terbentuk [Fe(SCN)] ).

a. Tuliskan reaksi pembuatan benzil klorida melalui klorinasi toluena! (2 Poin)


b. Tuliskan reaksi pemanasan campuran NaOH dan benzil klorida! (2 Poin)
c. Apa tujuan penambahan HNO3 sebelum penambahan AgNO3? (1 Poin)
d. Tuliskan reaksi pembentukan endapan setelah penambahan HNO 3
dan larutan AgNO3! (1 Poin)
e. Tuliskan reaksi pembentukan endapan selama titrasi berlangsung! (1 Poin)
2+
Bila warna merah [Fe(SCN)] muncul pada saat 6,75 mL larutan NH 4SCN 0,100
M telah dititrasikan:

f. Tentukanlah berapa berat persentase kemurnian benzil klorida dalam


contoh komersial tersebut. (4 Poin)
2+
g. Mengapa pada titik akhir terjadi warna merah [Fe(SCN)] ? (3 Poin)

Soal 2 – Fosfor dan Senyawanya (16 Poin)

Fosfor putih (P4) adalah unsur non-logam yang sangat reaktif dengan titik leleh
o
44 C dan berbentuk padat pada temperatur ruang serta dapat larut dalam CS 2 cair. Bila
kontak dengan oksigen di udara fosfor putih akan terbakar menimbulkan api dan
membentuk fosfor(V) oksida. Fosfor dalam bentuk kalsium fosfat merupakan komponen
penting pada pembentukan tulang. Fosfor terkandung dalam DNA yang menjadi ciri khas
makhluk hidup secara individu.

a. Gambarkan struktur geometri fosfor putih! (4 Poin)


b. Berapakah besarnya sudut P – P – P dalam fosfor putih? (2 Poin)
c. Mengapa fosfor putih dapat larut dalam CS 2 ? (2 Poin)
d. Tuliskan reaksi yang terjadi bila fosfor putih terbakar diudara! (2 Poin)
e. Tuliskan rumus kimia kalsium fosfat! (1 Poin)
f. Gambarkan struktur Lewis ion Fosfat! (3 Poin)
g. Molekul / ion apa saja yang terkandung dalam DNA ? (2 Poin)

Soal 3 – Kesetimbangan Gas ClF3 dalam Ruang Tertutup (16 Poin)

Seleksi Olimpiade Sains Nasional Kimia Tingkat Provinsi 2011 Halaman 13


Klorin trifluorida (ClF3) adalah gas tak berwarna, berbau manis, bersifat racun dan
o
bersifat korosif dengan titik didih 11,8 C. Gas ini digunakan sebagai komponen bahan
bakar roket dan sebagai pembersih untuk menghilangkan oksida silikon dan oksida
logam dalam industri semikonduktor.

a. Apa orbital hibrida atom Cl dalam ClF3 ? (2 Poin)


b. Tuliskan struktur Lewis dan bentuk geometri dari ClF 3 ! (4 Poin)
c. Apa jenis ikatan kimia yang terdapat dalam ClF 3 ? (1 Poin)

Didalam ruang tertutup dan temperatur tinggi gas ClF 3 akan terurai dan
menghasilkan kesetimbangan campuran gas ClF 3, ClF, dan F2. Bila 9,25 g ClF3
dimasukkan kedalam wadah hampa yang volumenya 2 liter dan temperatur 700 K maka,
sebanyak 19,8 % gas ClF3 akan terurai dan menghasilkan kesetimbangan campuran gas
ClF3, ClF, dan F2.

d. Berapa tekanan awal gas ClF3 ? (2 Poin)


e. Tuliskan reaksi kesetimbangan, dan bagaimana pernyataan
konstanta kesetimbangan konsentrasi (Kc) dan tekanannya (Kp) ! (3 Poin)
f. Berapa nilai kedua konstanta kesetimbangan tersebut pada 700 K ? (4 Poin)

Soal 4 – Asam Fosfat dan Sifatnya dalam Larutan (20 Poin)


-3
Asam fosfat adalah asam lemah berproton tiga dengan Ka 1 = 7 x 10 , Ka2 = 6 x
-8 -13
10 , dan Ka3 = 4 x 10 . Asam fosfat dapat dibuat dengan mereaksikan padatan mineral
batuan Ca5(PO 4)3F dengan asam sulfat encer, dengan menghasilkan produk samping
HF dan gipsum.

a. Ion – ion apa saja yang mungkin terbetnuk dalam larutan asam fosfat,
tuliskan reaksi kesetimbangan ionnya ! (4 Poin)
2. Hitung semua konsentrasi ion – ion tersebut dalam asam fosfat 1 M ! (8 Poin)
3. Berapa tingkat oksidasi kalsium dan fosfor dalam senyawa
Ca5(PO4)3F ? (2 Poin)
d. Apa rumus kimia gipsum?
e. Tuliskan persamaan reaksi pembuatan asam fosfat tersebut! (2 Poin)
f. Jika bahan baku awal adalah 1 kg Ca5(PO4)3F, berapakah produk
asam fosfat yang diperoleh? (Asumsikan keberhasilan sintesis 80 %) (3 Poin)

Soal 5 – Pelapisan Logam dan Pencegahan Korosi (14 Poin)

Teknik yang umum untuk mencegah terjadinya pengkaratan (korosi) logam adalah
dengan cara melapiskan logam lain (elektrodeposisi) di permukaan logam yang rentan
korosi tersebut. Anda mempunyai sebatang pipa logam besi yang berbentuk silinder
dengan panjang 0,260 m dan radius 17 mm. Untuk memperlambat korosi, anda hendak
melapiskan logam Ni dengan tebal 300 µm dipermukaan pipa besi silinder tersebut
(termasuk ujungnya) dengan cara elektrodeposisi (elektrolisis). Sel elektrokimia yang
digunakan untuk proses pelapisan ini mengandung 4,20 kg NiSO 4.7H2O dan 6,80 L air.
Arus yang digunakan untuk proses elektrodeposisi ini adalah 2,1 Ampere.
-3 -3
Diketahui densitas air = 1 g cm ; densitas Ni(s) = 8,9 g cm . Luas lingkaran A = π
2 2
r ; Isi silinder, V = π r h ; π = 3,142857.

Seleksi Olimpiade Sains Nasional Kimia Tingkat Provinsi 2011 Halaman 14


a. Tuliskan persamaan setengah reaksi yang terjadi di elektroda dan
reaksi keseluruhan yang terjadi dalam sel elektrolisis ini. (5 Poin)
b. Hitunglah persentase massa Nikel Sulfat dalam larutan elektrolit
pada saat awal dan akhir reaksi. (6 Poin)
c. Berapa jam waktu yang dibutuhkan untuk melapiskan batang
silinder tersebut. Anggaplah sel tersebut bekerja dengan efisiensi
88 %. (3 Poin)

Soal 6 – Senyawa Alkil Dihalida (24 Poin)

Suatu senyawa alkil dihalida (A) mempunyai rumus molekul C 4H8Cl2. Jika senyawa
(A) dihidrolisa akan terbentuk menjadi senyawa (B). Senyawa (B) dapat bereaksi dengan
hidroksil amin membentuk oksim. Senyawa (B) bila direaksikan dengan I 2 dalam NaOH(aq)
dapat menghasilkan iodoform, tetapi (B) tidak bereaksi jika diberi pereaksi Fehling.

a. Tentukan senyawa isomer dari C4H8Cl2 ! (8 Poin)


b. Tentukan struktur dari senyawa (A) dan (B) ! (6 Poin)
c. Tuliskan persamaan reaksi antara (B) dan hidroksil amin ! (4 Poin)
d. Bagaimana persamaan reaksi antara (B) dan I 2 dalam NaOH(aq) ? (4 Poin)
e. Mengapa (B) tudaj bereaksi dengan Fehling ? (2 Poin)

**SEMOGA BERHASIL**
OLIMPIADE SAINS NASIONAL 2012
SELEKSI TINGKAT PROVINSI

JAWABAN
(DOKUMEN NEGARA)

UjianTeori
Waktu: 180 menit

Kementerian Pendidikan Nasional Dan Kebudayaan


Direktorat Jenderal
Managemen Pendidikan Dasar dan Menengah Direktorat Pembinaan Sekolah Menengah
Atas

2012
Petunjuk :

3. Isilah Biodata anda dengan lengkap (di lembar Jawaban)


Tulis dengan huruf cetak dan jangan disingkat !

4. Soal Teori ini terdiri dari dua bagian:

A. 30 soal pilihan Ganda = 60 poin

jawaban benar = 2 poin


jawaban salah = -1 poin
tidak menjawab = 0 poin

B. 6 Nomor soal essay = 104 poin

TOTAL Poin = 164 poin

3. Waktu yang disediakan: 180 menit

4. Semua jawaban harus ditulis di lembar jawaban yang tersedia

5. Diperkenankan menggunakan kalkulator.

6. Diberikan Tabel periodik Unsur, formula dan tetapan yang diperlukan


7. Anda dapat mulai bekerja bila sudah ada tanda mulai dari pengawas.
8. Anda harus segera berhenti bekerja bila ada tanda berhenti dari Pengawas.
9. Letakkan jawaban anda di meja sebelah kanan dan segera meninggalkan ruangan.
10. Anda dapat membawa pulang soal ujian !!
NAMA :
No Peserta :
Asal Sekolah :
Provinsi :

LEMBAR JAWABAN
Bagian I
Beri Tanda Silang (X) pada Jawaban Yang Anda Pilih

No Jawaban No Jawaban

1 A B C D E 16 A B C D E

2 A B C D E 17 A B C D E

3 A B C D E 18 A B C D E

4 A B C D E 19 A B C D E

5 A B C D E 20 A B C D E

6 A B C D E 21 A B C D E

7 A B C D E 22 A B C D E

8 A B C D E 23 A B C D E

9 A B C D E 24 A B C D E

10 A B C D E 25 A B C D E

11 A B C D E 26 A B C D E

12 A B C D E 27 A B C D E

13 A B C D E 28 A B C D E

14 A B C D E 29 A B C D E

15 A B C D E 30 A B C D E
Tetapan dan rumus berguna

Tetapan (bilangan) Avogadro NA = 6.022∙1023 partikel.mol–1

R= 8,314 J.K-1.mol-1 = 8,314 x107 erg. Mol-1.K-1


Tetapan gas universal, R =1,987 cal.mol-1.K-1 = 0,082054 L.atm.mol-1.K-1
1 Pa= 1 N/m2= 1 kg/(m.s2)
1 atm. =760 mmHg =760 torr
Tekanan gas
= 101325Pa= 1,01325 bar
1 bar =105 Pa
Persamaan gas Ideal PV= nRT
Tekanan Osmosa pada larutan = M RT
Tetapan Kesetimbangan air (Kw) pada 25oC Kw= 1,0x10-14
Konstanta kesetimbangan dan tekanan Kp = Kc(RT)∆n
parsial gas
Hubungan tetapan kesetimbangan dan o ln = ൬ +

Temperatur dan konstanta kesetimbangan

energi Gibbs G = -RT ln K


Energi Gibbs pada temperatur konstan GH T S
Isotherm reaksi kimia G = G + RT∙ln Q
Potensial sel dan energi Gibbs Go=-nFEo
Konstanta Faraday F = 96500 C/mol elektron
Muatan elektron 1,6022 x 10-19 C
Ampere (A) dan Coulomb (C) A =C/det

Reaksi orde pertama: AB

Reaksi orde kedua: AB


Pilih Jawaban yang paling tepat

2 Bila Cu(CN)2 dipanaskan, dihasilkan C2N2(cyanogen) dan CuCN. Berapa massa


Cu(CN)2 dibutuhkan untuk menghasilkan C2N2 sebanyak 5,00 g? [Mr Cu(CN)2 = 115,6;
C2N2 = 52,04]
A. 20,2 g
B. 22,2 g
C. 24,2 g
D. 26,4 g
E. 28,6 g
3 Suatu silinder yang sudah dikosongkan yang isinya 5,00 L diisi dengan gas 25,5 g gas
NH3 dan 36,5 g gas HCl. Hitunglah berapa tekanan pada temoeratur 85 oC sesudaah
kedua gas tersebut bereaksi sempurna sesuai reaksi :
NH3(g) + HCl(g) →NH4Cl(s)
F 2.94 atm
G 5.88 atm
H 8.82 atm
11,76 atm
14.7 atm

3. Anda mempunyai 3 jenis larutan garam dan 1jenis larutan nonelektrolit yaitu:
I. 0.13 m BaCl2
II. 0.22 m MnSO4
III. 0.24 m ZnSO4
IV. 0.41 m Ethylene glycol (nonelektrolit)

Berikut ini, urutan yang benar berdasarkan berkurangnya titik leleh adalah:

1. I > II >III >IV


2. I > IV >II > III
3. IV > III > II > I
4. III > II > IV > I
5. IV > I > III > II

Total mole ion:


a = 3 x 0.13 = 0.39
b = 2 x 0.22 = 0.44
c = 2 x 0.24 = 0.48
d = 1 x 0.41 = 0.41

Semakin besar jumlah mol ion, semakin rendah titik lelehnya: a > d > b > c.

4. Suatu larutan komposisinya adalah campuran 50%/50% berat air (18 g/mol) dan
ethylene glycol (64 g/mol). Ethylene glycol merupakan cairan yang tidak menguap
(nonvolatile). Pada temperatur 25 oC. tekanan uap ai murni adalah 23.8 mm Hg.
Hitunglah berapa tekanan uap larutan.

11.6,68 mmHg
12.11,90 mmHg
13.18,6 mmHg
14.23,8 mmHg
E. 29,74 mmHg
Plarutan = Pair murnix mole fraction H2O

Asumsi 100 g larutan, berarti 50 g ethylene glycol + 50 g air

Fraksi mole H2O = (50/18,02)/[(50/18,02) + (50/64,04)] = 0,781

Plarutan = 23,8 mm Hg x 0,781 = 18,6 mm Hg

5 Pada atom atau ion berikut ini, manakah yang mengandung paling banyak elektron
yang tak-berpasangan:
A. F
B. N
C. S2–
D. Sc3+
E. Ti3+

F = [He]2s22p5 mempunyai 1 elektron tak-berpasangan dalam orbital 2p


N = [He]2s22p3 mempunyai 3 elektron tak-berpasangan dalam orbital 2p
S2– = [Ar] tidak mempunyai elektron tak-berpasangan (semua elektron berpasangan)
Mg2+ = [Ne] tidak mempunyai elektron tak-berpasangan (semua elektron
berpasangan) Sc3+ = [Ar] tidak mempunyai elektron tak-berpasangan (semua elektron
berpasangan) Ti3+ = [Ar]4s1 mempunyai 1 electron tak berpasangan dalam orbital 4s

8. Atom atau ion berikut ini, Ar, K+ dan Ca2+ adalah isoelektronik (mempunyai jumlah
elektron sama). Dari pernyataan berikut ini, manakah urutan kenaikan jari-jari
benar?
Terkecil  terbesar
A. Ar Ca2+ K+
+
B. Ar K Ca+
2+
C. Ca Ar K+
D. Ca2+ K+ Ar
E. K+ Ar Ca2+

7. Diantara molekul berikut ini, manakah yang merupakan contoh molekul dengan ikatan
kovalen 100%?
A. H2O
B. KOH
C. HCl
D. O2
E. Tidak ada molekul yang 100 % ikatan kovalen.

8. Berdasarkan kecenderungan keelektronegatifan masing masing unsur yang berikatan,


berikut ini manakah ikatan yang paling polar?
A. N—O
B. C—N
C. Si—F
D. P—S
E. B—C
9. Apa geometri dari untuk molekul dengan jumlah pasangan elektron pada atom pusat
masing masing 4 pasang, 3 pasang dan 2 pasang elektron?
tetrahedral, trigonal planar, linear
tetrahedral, trigonal piramidal, linear
tetrahedral, trigonal planar, bengkok
pyramidal, trigonal planar, linear.
bukan salah satu jawaban diatas

10. Dari reaksi reaksi ammonia berikut ini, manakah yang tidak melibatkan pasangan
elektron non-ikatan (non-bonding) pada atom nitrogen?
NH3(g) + H2O(l)  NH4+(aq) + OH-(aq)
NH3(g) + CH3I(g)  CH3NH3+I-(s)
NH3(g) + HCl(g)  NH4Cl(s)
2NH3(l) + 2Na(s)  2NaNH2(s) + H2(g)
2NH3(g) + Ag+(aq)  Ag(NH3)2+(aq)

11. Berikut ini, manakah pasangan yang mengandung oksida basa, oksida asam dan
oksida amfoter?
A. Al2O3 SiO2 P4O10
B. MgO Al2O3 P4O10
D. MgO P4O10 SO3
E. Na2O MgO Al2O3
F. Na2O MgO SO3

12. Diantara isomer senyawa hidrokarbon berikut ini, manakah yang lebih mungkin
mempunyai tekanan uap paling rendah?
A (CH3)3CCH2CH3
B (CH3)2CHCH(CH3)2
C (CH3)2CHCH2CH2CH3
D CH3CH2CH(CH3)CH2CH3
E CH3CH2CH2CH2CH2CH3

13. Berikut ini adalah Tabel tingkat oksidasi yang mungkin dari lima unsur blok-d (unsur
dinyatakan sebagai huruf , bukan symbol unsur)
unsur Bilangan oksidasi yang mungkin
P - - 3 - - - -
Q - 2 3 4 - - -
R 1 2 3 4 5 - -
S - - 3 4 5 6 -
T - 2 - 4 5 6 7

Berdasarkan kemungkinan tingkat oksidasi unsur unsur tersebut, berikut ini manakah
ion yang paling mungkin keberadaannya?
1. PO2+
2. QO3-
3. RO42-
4. SO4-
5. TO22+
14. Sebanyak 30 mL larutan 0,10 mol/L Ba(OH)2 ditambahkan ke dalam 30 mL larutan 0,10
mol/L H2SO4. Setelah diampurkan, terjadi kenaikan temperatur sebesar T1. Percobaan
ini kemudian diulang dengan mencampurkan larutan yang sama, tetapi mengunakan
volume masing masing campuran sebanyak 90 mL dari larutan yang sama, dan terjadi
kenaikan temperatur sebesar T2. Berikut ini manakah persamaan yang benar mengenai
kenaikan temperatur kedua campuran tersebut?
T2 = T1
T2 = 3 . T1
T2 = 6 . T1
T2 = 1/3 . T1.
Tidak dapat ditentukan.

15. Perhatikan reaksi berikut ini: 4NH3 + 7O2  4NO2 + 6H2O


Pada saat tertentu, laju awal hilangnya gas oksigen adalah X. Pada saat yang sama,
berapa nilai laju awal munculnya air?
1.2 X
1.1 X
0.86 X
0.58 X
tidak dapat ditentukan dari data yang tak lengkap
C. 0.86 X

16. Penguraian gas Nitryl chloride menghasilkan gas nitrogen dioksida dan gas chlor
sesuai reaksi:

2 NO2Cl(g) 2 NO2(g) + Cl2(g)

Untuk reaksi tersebut, telah diusulkan mekanisme melalui 2 langkah rekasi:

Langkah 1: ?  ? lambat

Langkah 2: NO2Cl(g) + Cl(g) NO2(g) + Cl2(g) sangat cepat


Usulan mengenai mekanisme reaksi pada langkah 1 adalah:
E. 2 NO2Cl(g)2 NO2(g) + Cl2 (g)
F. NO2Cl(g)NO(g) + ClO (g)
G. 2 NO2Cl(g)2 NO2(g) + 2 ClO (g)
H. NO2Cl(g)NO2(g) + Cl(g)
I. Semua usulan mekanisme diatas dapat diterima

17.Perhatikan reaksi: 2 NO(g) + Cl2(g) → 2 NOCl.


Berikut ini, manakah pernyataan yang tepat agar supaya reaksi antara molekul NO dan Cl 2
dapat berhasil?
I . Orientasi yang sesuai (tepat)
II. Perbandingan NO/Cl2 adalah 2 : 1
III Energi tumbukan yang cukup.
1. Hanya I
2. Hanya I dan II
3. Hanya I dan III
4. Hanya II dan III
5. I, II, dan III
18. Pada temperatur 25 oC, berapa konsentrasi [H+] dalam larutan yang mengandung [OH-]
3.99 x 10-5 M?
3.99 x 10-5 M
1.00 x 10-7 M
1.00 x 10-14 M
2.51 x 10-10 M
3.99 x 109 M

19. Berikut ini, manakah anion asamnya (basa konyugasinya) memberikan sifat basa yang
paling kuat?
HSO4-
Cl-
C2H3O2- (ion asetat)
NO3-
semuanya adalah basa konyugasi dari asam kuat dan mempunyai kekuatan
yang sama

20. Kolam renang umum sering diklorinasi untuk membunuh bakteri. Sebagai alternatif untuk
klorinasi, ion perak dapat dipakai dalam konsentrasi dari tidak lebih dari 10 -6 mol L-1 dan
tidak kurang dari 10-7 mol L-1. Semua larutan garam senyawa berikut ini adalah larutan
jenuh. Diantara larutan tersebut, manakah larutan yang dapat mengkasilkan
konsentrasi ion perak yang dibutuhkan?
Senyawa hasil kali kelarutan (Ksp)
A. AgBr 5 x 10-13 mol2 L-2
B. AgCl 2 x 10-10 mol2 L-2
C. AgIO3 2 x 10-8 mol2 L-2
D. Ag2CO3 5 x 10-12 mol2 L-2
E. Ag2SO4 2 x 10-5 mol2 L-2

21. Bahan pembakar padat untuk pendorong roket pesawat luar angkasa terdiri dari
campuran aluminium dan senyawa X. Senyawa X adalah senyawa yang mengandung
chlor dengan tingkat oksidasi +7.
Berikut ini, manakah yang merupakan senyawa X?
1. NCl3
2. NH4Cl
3. NH4ClO3
4. NH4ClO4
5. N2H5Cl

22. Berikut ini diberikan potensial reduksi setengah sel:


Mg2+ + 2e-  Mg, E = -2,38 V (1)
Cu2+ + 2e-  Cu E = 0,34 V (2)
Ag+: + e-  Ag E = 0,80 V (3)
Berdasarkan potensial setengah sel tersebut, berikut ini manakah pernyataan yang
benar?
9. Magnesium adalah reduktor lebih baik
35. Dalam keadaan standar, reaksi akan terjadi secara spontan bila setengah sel
(2) digabung dengan setengah sel hidrogen.
61. Bila logam tembaga ditambahkan ke larutan AgNO3, logam perak akan
mengendap
(IV) Jika 2Ag+ + 2e-  2Ag, maka E = 2(0,80) = 1,60 V

A. I dan II
B. II dan III
C. I, II dan III
D. I, III dan IV
E. I, II dan IV

23 Berikut ini adalah senyawa alkohol. Bila dilakukan test-Iodoform (I 2/NaOH) terhadap
semua senyawa alkohol tersebut, manakah yang memberikan hasil yang positif?
A. (CH3)2COH
B. (C2H5)2CHOH
C. CH3CH2COH
D. CH3CH2OH
E. C6H5CHOH

24 Senyawa propenaldehida dioksidasi dengan KMnO4 pada temperature ruang.


Apa produk reaksi yang terbentuk ?
A CH2 (OH)CH (OH)COOH
B CH2 (OH)CH (OH)COH
C CH2 =C(OH)COOH
D CH2 =CHCOOH .
E CH3CO + CO2

25 Di dalam pelarut air, senyawa senyawa fenol bersifat sebagai asam lemah. Diantara
senyawa fenol berikut ini, manakah yang mempunyai sifat asam paling kuat (paling
asam) ?

A. Fenol
B. m-aminofenol
C. p-metilfenol
D. p-nitrofenol
E. p-aminofenol

26 Dalam senyawa organik berikut ini, gugus fungsi apa saja yang terdapat didalamnya?

A. ester, aldehida, alkena


B. alkena, ester, keton
C. aldehida, asam karboksilat, ester
D. asam karboksilat, keton, alkena
E. alkena, ester, asam karboksilat

(aa) Senyawa formaldehida direaksikan dengan metil magnesium bromida dan kemudian
dihidrolisa dalam suasana asam. Produk yang dihasilkan dari reaksi tersebut adalah:
2-propanol
n-propanol
propanaldehida
aseton
Etanol

bb. Nilon-6 adalah suatu polimer organik yang mengandung unsur C, H dan N yang
mempunyai rumus molekul sebagai berikut:

Dalam suasana asam, senyawa polimer ini akan mengalami hidrolisis. Produk yang
dihasilkan dari hidrolisis polimer nilon-6 ini adalah:

HO(CH2)5COOH
HO(CH2)5OH
HO2C(CH2)4COOH
H3N+(CH2)5COOH
H3N+(CH2)5OH

cc. Suatu senyawa 2-metil butana direaksikan dengan Br 2, h (energi cahaya) pada
temperatur 125oC. Produk utama yang dihasilkan dari reaksi tersebut adalah:
3-metil-2-bromo butana
1,2-dibromo butana
2-metil-2-bromo butana
3-metil-1-bromo butana
2-metil-1-bromo butana

Dari lima senyawa organik berikut ini, tunjukkan mana senyawa yang mempunyai
isomer cis dan trans ?

Jawab: (D)
Bagian II. Essay

Soal 1. Reaaksi unsur dan senyawa non-logam


Senyawa A adalah suatu senyawa hidrida dari unsur non logam yang dalam Tabel
periodik terletak pada golongan 14. Pada temperatur kamar, senyawa A adalah gas
yang mudah terbakar. Senyawa A dapat bereaksi dengan leburan unsur B yang
berwarna kuning yang pada temperatur tinggi menghasilkan senyawa C dan D. Dalam
temperatur kamar, senyawa D berupa gas yang memiliki bau khas yang menyengat,
sedangkan senyawa C dapat bereaksi dengan gas E yang berwarna hijau muda yang
menghasilkan senyawa F dan unsur B. Senyawa F juga dapat dihasilkan dari reaksi
langsung antara A dan E. Semua senyawa di atas mengandung unsur non-logam.
Tentukan rumus senyawa yang sebenarnya dari A sampai F, dan kemudian tuliskan
persamaan reaksi untuk membuktikan senyawa tersebut. (Nilai 24 poin)

Jawab:

Senyawa A = CH4 Unsur B = S senyawa C = CS2


Senyawa D = H2S senyawa E = Cl2 Senyawa F = CCl4
Masing-masing senyawa mendapat 3 poin
Reaksi Kimia CH4 + 4S  CS2 + 2H2S
CS2 + 2Cl2 CCl4 +2S
CH4 + 2Cl2  CCl4+ 2H2
Masing-masing reaksi kimia mendapat 2 poin

Soal 2. Energi Reaksi Kimia (7 poin)

Bungkusan pendingin instant (Instant cold packs) yang digunakan para olahragawan
untuk menghilangkan rasa-sakit akibat cidera, didalamnya mengandung kantong
garam NH4NO3 padat dan kantong air. Bila bungkus tersebut diperas, kantong garam
akan sobek dan garam padat didalamnya akan larut sesuai reaksi berikut:
NH4NO3 (aq) NH4+ (aq) + NO3– (aq)
- Umumnya bungkusan mengandung 50,0 g NH4NO3 dan 125 mL air. Bila temperatur
awal dari air adalah 25,0 C dan temperatur akhir dari campuran larutan adalah –9.0 C,
hitunglah H untuk reaksi dalam kJ/mol NH4NO3 yang dikonsumsi. Anggaplah
pembungkus (pack) tidak mengabsorpsi panas dan larutan mempunyai kapasitas panas
molar 75,30 J/mol C. (3 poin)
q c m
surr ,air m,air air T
q
J 1.00 g 1 mol 4
surr ,air 75.30 125.0 g 9.0 C 25.0 C1.776 10 J

mol C mL 18.015 g
panas hilang oleh sekeliling ve panas diterima oleh reaksi ve

qrxn1.776 104 J1.776 104 J

H 1.77 6 104 J 80.044 g


rxn
50.00 g 1 mol 2.84 4 104 J28.4 kJ/mol

(b) Apakah reaksi tersebut endo- atau eksoterm?(1 poin)


endoterm

(c) Tentukan nilai H f untuk NH4NO3 (aq) dengan menggunakan data berikut ini (3 poin)

NO3–
NH4+ (aq)

ΔH f (kJ/mol) –132.5 –204.7

n Ho n Ho
H rxno f produk f reak tan

H rxno1 mol NH 4H o f ,NH 4


1 mol NO3H o f ,NO3
1 mol NH 4 NO3 H of ,NH
4
NO
3

28.4 kJ1 mol NH 4 132.5 kJ / mol1 mol NO3 204.7 kJ / mol1 mol NH 4 NO3 H of ,NH 4 NO3
H of ,NH 4 NO3 365.6 kJ / mol

Soal 3. Senyawa Metil Bromida dan Kinetika reaksi (21 poin)

Senyawa metil bromida (Bromo metana) berasal dari sumber alam dan manusia. Di
laut, organisme laut/lautan diperkirakan menghasilkan 1-2 milyar kilogram
pertahunnya. Di industri senyawa ini diproduksi untuk pertanian dan industri dengan
mereaksikan metanol dan hidrogen bromida.

e. Tuliskan rekasi antara metanol dan hidrogen bromida yang menghasilkan


metil bromida (2poin)
f. Bagaimana geometri senyawa metil bromida, jelaskan dan gambarkan
geometrinya (3 poin)
g. Bagaimana kepolaran molekul metil bromida, polar atau non-polar?(2 poin)
Didalam air, senyawa metil bromida bereaksi dengan ion hidroksida dan
menghasilkan methanol dan ion bromida. Laju reaksi ini adalah adalah reaksi orde-
1 terhadap metil bromida dan orde-1 terhadap ion hidroksida. Pada konsentrasi
metil bromida 0,005M dan konsentrasi ion hidroksida 0,05M, laju reaksinya =
0,043M/det pada 298K.
d. Tuliskan persamaan reaksi antara metil bromida dengan ion hidroksida
(3 poin)
e. Hitung tetapan laju reaksi metil bromida dengan ion hidroksida (5 poin)
vi. Jika konsentrasi awal metil bromida = ion hidroksida = 0,1M, hitung
konsentrasi metil bromida setelah reaksi berlangsung selama 0,1 detik (6
poin)

Jawab

a. CH3OH + HBr → CH3Br + H2O


b. hibridisasi sp3 (tetrahedral)

c. Polar
d. CH3Br +OH- CH3OH + Br-
e. r = k[CH3Br][OH-], maka k = 0,043Mdet-1/(0,005M*0,05M) = 172 M-1det-1
f. 1/[A]t -1/0,1 = 172*0,1, maka [A]t = 0,037 M

Soal 4. Asam Malat dan sistem Buffer (12 poin)

Asam Malat, C3H5O3COOH, adalah asam lemah yang memberikan rasa masam
dalam setiap minuman yang mengandung buah anggur atau apel. Asam ini juga
banyak digunakan sebagai larutan buffer.

(a) Tentukanlah pH larutan yang dibuat dari 8.20 g natrium malate (berat molekul = 156.07
g/mol) yang ditambahkan kedalam 500.0 mL larutan 0.125 M asam malat (Ka = 4.00
10–4). (7 poin)

C3H5O3COONa (aq)C3H5O3COO– (aq) + Na+ (aq)

1mol 0.05254 mol C3H5O3COONa 0.105 M C3H5O3COO


8.20 g C3H5O3COONa
156.07 g 0.5000 L
C3H5O3COOH (aq) + H2O(ℓ) C3H5O3COO– (aq) + H3O+(aq)

C3H5 O3 COO 4 0.105


pH pK a log log 4.00 10 log 3.32

C3H5 O3 COOH 0.125

b. Hitunglah pH larutan bila kedalam larutan (a) tersebut ditambahkan 45.0 mL larutan
0.500 M HBr. (3 poin)

C3H5O3COO– + H+(aq) C3H5O3COOH (aq)


(aq)
(0.0450 L)(0.500 mol/L) (0.5000 L)(0.125 mol/L) =
Awall (mol) 0.05254 mol = 0.0225 mol 0.0625 mol
perubahan – 0.0225 – 0.0225 + 0.0225
(mol)
Akhir (mol) 0.0300 0 0.0850

C3H5O3COO 4 0.0300 mol


pH pKa log log 4.00 10 log 2.95
C H O COOH 0.0850 mol
3 5 3

c. Apakah sistem buffer yang dibuat pada (a) merupakan buffer yang effektif? Jelaskan
dengan singkat (2 poin)
Karena penambahan sejumlah moderate/kecil asam kuat hanya menghasilkan
perubahan pH yang moderat dalam sistem buffer (perubahan pH sebesar 0.37 unit .
Perubahan pH buffer sebaiknya tidak melebihi 1 unit pH). Buffer ini dianggap merupakan
sistem buffer yang baik.
Karena rasio C3H5O3COOH terhadap C3H5O3COO– awalnya ≈ 1 menjadikan buffer
mempunyai kapasitas buffer yang baik.

Soal 5. Aniline (12 poin)


Aniline, C6H5NH2, bereaksi dengan air dan memberikan sifat basa sesuai
persamaan berikut:
C6H5NH2(aq) + H2O(l) C6H5NH3+(aq) + OH–(aq)
Dalam larutan aqueous aniline 0,180 M, konsentrasi [OH–] = 8,80x10–6.
a Tuliskan pernyataan konstanta ionisasi basa (Kb) untuk reaksi ini. (1 poin)
b Tentukan nilai konstanta ionisasi basa, Kb, untuk C6H5NH2(aq). (2 poin)
c Hitung persen ionisasi C6H5NH2 dalam larutan. (2 poin)
d Tentukan nilai konstanta kesetimbangan untuk reaksi netralisasi :
C6H5NH2(aq) + H3O+(aq) C6H5NH3+(aq) + H2O(l) (3 poin)
e.
a. Tentukan nilai [C6H5NH3+(aq)]/[C6H5NH2(aq)] yang dibutuhkan untuk menghasilkan
larutan pH = 7,75. (2 poin)
2. Hitung volume larutan 0,050M HCl yang harus ditambahkan ke 250,0 mL larutan
0,180 M C6H5NH2(aq) untuk mencapai rasio tersebut. (2 poin)

Jawab:
1. Kb = [C6H5NH3+] [OH–]/ [C6H5NH2]
2. Kb = (8,80x10–6)2/0,180 = 4,3x 10-10
3. % ionisasi = 8,8x10-6/0,180= 4,89x10-4 %
4. C6H5NH2(aq) + H3O+(aq)  C6H5NH3+(aq) + H2O(l)
K = [C6H5NH3+]/[C6H5NH2] [H3O+] = Kb/Kw= 4,3x10-10/10-14 = 4,3x 104
Artinya cenderung/mudah bereaksi dengan asam
e. i. [C6H5NH3+(aq)]/[C6H5NH2(aq)] untuk larutan pH = 7,75.
pOH =14-7,75= 6,25
[OH-] = 10-6,25 =5,623 M
Kb = [C6H5NH3+] [OH–]/ [C6H5NH2]
Kb = [C6H5NH3+] [OH–]/ [C6H5NH2]
4,3 x10-10 =[C6H5NH3+] 5,623x10-7/ [C6H5NH2]

[C6H5NH3+]/[C6H5NH2] = 7,65x10-4

Soal 6. Senyawa Fenol (20 poin)


Fenol, C6H5OH, mengalami reaksi substitusi dengan brom membentuk padatan kristalin
dengan rumus molekul C6HxBryOH. Percobaan berikut dilakukan untuk menentukan x dan y
pada rumus molekul tersebut. Brom dihasilkan dari reaksi antara larutan kalium bromat
dengan kalium bromida dan asam klorida berlebih. Reaksinya adalah sebagai berikut:
BrO3-(aq) + Br-(aq) + H+(aq) →Br2(aq) + H2O(l) (1)
Agar terjadi reaksi substitusi fenol dengan brom, sebanyak 25 mL larutan fenol 0,1 mol/L
ditambahkan ke dalam 30 mL larutan kalium bromat, kemudian ditambahkan kalium
bromida dan asam klorida berlebih yang akan menghasilkan brom sesuai reaksi (1).
Selanjutnya, brom yang dihasilkan dari reaksi (1) langsung bereaksi dengan fenol
menghasilkan padatan C6HxBryOH. Brom yang tersisa direaksikan dengan kalium iodida
berlebih, dan iod yang dihasilkan kemudian ditentukan dengan cara titrasi dengan
larutan natrium tiosulfat. Pada titrasi ini dibutuhkan 30,0 mL larutan natrium tiosulfat
0,100 mol/L untuk mencapai titik akhir titrasi. Reaksinya yang terjadi saat titrasi adalah:
I2(aq) + S2O32-(aq) → I-(aq) + S2O42-(aq) (2)

a. Perhatikan persamaan reaksi (1), jenis reaksi apakah yang terjadi dan jelaskan.(2
poin)
b. Setarakan persamaan reaksi (1) (1)
c. Setarakan persamaan reaksi (2) (1)
d. Berdasarkan percobaan di atas, hitung jumlah mol brom yang bereaksi dengan 1 mol
fenol! (5)
e. Berdasarkan perbandingan molnya, tuliskan persamaan reaksi antara fenol dengan
brom, serta tentukan nilai x dan y pada senyawa yang dihasilkan. (3)
vi. Gambarkan struktur fenol dan padatan kristalin yang terbentuk pada reaksi di atas
serta tuliskan nama IUPAC kedua senyawa tersebut! (3)
g. Gambarkan reaksi yang terjadi antara fenol dengan anhidrida asam asetat serta
tuliskan nama IUPAC produk yang terbentuk! (3)
h. Tuliskan persamaan reaksi dari uji kimia kualitatif yang dapat membedakan sifat
gugus hidroksi (-OH) yang terdapat pada fenol dan etanol. (2)
Jawab:
i. jenis reaksi: redoks, tepatnya disproporsionasi karena terjadi perubahan bilangan oksidasi
dari pereaksi menjadi produk, yaitu: reduksi Br(+5) dalam BrO 3 menjadi Br(0) dalam Br2; dan
oksidasi Br(-1) dalam Br menjadi Br(0) dalam Br 2.
ii. BrO3 (aq) + 5Br (aq) + 6H+(aq) →3Br2(aq) + 3H2O(l)
iii. I2(aq) + 2S2O32 (aq) → I2(aq) + S2O42 (aq)
iv. Mol fenol = 25 mL x 0,1 M = 2,5 mmol. Dari reaksi dan analisis data: mol I 2 = ½ mol S2O32 = ½
x 30 mL x 0,1 M = 1,5 mmol. Mol Br2 = mol I2 = 1,5 mmol; artinya mol Br2 yang tersisa = 1,5
mmol. Jadi, mol Br2 yang bereaksi = 9 mol Br2 untuk 2,5 mol fenol, sehingga 1 mol fenol
bereaksi dengan 3 mol Br2.
v. Persamaan reaksi: C6H5OH + 3Br2→ C6H2Br3OH + 3HBr. Sehingga nilai x = 2 dan y = 3 dalam
C6HxBryOH.

f.

g.

h. C6H5OH(aq) + NaOH(aq) (+indikator fenolftalein) → C6H5ONa(aq) + H2O(l) (warna


merah muda fenolftalein menghilang/memudar), tetapi
C2H5OH(aq) + NaOH(aq) (+indikator fenolftalein)(tidak bereaksi karena etanol adalah
asam yang jauh lebih lemah daripada fenol; warna merah muda fenolftalein tetap merah
muda dan tidak memudar/menghilang)
Soal 7. Identifikasi karbonil

Reaksi reaksi di bawah ini ditujukan untuk identifikasi senyawa karbonil:

a. Tuliskan semua struktur hasil reaksi berikut.


b. Berikan keterangan penggunaan pereaksi yang dipakai, apakah dipergunakan
untuk identifikasi, sintesis atau untuk membedakan diantara senyawa organik.

O
H2 I /NaOH
2 (3)
(i) H3C C C CH3 NHNH2

O 2N NO2 (3)
(ii) C6H5CHO

Tollen's
(iii) CH3CHO atau Fehling (2)
HCN (2)

(iv) C6H5COCH3
Jawab:
I
O O
H2 I2/NaOH H2 - +
(i) HC C C CH H3C C C O Na dan H C I
3 3

Ini merupakan test iodoform, untuk mengidentifikasi adanya:

O HO

C CH3dan C CH3

NHNH2 H
NHN C
O2N NO2 H

(ii) C6H5CHO
O 2N NO 2
2,4-
dinitrofenilhidrazin dipakai untuk mengidentifikasi atau menunjukkan adanya gugus karbonil.
O
Tollen's
(iii)CH3CHO atau Fehling CH3C
-
O
Pereaksi Tolen’s atau Fehling umumnya ditambahkan untuk membedakan antara keton
dan aldehid , hanya senyawa aldehid alifatik yang menghasilkan reaksi positif.

OH

C C N
HCN
(iv)C6H5COCH3 CH3
Reaksi ini dipakai untuk mensintesis senyawa yang mempunyai 2 gugus fungsi didalam satu molekul,
atau untuk membuat senyawa yang bertambah satu atom karbonnya.
PEMBAHASAN SOAL PILIHAN GANDA SELEKSI TIM OSN KIMIA TK PROVINSI
2013 Oleh Urip Kalteng (http://urip.wordpress.com) pada 6 Juni 2013

1. Padatan berikut ini, manakah atom-atom atau molekul-molekulnya yang berinteraksi hanya oleh gaya
van der Waals?
A. CO2
B. SiO2
C. Cu
D. MgO
E. CH3CH2OH
Pembahasan:
Zat yang bersifat non polar hanya memiliki interaksi gaya van der Waals antarmolekulnya. Di
antara spesies pada alternatif jawaban yang bersifat non polar hanyalah CO2 .
2. Dari pernyataan berikut ini, manakah yang menggambarkan fenomena adanya ikatan hidrogen
antar-molekular?
A. Titik leleh dari senyawa hidroksida golongan 1 meningkat dengan bertambahnya massa
molekul relatif (Mr)
B. Titik didih alkana meningkat dengan bertambahnya massa molekul relatif
C. CH3OCH3 (Mr: 46) mempunyai titik didih lebih tinggi dari CH3CH2CH3 (Mr:44)
D. Hidrogen klorida membentuk suatu larutan asam bila dilarutkan dalam air
E. Pada 0 oC , es mempunyai densitas lebih rendah dari air.
Pembahasan:
Fenomena adanya ikatan hidrogen antarmolekul yang paling tepat ditunjukkan pada fenomema H 2O
yang ketika dalam bentuk es pada suhu 0 oC memiliki kerapatan (densitas) yang lebih rendah
dibandingkan densitas air.
3. Jari-jari dan muatan masing-masing dari enam ion ditunjukkan dalam tabel ini:
+ +
Ion J L M2+ X– Y– Z2–
Jari-jari (nm) 0,14 0,18 0,15 0,14 0,18 0,15
Senyawa padatan ionik JX, LY, dan MZ masing-masing mempunyai jenis kisi yang sama. Manakah
urutan yang benar dari penembapatan energi kisinya mulai dari yang tertinggi hingga terendah?
B. JX > LY > MZ
C. JX > MZ > LY
D. LY > MZ > JX
E. MZ > JX > LY
F. MZ > LY > JX
Pembahasan:
Energi kisi secara sederhana dapat diperoleh dari energi potensial elektrostatis antara kation dengan
anion melalui persamaan:
E =nilai E akan semakin besar jika nilai makin besar.

JX LY MZ
Jarak kation-anion (nm) 0,14 + 0,14 = 0,28 0,18 + 0,18 = 0,36 0,15 + 0,15 = 0,30
Muatan kation-anion +1 dan -1 +1 dan -1 +2 dan -2

3,57 2,78 13,3

Soal diketik ulang oleh Urip Kalteng (urip.wordpress.com) dan pembahasan berdasarkan berbagai referensi 1 dari 13
4. Allicin, suatu senyawa organik yang mengandung sulfur yang terdapat dalam bawang putih, adalah suatu
senyawa yang berpotensi sebagai agen anti-bakteri. Sebanyak 5,00 mg allicin dibakar dan menghasilkan
8,13 mg CO2; 3,95 mg SO2; dan 2,76 mg H2O. Massa molar allisin adalah 162 g/mol. Bagaimana formula
molekul dari allicin?
C8H18OS
C7H14O2S
C6H10OS2
C6H10O3S
C5H6O2S2
Pembahasan:
CmHnOxSy + O2CO2 + SO2 + H2O
0,005/162 0,00813/44 0,00395/64 0,00276/18
0,00003 mol 0,00018 mol 0,00006 0,00015
1 6 2 5 ==> hasil pembagian dgn mol terkecil
1 CmHnOxSy + 10 O2  6 CO2 + 2 SO2 + 5 H2O
m = 6 --- n = 10 --- x = 1--- y = 2  C6H10OS2

5. Reaksi berikut ini:


A. 2 KHCO3 + panas  K2CO3 + CO2 + H2O
(ii) 6 CuCl + 16 HNO3  6 Cu(NO3)2 + 4 NO + 3 Cl2 + 8 H2O
3. 2 ClO2 + 2 NaOH  NaClO2 + NaClO3 + H2O
4. 12 MoCl6 + H3PO4 + 36 H2O  H3(MO12PO40) + 72 HCl
Dari keempat reaksi tersebut, manakah reaksi yang bukan reaksi redoks?
A. (i), (ii), (iii), dan (iv)
B. (i), (iii), dan (iv)
(i), (ii), dan (iii)
(ii), (iii), dan (iv)
(i) dan (iv)
Pembahasan:
Pada reaksi (i) dan (iv) tidak terdapat perubahan bilangan oksidasi ini menandakan bukan rekasi
redoks. Pada reaksi (ii):
Bilangan oksidasi Cu berubah dari +1(CuCl) menjadi +2(Cu(NO 3)2), bilangan oksidasi Cl berubah dari
–1(CuCl) menjadi 0(Cl2), bilangan oksidasi N berubah dari +5(HNO 3) menjadi
+2(NO). Pada reaksi (iii):

Bilangan oksidasi Cl berubah dari +4 (ClO 2) menjadi +3 (NaClO2) dan +5 (NaClO3) (reaksi redoks ~
disproporsionasi)

6. Suatu contoh merkuri (II) oksida ditempatkan dalam 5,00 L wadah yang hampa udara. Wadah tersebut
kemudian dipanaskan dan semua merkuri oksida terurai menjadi logam merkuri dan gas oksigen.
o
Sesudah wadah didinginkan hingga 25 C, tekanan gas didalamnya adalah 1,73 atm. Berapa massa
merkuri (II) oksida yang ditempatkan dalam wadah tersebut?
913 g
76,6 g
1,51 g
45,6 g
153 g
Pembahasan:
Jumlah mol O2 (n) yang dihasilkan dapat dihitung dengan hukum gas ideal P.V =
n.R.T n = (P.V):(R.T) = (1,73 × 5) : (0,0821 × (25 + 273)) = 0,354 mol O2
Persamaan reaksi yang terjadi: 2 HgO  2 Hg + O2
Berdasar perbandingan koefisien, mol HgO = 2 × mol O2
mol HgO = 2 × mol O2
mol HgO = 2 × 0,354 mol = 0,708 mol
Massa HgO = 0,708 mol × 216,6 g/mol = 153,3528 ~ 153 g.

Soal diketik ulang oleh Urip Kalteng (urip.wordpress.com) dan pembahasan berdasarkan berbagai referensi 2 dari 13
7. Gangguan atau kerusakan syaraf terjadi dari keracunan perkuri disebabkan oleh karena merkuri dapat
membentuk senyawa kompleks 1:1 dengan gugus liopil, yang merupakan enzim yang penting dalam
metabolisme glukosa. Dalam tubuh manusia, konsentrasi rata-rata gugus liopil dalam cairan tubuh adalah
–8
1,0 × 10 mol/kg cairan tubuh. Bila rata-rata tubuh manusia mengandung 5,0 kg cairan tubuh,
berapakah massa merkuri bila semua liopil dalam tubuh manusia membentuk kompleks
merkuri? Massa atom relatif Hg = 200)
–9
A. 2,5 x 10 g
–8
B. 4,0 x 10 g
–7
C. 1,0 x 10 g
–5
D. 1,0 x 10 g
–4
E. 1,0 x 10 g
Pembahasan:
–8 –8
Liopil dalam cairan tubuh = 1,0 × 10 mol/kg × 5,0 kg = 5 × 10 mol
–8 –8
Hg yang dapat membentuk kompleks dengan liopil = 1/1 × 5 × 10 mol = 5 × 10
–8 –5
mol Massa Hg = 5 × 10 mol x 200 g/mol = 1 × 10 g

8. Perubahan entalpi (∆H) untuk reaksi A + B + C  D + E + F digambarkan pada kurval entalpi di bawah
ini: Kompleks
teraktivas
e
n
D+E+F
t
a
A+B+C
l
p
i

Koordinat reaksi
Berdasar kurva tersebut, terlihat bahwa entalpi D + E + F lebih tinggi dari entalpi A + B + C. Hal ini
menunjukkan bahwa....
4 Reaksi ini adalah eksotermik
5 Reaksi ini adalah endotermik
6 Energi aktivasi dibutuhkan untuk reaksi kebalikan yang lebih tinggi daripada untuk reaksi ke kanan
7 Reaksi ini tidak memerlukan katalis
8 Kompleks teraktifkan untuk reaksi kebalikan adalah spesies yang berbeda dari reaksi ke kanan
Pembahasan:
Reaksi endotermik merupakan reaksi yang menerima kalor sehingga entalpi produk akan lebih tinggi
daripada entalpi reaktan. Reaksi endotermik akan mempunyai energi aktivasi yang lebih tinggi
daripada reaksi eksotermik kebalikannya. Reaksi ke kanan maupun reaksi ke kiri biasanya mempunyai
spesies kompleks teraktifkan yang sama.

9. Berdasarkan data pada 25 oC di bawah ini:


2NOCl(g)  2 NO(g) + Cl2(g) ΔH0 = +75,56 kJ
0
2NO(g) + O2(g)  2NO2(g) ΔH = –113,06 kJ
2 NO2(g)  N2O4(g) ΔH0 = –58,03 kJ
Hitunglah berapa ∆H0 pada 25 oC untuk reaksi:
N2O4 + Cl2(g)  2NOCl(g) + O2(g)
A. +95,52 kJ
B. –85,52 kJ
C. +299 kJ
D. –246,65 kJ
E. – 186,80 kJ

Soal diketik ulang oleh Urip Kalteng (urip.wordpress.com) dan pembahasan berdasarkan berbagai referensi 3 dari 13
Pembahasan:
0
Balik semua persamaan reaksi di atas dan ubah tanda ∆H setiap reaksi tersebut:
0
2 NO(g) + Cl2(g)  2 NOCl(g) ΔH = –75,56 kJ
0
2 NO2 (g)  2 NO(g) + O2(g) ΔH = +113,06 kJ
0
N2O4 (g)  2 NO2(g) ΔH = +58,03 kJ
0
N2O4(g) + Cl2(g)  2NOCl(g) + O2(g) ΔH = +95,52 kJ

10. Ion-ion berikut ini dapat bertindak sebagai asam Bronsted atau basa Bronsted, kecuali:
2-
A. CO3
-
B. HPO42
-
C. HSO3
D. H2O
-
E. HS
Pembahasan:
Secara taktis, ion akan dapat bertindak sebagai asam Bronsted jika masih memiliki H pada ion-nya, jika
tidak memilikinya maka ia tidak akan dapat bertindak sebagai asam Bronsted. Ion akan dapat bertindak
+.
sebagai basa Bronsted jika masih memungkinkan menerima ion H Kecuali CO32- semua dapat

bertindak sebagai asam maupun basa Bronsted.


11. Berikut ini adalah larutan garam 1 molar:
7. Zn(NO3)2
8. KBr
9. Na2HPO4
10. KNO2
Dari pernyataan berikut ini manakah yang benar?
6 Semua larutan garam tersebut bersifat netral
7 Larutan i, iii, dan iv bersifat basa, sedangkan larutan ii bersifat netral
8 Larutan i bersifat asam; larutan ii bersifat netral, larutan iii dan iv bersifat basa
9 Larutan i, dan ii bersifat netral, sedangkan larutan iii dan iv bersifat basa
Larutan i, ii bersifat netral, sedangkan larutan iii, iv dan v bersifat asam
Pembahasan:
Zn(NO3)2 garam ini berasal dari basa lemah (Zn(OH) 2) dan asam kuat (HNO3), maka larutannya
akan bersifat asam
KBr garam ini berasal dari basa kuat (KOH) dan asam kuat (HBr), maka larutannya akan bersifat
netral
Na2HPO4 garam ini berasal dari basa kuat (NaOH) dan asam lemah (H 3PO4), maka larutannya
akan bersifat basa
KNO2 garam ini berasal dari basa kuat (KOH) dan asam lemah (HNO 2), maka larutannya akan
bersifat basa
12. Suatu larutan basa lemah yang konsentrasinya tidak diketahui, dititrasi dengan larutan standar asam kuat, dan
digunakan pH meter untuk mengamati kemajuan titrasi. Berikut ini manakah percobaan yang benar?
pH pada titik ekivalen tergantung pada indikator yang digunakan
pH pada titik ekivalen adalah 7
pada awalnya, grafik pH versus volume asam yang ditambahkan meningkat secara perlahan
dan beraturan, kemudian meningkat lebih cepat
pada awalnya, grafik pH versus volume asam yang ditambahkan menurun secara
perlahan dan beraturan, kemudian menurun lebih cepat
+
pada titik ekivalen [H ] sama dengan konstanta ionisasi basa.

Soal diketik ulang oleh Urip Kalteng (urip.wordpress.com) dan pembahasan berdasarkan berbagai referensi 4 dari 13
Pembahasan:
pH

Volume HCl
13. Bila fenol merah digunakan sebagai indikator dalam titrasi larutan HCl dengan larutan NaOH, indikator
cenderung berubah warna dari kuning menjadi merah pada titik akhir titrasi. Perubahan warna ini
terjadi dengan tiba-tiba karena:
Fenol merah adalah suatu asam kuat yang mampu disosiasi cepat
Larutan yang sedang dititrasi cenderung mengalami perubahan pH yang besar mendekati titik
akhir titrasi
Fenol merah cenderung bereaksi irreversibel dalam larutan basa
Fenol merah adalah asam lemah yang berwarna merah dan basa konyugasinya berwarna kuning
+ –
Fenol merah terlibat dalam langkah penentu laju reaksi antara H3O dan OH
Pembahasan:
Perubahan warna yang terjadi secara tiba-tiba dan cepat ini dikarenakan pada saat itu terjadi perubahan
pH yang besar ketika mendekati titik akhir titrasi, saat itu pula indikator yang berwarna merah berubah
menjadi kuning ketika terbentuk basa onjugasinya.

16 Seiring dengan bertambahnya atom oksigen dalam setiap deret asam yang mengandung oksigen,
seperti HXO, HXO2, HXO3 dan seterusnya, berikut ini manakah aturan umum yang benar?
A. Variasi kekuatan asam tidak dapat diramalkan
B. Kekuatan asam semakin menurun
C. Kekuatan asam meningkat seiring dengan meningkatnya elektronegatifitas X
D. Kekuatan asam menurun seiring dengan menurunnya elektronegatifitas X
E. Kekuatan asam semakin meningkat.
Pembahasan:
Yang dijadikan perbandingan adalah jumlah atom O pada asam tersebut. Semakin banyak jumlah atom O
pada asam okso maka kekuatan asamnya akan semakin meningkat. Karena dengan meningkatnya jumlah
atom O pada asam okso akan menjadikan basa konjugat-nya akan semakin stabil. Dengan stabilnya basa
+
konjugat ini maka asam semakin kuat karena ion H jumlah-nya tetap. Jika tidak stabil tentu saja
akan dapat bereaksi balik kan?!

15. Berikut adalah reaksi yang berlangsung dalam larutan benzena yang mengandung piridin 0,1
M. CH3OH (A) + (C6H5)3CCl (B)  CH3OC(C6H5)2 (C) + HCl

Konsentrasi awal
Konsentrasi
(M) t/min
akhir [C] (M)
[A]o [B]o [C]o
0,10 0,05 0 2,5 0,0033
0,10 0,10 0 15,0 0,0039
0,20 0,10 0 7,5 0,0077

Hukum laju dan nilai tetapan laju untuk reaksi tersebut berturut-turut adalah
10. r= k [A]2[B] dan 4,6 x 10-3 L2mol-2s-1
-3 -1 -1
11. r = k [A][B] dan 3,8 x 10 Lmol s
12.r= k [A][B]2 dan 5,2 x 10-3 L2mol-2s-1
-3 -1
13. r = k [A] dan 2,6 x 10 s
14. r =k [B] dan 7,4 x 10-3 s-1
Soal diketik ulang oleh Urip Kalteng (urip.wordpress.com) dan pembahasan berdasarkan berbagai referensi 5 dari 13
Pembahasan:
Berdasarkan rujukan soal yang ada di internet (ingat banyak soal yang diambil dari referensi lain
sebelumnya, jadi kalau di cari pasti ketemu saja :) ), soal ini terjadi kekeliruan pada data t/min baris
pertama, tertulis 2,5 seharusnya adalah 25. Lagi pula kalau dipaksakan maka tidak ada alternatif
jawaban yang benar.
Oleh karena itu di sini akan digunakan data yang benar itu.

Konsentrasi awal
Konsentrasi Laju reaksi
(M) t/min
akhir [C] (M) (M/detik)
[A]o [B]o [C]o
-6
0,10 0,05 0 25,0 0,0033 2,2 × 10
0,10 0,10 0 15,0 0,0039 4,3 × 10-6
0,20 0,10 0 7,5 0,0077 17,1 × 10-6

Laju reaksi setiap bagian akan dihitung terlebih dahulu dengan rumus
R = ([C]/t) x (1 menit/60 detik)
dan hasilnya terdapat pada kolom ke enam tabel di atas ini.

Orde reaksi terhadap A dapat ditentukan dengan menggunakan data kedua dan ketiga (konsentrasi B
yang sama/tetap), dan sebaliknya.
x -6 -6 x
A. orde terhadap A: (0,2/0,1) = (17,1 × 10 : 4,3 × 10 ) ==> 2 = 4 ===> x = 2
B. orde terhadap B: (0,1/0,05)y = (4,3 × 10-6 : 2,2 × 10-6) ==> 2y = 2
2
===> y = 1 Jadi r = r= k [A] [B]

Menghitung tetapan laju reaksi (k)


Biasanya k bisa ditentukan dengan menggunakan salah satu data hasil percobaan, dalam pembahasan
ini digunakan data kedua.
2 -6 2 -3
k = r / [A] [B] ==> 4,3 × 10 / (0,1) × (0,1) = 4,3 × 10
Jawaban A.

16. Tabel berikut menunjukkan hasil eksperimen yang diperoleh dari reaksi : 2 XO + O 2  2XO2
Tekanan parsial XO (satuan bebas) 100 100 50 50
Tekanan parsial O2 (satuan bebas) 100 25 100 ?
Laju relatif 1,0 0,25 0,50 0,125
Berapakah nilai yang hilang dari tekanan parsial O2 dalam tabel ini?
5. 12,5
6. 25
7. 40
8. 50

9. 75
Pembahasan:
PV=nRT jika R dan T konstan maka P = n/V  konsentrasi

No [XO] [O2] Laju relatif


1 100 100 1,0
2 100 25 0,25
3 50 100 0,50
4 50 ? 0,125

Orde reaksi terhadap XO digunakan data percobaan 1 dan


x x
3: (100/50) =1,0/0,50  2 = 2  x = 1
Orde reaksi terhadap O2 digunakan data percobaan 1 dan
y y
2: (100/25) = 1,0/0,25  4 = 4  y = 1
Persamaan laju reaksi: v = k [XO][O 2],
Soal diketik ulang oleh Urip Kalteng (urip.wordpress.com) dan pembahasan berdasarkan berbagai referensi 6 dari 13

Penentuan k dengan salah satu data: k = v : ([XO][O2])  k = 1,0 : (100)(100)  k = 10
4
Penentuan nilai ? dapat ditentukan dengan menggunakan data ke empat:
–4 –3
[O2] = v : (k.[XO])  [O2] = 0,125 : (10 × 50) = 0,125 : (5 × 10 ) = 0,125 : 0,005 = 25
o
17. Tubuh manusia mempunyai temperatur normal 37 C. Bila nilai tetapan kesetimbangan H2O
o –14 + -
(Kw) pada 37 C adalah 2,7 × 10 maka pada temperatur tersebut konsentrasi [H ] dan [OH ]
dalam larutan netral masing-masing adalah
–7 –7
A. 1,0 × 10 M, 1,0 × 10 M
–7 –7
B. 1,6 × 10 M, 1,6 × 10 M
–7 –7
C. 1,3 × 10 M, 1,8 × 10 M
–7 –7
D. 1,2 × 10 M, 1,6 × 10 M
–7 –7
E. 1,1 × 10 M, 2,4 × 10 M

Pembahasan:
+ -
Kw = [H ] × [OH ]
Karena dalam peruraiannya jumlah H+ dengan OH- sama dimisalkan x maka
2 –14 –7 –7
Kw = x  x = √Kw  x = √(2,7 × 10 ) = 1,6431 × 10 ~ 1,6 × 10

18. Mengenai larutan berikut ini,


–4
15. Ke dalam larutan HNO2 (Ka = 4,5 × 10 ) ditambahkan garam NaNO2 padat
–5
16. Ke dalam larutan NaC2H3O2 (Ka HC2H3O2 = 1,8 × 10 ), dialirkan gas HCl.
Setelah penambahan, apakah pH larutan akan bertambah, berkurang atau tetap sama?
A. (i) berkurang; (ii) bertambah
2 (i) bertambah; (ii) bertambah
3 (i) berkurang; (ii) berkurang
4 (i) bertambah; (ii) berkurang
5 (i) bertambah; (ii) sama

Pembahasan:
A. HNO2 adalah asam lemah jika ditambahkan NaNO 2 (garam berasal dari basa kuat dan konjugat

NO2 ) maka pH semakin basa, artinya pH akan meningkat (bertambah besar)

B. NaC2H3O2 (garam yang berasal dari basa kuat dan konjugat C 2H3O2 ), jika ditambah dengan asam
kuat (HCl) maka akan semakin asam dengan kata lain pH akan turun atau berkurang.

19. Indium sulfida, In2S3 (325,8 g/mol) adalah suatu garam yang sukar larut. Dalam 20 L larutan
jenuhnya, ternyata larutan tersebut mengandung hanya 34 picogram In2S3. Tentukan berapa
–12
Ksp In2S3 (picogram = 10 g)
–75
1,1 × 10
–70
4,2 × 10
–68
3,2 × 10
–60
2,4 × 10
E. 5,2 × 10–56

Pembahasan:
Reaksi kesetimbangan:
In2S3(s) 2 In3+(aq) + 3S2– (aq)
x 2x 3x -----> x = kelarutan molar
3+ 2 2– 3 2 3 5
Ksp = [In ] ×[2S ] = (2x) × (3x) = 108 x
Kelarutan molar (X) = (34 × 10–12 )/325,8 g/mol : 20 L = 5,22 × 10–15M
–15 5
Ksp In2S3 = 108 (5,22 × 10 M) = 4,20 × 10–70

Soal diketik ulang oleh Urip Kalteng (urip.wordpress.com) dan pembahasan berdasarkan berbagai referensi 7 dari 13
20. Bila arus besar dialirkan melalui akua Cu(II)SO4 yang diasamkan, yang serentak dibebaskan, di
katoda x mol Cu dan y L hydrogen (diukur pada STP). Berapa mol elektron dialirkan ke larutan?
x + y/22,4
x + y/11,2
x + y/5,6
2x + y/11,2
2x + y/22,4

Pembahasan:
Mol H2
Mol Cu = x
mol Reaksi :
+
2H + 2e-  H2
2+
Cu + 2e-  Cu
Mol elektron = 2 × mol Cu + 2 × mol H2
= 2x + 2 (
= 2x + (

21 Sejumlah lelehan garam klorida, masing-masing MgCl2 , AlCl3, CaCl2,dan FeCl3


dielektrolisis dengan kuat arus sebesar 3,00 A. Jumlah endapan (deposit) logam yang
manakah akan memerlukan waktu elektrolisis paling lama?
A. 50 g Mg
B. 75 g Al
C. 100 g Ca
D. 125 g Fe
E. Tidak terbentuk endapan logam

Pembahasan:
Elektrolis suatu lelehan garam pada pada elektroda akan menghasilkan deposit logam (dari ion
logamnya) yang biasanya akan menempel di elektroda dan gas klor (dari ion klorida).
Untuk menghitung lama waktu bisa menggunakan rumus sebagai berikut:
W = (e . i . t)/96500 ==> t = (W x 96500) : (e.i)
W = massa deposit logam (g); e = massa ekivalen (g); i = arus listrik (A); t = waktu (detik).

Mg2+ → Mg + 2e- Ar Mg = 24 → massa ekivalen Mg = 24/2 = 12


Al3+ → Al + 3e- Ar Al = 27 → massa ekivalen Al = 27/3 =9
Ca2+ → Ca + 2e- Ar Ca = 40 → massa ekivalen Ca = 40/2 = 20
3+ -
Fe → Fe + 3e Ar Fe = 56 → massa ekivalen Fe = 56/3 = 18,67
2+ -
Fe → Fe + 2e Ar Fe = 56 → massa ekivalen Fe = 56/2 = 28

t Mg = (50 g  96500 C) : (12 g  3,00 A) = 134.027,78 detik


t Al = (75 g  96500 C) : (9 g  3,00 A) = 268.055,56 detik
t Ca = (100 g  96500 C) : (20 g  3,00 A) = 168.833,33 detik
t Fe = (125 g  96500 C) : (18,67 g  3,00 A) = 215.363,33 detik
t Fe = (125 g  96500 C) : (28 g  3,00 A) = 143.601,19 detik

Jadi yang memerlukan waktu paling lama adalah logam Al 75 g.


Jawaban B.

Soal diketik ulang oleh Urip Kalteng (urip.wordpress.com) dan pembahasan berdasarkan berbagai referensi 8 dari 13
22. Diketahui potensial elektroda standar untuk logam X dan Y sebagai berikut.:
2+ 0
X (aq)/X(s) E = +0,30 V
2+ 0
Y (aq)/Y(s) E = –0,40 V
Sel yang ditunjukkan dalam diagram berikut ini:

Manakah penjelasan yang benar mengenai sel tersebut di atas?


0
Elektroda di mana ion positif masuk ke larutan ∆E /V
A. X 0,10
B. X 0,70
C. X 0,75
D. Y 0,10
E. Y 0,70

Pembahasan:
0
Agar reaksi berlangsung spontan maka E harus positif sehingga susunannya adalah:
Katoda ~ Reaksi Reduksi: X2+ (aq) + 2e- X(s) E0 = +0,30 V
Anoda ~ Reaksi Oksidasi: Y  Y2+ + 2e-
0
E = +0,40 V
(s) (aq)
0
E sel = (+0,30 + 0,40) = +0,70 Volt
Elektroda yang melarut (menjadi ion positif akan masuk dalam larutan) tentunya ini terjadi
2+
pada anoda di mana Y melarut atau berubah menjadi ion positif Y .
k1
23. Pada reaksi: A + B C + D Kr
k–1
Apakah pengaruh suatu katalis pada tetapan laju reaksi, k1 untuk reaksi ke kanan, k–1 untuk
reaksi kebalikannya dan pada nilai tetapan kesetimbangan Kr, untuk reaksi kesetimbangan?
k1 k–1 Kr
A. Bertambah Berkurang Tidak ada pengaruh
B. Bertambah Berkurang Bertambah
C. Bertambah Bertambah Tidak ada pengaruh
D. Bertambah Bertambah Bertambah
E. Tidak ada pengaruh Tidak ada pengaruh Bertambah

Pembahasan:
Katalis akan meningkatkan (menambah) laju reaksi ke kanan dan laju reaksi ke kiri, namun
tidak mengubah (tidak mempengaruhi) tetapan kesetimbangan.

Soal diketik ulang oleh Urip Kalteng (urip.wordpress.com) dan pembahasan berdasarkan berbagai referensi 9 dari 13
24. Dari kelima isomer di bawah ini tentukan mana senyawa yang bernama spiropentana

Pembahasan:

25. Senyawa (CH3)2CHCH(CH3)2 jika dibuat lima konformasi sesuai dengan proyeksi Newman
manakah yang merupakan konformasi Anti dan manakah yang konformasi Gauche?

A. 1. Anti 2.Gauche 3.Gauche 4. Anti 5. Gauche


B. 1. Anti 2.Anti 3.Gauche 4. Anti 5. Gauche
C. 1. Gauche 2.Anti 3.Gauche 4. Gauche 5. Anti
D. 1. Gauche 2.Gauche 3.Anti 4. Anti 5. Gauche
E. 1. Anti 2.Gauche 3.Anti 4. Anti 5. Gauche

Pembahasan:
(CH3)2CHCH(CH3)2 = 2,3 dimetilbutana
Proyeksi Newman 1: Hidrogen pada C-2 dan C-3 adalah gauche.
Proyeksi Newman 2: Hidrogen pada C-2 dan C-3 adalah anti.
Proyeksi Newman 3: Hidrogen pada C-2 dan C-3 adalah gauche
Proyeksi Newman 4: Hidrogen pada C-2 dan C-3 adalah gauche
Proyeksi Newman 5: Hidrogen pada C-2 dan C-3 adalah anti
Jawaban yang tepat berturut-turut adalah gauche, anti, gauche, gauche, dan anti; alternatif C

Soal diketik ulang oleh Urip Kalteng (urip.wordpress.com) dan pembahasan berdasarkan berbagai referensi 10 dari 13
o
26. Pada temperatur reaksi 60 C, senyawa 2-metil-butanol-1 akan mengalami reaksi dehidrasi
jika ditambahkan dengan asam sulfat 75% sesuai reaksi:

Produk utama dari hasil reaksi tersebut adalah :

Pembahasan:
Reaksi dehidrasi pada alkohol adalah reaksi pelepasan molekul air (H2O) ketika direaksikan
dengan H2SO4 disertai pemanasan sehingga akan menghasilkan senyawa alkena. Jadi
alternatif jawaban yang tepat adalah A dan atau C.
27. Produk utama apa yang akan dihasilkan jika benzaldehida direaksikan dengan aseton dalam
o
20% NaOH pada 100 C?

Pembahasan:

Alternatif jawaban D yang tepat.

Soal diketik ulang oleh Urip Kalteng (urip.wordpress.com) dan pembahasan berdasarkan berbagai referensi 11 dari 13
28. Pereaksi mana yang akan bereaksi dengan benzaldehida menghasilkan senyawa
semikarbazol-benzaldehida?
NH2-NH2
H2N-NH-CO(NH2)
C6H5NH-NH2
NH2OH
C6H5NH2

Pembahasan:
Aldehid atau keton jika direaksikan dengan semikarabazid (H2N-NH-CO(NH2)) melalui
reaksi kondensasi akan menghasilkan semikarbazol (semicarbazon) sambil melepaskan
molekul air. Benzaldehid adalah salah satu aldehid, jJadi pada soal ini pereaksi yang dapat
digunakan untuk menghasilkan senyawa semikarbazol-benzaldehid adalah pereaksi
semikarbazida (H2N-NH-CO(NH2)) sebagaimana reaksi simpel berikut:

CC. Tentukan pereaksi-pereaksi dari persamaan reaksi berikut:


1. NH2 dan 2. H2SO4

1. NaNO2 / HCl dan 2. NH3 / NH2

1. H2SO4 /HNO3 dan 2. NH3 / NH2

1. (CH3CO)2O dan 2. NH3 / NH2
Semua jawaban di atas tidak benar

Pembahasan:

Cukup jelas atau gak jelas ?

Soal diketik ulang oleh Urip Kalteng (urip.wordpress.com) dan pembahasan berdasarkan berbagai referensi 12 dari 13
30. Reaksi substitusi elektrofilik aromatik berikut ini yang menghasilkan produk yang benar
adalah ...

A. Hanya 1 dan 2
4. 1, 2, dan 3
5. 1, 4, dan 3
6. 2, 3, dan 4
E. Hanya 1 dan 4

Pembahasan:
–NH2 adalah gugus pengarah orto (posisi C-2 dari –NH2) dan para (posisi C-4 dari –
NH2)
–NO2 adalah gugus pengarah meta (posisi C-3dari –NO2)

Hasil (1) benar, –HSO3 meta terhadap –NO2 dan orto terhadap –NH2 ;
Hasil (2) seharusnya posisi –NO2 (hasil substitusi) adalah meta terhadap NO 2 atau orto
terhadap –
NH2
Hasil (3) seharusnya posisi –CH3 (hasil substitusi) adalah meta terhadap NO2 atau orto
terhadap –
NH2
Hasil (4) andai pereaksinya benar maka produk ini

benar Jadi hanya (1) yang benar.
SOAL UJIAN
SELEKSI CALON PESERTA OLIMPIADE SAINS NASIONAL 2015
TINGKAT PROVINSI

Waktu : 180 menit

KEMENTERIAN PENDIDIKAN DAN KEBUDAYAAN


DIREKTORAT JENDERAL PENDIDIKAN MENENGAH
DIREKTORAT PEMBINAAN SEKOLAH MENENGAH ATAS
TAHUN 2015

1
OSP-2015
Petunjuk :

9 Isilah Biodata anda dengan lengkap (di lembar Jawaban)


Tulis dengan huruf cetak dan jangan disingkat!

10 Soal Teori ini terdiri dari dua bagian:


A. 30 soal pilihan Ganda @ 3 poin = 90 poin
jawaban benar = 3 poin
jawaban salah = -1 poin
tidak menjawab = 0 poin
B. 6 soal essay= 110 poin

TOTAL Poin = 200 poin

3. Tidak ada ralat soal

4. Waktu yang disediakan : 180 menit

5. Semua jawaban harus ditulis di lembar jawaban yang tersedia

6. Jawaban soal essay harus dikerjakan dalam kotak yang tersedia (jawaban tidak boleh tersebar)

7. Diberikan Tabel Periodik Unsur, Rumus, dan Tetapan yang diperlukan

8. Diperkenankan menggunakan kalkulator

9. Tidak diperbolehkan membawa Hand Phone (HP) atau peralatan Komunikasi lainnya

10. Anda dapat mulai bekerja bila sudah ada tanda mulai dari Pengawas

11. Anda harus segera berhenti bekerja bila ada tanda berhenti dari Pengawas

12. Letakkan jawaban anda di meja sebelah kanan dan segera meninggalkan ruangan

13. Anda dapat membawa pulang soal ujian!!

2
OSP-2015
3
OSP-2015
4
OSP-2015
5
OSP-2015
1 Pilihan Berganda: pilihlah jawaban yang paling tepat

A. Bila Cu(CN)2 dipanaskan, dihasilkan C 2N2 (sianogen) dan CuCN. Massa Cu(CN) 2
yang dibutuhkan untuk membuat C 2N2 sebanyak 5,00 g adalah
20,2 g
22,2 g
24,2 g
26,4 g
28,6 g

B. Bila persen hasil reaksi:



3NO2(g) + H2O (l) 2HNO3(aq) + NO(g)
Adalah 75,0%, dan dalam reaksi tersebut dikonsumsi sebanyak 45,0 g gas NO 2, maka
massa (dalam satuan gram) asam nitrat, HNO 3(aq) yang dihasilkan adalah
A 22,5 g
B 30,8 g
C 41,1 g
D 54,8 g
E 69,3 g

3. Suatu pil sakit kepala mengandung 200 mg ibuprofen (C 13H18O2) diminum dengan 0,5 L air
oleh siswanya yang perutnya kosong. Bila semua pil tersebut larut, maka konsentrasi larutan
(dalam satuan molal) yang terbentuk dalam perut siswa tersebut adalah
2,3 x 10-3 m
4,1 x 10-3 m
9,7 x 10-4 m
1,9 x 10-3 m
1,7 x 10-2 m

4. Pada tekanan 50 kPa dan 127 oC, sebanyak 100 cm3 gas pada mempunyai massa 0,120
g. Massa molekul relatif gas tersebut adalah
1,2
25
80
120
160

5. Diketahui terdapat larutan zat dalam air sebagai berikut:


KCl, CH3CH2COOH, CH3CH2CH3, CH3CH2CH2OH, dan CH3C(O)CH3
Urutan yang paling tepat untuk kelarutan zat-zat tersebut di dalam air adalah
KCl < CH3CH2COOH < CH3CH2CH3 < CH3CH2CH2OH < CH3C(O)CH3
KCl < CH3CH2CH2OH < CH3CH2CH3 < CH3CH2COOH < CH3C(O)CH3
CH3CH2CH3 < KCl < CH3C(O)CH3 < CH3CH2CH2OH < CH3CH2COOH
CH3CH2COOH < CH3CH2CH2OH < CH3C(O)CH3 < CH3CH2CH3 < KCl
CH3CH2CH3 < CH3C(O)CH3 < CH3CH2CH2OH < CH3CH2COOH < KCl

6
OSP-2015
o
6. Suatu zat padat mempunyai titik leleh yang tajam dan jelas di atas 100 C. Zat padat tersebut
tidak dapat menghantarkan listrik bahkan dalam keadaan lelehan. Zat padat tersebut larut
dalam pelarut hidrokarbon. Struktur yang paling tepat mengenai zat padat tersebut adalah
Kristal atom
Kristal ion
Kristal molekul raksasa
Kristal molekul
Logam

7. Suatu sampel dari senyawa X, bila dipanaskan dengan larutan natrium hidroksida akan
menghasilkan gas A. Bila X dipanaskan dalam asam sulfat pekat, akan dihasilkan gas B. Bila gas
dan B direaksikan, maka akan dihasilkan kembali senyawa X. Berdasarkan informasi tersebut
maka senyawa X adalah
A. CH3CO2C2H5
B. NH2CH2CO2CH3
C. NH4CI
D. NH4I
E. (NH4)2SO4

8. Pernyataan paling tepat yang dapat menjelaskan bahwa endapan magnesium hidroksida
dapat larut dalam larutan NH4Cl(aq), tetapi tidak larut dalam larutan NaCl(aq) adalah
A Dalam air, larutan NH4Cl menghasilkan NH4OH, dan ion OH- yang terbentuk
kemudian memberikan efek ion sejenis
B Ion NH4+ dalam larutan NH4Cl akan menurunkan nilai hasil kali kelarutan Mg(OH) 2
C Larutan garam NH4Cl kurang berdisosiasi sempurna dibandingkan larutan NaCl
D Ion Na+ dan ion Mg2+ adalah isoelektron (mempunyai jumlah elektron sama)
E Ion NH4+ dalam air akan menghasilkan sejumlah H3O+

9. Alanin. H2NCH(CH3)CO2H adalah suatu asam amino dengan nilai Ka = 4,5 x 10 -3 dan nilai Kb
= 7,4 x 10-5. Di dalam air, spesi yang mempunyai konsentrasi paling tinggi pada pH 7 adalah
A H2NCH(CH3)CO2H
B +H3NCH(CH3)CO2H
C H2NCH(CH3)CO2-
D +H3NCH(CH3)CO2-
E Semua jawaban, A, B, C dan D benar

10. Pada molekul berikut ini.

Jumlah atom karbon yang mempunyai hibridisasi sp 2 adalah


15. 0
16. 1
17. 2
18. 3

7
OSP-2015
E. 4

K. Perhatikan reaksi pembentukan glukosa (C 6H12O6) berikut ini:

CO2(g) + 2C2H5OH(l) + energi panas ↔ C6H12O6(aq)


Di antara pernyataan berikut yang paling tepat mengenai persen hasil C 6H12O6 adalah
A. Persen hasil C6H12O6 bertambah besar jika tekanan parsial CO2 diturunkan
B. Persen hasil C6H12O6 naik dua kali lipat jika tekanan parsial CO 2 diduakalikan
C. Persen hasil C6H12O6 bertambah besar jika suhu dinaikkan
D. Persen hasil C6H12O6 berkurang jika suhu diturunkan
E. Persen hasil C6H12O6 berkurang jika bila tekanan total sistem reaksi dinaikkan

12. Di dalam reaksi kimia perubahan senyawa X menjadi senyawa Z, melalui mekanisme
reaksinya ditemukan bahwa tahap reaksinya berlangsung melalui pembentukan senyawa Y,
yang dapat diisolasi. Tahap yang dilalui adalah:
X --> Y , ∆H = positif
Y --> Z , ∆H = negatif
Berdasarkan informasi tersebut, profil reaksi yang sesuai dengan data tersebut adalah

13. Perhatikan reaksi gas pencemar NO2 dan ozon berikut ini:

2NO2(g) + O3(g) N2O5(g) + O2(g)
Reaksi tersebut diamati lajunya dan diperoleh data berikut ini:
Percobaan NO2(g), M O3(g), M Laju awal, Ms-1
1 0,0015 0,0025 4,8 x 10-8
2 0,0022 0,0025 7,2 x 10-8
3 0,0022 0,0050 1,4 x 10-7
Dari percobaan tersebut, penyataan paling tepat mengenai hukum laju reaksi (r) adalah
17. r = k[NO2]2[O3]
18. r = k[NO2][O3]2

8
OSP-2015
D. r = k[NO2][O3]
E. r = k[NO2]
F. r = k[O3]

14. Reaksi berikut ini, 3ClO- (aq) ClO3-(aq) + 2Cl-(aq) telah disusulkan berlangsung melalui
mekanisme berikut ini:

ClO-(aq) + ClO-(aq) ClO2-(aq) + Cl-(aq) (lambat)

ClO2-(aq) -
ClO3-(aq) -
+ ClO (aq) + Cl (aq) (cepat)
Hukum laju yang konsisten dengan mekanisme tersebut adalah:
B. Laju = k[ClO-]2
C. Laju = k[ClO-]
D. Laju = k[ClO-][ClO-]
E. Laju = k[ClO-][Cl-]
F. Hukum laju harus ditentukan secara eksperimen, bukan dari stoikiometri

15. Reaksi kesetimbangan berikut terjadi dalam campuran asam nitrat pekat dan asam sulfat
pekat:
+ - +
HNO3(aq) + 2H2SO4(aq) ↔ NO2 (aq) + 2HSO4 (aq) + H3O (aq)
Pernyataan yang paling tepat mengenai reaksi kesetimbangan ini adalah
Penambahan H2O akan mengurangi konsentrasi NO2+
HNO3 dan NO2+ adalah pasangan asam-basa konjugasi
Asam nitrat bertindak sebagai suatu oksidator
Asam sulfat bertindak sebagai dehidratator
Asam sulfat bertindak sebagai suatu basa

16. Tetapan kesetimbangan reaksi berikut ini masing-masing adalah K 1, K2, dan K3

HNO2(aq) + H2O(l) ↔ NO2-(aq) + H3O+(aq) K1


2 H2O(l) ↔ H3O+(aq) + OH-(aq) K2
NH3(aq) + H2O(l) ↔ NH4+(aq) + OH-(aq) K3
Tetapan kesetimbangan untuk reaksi di bawah ini adalah
HNO2(aq) + NH3(aq) ↔ NO2-(aq) + NH4+(aq)
A. K1 – K2 + K3
B. K1K3
C. K1K3/K2
D. K1K2K3
E. K2/(K1K3)

-
17. Setengah reaksi yang terjadi di anoda pada reaksi setara di bawah ini 3MnO 4
+ 2+ 3+
(aq) + 24H (aq) + 5Fe(s) ↔ 3Mn (aq) + 5Fe (aq) + 12H2O(l)
Adalah

6. 2MnO4-(aq) + 12H+(aq) + 6e- 2Mn2+(aq) + 3H2O(l)
- + -
7. MnO4 (aq) + 8H (aq) + 5e Mn2+(aq) + 4H2O(l)
 3+ -
8. Fe(s) Fe (aq) + 3e
9. 2+ 3+
Fe (aq)Fe (aq) + e
-


10. Fe(s) Fe2+(aq) + 2e-

9
OSP-2015
18. Perhatikan sel volta berikut ini

Cu2+(aq) + 2e- --> Cu(s) o


E = 0,340 V
Potensial sel volta ini, Esel, adalah
+0,0296
-0,0370 V
+0,0592 V
-0,399 V
0V

19. Bentuk geometri, bilangan oksidasi, bilangan koordinasi tembaga, untuk ion
kompleks. [Cu(NH3)4(OH2)2]2+ adalah
Tetrahedral ; +2 ; 6
Square planar ; -2 ; 4
Oktahedral ; +2 ; 6
Linier ; +3 ; 2
Trigonal Planar ; +1 ; 4

20. Mengenai garam kompleks [Co(NH3)5Cl]Cl2, pernyataan yang tidak tepat adalah
Larut dalam air
Dapat menghantarkan listrik
Larutan 1 mol [Co(NH3)5Cl]Cl2 menghasilkan 1 mol kation dan 3 mol anion
Dalam air, kation kompleks yang terbentuk adalah [Co(NH 3)5Cl]+2
Mengandung ligan NH3 DAN Cl-

21. Berikut ini adalah asam-asam karboksilat


I. CHF2CH2CH2CO2H
35. CH3CF2CH2CO2H
III. CH3CH2CF2CO2H
IV. CH3CH2CH2CO2H
Dari keempat asam karboksilat tersebut, urutan yang paling tepat berdasarkan kenaikan
keasaman, mulai dari yang paling asam hingga yang paling lemah asamnya, adalah
6. I > II > III > IV
7. I > IV > III > II
8. III > II > I > IV
9. III > IV > I > II
10. IV > I > II > III

10
OSP-2015
22. Di antara pernyataan mengenai senyawa berikut yang sesuai dengan aturan Huckle adalah

Naftalen Pirol Sikloheptatriena Piridin Stirena

1. Naftalen bukan senyawa monosiklik, oleh karena itu bukan suatu senyawa aromatik
2. Pirol bukan senyawa hidrokarbon, dan bukan termasuk senyawa aromatik
3. Sikloheptatriena bukan senyawa konjugasi sempurna, yang bukan senyawa aromatik
4. Piridin merupakan basa lemah, dan juga bukan senyawa aromatik
5. Stirena mempunyai 8 π elektron, dan juga bukan senyawa aromatik

HBr berlebih
C CH

23. Dalam reaksi adisi berikut ini, yang merupakan produk utama
adalah:
Br

A.
CH3

Br

Br Br
D.
B. C

CH2 CHBr
Br
CHBr2

C. E. CH
CH2

CH2Br

24. Senyawa 2-bromobutana jika direaksikan dengan metanol, seperti pada


persamaan reaksi berikut
H2
C CH3 CH3OH
CH3 CH

Br
Maka produk utama yang dihasilkan adalah
H
2 H
A. C CH3 D. C2 CH2
H3C CH H3C C
H
Br
H
2
C CH3
B. E. C CH3
H3C CH
H3C C
OCH3
H2 OCH3
C CH3
C. H3C C
OCH3

11
OSP-2015
25. Urutan yang paling tepat berdasarkan kenaikan kereaktifan senyawa alkohol di bawah ini
terhadap reaksi dehidrasi dalam suasana asam adalah:
HO CH3
H3C OH
H 3C CH OH
H3C C H3C CH
CH CH3
C C

H
C CH3 2 H2
H
CH3 2

I II III
I < II < III
I < III < II
II < III < I
III < I < II
III < II < I

26. Senyawa yang terbentuk dari reaksi kesetimbangan antara siklopentanon dengan HCN berikut
O

HCN

Adalah
H CN
O H
CN
A. C. E.
CN

OH
NC
CN CN
D.
B.

27. Perhatikan rangkaian reaksi berikut ini:

O O
H
2 1. NaOCH2CH3 etanol KOH, H2O H30+
C C C
H 3C C O CH3 panas panas
H2 2 CH2Cl
Senyawa yang merupakan produk dari rangkaian reaksi di atas adalah :

12
OSP-2015
O O
A. D. H2 O CH3

C
C C CH3
C C O C C C
H2 H2 H2 H2
O H2 O

B. C
C C CH3 E. H
2
H2 C CH3
O C C
H2
O O

C. C CH3

C C
H2 H2

28. Persamaan reaksi berikut ini adalah perubahan dari suatu alkena menjadi alkohol
H2 H2
H2 pereaksi ??
C CH2 C C
3HC C 3HC C OH
H H2
Pereaksi yang dipakai untuk reaksi perubahan tersebut adalah
KOH
BH3/THF kemudian H2O2, NaOH
Hg(O2CCH3)2/ H2O lalu NaBH4
H2O , H2SO4
H2O , OH-

29. Produk dari reaksi Wittig di bawah ini adalah


O

H3C
3(C6H5)P CHCH C H
2 6 5 + H

A. C6H5
D. OH

OH P(C6H5)3
H
CH
B. E. 6 5

H
C.
O
P O (C6H5)3

30. Produk utama dari reaksi di bawah ini adalah


13
OSP-2015
NH2

CuCN
NaNO2 , H2SO4
H2O , 0 celcius

Br
1. p-cyano aniline
2. p-cyano nitro benzene
3. p-bromo cyano benzene
4. 2-nitro-4-bromo cyanobenzene
5. 2-cyano-4-bromo aniline

14
OSP-2015
B. Essay

8 Mineral dan Senyawa Mangan (20 poin)

Pyrolusite adalah suatu mineral mangan dioksida yang berwarna kehitaman atau coklat
yang merupakan sumber utama bijih mangan

Gambar 1 Padatan mineral Pyrolusite


Carl Scheele di tahun 1774 melakukan percobaan dengan menambahkan sejumlah asam sulfat ke
dalam mineral pyrolusite, ternyata dia memperoleh suatu gas A yang berupa suatu unsur. Pada
temperatur ruang, gas A tersebut tidak berwarna, serta tidak mempunyai bau dan rasa. Pada
percobaan berikutnya, ke dalam mineral tersebut dia menambahkan larutan asam klorida, dan
menghasilkan gas B, yang juga berupa unsur berwarna kuning-kehijauan dengan bau yang kuat dan
khas, serta gas tersebut dikenal luas banyak dihasilkan dalam kebanyakan zat pemutih rumah tangga.

1. Tuliskan persamaan reaksi untuk eksperimen Scheele tersebut, yaitu reaksi antara mineral
pyrolusite dengan asam sulfat dan dengan asam klorida, serta tuliskan nama gas A dan gas B
(4 poin)

b. Jelaskan jenis reaksi apakah yang terjadi (3 poin)

Ke dalam mineral MnO2 ditambahkan campuran larutan BaCl2 dan larutan H2SO4
c. Tuliskan reaksi yang terjadi dalam campuran tersebut (3 poin)

Pyrolusite mempunyai struktur tetragonal ( Gambar 2)

Dalam sel unitnya (sel satuannya) , a = b = 4,4 Å dan c = 2,9 Å


d. Hitunglah rapat massa pyrolusite (dalam g/cm 3) (5 poin)

Mangan dioksida yang terkandung dalam mineral pyrolusite dapat digunakan sebagai sumber untuk
pembuatan KMnO4 berdasarkan reaksi:

2MnO2 + 2KOH + O2 2KMnO4 + H2
15
OSP-2015
Untuk membuat KMnO4 ke dalam 1 kg bubuk pyrolusite ditambahkan sejumlah larutan pekat KOH
berlebih dan kemudian ke dalamnya dialirkan gas O2 yang juga berlebih. Setelah reaksi berakhir, dan
kemudian produk KMnO4 diisolasi, ternyata diperoleh sebanyak 1,185 kg KMnO4 murni. Bila dianggap
dalam proses tersebut hanya MnO2 dalam pyrolusite yang bereaksi, maka:
e. Hitunglah persen berat MnO2 dalam pyrolusite (3 poin)

7 Hitunglah berapa liter volume gas H2 yang dapat dihasilkan reaksi tersebut (diketahui volume
gas pada STP = 22,4 L/mol) (2 poin)

2. Medali Emas Hadiah Nobel (14 poin)

Di tahun 1940 ketika Nazi Jerman menginvasi Denmark, ahli kimia Hungaria George de Hevesy,
melarutkan medali emas dari pemegang Hadiah Nobel Max von Laue dan James Franck dalam suatu
larutan asam tertentu untuk mencegah penyitaan oleh penguasa Nazi Jerman selama pendudukan
Denmark. Dia menempatkan larutan kuning kebiruan tersebut di rak laboratoriumnya di Niels Bohr
Institute

Setelah perang selesai, de Hevesy kembali ke laboratorium, dan ternyata larutan tersebut masih
berada di raknya. Untuk mendapatkan kembali, emas yang terdapat dalam larutan asam tersebut
diendapkannya. Emas yang diperoleh kembali tersebut, diserahkan ke Royal Swedish Academy of
Sciences

Nobel Foundation kemudian membuat-ulang medali Nobel dengan menggunakan emas semula yang
asli, dan pada tahun 1952 diserahkan kembali kepada Max von Laue dan James Franck.

a. Jelaskan dalam pelarut apakah George de Hevesy melarutkan medali ini? Tuliskan komposisi
yang tepat untuk pelarut tersebut (3 poin)

b. Tuliskan persamaan reaksi untuk proses pelarutan emas dengan pelarut asam yang digunakan
(4 poin)

Emas sebagai salah satu logam paling mulia melarut hanya dalam pelarut ini karena energi
pelarutannya yang sangat kuat
o 3+
c. Berikan alasan secara kualitatif, mengapa pelarut asam ini yang dipilih. (Diketahui: E Au/Au
= 1,5 V) (3 poin)

Emas dalam larutan kuning kebiruan tersebut diendapkan (direduksi) dengan menggunakan larutan
Na2S2O5 segar sebagai sumber ion SO32-
d. Tuliskan reaksi pengendapan emas tersebut. (4 poin)

(f) Vanadium dan Senyawanya (20 poin)

Logam Vanadium mengkristal dengan sel satuan kubus berpusat badan (bcc).
Diketahui rapat massa vanadium = 6,11 g/cm 3

16
OSP-2015
a. Gambarkan satu sel satuan vanadium kemudian jelaskan posisi atom-atom vanadium dalam
sel satuan tersebut,
dan hitung jumlah atom Vanadium dalam satu sel satuan (6 poin)

b. Hitung jari-jari atom Vanadium (dalam pm) (4 poin)

Natrium vanadat, Na3VO4 adalah salah satu senyawa yang mengandung ion vanadium.
c. Tuliskan konfigurasi elektron spesi vanadium pada senyawa natrium vanadat dan tentukan
bilangan oksidasi spesi vanadium tersebut. (3 poin)

Ion vanadat dapat direduksi dengan penambahan logam Zn dalam suasana basa menjadi ion V 3+

d. Tuliskan persamaan reaksi yang setara untuk proses tersebut (3 poin)

e. Tuliskan orbital atom manakah pada vanadium yang menerima elektron pada reaksi reduksi
tersebut (2 poin)

6. Jika sebanyak 100 mL larutan natrium vanadat 0,2 M direaksikan dengan 6,54 g Zn, tentukan
konsentrasi V3+(aq) dalam larutan setelah reaksi (dalam Molar) (2 poin)

d. Senyawa Pembentuk Tulang dan Gigi (20 poin)


Senyawa hidroksi apatit adalah senyawa pembentuk tulang dan gigi yang memiliki rumus kimia:
Ca5(PO4)3OH. Dalam air, senyawa tersebut terionisasi menjadi 3 jenis ion (1 jenis kation dan 2
jenis anion)

a. Tuliskan persamaan reaksi kesetimbangan dalam proses pelarutan hidroksi apatit dalam air
dan tuliskan rumusan Ksp untuk pelarutan senyawa hidroksi apatit (3 poin)

b. Jika diketahui Ksp untuk hidroksi apatit pada T = 25 oC adalah 6,8 x 10-37, tentukan
solubilitas molar (kelarutan molar) dari senyawa hidroksi apatit tersebut. (4 poin)

- Sebuah sampel gigi dengan massa 0,100 gram (diasumsikan seluruhnya terdiri dari hidroksi
apatit) dimasukkan dalam 1,000 L air murni dan dibiarkan hingga mencapai keadaan
setimbang. Tentukan massa sampel gigi yang tidak larut. (5 poin)

d. Jelaskan mengapa jika mulut kita keasamannya meningkat, akan menyebabkan gigi
berlubang? (3 poin)

Untuk mencegah lubang pada gigi, pasta gigi mengandung senyawa yang dapat melepaskan ion F - dan
membentuk senyawa Ca5(PO4)3F pada gigi.

Reaksi pertukaran F- dengan OH- adalah sebagai berikut:


Ca5(PO4)3OH + F- ↔ Ca5(PO4)3F + OH- K = ?
e. Tentukan nilai tetapan kesetimbangan, K, jika diketahui Ksp untuk Ca5(PO4)3F adalah 2,0 x 10-61
(5 poin)

17
OSP-2015
5. Aspirin dan Minyak Winter (19 poin)

Senyawa A (asam o-hidroksi benzoat, atau asam salisilat) merupakan prekursor (zat
intermediet) untuk membuat obat analgesik (Aspirin) dan minyak winter (untuk salep)

COOH COOH CO2CH3

I II

OCOCH3 OH OCOCH
Aspirin
3

Minyak Winter

a. Tentukan pereaksi dan kondisi pada reaksi I dan II (4 poin)

b. Tentukan struktur produk melalui hasil reaksi jika senyawa A direaksikan dengan:
i.) Larutan natrium karbonat, Na2CO3 (2 poin)
ii.) Larutan NaOH (2 poin)
iii.) Asam nitrat encer (2 poin)

Bila sebuah tablet Aspirin dihaluskan, ditambahkan air (dipanaskan sampai larut), kemudian dititrasi
dengan 0,1 mol L-1 NaOH, maka dibutuhkan sebanyak 13,9 mL larutan alkali untuk menetralkan
aspirin.

c. Hitung berapa gram Aspirin yang ada dalam satu tablet Aspirin (4 poin)
d. Aspirin yang mudah larut biasanya sebagai garam kalsium aspirin
i.) Tuliskan reagen apa yang dipakai untuk mengubah Aspirin menjadi garam kalsium,
dan tuliskan reaksinya. (3 poin)
ii.) Jelaskan mengapa Aspirin kurang larut dalam air (2 poin)

f. Polimer PET (Poli Etilen Tereftalat) dan Turunannya

Senyawa poli etilen tereftalat (PET) banyak diaplikasikan untuk serat sintesis, seperti Dacron, film tipis
seperti Mylar dan sebagai bahan pembuat botol minuman bersoda.

O O
H
2

* O C C O C CH2 *

Poli Etilen Tereftalat (PET)


n

18
OSP-2015
a. PET dibuat dari monomer etilen glikol (1,2-etanadiol) dan monomer lainnya. Gambarkan
struktur etilen glikol dan struktur monomer lainnya tersebut. (4 poin)

b. Tuliskan nama senyawa monomer penyusun PET selain etilen glikol pada soal (a)
di atas (2 poin)

iii. Apabila senyawa monomer PET selain etilen glikol pada jawaban soal (b) di atas direaksikan
dengan 1,2-etanadiamina (etilen diamina), maka akan terbentuk polimer lain. Gambarkan
struktur polimer tersebut. (merujuk pada cara penggambaran PET di atas)! (3 poin)

d. Apabila etilen glikol direaksikan dengan asam 1,4-butanadioat, maka akan terbentuk
polimer lain. Gambarkan struktur polimer tersebut. (merujuk pada cara penggambaran PET
di atas)
(3 poin)

e. Etilen glikol dapat dioksidasi lebih lanjut menjadi senyawa turunan aldehidnya dan
kemudian mengalami oksidasi lebih lanjut menjadi turunan asam karboksilatnya.
Gambarkan struktur aldehid dan asam karboksilat hasil oksidasi total etilen glikol. Tuliskan
nama senyawa asam karboksilat hasil oksidasi total etilen glikol tersebut (5 poin)

SEMOGA BERHASIL
HAK CIPTA
DILINDUNGI UNDANG-UNDANG

SOAL SELEKSI
OLIMPIADE SAINS TINGKAT KABUPATEN/KOTA 2015
CALON TIM OLIMPIADE KIMIA INDONESIA 2016

Bidang Kimia

Waktu : 120 menit

KEMENTERIAN PENDIDIKAN DAN KEBUDAYAAN


DIREKTORAT JENDERAL PENDIDIKAN MENENGAH
DIREKTORAT PEMBINAAN SEKOLAH MENENGAH ATAS
TAHUN 2015
Petunjuk

1 Isilah Biodata anda dengan lengkap di lembar yang tersedia

2 Soal Teori ini terdiri dari dua bagian :

A. 25 soal pilihan Ganda = 50 poin

B. 5 Nomor soal essay = 103 poin

TOTAL = 153 poin

3. Waktu yang disediakan: 120 menit.

4. Semua jawaban harus ditulis di lembar jawaban yang tersedia

5. Diperkenankan menggunakan kalkulator.

6. Diberikan Tabel periodik Unsur.

7. Anda dapat mulai bekerja bila sudah ada tanda mulai dari pengawas.

8. Anda harus segera berhenti bekerja bila ada tanda berhenti dari Pengawas.

9. Letakkan jawaban anda di meja sebelah kanan dan segera meninggalkan ruangan.

10. Anda dapat membawa pulang soal ujian !!

OSK 2015 ii
This file was downloaded from
http://ivanjoannes.wordpress.com
OSK 2015 iv
This file was downloaded from http://ivanjoannes.wordpress.com
Tetapan dan Rumus
Coulomb
(C)

Bilangan
Avogadro

Tetapan gas
universal, R

Tekanan
gas

Volume gas
ideal (S,T,P)

Energi

Persamaan
gas Ideal
Tekanan
Osmosis
pada
larutan
Tetapan
Kesetimban
gan air (Kw)
pada 25oC
Tetapan
kesetimban
gan dan
tekanan
parsial gas

Temperatur
dan
Tetapan
kesetimban
gan

Tetapan
Faraday
Muatan
elektron
Ampere (A)
dan
1 Pa
=1
N/m
2
NA = =1
kg/(
6.022∙1023
m.s2
partikel.mo )
l-1 22,4
18. = liter
0,08205 /mo
L·atm/m l=
ol·K 22,4
8,3145 dm3
L·kPa/ /mo
mol·K l
A. 8,3145 1 kal =
x107 4,182 J
erg/mol· 1J=1
K L·kPa
8,3145
J/mol·K PV = nRT
1,987 =M
kal/mol RT
·K
62,364 Kw=
L·torr/ 1,0x10-14
mol·K Kp = Kc
1 atm = (RT)∆n
101,32 kPa ln ‫∆ܪ‬− = ‫ܭ‬
)+‫ݐݐ‬
1
(

1 atm = F=
760 mmHg 96500
= 760 torr C/mol
= elektron
101325 Pa
= 1,01325 1,6022 x
bar 1 torr = 10-19 C
133,322 Pa
1 bar = 105 A =C/det
Pa

Reaksi orde pertama:



A B

Reaksi orde kedua:



A B
OSK 2015
This file was downloaded from
http://ivanjoannes.wordpress.com
Pilih jawaban yang paling tepat ( 25 soal @ 2 poin)

Hasil analisis senyawa berupa gas menunjukkan kandungan (% massa) 33,0% Si dan
67,0% F. Pada temperatur 35oC, sebanyak 0,210 L senyawa tersebut memberikan
tekanan 1,70 atm. Jika massa 0,210 L senyawa tersebut adalah 2,40 g, maka rumus
molekulnya adalah :
SiF4
SiF3
Si2F8
Si2F6
Si3F9

B. Dari persamaan reaksi berikut ini, yang merupakan reaksi oksidasi-reduksi adalah :
A. 2HCl(aq) + Mg(s) MgCl2(aq) + H2(g)
B. Na2O(s) + H2O(l) 2NaOH(aq)
C. CO2(g) + H2O(l) H2CO3(aq)
D. CaO(s) + SO3(g) CaSO4(s)
E. NH3(g) + HCl(g) NH4Cl(s)

Konsentrasi (dalam satuan molalitas) senyawa para-diklorobenzena (C6H4Cl2) dalam


suatu larutan yang dibuat dengan cara melarutkan 2,65 g C6H4Cl2 dalam 50 mL benzena
(densitas = 0,879 g/mL) adalah :
0,018 m
0,041 m
0,180 m
0,410 m
1,810 m

4. Titanium(IV)oksida secara luas digunakan dalam industri cat sebagai pigmen putih.
Senyawa ini larut dalam asam sulfat pekat panas sesuai persamaan reaksi berikut,
TiO2 (s) + H2SO4 (aq) (TiO)2+ (aq) + SO42- (aq) + H2O (l).
Pernyataan yang tepat untuk menyatakan jenis reaksi tersebut adalah :
Asam-basa
pembentukan ion kompleks
dehidrasi
pertukaran
redoks.

5. Di antara pernyataan berikut yang benar adalah :


A. 18O dan 19F mempunyai jumlah neutron sama.
B. 14C dan 14N adalah isotop karena nomor massanya sama .
C. 18O-2 mempunyai jumlah elektron yang sama dengan 20Ne.
D. A dan B benar.
E. A dan C benar

OSK 2015 1
This file was downloaded from
http://ivanjoannes.wordpress.com
6 Dari serangkaian diagram berikut ini, diagram yang terbaik untuk menjelaskan jari-jari
relatif dari atom dan ion natrium serta atom klor dan ion klorida adalah :

7. Energi ionisasi kedua kalsium adalah 1150 kJ.mol-1. Di antara persamaan reaksi
termokimia berikut yang menggambarkan energi ionisasi kedua kalsium adalah :

A. Ca(g) Ca2+(g) + 2e- ∆H° = + 1150 kJ/mol.
+ 2+ -
B. Ca (g)Ca (g) + e ∆H° = + 1150 kJ/mol.
+ 2+ -
C. Ca (g)Ca (g) + e ∆H° = - 1150 kJ/mol.
 2+ -
D. Ca(s) Ca (g) + 2e ∆H° = + 1150 kJ/mol.
+ 2+ -
E. Ca (s)Ca (g) + e ∆H° = - 1150 kJ/mol
8. Diagram berikut ini menggambarkan suatu percobaan untuk pembuatan dan
pengumpulan gas SO2. Ternyata setelah dilakukan percobaan, percobaan dengan
sistem tersebut gagal (tidak dapat mengumpulkan gas SO2).

Di antara modifikasi berikut yang akan membuat eksperimen ini berhasil untuk
memperoleh gas SO2 adalah :
11. Menghilangkan labu P seluruhnya.
12. Menghilangkan labu Q seluruhnya
13. Memakai asam sulfat encer pengganti asam hidroklorida.
14. Memakai air dalam labu P pengganti akua kalium hidroksida.
15. Pengumpulan dengan cara pemindahan ke atas.

OSK 2015 2
This file was downloaded from
http://ivanjoannes.wordpress.com
9 Anda mempunyai zat berikut ini:
A. Kristal garam NaCl
B. Lelehan garam NaCl
C. Larutan garam NaCl

Yang dapat menghantarkan listrik adalah :


F i dan ii
G i dan iii
H ii dan iii
I i, ii, dan iii
J hanya iii

10. Yang merupakan diagram orbital hibrida dari boron dalam BF3 adalah :

11. Dalam molekul 1-butena, atom karbon yang diberi label 1 mempunyai hibridisasi

5. sp2
6. sp3
7. sp4
8. sp
9. sp3d

OSK 2015 3
This file was downloaded from
http://ivanjoannes.wordpress.com
Di antara pasangan molekul berikut yang mempunyai geometri sama adalah :
AlCl3 dan BCl3
AlCl3 dan PCl3
BF3 dan NH3
BeCl2 dan H2O
CO2 dan SO2.

Di antara molekul atau ion berikut : SO2, CO2, NO2+, ClO2-, molekul atau ion yang
isoelektronik adalah :
SO2, NO2+, dan ClO2-
SO2 dan CO2
SO2 dan NO2+
CO2 dan ClO2-
CO2 dan NO2+

Satu dari struktur resonansi ion OCN- yang digambarkan di bawah ini

Muatan formal untuk setiap atom dalam struktur resonansi di atas adalah :

atom O = -1, atom C atom = -1, dan atom N = +1


atom O = 0, atom C = 0, dan atom N = -1
atom O = -1, C atom = 0, dan atom N = 0
atom O = 0, C atom = 0, dan atom N = 0
atom O = +1, atom C = 0, dan atom N = -2

15 Persamaan reaksi kimia yang menunjukkan hubungan Kp = Kc adalah :


MgCO (s) + 2 HCl (g) ⇌MgCl (s) + CO (g) + H O (l)
3 2 2 2
C (s) + O2 (g) ⇌ CO2 (g)
CH4 (g) + 2O2 (g) ⇌ CO2 (g) + 2H2O (l)
Zn (s) + 2 HCl (aq) ⇌ H2 (g) + ZnCl2 (aq)
2N2 (g) + 5O2 (g) ⇌ 2N2O5 (g)

O. Dalam suatu wadah tertutup yang suhunya 25 oC, sejumlah ammonium karbamat
(N2H6CO2) menyublim dan terdisosiasi menjadi ammoniak (NH3) serta karbondioksida
(CO2) sesuai persamaan reaksi berikut :
N2H6CO2 (s) ⇌ 2 NH3 (g) + CO2 (g).
Setelah didiamkan beberapa lama, terjadi kesetimbangan dengan tekanan total gas
sebesar 0,116 atm. Nilai Kp untuk reaksi tersebut adalah :
8. 4,20 x 10-3
9. 2,99 x 10-3
C.4, 64 x 10-4
D.3,40 x 10-4
E. 2,31 x 10-4

OSK 2015 4
This file was downloaded from
http://ivanjoannes.wordpress.com
17. Larutan garam-garam di bawah ini masing-masing konsentrasinya adalah0,1 M.
Larutan yang memiliki nilai pH paling tinggi adalah :

A. Larutan NH4Cl (Kb NH4OH = 2,0 x 10-5)


B. Larutan (NH4)2SO4 (Kb NH4OH = 2,0 x 10-5)
C. Larutan NaClO (Ka HClO = 3,4 x 10-8)
D. Larutan NaCN (Ka HCN = 4,0 x 10-10)
E. Larutan Na2S (Ka H2S = 1,3 x 10-20)

18. Dalam fotografi, padatan AgBr yang tersisa dilarutkan dalam larutan Na 2S2O3. Ion
Ag+ bereaksi dengan ion S2O32 membentuk senyawa kompleks [Ag(S2O3)2]3 ,
dengan persamaan reaksi sebagai berikut :
AgBr (s) ⇌ Ag+ (aq) + Br (aq) Ksp = 5,4 x 10 13
Ag+ (aq) + 2 S2O32 (aq) ⇌ [Ag(S2O3)2]3 (aq) Kf = 2,0 x 1013
Jumlah padatan AgBr yang dapat larut dalam 125 mL larutan Na2S2O3 1,20 M adalah :
Q.7,14 g
R. 12,22 g
S. 14,08 g
T. 16,72 g
E. 40,65 g

19. Percobaan yang melibatkan reaksi oksidasi NO menjadi NO2 berlangsung sesuai
persamaan reaksi berikut :
2NO(g) + O2(g) ⇌ 2NO2(g)

Data yang diperoleh dari percobaan tersebut adalah sebagai berikut :

Percobaan [O2], M [NO], M Laju NO2 (M/det)


1 0,001 0,001 7,10
2 0,004 0,001 28,40
3 0,004 0,003 255,6
4 0,002 0,002 X

Nilai X dalam tabel adalah :


19. 3,65
20. 14,20
21. 28,40
22. 56,80
23. 85,20

OSK 2015 5
This file was downloaded from
http://ivanjoannes.wordpress.com
20. Di dalam reaksi yang digambarkan oleh persamaan reaksi berikut

ini : C6H6 + Cl2 C6H5Cl + HCl.
Jenis reaksi yang terjadi pada benzena adalah :
3 Addisi elektrofilik
4 Substitusi elektrofilik
5 Substitusi radikal bebas
6 Addisi nukleofilik
E. Substitusi nukleofilik

21. Aldehida dan keton dihasilkan secara industri oleh katalitik oksidasi alkena, seperti
etanal dibuat di industri petrokimia dari etena seperti persamaan reaksi berikut:

H2C=CH2 + ½ O2 ---katalis CH3CHO
Proses ini juga dipakai di industri untuk membuat 2-butena. Yang menggambarkan
struktur senyawa yang dapat dihasilkan 2-butena adalah :
CH3CH2CHO
CH3CH2CH2CHO
CH3COCH2CH3
CH3COCH3
(CH3)2CHCHO

22 Di antara senyawa berikut yang dapat dibuat dari reaksi antara bromoetana dengan kalium
sianida dan kemudian produk yang terbentuk direduksi lebih lanjut, adalah :
CH3CH3
CH3CH2NH2
CH3CH2CH3
CH3CH2CH2NH2
CH3CH2CH2CH2NH2

23 Produk yang akan terbentuk jika propenaldehida dioksidasi dengan KMnO 4 pada suhu
ruang adalah :
A. CH2 (OH)CH (OH)COOH
B. CH2 (OH)CH (OH)COH
C. CH2 =C(OH)COOH
D. CH2 =CHCOOH .
E. CH3CO + CO2

OSK 2015 6
This file was downloaded from
http://ivanjoannes.wordpress.com
27. Produk yang akan dihasilkan jika 3-metil-1-pentena direaksikan dengan Cl 2 dengan adanya
sinar UV adalah :
A 5-kloro-3-metil-1-pentena
B 4-kloro-3-metil-1-pentena
C 3-kloro-3-metil-1-pentena
D 1,2-dikloro-3-metilpentana
E 1-kloro-3-metilpentana

28. Senyawa ester P yang berbau buah-buahan mempunyai struktur seperti berikut:

Senyawa yang dihasilkan jika P dihidrolisis dengan asam hidroklorida adalah


11. CH3COCl dan (CH3)2CHCH2CH2OH
12. CH3CHO dan (CH3)2CHCH2CH2OH
13. CH3CO2H dan (CH3)2CHCH2CHO
14. CH3CO2H dan (CH3)2CHCH2CH2OH
15. CH3CH2CO2H dan (CH3)2CHCH2CHO

OSK 2015 7
This file was downloaded from
http://ivanjoannes.wordpress.com
B. Soal Esai

Soal 1. (23 poin)

Suatu senyawa organik hanya mengandung karbon, hidrogen dan sulfur. Sampel senyawa
organik tersebut dengan massa 1,045 g dibakar dalam oksigen menghasilkan gas CO2, H2O dan
SO2. Gas-gas tersebut dialirkan ke dalam 500 mL larutan KMnO 4 0,0200 M yang diasamkan,
sehingga menyebabkan gas SO2 teroksidasi menjadi spesi SO42-. Hanya sebagian dari KMnO4
tersebut yang tereduksi menjadi Mn2+.

Selanjutnya, sebanyak 50 mL larutan SnCl 2 0,0300 M ditambahkan ke dalam 50 mL larutan


sebelumnya yang masih mengandung KMnO4 yang belum tereduksi. SnCl2 yang digunakan
berlebih sehingga dapat mereduksi semua KMnO4 yang masih tersisa menjadi Mn2+. Kelebihan
Sn2+ yang masih tersisa dalam larutan kemudian dioksidasi menjadi Sn 4+ dengan cara dititrasi
oleh 27,28 mL larutan KMnO4 0,0100 M untuk mencapai titik akhir titrasi.

Kandungan gas CO2 dan H2O yang dihasilkan dari proses pembakaran sampel senyawa organik
tersebut setelah dianalisis adalah 1,660 g CO2 dan 0,4590 g H2O.

a. Tuliskan persamaan reaksi yang setara untuk reaksi oksidasi gas SO2 oleh KMnO4.
(2 poin)
b. Tuliskan persamaan reaksi yang setara untuk reaksi dalam titrasi Sn2+ oleh KMnO4.
(2 poin)
c. Hitung mol Sn2+ yang bereaksi dalam tahap mereduksi semua sisa KMnO4 menjadi Mn2+.
(3 poin)
d. Hitung mol KMnO4 yang bereaksi dalam tahap oksidasi gas SO2 menjadi SO3.
(4 poin)
e. Hitung massa SO2 yang dihasilkan dari reaksi pembakaran sampel senyawa organik
tersebut. (3 poin)
f. Hitung persentase sulfur (%S) dalam sampel senyawa organik tersebut. (3 poin)
g. Tentukan rumus empiris senyawa organik tersebut. (3 poin)
Dalam analisis spektometri massa diketahui sampel senyawa organik tersebut memiliki massa
molekul 247,98. Tentukan rumus molekul senyawa tersebut.

Soal 2. ( 26 poin)

Kalium superoksida, barium peroksida dan titanium dioksida adalah oksida yang memiliki rumus
empiris yang sama, yaitu setiap atom logam bergabung dengan 2 atom oksigen.

a. Tuliskan rumus oksigen yang ada pada titanium dioksida, gambarkan struktur Lewisnya
dan tentukan bilangan oksidasi titan pada senyawa tersebut. (4 poin)

OSK 2015 8
This file was downloaded from
http://ivanjoannes.wordpress.com
b. Tuliskan rumus oksigen yang ada pada barium peroksida, gambarkan struktur Lewisnya
dan jelaskan ikatan yang terbentuk di antara kedua atom oksigen. (5 poin)

3. Tuliskan rumus oksigen yang terdapat pada kalium superoksida, hitung jumlah elektron terluar
yang terdapat pada oksigen dalam kalium superoksida dan gambarkan struktur Lewis
serta resonansinya. (6 poin)

d. Dari ketiga senyawa tersebut, oksida mana yang dapat larut dalam air? Tuliskan
persamaan reaksinya dan jelaskan pH larutan yang dihasilkan. (6 poin)

5. Jika 7,39 kg Titanium dioksida bereaksi dengan karbon berlebih dan gas klorin membentuk
14,24 kg titanium tetraklorida. Tuliskan persamaan reaksinya dan hitung persen rendemen
hasil sintesis tersebut. (5 poin)

Soal 3 (12 poin)

Suatu padatan A dipanaskan dalam wadah tertutup. Pada suhu T, padatan A tersebut terurai
membentuk gas B dan gas C. Penguraian A tersebut mencapai keadaan kesetimbangan pada
saat perbandingan molar B : molar C = 1 : 2 , menurut persamaan :
A(s) → B(g) + 2C(g).

a. (i) Tuliskan persamaan tetapan kesetimbangan, Kp dalam tekanan gas parsial.


(2 poin)
(ii) Jelaskan dengan singkat mengapa persamaan ini tidak melibatkan A. (3 poin)

b. Tekanan kesetimbangan untuk sistem di atas pada suhu T adalah 3x103 kPa.
(i) Hitung tekanan parsial masing-masing komponen. (4 poin)
(ii) Tentukan nilai Kp lengkap dengan satuannya. (3 poin)

Soal 4 (20 poin)

Tersedia 5 buah botol berlabel A, B, C, D dan E masing-masing berisi garam tunggal berupa
padatan berwarna putih. Semua padatan tersebut mengandung unsur-unsur golongan utama
(non-transisi). Semua zat tersebut dapat larut dalam air menghasilkan larutan jernih tidak
berwarna.
Dari percobaan yang dilakukan, diperoleh data berikut ini:
Larutan E dapat mengubah warna lakmus biru menjadi merah, larutan D dapat mengubah
warna lakmus merah menjadi biru.

Larutan A bereaksi dengan larutan E membentuk endapan putih yang sangat halus.

OSK 2015 9
This file was downloaded from
http://ivanjoannes.wordpress.com
Larutan A bereaksi dengan larutan B membentuk endapan berwarna putih yang larut
dalam air panas.

Larutan B bereaksi dengan larutan C membentuk endapan kuning.

Larutan B bereaksi dengan larutan D membentuk endapan berwarna hitam.

Larutan C dicampur dengan larutan D dan larutan E tidak menghasilkan endapan

Padatan E bereaksi dengan basa kuat membentuk gas yang berbau khas dan mengubah
lakmus merah basah menjadi biru.

Test nyala untuk larutan C dan D menunjukkan warna kuning

Dari data tersebut, perkirakan senyawa kimia yang mungkin dari senyawa A, B, C, D
dan E. Buktikan dengan persamaan reaksi selengkapnya. (20 poin)

Soal 5. (22 poin)

Cyanidin (Cy) adalah pigmen organik alamiah yang terdapat dalam buah blackberry, redberry,
anggur, ceri, dan lain lain. Cyanidin mempunyai warna ungu kemerah-merahan, dan dapat
berubah warna seiring dengan perubahan pH. Dalam larutan, pada pH rendah, cyanidin (Cy)
berada sebagai CyH+, yang berwarna merah, sedangkan pada pH tinggi berada sebagai Cy yang
berwarna ungu.

CyH+ ⇌ Cy + H+
Merah ungu

a. Tuliskan persamaan untuk tetapan dissosiasi asam, Ka dari CyH+. (2 poin)


b. Dalam suatu buffer pH = 5,00, rasio dari bentuk merah ke bentuk ungu adalah 1 :
5. Hitunglah nilai Ka. (4 poin)
c. Hitunglah rasio bentuk merah terhadap bentuk ungu dalam buffer jus buah pada pH
= 3,00, dan perkirakanlah warnanya. (5 poin)

Jus buah sering diawetkan dengan penambahan sedikit sulfur dioksida yang mengkibatkan
terjadinya reaksi, dan menghasilkan senyawa tambahan yang tidak berwarna. Untuk
perubahan warna merah menjadi tidak berwarna digambarkan dengan reaksi berikut:

CyH+ + SO2 + H2O ⇌ CySO2H2 + H+ (1)


Merah tak-berwarna

Bila sulfur dioksida secukupnya ditambahkan ke buffer jus buah pada pH = 3,00
menghasilkan konsentrasi kesetimbangan, [SO2], 1,0 x 10-2 M, dan intensitas warna merah
berkurang sepersepuluh dari nilai intensitas awalnya.

OSK 2015 10
This file was downloaded from
http://ivanjoannes.wordpress.com
d. Tuliskan persamaan tetapan kesetimbangan reaksi (1) dan pakailah data yang
tersedia untuk menghitung nilainya. (11 poin)


SEMOGA BERHASIL

OSK 2015 11
This file was downloaded
from
http://ivanjoannes.wordpress.
com
\
Nama :
Nomor Peserta :
Kota/Provinsi :
SMA :

Soal 1. Asal mula dari Atmosfir (18 poin)

Pada awal mula kehidupan dimuka bumi, komposisi atmosfir sangat berbeda dengan keadaan
sekarang ini. Pada saat itu, gas A, metana, ammonia, dan gas gas lain kandungannya sangat
tinggi, sedangkan gas sederhana B keberadaannya hampir tidak ada sama sekali. Karena proses
kimia di dalam organisme, jumlah gas A mulai berkurang sedangkan jumlah gas B jumlahnya
mulai meningkat.

Sekarang ini, keberadaan dan kandungan gas B di atmosfir bumi dihasilkan melalui proses
fotosintesis yaitu nA + nH2O → nB + (CH2O)n. Pada mulanya, B mulai terakumulasi (terkumpul)
di atmosfir dapat menyebabkan kandungan Fe 2+ yang terdapat di dalam air laut teroksidasi
menjadi Fe3+. Lapisan gas di atmosfir yang melindungi bumidari radiasi sinarultra violet (UV)
terbuat dari C, yang merupakan allotropi dai B. Semua perubahan ini membantu meningkatkan
keanekaragaman hayati di permukaan bumi.

Dalam kondisi tertentu, senyawa D menghasilkan radikal, yang dapat menimbulkan penuaan.
Zat D hanya mengandung hindrogen dan oksigen serta mempunyai sifat mereduksi ataupun
mengoksidasi.

a) Tuliskan formula A-D, dan sebutkan nama zat tersebut (4 poin)


b) Tuliskan persamaan reaksi setara dari perubahan berikut ini. (4 poin)
i. nA + nH2O → nB + (CH2O)n
ii. D→B
iii. Fe(OH)2 + B + H2O →
iv. B↔C
c) Berdasarkan sifat reduksi –oksidasi dari D, tuliskan persamaan reaksi keseluruhan,
setengah reaksi reduksi dan setengah reaksi oksidasi, kemudian tunjukkan pada reaksi
mana D sebagai reduktor dan D sebagai oksidator (10 poin)
i. D + KI + H2SO4 →
ii. D + K2Cr2O7 +H2SO4 →

Soal 2. Struktur Molekul (22 poin)


Penentuan struktur molekul bermanfaat untuk mengetahui berbagai sifat fisik maupun kimia
suatu senyawa. Misalnya sifat kepolaran suatu senyawa ditentukan oleh konfigurasi elektron
dan orientasi atom-atom penyusunnya dalam ruang.
Berikut ini adalah contoh senyawa yang menunjukkan hubungan antara struktur molekul
dengan sifat fisik dan kimianya, yaitu : SO 2 , HBF2 , CH2Cl2 , NF3.
a) Tentukan atom-atom pusat (beri tanda garis bawahnya) dari keempat senyawa
tersebut. (2 poin)
b) Tentukan Jumlah total elektron valensi dalam masing masing senyawa tersebut
(2 poin)

1
teori
Nama :
Nomor Peserta :
Kota/Provinsi :
SMA :

c) Gambarkan struktur lewis keempat senyawa tersebut (8 poin)


d) Perkirakan sudut ikatan pada tiap senyawa (2 poin)
e) Tentukan Jenis hibridisasi pada atom pusat dari setiap senyawa (4 poin)
f) Tentukan apakah semua senyawa itu polar, atau nonpolar (2 poin)
g) Sebutkan nama geometri struktur molekul tersebut (2 poin)

Soal 3. Gas NOCl (15 poin)


Nitrosil klorida, NOCl adalah gas berwarna kuning yang sangat beracun dan dapat mengiritasi
paru-paru, mata dan kulit. Gas ini sering digunakan untuk sintesis produk farmasi dan dapat
dihasilkan campuran asam klorida dan asam nitrat pekat. Senyawa NOCl dapat mengurai
menjadi radikal NO dan Cl melalui proses fotosintesis, dan pembentukan radikal ini berbahaya
bagi lingkungan.
a) Tuliskan persamaan reaksi pembentukan nitrosil klorida sesuai wacana di atas
(1 poin)
b) Tuliskan reaksi fotolisis NOCl tersebut (1 poin)
Sejumlah gas NOCl murni dimasukkan ke dalam suatu wadah yang volumenya 1,00 L dan
kemudian dipanaskan hingga 240 oC, sehingga membentuk produk berupa gas NO dan gas klor
hingga tercapai kesetimbangan.
Dalam keadaan setimbang ternyata tekanan totalnya adalah 1,00 atm dan tekanan parsial
NOCl adaah 0,70 atm.
c) Tuliskan reaksi kesetimbangan tersebut (1 poin)
d) Tuliskan pernyataan Kp untuk keseimbangan tersebut (2 poin)
e) Hitunglah tekanan parsial gas NO dan gas klor pada kesetimbangan (4 poin)
o
f) Hitunglah nilai Kp untuk proses tersebut pada 240 C (2 poin)
-1
g) Diketahui entali pembentukan standart, ∆H◦ f(NOCl(g))= 51,7 kJ mol dan ∆H◦f(NO(g))=
90,4 kJ mol-1, tunjukan dengan perhitungan apakah reaksi kesetimbangan di atas
eksoterm atau endoterm. (4 poin)

Soal 4. Garam Natrium dan Larutannya (27 poin)


Garam Natrium adalah termasuk garam yang paling banyak ditemukan dalam mineral di bumi.
Berbagai variasi anion dapat bergabung dengan kation natrium menghasilkan garam dengan
keasaman yang berbeda bila dilarutkan dalam air. Diketahui beberapa garam natrium berikut
NaCN, CH3COONa, NaF, NaCl, dan NaOCl. Jika kosentrasi setiap larutan garam natrium tersebut
dalam air adalah 0,1 M dan diketahui K a HCN = 6,2 x 10-10, Ka CH3COOH=1,8 x 10-5, Ka HF=7,2 x
10-4, Ka HOCl =4,0 x 10-8, maka:
a) Susunlah urutan mulai dari yang paling asam hingga yang paling basa dari larutan 0,1
M garam tersebut. Berikan alasannya berdasarkan teori asam basa Bronsted Lowry
(5 poin)

2
teori
Nama :
Nomor Peserta :
Kota/Provinsi :
SMA :

Suatu garam natrium tak dikenal kemungkinananya adalah salah satu dari kelima garam di
atas. Bila 0,100 mol gara tersebut dilarutkan dalam air hingga volumenya 1,00 L, diperoleh
larutan yang pH-nya 8,07.
b) Tentukan rumus kimia gara tersebut dan buktikan dengan perhitungan. (5 poin) Garam
natrium hipoklorit (NaOCl) adalah bahan aktif dalam cairan pakaian, umumnya konsentrasinya
sebagai bahan aktif adalahh 5,0 %
c) Hitung konsentrasi garam NaOCl dalam pemutih dengan satuan molaritas
(3 poin)
d) Hitunglah pH bila 100 mL larutan pemutih ini dititrasi dengan 50,00 mL larutan
HNO3(aq) 0,67 M (3 poin)
Anda mempunyai senyawa padatan FA1, larutan FA2, larutan FA3 serta larutan FA4. Padatan
FA1 merupakan campuran natrium klorida dan natrium hidrogen karbonat. Larutan FA22
mengandung FA1 sebanyak 20,00 g per liter. Larutan FA3 mengandung asam klorida sebanyak
0,20 ml per liter. Larutan FA4 mengandung natrium hidroksida sebanyak 5,00 g per liter. Pada
suatu eksperimen, sebanyak 25,00 mL larutan FA2 direaksikan dengan 50,00 mL larutan FA3.
Kemudian, kelebihan asam dititrasi dengan larutan FA4 sebanyak 48,00 mL
e) Tuliskan persamaan reaksi larutan FA2 dengan larutan FA3 (1 poin)
f) Tuliskan pula persamaan reaksi larutan FA3 dengan larutan FA4 (1 poin)
g) Hitung konsentrasi FA4, nyatakan dalam M(mol/L) (2 poin)
h) Hitung volume FA3 yang bereaksi dengan FA4 sebanyak 48,00 mL (2 poin)
i) Hitung volume FA3 yang bereaksi dengan FA2 sebanyak 25,00 mL (2 poin)
j) Hitung persen natrium karbonat dalam FA1 (3 poin)

Soal 5. Hidrolisis Sukrosa dalam larutan asam (17 poin)


Di dalam larutan asam klorida, skurosa (C12H22O11) bereaksi membentuk gula sederhana
sesuai reaksi:

sukrosa→ glukosa + fruktosa

pada 23◦C, sukrosa dengan konsentrasi awal 0,316 M dalam larutan HCL 0,5 M diamati laju
hilangnya sukrosa dan dikumpulkan data yang kemudian di plot dan diperoleh grafik seperti
gambar 1. Selanjurnya data ini ditransformasi dan diperoleh grafik seperti gambar 2.

3
teori
Nama :
Nomor Peserta :
Kota/Provinsi :
SMA :

Sukrosa (M)

Waktu (min)

Gambar 1.

Waktu (min)

ln [Sukrosa]

Gambar 2.

4
teori
Nama :
Nomor Peserta :
Kota/Provinsi :
SMA :

Berdasarkan grafik yang diperoleh jawablah pertanyan berikut ini.


a) Bila reaksi terhadap HCL adalah oreder nol, tentukan hukum laju reaksinya.
(2poin)
b) Berapa nilai konstanta laju reaksinya (k)? (3 poin)
c) Hitunglah waktu paruhnya (3 poin)
d) Berapa konsentrasi sukrosa yang tersisa setelah 255 menit? (3 poin)
e) Bila konsentrasi HCL yang digunakan adalah 1M, bagaimana laju reaksinya
(2 poin)
Pada pengukuranlaju reaksi pada temperatur yang berbeda, ternyata pada 35◦C laju reaksinya
4,1 lebih cepat dibandingkan pada 25◦C.
f) Hitunglah berapa energi aktivasi reaksi hidrolisis tersebut (4 poin)

Soal 6. Sel Edison (21 poin)


Sel sekunder adalah suatu baterai yang dapat diidi ulang. Baterai timbal dalam asam sulfat
yang sering dipakai di kendaraan bermotor adalah salah satu contoh dari sel sekunder, mirip
dengan baterai timbal, baterai yang menggunakan besi dan nikel dalam suasana basa
dinamakan sel edison ketika bekerja, di anoda terjadi oksidasi besi menjadi FeO dan di katoda
terjadi reduksi Ni2O3(s) menjadi NiO(s).

Wadah porselin Tutup porselin

Pelat besi

Basa Pelat nikel oksida

5
teori
Nama :
Nomor Peserta :
Kota/Provinsi :
SMA :

a) Tuliskan setengah reaksi di anoda dan katoda, serta reaksi sel-nya (3 poin)
3
b) Bila suatu sel edison berukuran x cm , memberika potensial 1,2 V dan energi sebesar
115,8 kJ. Bila ukuran sel dilipatkan dua (masa katoda da anodanya dilipatkan 2), maka:
(2 poin)
(i) Potensial selnya akan:
Menjadi dua kali
Menjadi setengahnya
tetap

(ii) Energi yang dihasilkan sel:


Menjadi dua kali
Menjadi setenganya
tetap

3. Energi yang dihasilkan oleh suatu baterai dapat merupakan hasil kali jumlah mol
elektron yang diserah terimakan, muatan 1 mol elektron dan potensial baterai.
(i) Tentukan muatan satu mol elektron dalam Coulomb (C) (2 poin)
3. Tentukan jumlah mol elektron yang diserah terimakan dalam sel berukuran x
cm3 seperti dinyatakan pada soal di atas (3 poin)
(iii) Bila baterai ditarik sebanyak 0,05 C/detik, berapa lama waktu yang dibutuhkan
untuk mereduksi 0,5 mol NI2O3(s) (3 poin)
d) Tetapan keseimbangan reaksi yang berlangsung dalam sel dapat dihitung dengan
persamaan K = e (nFE◦/RT). Hitung tetapan kesetimbangan reaksi yang berlangsung dalam
sel Edison, pada suhu ruang (298 K) (4 poin)
e) Tuliskan reaksi yang berlangsung saat sel Edison diisi ulang (2 poin)
F. Berapa potensial minimal yang harus diberikan pada sel saat pengisian ulang?
(2 poin)

Soal 7. Asam Benzoat (18 poin)


Asam benzoat C6H5COOH, diapakai sebagai pengawet dalam makanan dan beberapa produk
buah-buahan dalam kaleng. Asam benzoat tidak terlalu berbahaya bagi manusia jika
dikonsumsi dalam jumlah sedikit, sebab dapat disekresi melalui urin sebagai asam hipurat (D).

(D)
Asam Hipurat

6
teori
Nama :
Nomor Peserta :
Kota/Provinsi :
SMA :

a) Sebutkan gugus fungsi apa saja dalam senyawa (D). (1 poin)


b) Tentukan dua rumus senyawa yang dapat bereaksi dalam laboratorium untuk
menghasilkan (D). (2 poin)
c) Sebutkan peraksi dan kondisi yang dibuthkan untuk membuat asam benzoat dari
senyawa (D). (2 poin)
d) Jika senyawa (d) direaksikan dengan gas klor dengan kaatalis FeCl3, gambarkan
struktur produk yang terbentuk. (3 poin)
Metilbenzen jauh kurang bersifat racun dibandingkan dengan benzena, sebab tubuh dapat
merubah metilbensena menjadi asam benzoat.
e) Termasuk jenis reaksi apa perubahan dari metilbenzena menjadi asam benzoat dan
tuliskan pereaksi untuk menghasilkan asam benzoat tersebut. (2 poin)
f) Sarankan skema pembuatan asam benzoat di laboratorium. (4 poin)
g) Berapa berat maksimum asam hipurat, jika 1,0 g metilbenzen tertelan ke dalam
tubuh? (4 poin)

Soal 8. Tripeptida (22 poin)


Tripeptida adalah suatu senyawa yang tersusun dari tiga jenis asam amino. Biasanya tripeptida
dan oligomer peptida rantai pendek tergantung dalam mahluk hidup famili Mollusca yang
fungsinya untuk penyangga bagian vital tubuh mahluk tersebut. Pada reaksi hidrolisis suatu
tripeptida dihasilkan asam amino dengan jumlah yang sama sebagai berikut:

B C D

a) Lingkari dan beri nama gugus-gugus fungsi yang terdapat pada ketiga asam amino
tersebut! (3 poin)
b) Ada beberapa kemungkinan ketiga asam amino tersebut berikatan menjadi suatu
tripeptida? (1 poin)
c) Gambarkan salah satu struktur molekul dari tripeptida tersebut. (2 poin)

7
teori
Nama :
Nomor Peserta :
Kota/Provinsi :
SMA :

d) Tunjukkan apa yang terjadi jika tripeptida tersebut


direaksikan dengan pereaksi di bawah ini tetapi
tidak terjadi hidrolisis (6 poin)
A. Ditambahkan dengan NaOH
B. Ditambahkan dengan HCl
5. Tunjukkan manakah di antara ketiga asam amino
tersebut yang paling bersifat basa
(1 poin)
6. Jika asam amino C direaksi dengan asam nitrat
pekat dalam suasana asam sulfat pekat,
gambarkan produk yang terbentuk. (3 poin)
g) Jika asam amino B dioksidasi dengan suatu
oksidator kuat, gambarkan produk yang terbentuk.
(3 poin)
8. Jika asam amino D bereaksi dengan tionil klorida
( SOCl2), kemudian produk yang terbentuk
direaksikan dengan etil amina, gambarkan produk
yang terbentuk
(3 poin)
OLIMPIADE SAINS NASIONAL 2010

Medan, 1-7 Agustus 2010

BIDANG KIMIA

UjianTeori
Waktu 210 menit
Kementerian Pendidikan Nasional
Direktorat Jenderal
Manajemen Pendidikan Dasar dan Menengah
Direktorat Pembinaan Sekolah Menengah Atas

2010

OSN-2010 Pag
Petunjuk :

a) Isilah Biodata anda dengan lengkap (di lembar Jawaban)


Tulis dengan huruf cetak dan jangan disingkat !

ii Soal Teori ini terdiri 8 Nomor soal essay

TOTAL Poin = - poin

iii. Waktu yang disediakan: 210 menit

iv. Semua jawaban harus ditulis di lembar jawaban yang tersedia

v. Diperkenankan menggunakan kalkulator.

vi. Diberikan Tabel periodik Unsur.


vii. Anda dapat mulai bekerja bila sudah ada tanda mulai dari pengawas.
viii.Anda harus segera berhenti bekerja bila ada tanda berhenti dari Pengawas.
ix. Letakkan jawaban anda di meja sebelah kanan dan segera meninggalkan
ruangan.
x. Anda dapat membawa pulang soal ujian !!

OSN-2010 Page 2
OSN-2010 Page 3
Tetapan dan rumus berguna
23 –1
Tetapan (bilangan) Avogadro NA = 6.022∙10 partikel.mol
-1 -1 7 -1 -1
R= 8,314 J.K .mol = 8,314 x10 erg. Mol .K
-1 -1 -1 -1
Tetapan gas universal, R =1,987 cal.mol .K = 0,082054 L.atm.mol .K
2 2
1 Pa= 1 N/m = 1 kg/(m.s )
1 atm. =760 mmHg =760 torr
Tekanan gas
= 101325Pa= 1,01325 bar
5
1 bar =10 Pa
Persamaan gas Ideal PV= nRT
Hubungan antara tetapan
o
kesetimbangan dan energi G = -RTlnK
Gibbs
Energi Gibbs pada temperatur
GH T S
konstan
Isotherm reaksi kimia G = G + RT∙ln Q
Temperatur dan konstanta
kesetimbangan
Tekanan Osmosa pada
p =c RT
larutan

o 0,0592 logQ
Persamaan Nernst pada 298K, E E
n
Faraday 1 F = 96450 C/mol e-
Muatan elektron 1,6022 x 10-19 C

Reaksi orde pertama: AB

Reaksi orde kedua: AB


OSN-2010 Page 4
Soal 1. Pemanasan garam magnesium oksalat (13 poin)
Sebanyak 1,0 g magnesium oksalat padat (MgC 2O4.nH2O , Kode A )
dipanaskan dengan penaikan temperatur secara teratur. Selama proses
pemanasan zat tersebut, dialirkan gas nitrogen kering (inert) untuk menjaga
atmosfir pemanasan. Proses perubahan berat selama pemanasan diamati,
dan berat A berkurang. Selama proses pemanasan tersebut, perubahan
berat padatan diamati, dan sampel A berubah menjadi B dan C, seperti
diperlihatkan pada grafik. Telah diketahui bahwa selama pemanasan antara
o
100 dan 250 C, H2O didalam sampel kristal garam akan dibebaskan. Pada
pemanasan dari BC dibebaskan sejumlah gas.

Berdasarkan grafik perubahan berat dan temperatur yang diberikan, (unjukkan


perhitungannya):
a. Tuliskanlah reaksi perubahan dari A  B (sebutkan fasanya), dan hitunglah
jumlah air, n, dalam garam MgC2O4.nH2O (4 poin)
b. Tuliskan reaksi pada pemanasan B C (sebutkan fasanya) (5 poin)
c. Tuliskan formula Lewis dari gas gas yang dibebaskan pada proses
pemanasan BC . (4poin)

Jawab:
a. AB:MgC2O4. nH2O(s)  MgC2O4(s) + n H2O(g) (1poin)
Dari grafik:

OSN-2010 Page 5
a)= MgC2O4. nH2O(s) = 1.00 g
b)= MgC2O4(s) =0,76 g = (0,76/112) mol
H2O = 1.00-0,76 = 0,24 g = (0,24/18) mol (1 poin)

n.(0,76/112) = (0,24/18)
n (19/2800) = 1/75
n= 112/57 ≈ 2 (2 poin)

b. BC: MgC2O4(s)  X + gas gas

X= 0,27 g
MgC2O4 = (0,76/112)
X = 0,27/x
(0,76/112) = 0,27/x
x =39,79 (2 poin)

massa molar X =39,79 ≈ 40 g/mol


Mg =24
X = 40 =24 (Mg)+ 16 (O)
Maka kemungkinan X adalah MgO (1 poin)
Sehingga: BC: MgC2O4(s)  MgO(s) + CO2(g) + CO(g) (2poin)

c) Formula Lewis: CO2


CO:

Soal 2. Analisis campuran garam nitrat (12 poin)

Suatu sampel serbuk bermassa 0,3657 g mengandung barium nitrat (Ba(NO 3)2)
dan kalsium nitrat (Ca(NO3)2). Sampel ini dilarutkan dalam 50 mL air. Kemudian
pada larutan yang dihasilkan ditambahkan amonia untuk meningkatkan pH, setelah
itu natrium oksalat (Na2C2O4) dalam jumlah berlebih ditambahkan untuk
mengendapkan ion logam-logamnya. Endapan yang dihasilkan kemudian disaring,
dicuci dan ditransfer ke gelas kimia yang mengandung 50,00 mL air lalu diasamkan
untuk melarutkan endapannya. Larutan yang dihasilkan dititrasi dengan larutan
kalium permanganat (KMnO4) 0,0500 mol L-1. Diperlukan 13,94 mL larutan
permanganat untuk mencapai titik akhir.

a. Bila jumlah ion barium dan kalsium (dalam mmol) berturut-turut adalah x dan y,
nyatakan hubungan massa sampel sebagai fungsi x dan y. (3 poin)

Massa Sample= mSample = 0,3657 g = 365, 7 mg


Massa Ba(NO3)2 = mBa = x(261) = 261 x
Massa Ca(NO3)2 = mCa = y(164) = 164 y
mSample= mBa + mCa = (261) x + (164) y = 365,7 mg (3 poin)
OSN-2010
Page 6
b. Tentukan jumlah mmol ion oksalat yang diperlukan untuk mengendapkan
sempurna kedua ion tersebut dalam x dan y. (2 poin)

Ba2+ (aq) + C2O4= (aq)  BaC2O4 (s)


x mmol x mmol x mmol

Ca2+ (aq) + C2O4= (aq) CaC2O4 (s)


y mmol y mmol y mmol

mmol ion C2O4= = (x + y) mmol (2 poin)

c. Tuliskan reaksi setara titrasi C2O42 dengan MnO4 bila diketahui hasil reaksinya
adalah Mn2+ dan CO2. (3 poin)

(C2O4=  2 CO2 + 2e) x5 (1 poin)


(MnO4- +
+ 8 H + 5e  Mn 2+
+ 4H2O) x2 (1 poin)
5 C2O4= + 2 MnO4- + 16 H+ 2Mn2+ + 10 CO2 + 4H2O (1 poin)

d. Tentukan % massa barium nitrat dalam sampel. (4 poin)

MnO4- = 0,0500 mol L-1 x13,94 mL = 0,697


mmol C2O4= = 5/2 x0,697 mmol =1,7425 mmol
mol C2O4= = x + y = 1,7425
mmol x= 1,7425-y
x + (164) y =365,7
aaaaaaaaaaa)(1,7425-y) +(164) y = 365,7
454,7925 -261y + 164 y = 365,7
97y = 89,0925 (2 poin)
Ca(NO3)2 = y = 89,095/97 = 0,9185 mmol
Ca(NO3)2 = 0,9185 x 164 mg =150,634 mg (1 poin)

Ba(NO3)2 = x = 1,7425-0,9185 = 0,8240 mmol

Ba(NO3)2 = 365,7-150,634 = 215,066 mg


Atau:
Ba(NO3)2 =0,8240 x261 = 215,064 mg (1 poin)

Soal 3. Titrasi asam lemah-basa kuat (14 poin)

Berikut ini adalah dua grafik kurva titrasi dari asam lemah monoprotik HA yang dititrasi
dengan basa kuat.

OSN-2010 Page 7
Dengan menggunakan grafik kurva titrasi tersebut, tentukan :)

Tentukanlah nilai berikut ini:


a.Tuliskan ionisasi larutan aqua asam lemah tersebut (1 poin)
b. pH awal larutan asam lemah (1 poin)
c. nilai Ka asam lemah monoprotik (HA). (6 poin)
d. (molarita asam lemah monoprotik (6 poin)
+ -
a. HA(aq)  H (aq) + A (aq) (1 poin)
b. Grafik: pH awal =3 (1 poin)

(iii) HA (aq) + OH-(aq) A-(aq) + H2O(l)


Selama titrasi, sebelum TE , sistem membentuk buffer. (1 poin)
Dari grafik: TE = 15 mL (1 poin)
Mk: pada sistem buffer : pH = pKa – log ([HA]/[A-]
Sehingga: nilai Ka asam lemah dapat ditentukan dari setengah titik ekivalen dimana
: [HA] = [A-] sehingga pH = pKa (1 poin)
Grafik: setengah titik ekivalen , pH = pKa
= 15/2 = 7.5 mL titrant. [OH-] yang ditambahkan (2 poin)
Dar grafik:pada volume setengah ekivalen (7,5 mL), pH = pKa = 6.5 (3 poin)

Maka: Ka = 10-pKa = 3.162 × 10-7 = 3.2 × 10-7 (2 poin)

d. Molaritas asam lemah dapat diperoleh dari pH awal: (6 poin)

HA(aq)  H+(aq) + A-(aq


Awal: m 0 0
C -x +x +x
Ekiv: m–x x x

Diketahui dari grafik: pH awal = 3.0, sehingga:


x = [H3O+] = 0.0010 M (2 poin)

Mk pernyatan kesetimbangan adalah:

. (2 poin)
OSN-2010 Page 8
Diperoleh konsentrasi asam lemah = m = 3.2 M (2 poin)

Soal 4. Struktur intan ( poin)

Dua gambar berikut adalah identik untuk menggambarkan struktur intan:

(i) Beri nomor 1 untuk atom yang terikat dengan A, kemudian nomor 2, 3 dan 4 untuk
atom yang terikat pada 1. Selanjutnya atom 5 terikat pada atom B dan atom 8; atom
6 terikat pada atom C dan atom 9; atom 7 terikat pada atom D dan atom 10.

(ii) Gambar disebelah kanan sama dengan gambar di kiri, oleh karena itu beri nomor
atom-atom pada gambar kanan yang sesuai dengan nomor atom-atom pada
gambar di sebelah kiri.
(iii) Tuliskan semua koordinat untuk setiap atom 1-10 pada gambar kedua dengan
asumsi A(0,0,1) B(1,0,0) dan D(0,1,0).
JAWAB:
a.

b.

c.

Soal 5. Disproporsionasi Hypobromite


Studi mengenai laju disproporsionasi ion hipobromit, (BrO-) didalam larutan aqua pada 80
o
C, menghasilkan ion bromat (BrO3-). Pembentukan ion bromat tersebut memberikan
perubahan konsentrasi bromit terhadap waktu, sesuai dengan grafik berikut ini:

OSN-2010 Page 9
Berdasarkan grafik dan informasi yang anda peroleh maka:

a. Tuliskan persamaan reaksi disproporsionasi ion hipobromit (2 poin)


b. Tentukan order reaksi dan tunjukkan dengan perhitungan. (6 poin)
c. Tentukan konstanta laju (k) untuk reaksi ini. (3 poin)
4. Berikan usulan mekanisme reaksi ini, yang berhubungan dengan data eksperimen.
5. Metoda manakah yang dapat digunakan untuk memperoleh kurva (seperti pada
gambar), dalam laboratorium, bila tersedia semua perlengkapan gelas, semua
pereaksi, tetapi tidak ada instrumen untuk pengukuran fisika (kecuali neraca dan
termometer dengan satu tanda pada 80 ºC)?

Solusi :
1. 3BrO-(aq)  BrO3-(aq) + 2 Br-(aq)

2. Berdasarkan grafik:
Untuk menentukan order reaksi dari grafik adalah dengan melihat ketergantungan
waktu paruh , t1/2 terhadap konsntrasi BrO-

Untuk reaksi order pertama:


t1/2 tidak tergantung terhadap konsentrasi.
Ambil sembarang 2 pasang titik dari grafik , dimana c2 = c1/2,
contohnya:
Pasangan pertama:
c1 = 0.68 M, t1 = 4,0 min.
c2 = 0.68/2 = 0.34 M,
dari grafik c vs t diperoleh t2 = t1/2 =30.0 min. (2 poin)

Pasangan kedua:
c1 = 0.52 M, t1 = 12.0 min.
c2 = 0.52/2 = 0.26 M,
OSN-2010 Page
10
dari grafik c vs t, diperoleh t2 = t1/2 = 46,5 min. (2 poin)

melalui data t1/2 yang diperoleh :


menunjukkan bahwa reaksi adalah order kedua (2 poin)
(karena t1/2 tergantung pada konsentrasi)

c. reaksi orde kedua: k = 1/(cot1/2);


k = 0.057 L/(mol·min) berdasarkan pada 2 titik pada pasangan pertama , dan
k =0.056 L/( mol·min) berdasarkan 2 titik pada pasangan kedua.

d. Reaksi order ke 2:
langkah elemeter akan melibatkan tumbukan dua partikel hiperbromit . Dalam
tumbukan ini elektron dan ion oksigen ditransfer, tetapi partikel yang terbentuk lebih
reaktif, dan intermediete ini tidak stabil dan cepatt bereaksi dengan ion hypobromit
lainnya:

2BrO-Br-+ BrO2 (lambat) BrO2-


+ BrO- Br-+ BrO3-(cepat)
Mekanisme ini sesuai dengan data eksperimen.

E. Untuk mendapatkan kurva, maka harus dimungkinkan metoda untuk menentukan


ketergantungan konsentrasi setiap reagen terhadap waktu.
Karena mengandung ion Br-, maka cara paling sederhana adalah menentukan
konsentrasi Br- melalui titrasi menggunakan larutan AgNO3 sesuai reaksi:
AgNO3 + KBr = AgBr + KNO3

Sample diambil dari campuran reaksi, didinginkan segera dari 80ºC menjadi
temperatur kamar dan dititrasi dengan larutan AgNO3 membentuk endapan AgBr

Soal 6. Elektrokimia kluster nano perak

Kluster berukuran nanometer memiliki sifat khas yang berbeda dari material berukuran
normal. Untuk meneliti perilaku elektrokimia kluster nano perak, diperlukan data berikut:
Ag(s)| AgCl(jenuh) | | Ag+(0,01M) | Ag(s) E1 = 0,170 V
Pt(s) | Ag5(s, nano), Ag+(0,01M) | | AgCl(jenuh) | Ag(s) E2 = 1,030 V
R = 8,314 J/mol K, T = 298,15 K, F = 96485 C/mol

a. Hitung Ksp AgCl

Nano Ag5 mengandung logam perak tetapi potensial standar nya berbeda
2. Hitung potensial standar nano Ag5
3. Apa yang terjadi bila nano Ag5 dimasukkan dalam larutan dengan pH = 13
4. nano Ag5 dimasukkan dalam larutan dengan pH = 5
2+
5. nano Ag5 dimasukkan dalam larutan pH = 7 yang mengandung [Cu ] =
+ -10 o + o 2+
0,001M dan [Ag ] = 10 M. Diketahui E Ag | Ag = 0,8 V, E Cu | Cu =
0,345 V, T = 298,15K

SOLUSI 6.
o -1 +
a. E = E + RTF ln [Ag ]
-1
E1 = 0,170 V= E2-E1 =RTF ln [0,01]/x = 8,314*298.15/96485 *ln0,01/x
OSN-2010 Page 11
+ -5 - -5
[Ag ] = x = 1,337x 10 M, dalam lar jenuh = Cl = 1,337x 10 M.
-10
Maka Ksp = 1,778x 10 M
-1 -5
b. Untuk sel kanan pada reaksi (2): E(AgCl) = 0,8V + RTF ln [1,337x 10 ] = 0,512
+
V, maka E = E (AgCl) – E(Ag5, Ag )
+ o + -1
dan E(Ag5, Ag )= E (Ag5, Ag )+RTF ln [0,01]
+
Ag5 E(Ag5, Ag )= 0,512V –1,030V = 0,518V
o + -1
E (Ag5, Ag )=0,518- RTF ln [0,01]= -
+ -13
0,400V c. E(H2| 2H )= 0,059 log 10 = - 0,769V
Dibanding dengan E perak nanocluster yang telah dihitung di atas, ternyata lebih
positif dari potensial hidrogen, maka nano perak berperilaku sebagai logam dan tidak
teroksidasi dalam larutan, artinya tidak ada reaksi.
-5
d. Untuk larutan dengan pH=5 maka [H+] =10
+ -5
E(H2| 2H )= 0,059 log 10 = - 0,269V
+
Potensial standar Ag5 lebih rendah dari potensial hidrogen maka ion H tereduksi dan Ag5
+
teroksidasi menjadi Ag artinya cluster Ag5 larut
-7
e. Untuk larutan dengan pH=7 maka [H+] =10
2+ -3
E(Cu| Cu )= 0,345 + (0,059/2) log 10 = 0,256 V
+ -10
E(Ag| Ag )= 0,800 + (0,059) log 10 = 0,208 V
+ -10
E(Ag5| Ag )= - 0,400 + (0,059) log 10 = - 0,992 V
+ -7
E(H2| 2H )= (0,059) log 10 = - 0,414 V
2+ 2+
Yang paling positif E(Cu| Cu ), jadi ion Cu akan tereduksi jadi Cu dan Ag5
larut membentuk Ag+.
+ 2+
Setelah beberapa saat ion Ag jadi bertambah dan konsentrasi ion Cu menurun, berikutnya
+ +
setelah konsentrasi ion Ag meningkat maka potensial E(Ag| Ag ) akan meningkat melebihi
2+
potensial E(Cu| Cu ), ini menyebabkan ion Ag+ tereduksi menjadi logam Ag.

Soal 7.
Kromium membentuk 3 senyawa isomerik A, B dan C dengan komposisi Cr = 19,52%, Cl =
39,91% dan Air = 40,47%. Setiap sampel larut dalam air dan ketika ditambahkan larutan
perak nitrat terbentuk endapan putih. Sebanyak 0,225g sampel A menghasilkan 0,363 g
endapan, untuk 0,263 g sampel B menghasilkan endapan 0,283g dan untuk 0,358g sampel
C menghasilkan endapan 0,193g. Salah satu sampel berwarna violet sedangkan yang
lainnya berwarna hijau. Semua sampel merupakan senyawa kompleks dengan bilangan
kordinasi = 6. Diketahui massa atom relatif Cr = 52, Ag = 108, Cl = 35,5 dan massa
molekul realtif H2O =18.
1. Tuliskan rumus empirik senyawa kompleks tersebut.
2. Tuliskan rumus molekul sesungguhnya dari sampel A, B, dan C.
3. Tentukan dan jelaskan sampel mana yang berwarna violet?

SOLUSI:

a. CrCl3(H2O)6
b. A= Cr[(H2O)6]Cl3 B= Cr[Cl(H2O)5]Cl2 H2O C= Cr[Cl2 (H2O)4]Cl.2
H2O
c. senyawa A yang violet karena tidak ada Cl yang terkoordinasi

OSN-2010 Page 12
Kimia Organik

Provinsi Sumatera Utara, memiliki banyak sekali potensi alam yang mengandung bahan
kinia yang dapat dikembangkan untuk kemaslahatan umat manusia. Sebagai contoh dari
kekayaan alam tersebut adalah karet alam atau 1,4-cis-poliisoprena yang berasal dari
getah pohon karet Hevea brasiliensis yang digunakan sebagai bahan baku pembuatan
ban. Kekayaan alam lainnya yang tak kalah pentingnya adalah minyak kelapa sawit yang
merupakan sumber utama asam-sama lemak dan fatty alcohol untuk dimanfaatkan di
berbagai industri seperti industri surfaktan dan oleokimia. Minyak kelapa sawit juga
merupakan ssumber -karoten yang banyak digunakan sebagai zat aditif makanan dan
obat suplemen.
Soal-soal berikut berhubungan dengan senyawa-senyawa yang bersumber dari
keanekaragaman hayati yang ada di Sumatera Utara.

Soal 8. (14 poin)


Karet alam atau 1,4-cis-poliisoprena merupakan polimer dari isoprena (2-metilbuta-1,3-
diena).
1. Gambarkan struktur isoprena! (2)
2. Gambarkan segmen polimer 1,4-cis-poliisoprena! (2)
3. Gambarkan mekanisme pembentukan polimer 1,4-cis-poliisoprena dari isoprena
melalui reaksi polimerisasi adisi radikal! (4)
4. Tuliskan satu reaksi yang dapat menunjukkan adanya ikatan rangkap pada
senyawa turunan isoprena! (3)
5. Tuliskan reaksi dan produk yang terbentuk ketika 1,4-cis-poliisoprena direaksikan
dengan belerang (S8) yang dikenal dengan reaksi vulkanisasi karet! (3)

Soal 9. (14 poin)


Salah satu senyawa potensial dalam minyak kelapa sawit adalah asam oleat (asam cis-9-
oktadekenoat). Senyawa ini diperoleh dari hidrolisis trigliserida yang 93% terkandung
dalam minyak kelapa sawit.
6. Gambarkan struktur asam oleat (asam cis-9-oktadekenoat)! (2)
7. Gambarkan reaksi dan produk hidrolisis dalam suasana basa trigliserida (trioleoil
gliserat)! (3)
8. Tuliskan reaksi dan produk reaksi antara asam oleat dengan LiAlH4 yang
dilanjutkan dengan hidrolisis dalam suasana asam (H+/H2O)! (3)
9. Tuliskan reaksi dan produk reaksi antara asam oleat dengan reagen Br2, P (atau
PBr3) yang dilanjutkan dengan hidrolisis dalam suasana asam (H+/H2O)! (3)
10. Tuliskan reaksi dan produk transesterifkasi trioleil gliserat dengan metanol dalam
suasana asam yang merupakan zat untuk biodisel! (3)

Soal 10. (13 poin)


Senyawa -karoten merupakan prekursor vitamin A. Senyawa ini diubah secara enzimatis
dalam tubuh menjadi vitamin A (Retinol) yang berguna bagi penglihatan. Salah satu
komponen penting pada mata adalah zat Retinal, yaitu analog aldehid Retinol, yang
terdapat pada retina mata. Berikut adalah struktur -karoten dan vitamin A.

OSN-2010 Page 13
-Karoten

OH

Vitamin A

1. Gambarkan struktur Retinal yang merupakan hasil oksidasi Vitamin A. (2)


2. Gambarkan mekanisme reaksi pembentukan senyawa -karoten dari Retinal
melalui reaksi Wittig Retinal oleh suatu reagen ilida yang sesuai! (struktur utuh
tidak harus digambarkan semua, yang penting transformasi pada gugus
fungsinya) (5)
3. Tuliskan reaksi dan reagen kimia yang sesuai untuk mengubah -karoten menjadi
Retinal di laboratorium! (struktur utuh tidak harus digambarkan semua, yang
penting transformasi pada gugus fungsinya) (3)
4. Tuliskan reaksi dan reagen kimia yang sesuai untuk mengubah Retinal menjadi
Vitamin A! (3)

OSN-2010 Page 14
JAWAB:

Soal 8.
a. Struktur isoprena:

b. Segmen polimer 1,4-cis-poliisoprena:

*
*
n
c. Mekanisme polimerisasi adisi radikal:

dst.
dst.

dst.

d. Reaksi identifikasi ikatan rangkap pada isoprena


Br
Br2/CCl4 Br (larutan coklat)

Br
Br
Warna coklat Br2 hilang apabila sudah bereaksi dengan ikatan rangkatp pada isoprena.

e. Reaksi Vulkanisasi karet alam:


OSN-2010 Page
15
(Sumber: http://www.chemistrydaily.com/chemistry/upload/5/58/Vulcanization.png)
Atau

(Sumber: http://chem.chem.rochester.edu/~chem421/polmod1.htm)

Soal 9.
a. Struktur asam oleat:
O

OH

b.Reaksi hidrolisis trioleiol gliserat dalam suasana basa:

OSN-2010
Page 16
O
CH CH
7 14 8 17 O
CH
O O OH +3 8 17

CH CH
O 7 14 7 14

O C H
8 17
NaOH/H2O OH O

Na
O
CH CH
7 14 8 17 OH

c. Reaksi asam oleat dengan LiAlH4 dilanjutkan hidrolisis dalam asam:


O 1) LiAlH4
+
2) H /H O
C H
2 7 14
CH
7 14
OH
OH C H
8 17

CH
8 17

d. Reaksi asam oleat dengan Br2, P atau PBr3:


CH
O 1) Br2, P 8 17 BrO
+
2) H /H2O OH
CH
7 14
CH
OH Br 6 12
CH
8 17
Br

e. Reaksi transesterifikasi trioleilgliserat dengan metanol:


O

CH CH
7 14 8 17
O
O katalis asam CH O
OH +3
7 14

+ 3 CH3OH
CH CH
7 14 8 17 OH

O O OCH3

C H
O OH 8 17
CH CH
7 14 8 17

Soal 10.
a. Struktur Retinal:
O
H

OSN-2010 Page
17
b. Mekanisme reaksi dari retinal menjadi -karoten:
H

Retinal (Ph)3P

PH(Ph)3
O

+
3
PH(Ph)
O
-Karoten

c. Sintesis Retinal dari -karoten:

-Karoten

1. O3
2. Me2S atau Zn dalam asam asetat

2 H

Retinal

d. Sintesis Vitamin A dari Retinal:


1
)

N
a
B
H
4

a
t
a
u

L
i
A
l
H
4
+
2) H /H2O
OH
H
Vitamin A
OLIMPIADE SAINS NASIONAL 2011

Manado
11-16 September 2011

JAWABAN
(DOKUMEN NEGARA)

UjianTeori
Bidang Kimia
Waktu 210 menit

Kementerian Pendidikan Nasional


Direktorat Jenderal
Managemen Pendidikan Dasar dan Menengah Direktorat
Pembinaan Sekolah Menengah Atas

2011
OSN2011 Page 1
Petunjuk :
iii. Isilah Biodata anda dengan lengkap (di lembar Jawaban)
Tulis dengan huruf cetak dan jangan disingkat !

b) Soal Teori berjumlah 8 soal essay

TOTAL Poin = 160

c) Waktu yang disediakan: 210 menit.

d) Semua jawaban harus ditulis di lembar jawaban yang tersedia

e) Tuliskan nama, nomor peserta dan asal SMA anda pada setiap halaman lembar
jawab

f) Diperkenankan menggunakan kalkulator.

g) Tabel Periodik Unsur sudah disediakan.


h) Anda dapat mulai bekerja bila sudah ada tanda mulai dari pengawas.
i) Anda harus segera berhenti bekerja bila ada tanda berhenti dari Pengawas.
j) Letakkan jawaban anda di meja sebelah kanan dan segera meninggalkan ruangan.
k) Anda dapat membawa pulang soal ujian !!

OSN2011 Page 2
Tetapan dan Rumus
23 –1
Bilangan Avogadro: NA = 6,0221×10 mol Persamaan gas Ideal: PV = nRT

Tetapan Gas: R = 8,314 J K mol


–1 –1
hc
0,08205 atm L K mol
–1 –1 Energi foton: E
–1 Energi bebas Gibbs: G = H – TS
Tetapan Faraday: F = 96485 C mol

r G oRT ln KnFEcello
–34
Tetapan Planck: h = 6,6261×10 Js
H = E + nRT
8 –1 Persamaan Faraday: Q = it
Kecepatan cahaya: c = 3,000×10 m s
Skala Nol Celsius: 273,15 K Persamaan Arrhenius: k=A
-14
1 N = 1 kg m s 1 eV = 1,602×10-19 J Kw = = 1.0×10 at 25 C
5
1 atm = 760 torr = 1.01325×10 Pa
Hukum laju terintegrasi orde ke-nol: [A] = [A]o- kt

Hukum laju terintegrasi orde pertama: ln [A] = ln [A]o- kt

Tabel Periodik Unsur dengan Massa Atom Relatif


1 18
1 2
H He
1.008 2 13 14 15 16 17 4.003
3 4 5 6 7 8 9 10
Li Be B C N O F Ne
6.941 9.012 10.81 12.01 14.01 16.00 19.00 20.18
11 12 13 14 15 16 17 18
Na Mg Al Si P S Cl Ar
22.99 24.31 3 4 5 6 7 8 9 10 11 12 26.98 28.09 30.97 32.07 35.45 39.95
19 20 21 22 23 24 25 26 27 28 29 30 31 32 33 34 35 36
K Ca Sc Ti V Cr Mn Fe Co Ni Cu Zn Ga Ge As Se Br Kr
39.10 40.08 44.96 47.87 50.94 52.00 54.94 55.85 58.93 58.69 63.55 65.38 69.72 72.64 74.92 78.96 79.90 83.80
37 38 39 40 41 42 43 44 45 46 47 48 49 50 51 52 53 54
Rb Sr Y Zr Nb Mo Tc Ru Rh Pd Ag Cd In Sn Sb Te I Xe
85.47 87.62 88.91 91.22 92.91 95.96 [98] 101.07 102.91 106.42 107.87 112.41 114.82 118.71 121.76 127.60 126.90 131.29
55 56 57 72 73 74 75 76 77 78 79 80 81 82 83 84 85 86
Cs Ba La Hf Ta W Re Os Ir Pt Au Hg Tl Pb Bi Po At Rn
132.91 137.33 138.91 178.49 180.95 183.84 186.21 190.23 192.22 195.08 196.97 200.59 204.38 207.2 208.98 (209) (210) (222)
87 88 89 104 105
Fr Ra Ac Rf Ha
(223) 226.0 (227) (261) (262)

58 59 60 61 62 63 64 65 66 67 68 69 70 71
Ce Pr Nd Pm Sm Eu Gd Tb Dy Ho Er Tm Yb Lu
140.12 140.91 144.24 (145) 150.36 151.96 157.25 158.93 162.50 164.93 167.26 168.93 173.05 174.97
90 91 92 93 94 95 96 97 98 99 100 101 102 103
Th Pa U Np Pu Am Cm Bk Cf Es Fm Md No Lr
232.04 231.04 238.03 237.05 (244) (243) (247) (247) (251) (254) (257) (256) (254) (257)
OSN2011 Page 3
Soal 1 Paduan Logam (18 poin)

Suatu paduan logam terdiri dari aluminium sebanyak 93,7% (berat/berat) dan tembaga 6,3%
(berat/berat). Densitas paduan logam tersebut sebesar 2,85 g/mL. Jika sepotong paduan logam
tersebut dengan volume 0,691 mL, bereaksi dengan larutan asam klorida secara sempurna
menghasilkan gas hidrogen, jawablah pertanyaan berikut:

a. Tuliskan reaksi kimia paduan logam tersebut dengan asam klorida. ( 4poin)

Jawab: 2Al(s) + 6HCl(aq)  2Al3+(aq) + 6Cl-(aq) + 3H2 (g) (2 poin)


Cu + HCl ≠ (2 poin)

b. Berapa massa (g) paduan logam dengan volume 0,691 mL tersebut? (3 poin)

Jawab: massa paduan logam = (0,691 mL) x 2,85g/mL = 1,97 g (3 poin)

c. Berapa massa (g) aluminium yang terdapat dalam paduan logam tersebut? (3 poin)

Jawab: massa aluminium = 0,937 x 1,97g = 1,85 g (3 poin)

d. Berapa volume (L) gas hidrogen yang terbentuk jika diukur pada keadaan STP? (3 poin)

Jawab: volume gas hidrogen = (3/2) x (1,85/26,98) x 22,4 L = 2,30 L (3 poin)

e. Jika paduan logam tersebut bereaksi dengan asam nitrat encer, tuliskan reaksi yang terjadi.
(5 poin)

Jawab: 2Al(s) +2HNO3(aq)  2Al3+(aq) + 6NO3-(aq)+ 3H2 (g) (2 poin)


3Cu(s) + 8H+ + 2NO3-(aq)  3Cu2+(aq)+ 2NO(g)+ 4H2O (l) (3 poin)

OSN2011 Page 4
Soal 2. Kimia Air Laut (15 poin)

Menado terkenal dengan lautnya yang indah. Kandungan kimia air laut bergantung pada lokasi
diambilnya sampel air laut itu, namun rata-rata air laut mengandung 3,5 % (berat/berat) senyawa
ion dalam air. Pada suhu rata-rata air laut, massa jenis air laut adalah 1,0 g/mL.

b) Dengan menganggap senyawa ion yang terlarut dalam air laut adalah NaCl, tentukan
konsentrasi NaCl dalam air laut dalam satuan molar, tunjukkan bahwa pernyataan: “dalam
tiap liter air laut terkandung sekitar 1 mol ion” dapat dianggap benar. (3 poin)

Jawab:
Dalam 1L (1000g) air larut terdapat 35 g garam, karena semua dianggap NaCl
maka jumlah mol NaCl = (35/58,5)mol = 0,6mol. Jumlah ion sekitar 1,2 mol/ L
larutan (asumsi di atas “sekitar 1 mol” benar)

b. Dengan tetap menggunakan asumsi yang sama seperti di atas, tentukan konsentrasi NaCl
dalam air laut yang tepat dalam satuan molal. (3 poin)

Jawab:
Dengan asumsi tersebut, 1000g air laut mengandung 35 g NaCl dan 965 g air,
maka konsentrasinya = (35/58,5)mol/0,965 kg = 0,62 m

c) Air laut dapat diubah menjadi air tawar melalui proses desalinasi, salah satu proses tsb
menggunakan osmosis balik yakni memberikan tekanan luar minimal sebesar tekanan
osmosis larutan tersebut. Dari jawaban (a) di atas, perkirakan tekanan yang diperlukan
untuk mendapatkan air murni dari air laut pada suhu 298K.
(3 poin)

Jawab:
Tekanan = cRT = 1 x 0.082 x 298 atm = 24,4 atm

d. Perkirakan titik didih air laut pada tekanan 1 atmosfer (Kb = 0,51°C/molal) (2
poin)

Jawab:
Titik didih = 100 + (0,5 ) = 100,5oC

OSN2011 Page 5
Pada Tabel berikut dirangkum konsentrasi gas nitrogen, oksigen dan karbondioksida di atmosfer
dan di air laut:
gas % di mg/kg (ppm)
atmosfer di air laut
Nitrogen, N2 78% 12.5
Oksigen, O2 21% 7
Karbon dioksida, CO2 0,03% 90

Bila suatu gas tidak bereaksi dengan air, maka kelarutan gas tersebut dalam air berbanding
lurus dengan tekanan gas tersebut.
e. Jelaskan dengan kalimat singkat mengapa konsentrasi gas oksigen lebih rendah daripada
konsentrasi gas nitrogen di dalam air laut. (2 poin)

Jawab: Karena di atmosfer pN2 >>> pO2 maka:


[O2] = kpO2 akan lebih kecil dari [N2] = k pN2, kO2 dan kN2 berdekatan
nilainya karena keduanya non-polar.

f) Apakah data di atas menunjukkan bahwa karbondioksida bereaksi dengan air? Jelaskan
(2 poin)

Jawab: Ya karena [CO2] >>> [N2] padahal pCO2 << pN2


Soal 3. Respirasi Aerobik dan Anaerobik pada Mikroorganisme (20 poin)

Dalam respirasi (pernafasan) anaerobik (tanpa udara), mikroorganisme menggunakan anion A dari
asam kuat B sebagai akseptor elektron, dan bukan mengunakan oksigen. Dalam A dan B, keduanya
mengandung unsur X dengan tingkat oksidasi paling tinggi. Dalam proses respirasi ini terjadi reduksi
А secara sistematik. Pertama, tingkat oksidasi Х turun 2 satuan menghasilkan anion C dari asam
lemah D. Kemudian C direduksi menjadi gas tak berwarna F yang berbau tajam, yang membentuk
kation Е dalam larutan air (aqueous). Bakteri lain dapat mereduksi C secara bertahap, menjadi oksida
G yang berupa gas tak berwarna, yang selanjutnya dapat diubah menjadi oksida I berwujud gas, dan
akhirnya menjadi molekul K sederhana yang berupa gas inert.

Pada saat yang sama, unsur X digunakan sebagai sumber energi dalam respirasi aerobik
(menggunakan oksigen). Bakteri tertentu dapat mengoksidasi F menjadi С, bakteri lainnya dapat
mengoksidasi С menghasilkan А yang masuk ke dalam tanah. Asam В dapat menguraikan mineral
dalam tanah dan dapat mempercepat terjadinya korosi (karat) pada bahan bangunan. Hingga kini,
diperkirakan bahwa X hanya terlibat dalam nutrisi untuk organisme aerobik. Tetapi, Brocadia
anammoxidans menggunakan F sebagai sumber energi dan C untuk respirasi. Sebagai hasil pada
akhir proses terbentuk K.
a. Tuliskan rumus kimia yang sebenarnya untuk A, B, C, D, E, F, G, I, K, dan X; serta tuliskan
namanya masing-masing. (10 poin)

A – NO3 , ion nitrat


B – HNO3, asam nitrat
(iv) – NO2 , ion nitrit
(v) – HNO2, asam nitrit
(vi) – NH4+, kation ammonium
(vii) – NH3, ammonia
(viii) – NO, nitrogen monoksida
I – N2O, dinitrogen monoksida
K – N2, nitrogen
– N, atom nitrogen

b. Tuliskan persamaan reaksi rumus kimia yang sebenarnya pada reaksi berikut: (tambahkan
air bila diperlukan): (8 poin)
+
1. A + e + H → C,
2. C + e + H+ → F,
3. C + e + H+ → G,
4. G + e + H+ → I,
5. I + e + H+ → K,
6. E + C → K,
7. F + O2 → C + H+ dan
8. C + O2 → A.
B. i) NO3 + 2e + 2H+ → NO2 + H2O
NO2 + 6e + 7H+ → NH3 + 2H2O
NO2 + e + 2H+ → NO + H2O
2NO + 2e + 2H+ → N2O + H2O
N2O + 2e + 2H+ → N2 + H2O
NH4+ + NO2 → N2 + 2H2O
2NH3 + 3O2 → 2 NO2 + 2H+ + 2H2O
viii)2 NO2 + O2 → 2 NO3

OSN2011 Page 7
3. Identifikasi satu spesi kimia apa yang berfungsi sebagai donor elektron dan satu spesi kimia
apa yang berfungsi sebagai akseptor elektron dalam proses yang melibatkan Brocadia
anammoxidans tersebut. (2 poin)
Dalam Brocadia anammoxidans, ion nitrit (C) adalah akseptor elektron , sedangkan
ammonia (F) adalah donor elektron.

OSN2011
Page 8
Soal 4. Industri Pembuatan Gas Hidrogen (24 poin)

Hidrogen dapat dibuat dalam suatu proses industri dengan pemanasan campuran hidrokarbon
dan uap air, sesuai reaksi:
CH4(g) + H2O(g)  3H2(g) + CO(g)

Pada soal-soal berikut, semua gas–gas dianggap ideal dan nilai H dan S tidak bergantung pada
temperatur. Diketahui pada 298,15 K tetapan kesetimbangan K p = 1,450.10 25 , dan pada 1580 K,
nilai Kp = 26640.

a. Berdasarkan nilai Kp tersebut, tentukan apakah reaksi pembuatan gas hidrogen tersebut
merupakan reaksi eksoterm atau endoterm. Jelaskan (2 poin)

Nilai Kp bertambah besar dengan naiknya temperatur, sehingga reaksi ini adalah reaksi
endoterm.

B. Hitung H, S juga G dan K p untuk reaksi ini pada 1000 K. Gunakan angka berarti yang
sesuai pada hasil perhitungan berdasarkan data di atas. (10 poin)

ln(Kp1/Kp2) = - H /R.(T1-1 – T2-1) (1)


H = -8,314.ln(1,450.10-25 / 26640).(298,15-1 – 1580-1)-1J
H = 205,9 kJ.mol-1
G = -R.T.lnKp G 1580K = -8,314 J/mol.1580.ln26640
G 1580K = -133,9 kJ/mol
G = H - T. S S = (205,9 + 133,9) kJ/mol / 1580 K
-1 -1
S = 215,1 K.K .mol
G 1000K = 205,9 kJ.mol-1 – 1000 K.215,1 JK-1 mol-1
G 1000K = -9200 kJ.mol-1
ln Kp.1000 K = -9200 kJ.mol-1 / (-8,314 J.mol-1.K-1.1000 K) ; ln Kp 1000 K = 1,107
Kp 1000 K = 3,025

Dalam suatu wadah tertutup dengan volume konstan pada 400 K terdapat campuran 1,000 mol
CH4 dan 1,000 mol H2O dengan tekanan total adalah 1,600 bar. Wadah tersebut dipanaskan
sampai temperatur 1100 K. Pada temperatur ini nilai tetapan kesetimbangan adalah 28,50.

c. Hitunglah tekanan dalam wadah ini pada saat tercapai kesetimbangan. Tentukan persen
metana yang berubah menjadi gas H 2. (4 poin)

OSN2011
Page 9
Pada volume konstan p1/T1 = p2/T2 p2 = T2.p1/T1
CH4 H2O H2 CO
Awal pada 400 K 0,800 bar 0,800 bar 0 bar 0 bar
Awal pada 1100 K 2,200 bar 2,200 bar 0 bar 0 bar
Kesetimb pada (2,200-x) bar (2,200 – 3.x bar X bar
1100 K x)bar

Tekanan total dalam kesetimbangan = (4,400 + 2.x) bar Ptotal,1100 K = 6,550 bar
Rasio dari jumlah = rasio tekanan parsial
Konversi (metana) = x / 2,200.100% konversi (metana) = 49 %

Reaksi yang berlangsung dengan jumlah pereaksi yang sama pada 100 K dalam wadah dengan
tekanan konstan 1,600 bar ternyata menghasilkan jumlah konversi metana yang berbeda dari
reaksi sebelumnya.
d. Jelaskan apakah jumlah konversi metana pada reaksi tersebut lebih rendah atau lebih
tinggi daripada reaksi sebelumnya? (2 poin)

Konversi akan lebih tinggi, karena tekanan berubah dari 6,550 bar menjadi lebih rendah
(1,6 bar) maka sistem tersebut mengarah pada jumlah mol yang besar (produk).

Pada suatu reaksi dengan kondisi ini, volume akan berubah dari V awal ke Vksetimb = 1.750 xVawal.
e. Tentukan persen jumlah konversi metana pada kondisi ini. (4 poin)

CH4 H2O H2 CO
Awal pada 1100 K 1 mol 1 mol 0 0 2 mol
Kstimb pada 1100 K (1-a) mol (1-a) mol 3.a mol a mol 2.(`1+a) mol

Jumlah dan volume sebanding pada tekanan dan suhu yang sama 2.(1+a) a = 0,750
Konversi (metana) = a/1.100%; Konversi (metana) = 75 %

f. Bagaimana cara memisahkan gas CO dari suatu campuran yang mengandung gas H 2 dan
CO? (2 poin)
Konversi dari CO ke CO2 dengan penambahan uap air: CO + H2O  CO2 + H2
Pada tekanan tinggi CO2 dapat dipisahkan dengan mudah dengan air atau ia
diabsorp oleh methanol atau dengan basa (seperti K2CO3 atau amina organik):
CO2 + H2O + K2CO3  2 KHCO3.
OSN2011 Page
10
Soal 5. Buffer Fosfat Darah (20 poin)

Asam Fosfat (H3PO4) adalah suatu asam lemah triprotik dengan tiga tetapan ionisasi Ka, yaitu Ka1 =
7x10 3, Ka2= 6 x10 8 dan Ka3 = 4x10 13 yang dapat membentuk sistem buffer yang bervariasi.

a. Di dalam larutan asam fosfat 0,1M, tentukan spesi kimia asam fosfat yang konsentrasinya
paling tinggi selain air. (2 poin)

Jawab:
karena asam lemah dan Ka sangat kecil, maka sebagian besar molekul asam
fosfat tidak mengion. Spesi yang konsentrasinya paling tinggi adalah H 3PO4

Salah satu sistem buffer yang mengatur pH darah (≈ 7,4) adalah buffer fosfat, yaitu sistem buffer
yang terdiri dari ion H2PO4 dan HPO42 .
b. Gambarkan struktur Lewis ion HPO42 termasuk struktur resonansinya. (3
poin)

Di dalam darah, ion ion yang berasal dari asam fosftat ini merupakan sistem buffer yang dapat
mempertahankan pH darah terhadap penambahan sedikit asam atau basa.

3. Tuliskan persamaan reaksi kesetimbangan ion fosfat dengan air yang sesuai dengan pH
darah =7,40.
(2 poin)
Jawab:
Pada pH 7,4 yang dekat dengan nilai pKa2, kesetimbangan yang terjadi adalah
H2PO4 (aq) + H2O(l) ⇌ HPO4 (aq) + H3O+(aq)

Nilai pH dari larutan buffer fosfat dapat dihitung dari persamaan Henderson–Hasselbach:
pH = pKa – log ([H2PO4 ]/[ HPO42 ]), dimana pKa = 6,86.
Kemampuan untuk menjaga pH yang konstan didefinisikan sebagai kapasitas buffer, . Kapasitas
buffer ini setara dengan jumlah ion Н + ( +) atau ОН ( ) yang ditambahkan ke dalam satu liter larutan
yang dapat mengubah pH sebesar satu satuan.

OSN2011 Page 11
4. Hitung:

i) jumlah relatif (dalam %) H2PO 4 dan HPO 24 . (4 poin)


[H PO ] [H PO] 0, 288
2
7, 4 6,86 log 2
4 2 4

2
[HPO 4 ] [HPO 4 ]
%(H PO ) 0, 288 100 = 22,4 ≈ 22
2 4 1 0,288
1 100 = 77,6 ≈ 78
%(HPO 24 )
1 0,288

ii) konsentrasi (mmol/L) H2PO 4 dan HPO 24 yang tersedia dalam darah dimana
keseluruhan konsentrasi fosfat adalah 1,2 mmol/L. (3 poin)
[H2PO 4 ] = 0,224∙x 1,2 mmol/L = 0,269 mmol/L ≈ 0,27 mmol/L
[HPO 24 ] = 0,776∙x 1,2 mmol/L = 0,931 mmol/L ≈ 0,93 mmol/L

e. Hitung kapasitas buffer dari buffer fosfat darah , + dan . (4 poin)

[H PO ] 0, 269
2 4 H H 2, 88 0,931- H
6, 4 6, 86 log [HPO 2 ]
4 H
+= 0,65 mmol/L
0, 269
OH 0, 0288 0, 931 OH
[H PO] OH
2 4

8, 4 6, 86 log [HPO 2- ] O
4 H

= 0,24 mmol/L

7. Jelaskan manakah buffer darah yang baik, ion asam laktat (pK a = 3,9) atau ion karbonat
(pKa = 10,2)? (2 poin)

Asam laktat karena kapasitas buffer untuk buffer fosfat lebih besar untuk asam ( +>

)
OSN2011

Page 12
Soal 6. Ekstraksi Bijih Emas (20 poin)
Sulawesi Utara adalah salah satu wilayah Indonesia yang kaya deposit bijih emas. Emas merupakan
logam inert dan salah satu logam tertua yang banyak dimanfaatkan manusia, dan penambangan
emas sudah dilakukan sejak 3500 SM.
a. Tuliskan konfigurasi elektron logam emas. (1 poin)

Konfigurasi elektron Au: 1s2 2s2p6 3s2p6d10 4s2p6d10f14 5s2p6d10 6s1


Atau ([Xe] 4f14 5d10 6s1 .)
Kandungan emas dalam bijih emas umumnya sangat rendah (<10 mg/kg bijih). Hingga kini, metode
yang paling umum digunakan dalam ekstraksi emas dengan skala besar menggunakan larutan
natrium sianida, suatu zat kimia yang sangat beracun. Walaupun proses ini tidak bersahabat
terhadap lingkungan, sebagian besar penambangan emas tetap menggunakannya, sehingga
menghasilkan limbah beracun yang sangat besar.
Ekstraksi emas dengan menggunakan larutan sianida melibatkan reaksi kimia yang sangat rumit dan
perlu dipelajari secara mendalam. Larutan yang digunakan untuk melarutkan emas mengandung CN
dan O2 terlarut, dengan pH 10,2 agar larutan mengandung ion CN yang cukup besar (99%). Bila
kondisi pH kurang dari 10,2 , larutan ion CN - akan membentuk asam sianida (HCN) yang bersifat
racun. Menurut mekanisme yang diusulkan, selama proses pelarutan, pada permukaan partikel
emas terbentuk suatu sel Volta mikro dengan mekanisme berikut:
Anoda: Au  Au+ + e
Au+ + CN  AuCN
AuCN + CN  Au(CN)2
Katoda: O2 + 2H2O + 2e  H2O2 + 2 OH
H2O2 + 2 e  2 OH
b. Tuliskan reaksi total pada Anoda, reaksi total pada Katoda dan reaksi sel Total pada sel
Volta mikro pelarutan emas tersebut. (6 poin)
Larutan emas dalam larutan garam sianida:
Anoda (oksidasi): Au  Au+ + e-
Au+ + CN  AuCN
AuCN + CN  Au(CN)2
----------------------------------------------------------------- +
Anoda: Au(s) + 2 CN  Au(CN)2 (2 poin)

Katoda (reduksi): O2 + 2 H2O + 2 e-  H2O2 + 2 OH


H2O2 + 2 e-  2 OH
----------------------------------------------------------------- +
Katoda : O2+ 2 H2O+ 4 e 4OH (2 poin)

Anoda Au(s) + 2 CN  Au(CN)2 x4

Katoda
-------------------------------------------------------------------:
O2+ 2 H2O+ 4 e 4OH------------- ----------+
Reaksi sel: 4Au + 8 [CN] + H2O +O24[Au(CN)2] + 4OH- (2 poin)

Untuk memperoleh logam emas, ke dalam larutan sianida yang diperoleh dari proses pelarutan
emas dilakukan proses de-aerasi (tanpa oksigen), kemudian ditambahkan sejumlah serbuk logam Zn.
Pada proses ini terbentuk endapan emas dan ion kompleks Zn(CN) 42 yang larut.
OSN2011 Page 13
c. Tuliskan reaksi penambahan serbuk Zn pada larutan emas sianida tanpa oksigen tersebut.
(2 poin)
Jawab: Reaksi 2 Au(CN)2 +Zn 2Au + Zn(CN)42

4. Bila dalam larutan emas sianida terdapat oksigen terlarut, maka jumlah endapan emas
yang diperoleh ternyata semakin sedikit dan pH larutan meningkat. Jelaskan mengapa
emas yang diperoleh semakin sedikit. (3 poin)
+ o
Diketahui: Au +e Au(s) E = 1,68 volt
2+
Zn + 2e Zn(s) Eo = -0,76 volt
Karena oksigen dapat mengoksidasi Zn, sedangkan yang diharapkan kompleks
Au(CN)2- tereduksi menjadi Au.
Reduksi (katodik): O2 + 2H2O + 4e  4OH-
Oksidasi (anodik): ZnZn2+ + 2e x2
2+ - 2
Zn + 8 CN  2[Zn(CN)4] x2
------------------------------------------------------------------------------------- +
2
2Zn + 8 [CN] + 2H2O +O22[Zn(CN)4] + 4OH
Larutan emas sianida harus di de-oksigenasi terlebih dahulu untuk mencegah terjadinya
oksidasi Zn oleh oksigen.

e. Gambarkan struktur geometri ion Au(CN) 2 dan Zn(CN)42 dan tuliskan hibridisasinya.
(4 poin)

Jawab: Au+ : [Xe] 4f14 5d10. (2 poin) Zn2+ : 1s2 2s2p6 3s2p6d10

(2 poin)

Tetapan pembentukan kompleks K f, untuk [Au(CN)2] adalah = 2 x 10 38. Dalam larutan kompleks emas
sianida, konsentrasi CN = 0,100 M dan konsentrasi total spesi spesi Au(I) = 5,10×10 -3 M pada
kesetimbangan.
f. Tuliskan reaksi kesetimbangan pembentukan kompleks tersebut dan hitung persentase ion
Au(I) yang ada dalam bentuk ion kompleks sianida. (4 poin)

OSN2011 Page 14
Au+ + 2CN  [Au(CN)2] Kf [Au(CN)2] = 2 x 1038 (1 poin)
Kf= [Au(CN)2] / [Au+].[CN ]2= 2 x 1038
Misalkan [Au+]= a
Maka: 2 x 1038= (5,10×10-3-a)/(a)(0,100)2
2 x 1038 (a)(0,100)2= 5,10×10-3-a
2 x 1034a=.5,10.10-3-a 2 x 1034a≈.5,10.10-3
a = Au+= (5,10.10-3)/ (2 x 1034) =2,55x10-37M (sangat kecil sekali).
Maka [Au(CN)2] dalam larutan≈ 100 % (3 poin)
OSN2011

Page 15
Soal 7. Senyawa Prekursor untuk Obat Sakit Maag (20 poin)

James M. Schlatter adalah seorang ahli kimia dari Amerika yang berhasil mensintesis senyawa
prekursor untuk obat sakit maag (A).

O
HO O
N CH3
H

O NH2 O

(A)

a. Tuliskan semua gugus fungsi yang ada dalam senyawa (A). (2 poin)

Gugus Amida Gugus Fenil


a).
O
HO O
Gugus karboksilat N CH3
H
O NH2 O
Gugus Ester
Gugus amina
(A)

2. Senyawa A berasal dari dipeptida yang dibentuk dari dua asam amino fenilalanin dan
asam amino(B). Asam amino (B) dalam industri dibuat dari reaksi berikut:
O O O hv
H 2O NH3
D E katalis enzim B

Gambarkan struktur molekul B , D dan E dan berikan nama trivialnya. (10


poin)
O O

O O O OO OH OH
H2O HO OH hv HO NH HO
3
katalis enzim NH2
O O
D= Asam maleat E= Asam Fumarat B= L-Asam aspartat

c. Gambarkan struktur (R)-fenilalanin dan (S)-fenilalanin dalam proyeksi Fisher. (4 poin)

OSN2011 Page 16
COOH COOH
HN H H NH2
2

CH2 CH2

(S )-f enilalanin (R)-fenilalanin

d. Asam amino fenilalanin dapat disintesis melalui metode Gabriel. Tentukan pereaksi yang
digunakan (E dan F) dalam sintesis fenilalanin melalui metode Gabriel sesuai skema reaksi
berikut. (4 poin)

H2 H2 (E) H2 H (F) H2 H
C C COOH C C COOH C C COOH

Br NH2

Pereaksi (E) = 1. Br2, PBr3


2. H2O O

- +
1. N K

Pereaksi (F) = O
Kalium f talamida
2. OH /H2O
OSN2011 Page 17
Soal 8. Eter Siklik dari Alga Merah (23 poin)

Salah satu sumber daya hayati bahari yang dapat ditemukan dan dibudidayakan di Sulawesi Utara
adalah alga merah genus Laurencia. Apabila alga merah ini dipelajari lebih lanjut, maka di dalamnya
terkandung senyawa (+)-Laurenyne. Senyawa (+)-Laurenyne diisolasi pertama kali dari alga merah
oleh Thomson dan rekannya pada tahun 1980, yang ternyata memiliki toksisitas terhadap bakteri
(bersifat antibakteria dan antimikrobial). Berikut adalah pertanyaan seputar senyawa (+)-Laurenyne.

Cl

(+)-Laurenyne

Berdasarkan struktur (+)-Laurenyne, tentukan:


a. Jumlah karbon kiral yang terdapat dalam struktur senyawa ini! (2 poin)

Jumlah karbon kiral (2 poin) : 3 (tiga)

b. Jumlah stereoisomer maksimum yang dimiliki senyawa ini! (2 poin)

Jumlah stereoisomer maksimum (2 poin) : 23 = 8 (maksimum 8 stereoisomer)

3. Konfigurasi absolut pada masing-masing karbon kiral pada struktur senyawa (+)-Laurenyne
(silakan beri nomor pada masing-masing karbon kiral untuk membedakan masing-masing
atom kiral tersebut)! (3 poin)
Konfigurasi absolut (3 poin= @ konfigurasi yang benar 1 poin):
Cl
1
R 2
R
R
O
3

Jika senyawa (+)-Laurenyne. direaksikan dengan BH3 dalam THF, kemudian direaksikan dengan OH
dalam H2O2 maka:
d. Gambarkan struktur produk yang terbentuk! (3 poin)

OSN2011 Page 18
Cl Cl
1) BH3/THF
2) H2O2, OH
OH
O O O

OH H

e. Gambarkan struktur produk yang terbentuk dari tahapan reaksi berikut pada kotak yang
tersedia berikut. (nilai @ kotak = 2 poin)
Cl
O OH
1) NaOH, kalor +
+ +
2)H O KMnO4/H O OH
O HO O
+ O + O
O
O HO
(+)-Laurenyne OH H
OH

6 Gambarkan struktur produk yang terbentuk jika senyawa (+)-Laurenyne. direaksikan


dengan gas hidrogen dalam katalis Lindlar (katalis Pd yang teracuni).
(3 poin)

Cl

H2, Katalis Lindlar


(Pd teracuni)
O O

(+)-Laurenyne

OSN2011 Page 19
7. Tuliskan dan jelaskan reagen apa saja yang dapat mengidentifikasi keberadaan gugus-gugus
fungsi pada senyawa (+)-Laurenyne secara uji kualitatif dan tuliskan reaksi yang
terjadi! (6 poin)

Reagen untuk identifikasi gugus fungsi ikatan rangkap pada senyawa (+)-Laurenyne yaitu dengan
mereaksikan senyawa (+)-Laurenyne dengan Br2 dalam pelarut CCl4 yang akan mengadisi ikatan
rangkap. Sehingga indikasinya adalah warna larutan brom yang berwarna coklat ketika ditambahkan
ke dalam larutan yang mengandung senyawa (+)-Laurenyne akan berubah menjadi tidak berwarna.
(nilai untuk reagen yang benar = 1 poin; penjelasan = 2 poin). Reaksinya sebagai berikut (nilai
untuk reaksi = 3 poin):
Cl Cl
Br2 dalam CCl4
(warna coklat) Br
O O Br
Br Br
Br
(+)-Laurenyne Br Br
Br
(Dalam larutan berwarna bening) (Dalam larutan berwarna bening)

Reagen lainnya adalah lelehan natrium untuk identifikasi gugus alkil halida pada senyawa (+)-
Laurenyne yaitu dengan mereaksikan senyawa (+)-Laurenyne dengan lelehan natrium yang
merupakan reduktor kuat dan kemudian akan membentuk garam natrium halida pada akhir
reaksinya. Sehingga indikasinya adalah terbentuknya endapan garam natrium halida berwarna putih
pada akhir reaksi. (nilai untuk reagen yang benar = 1 poin; penjelasan = 2 poin). Reaksinya sebagai
berikut (nilai untuk reaksi = 3 poin):

SEMOGA BERHASIL
HAK CIPTA
DILINDUNGI UNDANG UNDANG

OLIMPIADE SAINS NASIONAL 2012


Jakarta
2- 7 September 2012
SOAL UJIAN

Bidang Kimia
Ujian Teori
Waktu: 210 menit

Kementerian Pendidikan Nasional dan Kebudayaan


Direktorat Jenderal Pendidikan Menengah
Direktorat Pembinaan Sekolah Menengah Atas
2012
Petunjuk :
1. Isilah Biodata anda dengan lengkap (di lembar Jawaban)
Tulis dengan huruf cetak dan jangan disingkat!
2. Ujian Teori terdiri dari 8 Soal:
Soal 1 = 31 poin
Soal 2 = 20 poin
Soal 3 = 24 poin
Soal 4 = 17 poin
Soal 5 = 21 poin
Soal 6= 24 poin
Soal 7 = 21 poin
Soal 8 = 24 poin

TOTAL Poin = 182 poin

3. Waktu yang disediakan: 210 menit.

4. Semua jawaban harus ditulis di dalam kotak di lembar jawaban yang tersedia.

5. Diperkenankan menggunakan kalkulator.

6. Diberikan Tabel Periodik Unsur, rumus dan tetapan yang diperlukan.


7. Mulailah bekerja ketika ada tanda “MULAI” dari Pengawas.
8. Anda harus segera berhenti bekerja bila ada tanda “BERHENTI” dari
Pengawas.
9. Letakkan jawaban anda di atas meja sebelah kanan dan segera tinggalkan
ruangan.
10. Anda dapat membawa pulang soal ujian !!

OSN2012 Halaman ii
OSN2012 Halaman iii
Tetapan dan Rumus

Bilangan Avogadro NA = 6.022∙1023 partikel.mol–1

R = 0,08205 L·atm/mol·K
= 8,3145 L·kPa/mol·K
= 8,3145 x107 erg/mol·K
Tetapan gas universal, R = 8,3145 J/mol·K
= 1,987 kal/mol·K
= 62,364 L·torr/mol·K
1 atm = 101,32 kPa
1 atm =760 mmHg = 760 torr
= 101325 Pa = 1,01325 bar
Tekanan gas 1 torr = 133,322 Pa
1 bar = 105 Pa
1 Pa = 1 N/m2= 1 kg/(m.s2)
1 kal = 4,182 J
Energi 1 J = 1 L·kPa

Persamaan gas Ideal PV= nRT


Tekanan Osmosis pada larutan = M RT
Tetapan Kesetimbangan air (Kw) pada 25oC Kw= 1,0x10 14
∆n
Tetapan kesetimbangan dan tekanan parsial Kp = Kc(RT)
gas
Temperatur dan Tetapan kesetimbangan
൬ +
Hubungan tetapan kesetimbangan dan ln o=
G=RTlnK
energi Gibbs
Energi Gibbs pada temperatur konstan GH T S
Isoterm reaksi kimia G = G + RT∙ln Q
Potensial sel dan energi Gibbs Go=nFEo
Tetapan Faraday F = 96500 C/mol elektron
Muatan elektron 1,6022 x 10 19 C
Ampere (A) dan Coulomb (C) A =C/det

Reaksi orde pertama: AB

Reaksi orde kedua: AB

OSN2012 Halaman iv
Soal 1. Alloy (31 poin)

Suatu alloy dibuat dengan komposisi yang mengandung aluminium, zinc, silikon dan
tembaga. Jika 1 gram alloy ini bereaksi dengan asam hidroklorida berlebih, terbentuk gas
hidrogen sebanyak 899 mL (21 C, 102,25 kPa). Pada penambahan larutan asam
hidroklorida berlebih ini, terdapat sisa zat yang tak-larut (residu) sebanyak 170 mg.
Selanjutnya, pada kondisi tekanan dan temperatur yang sama, jika 500 mg alloy ini
bereaksi dengan larutan natrium hidroksida berlebih, maka terbentuk gas hidrogen
sebanyak 552 mL. Pada percobaan ini juga diperoleh residu.
a. Tuliskan unsur dari alloy yang bereaksi dengan asam hidroklorida, tuliskan
persamaan reaksinya dan tuliskan pula rumus kimia dari residu pada proses ini.
(6 poin)
b. Tuliskan unsur dari alloy yang bereaksi dengan dengan larutan natrium hidroksida,
tuliskan persamaan reaksinya dan tuliskan pula rumus kimia dari residu pada
proses ini. (9 poin)
c. Hitunglah komposisi dari alloy ini dalam persentase massa (2 angka desimal).
(16 poin)
Catatan: Penulisan persamaan reaksi yang tidak setara tidak mendapatkan nilai.

Soal 2. Senyawa Oksida (20 poin)

Oksida adalah senyawa yang dibentuk dari suatu unsur dengan oksigen. Jenis unsur
yang membentuk oksida dapat berasal dari unsur non-logam, semilogam maupun logam.
Oksida yang terbentuk memiliki sifat unik, baik sebagai asam, basa, oksidator maupun
reduktor. Penelitian mengenai oksida sangat berkembang pesat karena banyak material
baru dapat dibuat dari beragam oksida. Pada soal ini dipelajari aspek kimia yang
mendasari karakteristik oksida.
Timah dan timbal memiliki oksida yang khas, yaitu SnO, SnO 2, PbO dan PbO2. Ada dua
aturan umum yang diamati pada oksida-oksida dari unsur-unsur di golongan 14 Tabel
Periodik Unsur modern, yaitu:
• Dengan meningkatnya sifat logam unsur dari atas ke bawah, oksidanya
menjadi lebih basa.
• Dengan meningkatnya bilangan oksidasi unsur yang sama, oksidanya menjadi
lebih asam.
b. Gunakan aturan umum di atas untuk menyarankan oksida timah atau oksida
timbal mana yang cenderung bereaksi dengan masing-masing reagen berikut:
(catatan: dalam setiap kasus tuliskan persamaan reaksi setaranya)
(i) dengan NaOH (aq)
Oksida yang cenderung bereaksi: …………………………… … (2 poin)
Persamaan reaksi: …………………………………………………… (3 poin)
(ii) dengan HCl (aq)
Oksida yang cenderung bereaksi: ……………………………… (2 poin)
Persamaan reaksi: ………………………………………………… (3 poin)

b. Oksida hafnium HfO2 pertama kali digunakan pada chip komputer Intel® super
cepat 45 nm Core-2 yang berdasarkan pada teknologi Penryn. Jika diketahui
keelektronegatifan O= 3,61, Hf = 1,16, maka:

OSN2012 Halaman 1
(i) Tuliskan koordinat titik untuk oksida hafnium, HfO2, dalam segitiga van Arkel
berikut (4 poin)

(ii) Tuliskan persamaan reaksi setara untuk reaksi antara HfO2 dengan asam
klorida (3 poin)
(iii) Tuliskan pula persamaan reaksi untuk reaksi reduksi produk reaksi (ii) oleh
magnesium (3 poin)

Catatan: Penulisan persamaan reaksi yang tidak setara tidak mendapatkan


nilai.

Soal 3. Reaksi Redoks (24 poin)

Reaksi redoks adalah reaksi kimia yang terjadi antara suatu zat yang disebut oksidator
dengan zat lain yang berfungsi sebagai reduktor. Oksidator adalah suatu zat yang
mengalami reaksi reduksi yaitu yang menggunakan sejumlah elektron sebagai pereaksi;
sedangkan reduktor adalah zat yang mengalami reaksi oksidasi atau menghasilkan
elektron sebagai salah satu produknya. Pada reaksi redoks tidak dijumpai elektron baik
pada pereaksi maupun pada produk reaksi.
Ketika gas asam sulfida dialirkan ke dalam larutan asam nitrat, terjadilah reaksi redoks
yang ditandai dengan terbentuknya endapan belerang dan berbagai spesi kimia yang
mengandung nitrogen seperti gas nitrogen, gas nitrogen dioksida, gas nitrogen
monoksida dan ion ammonium.
a. Tuliskan reaksi oksidasi untuk gas asam sulfida menghasilkan endapan belerang. (2
poin)

b. Tuliskan 4 reaksi ion untuk reduksi asam nitrat yang masing-masing menghasilkan
gas nitrogen, gas nitrogen dioksida, gas nitrogen monoksida dan ion ammonium.
(8 poin)
c. Tuliskan 4 reaksi redoks antara asam sulfida dengan asam nitrat yang
menghasilkan 4 spesi kimia yang mengandung nitrogen tersebut. (8 poin)

d. Dari reaksi redoks tersebut, gambarkan struktur 3 spesi kimia yang paling
berbahaya bagi lingkungan. (6 poin)

Catatan: Penulisan persamaan reaksi yang tidak setara tidak mendapatkan nilai.

OSN2012 Halaman 2
Soal 4. Pembuatan Minuman Anggur (17 poin)

Di negara yang memiliki empat musim, penduduknya sering mengkonsumsi anggur


sebagai penghangat tubuh ketika musim dingin tiba. Di pabrik minuman anggur, untuk
menstabilkan anggur sebelum dimasukkan ke dalam botol, drum-drum yang berisi
minuman anggur harus didinginkan terlebih dahulu. Dalam proses pendinginan, ion
tartarat, HOOC(C2H4O2)COO–, yang terkandung di dalamnya dapat membentuk kristal
padat putih kalium tartarat yang tak diinginkan. Dalam proses pendinginan, ion tartarat
akan mengendap sebagai kalium tartarat bila konsentrasinya lebih besar daripada 1,5 10–
4
M.
Seorang analis kimia yang bekerja di pabrik tersebut, dalam suatu contoh drum minuman
3
anggur, mendapatkan konsentrasi asam tartarat di dalamnya sebesar 1,75x10 M. Dia
harus mengambil keputusan apakah langsung mendinginkan atau harus menurunkan
konsentrasi asam tartarat terlebih dahulu sebelum didinginkan.
a Menurut pendapat anda, apakah analis tersebut harus segera mendinginkan anggur
3
yang konsentrasi asam tartaratnya 1,75x10 M atau menurunkan terlebih dahulu
konsentrasi asam tartarat sebelum menstabilkan minuman anggur tersebut? Uraikan
penjelasan anda berdasarkan perhitungan. (7 poin)
3
b. Hitung pH larutan asam tartarat 1,75x10 M. (3 poin)
c. Hitung berapa konsentrasi awal asam tartarat (HOOC(C2H4O2)COOH) maksimum
dalam drum minuman anggur sehingga dalam proses pendinginan tidak
menimbulkan endapan kristal yang tak-diinginkan. Tentukan pula apakah sang analis
harus mendinginkan untuk menstabilkan anggur tersebut. (7 poin)

Diketahui:
Tetapan kesetimbangan asam tartarat, HOOC(C2H4O2)COOH, Ka = 4,60 10–5.

Soal 5. Gas Nitril Klorida (21 poin)

Nitril klorida (NO2Cl) adalah gas yang dikenal sebagai zat yang digunakan untuk
memasukkan gugus nitro ke dalam struktur suatu senyawa. Gas ini bersifat reaktif dan
mendidih pada 16 oC dalam tekanan atmosfer.
a) Gambarkanlah struktur Lewis nitril klorida beserta resonansinya (bila ada), dan
tentukan muatan formal pada setiap atom dalam struktur yang anda gambarkan.
(6 poin)
b. Tuliskan hibridisasi orbital atom N dalam struktur yang anda gambarkan?
(2 poin)

Bila terdekomposisi, gas ini akan menghasilkan gas nitrogen dioksida dan gas klor sesuai
reaksi:
2NO2Cl(g) 2NO2(g) + Cl2(g)
Untuk reaksi tersebut, telah diusulkan mekanisme reaksi melalui 2 langkah reaksi, yaitu:

Langkah 1: ? lambat

Langkah 2: NO2Cl(g) + Cl(g)NO2(g) + Cl2(g) sangat cepat

c. Berikan usulan untuk mekanisme reaksi pada langkah 1. (2 poin)


d. Tuliskanlah persamaan laju reaksi yang mungkin. (2 poin)

OSN2012 Halaman 3
e. Tuliskan pendapat anda mengenai nilai relatif dari energi pengaktifan (Ea) untuk
setiap langkah reaksi tersebut? (3 poin)
f. Dengan menggunakan diagram energi di bawah ini, identifikasikan manakah spesi
dari langkah di atas yang terdapat pada posisi yang ditunjukkan oleh nomor 1, 2 dan
3 pada diagram energi di bawah ini. (6 poin)

Soal 6. Kadar Air dalam Makanan (24 poin)

Salah satu prosedur analisis yang sering dilakukan pada industri makanan adalah
penentuan kadar air di dalam makanan. Metode yang paling umum digunakan adalah
titrasi Karl Fischer. Sampel makanan ditambahkan ke dalam pereaksi Fischer yang terdiri
dari imidazol, metanol dan SO2. Metanol pada pereaksi Fischer bertindak sebagai
reaktan dan pelarut. Sampel makanan di dalam pereaksi Fischer tersebut kemudian
ditrasi dengan larutan I2 dalam pelarut metanol. Titik ekivalen ditentukan secara
elektrokimia, yaitu dengan mengukur muatan yang mengalir selama reaksi terjadi
menggunakan elektroda Pt.
Tahap pertama pada proses ini adalah pembentukan ion [CH3COSO2] dari reaksi:

+
NH + CH OH + SO NH + [CH COSO ] (1)
3 2 2 3 2

N N
imidazol

Ion [CH3COSO2] kemudian bereaksi dengan I2 dan air dari sampel makanan sebagai
berikut:

[CH3COSO2] + I2 + H2O2I + [CH3COSO3] + 2H+ (2)


Karena pereaksi Fischer dibuat berlebih, maka reaksi (2) akan terus berlangsung sampai
semua air yang terkandung dalam bahan makanan habis bereaksi.

OSN2012 Halaman 4
a. Tentukan hibridisasi orbital atom N pada molekul imidazol. (4 poin)
b. Reaksi (1) diperkirakan terjadi melalui zat antara yang merupakan adduct dari
metanol dan SO2. Gambarkan struktur Lewis dari adduct tersebut. (4 poin)
c. Reaksi pembentukan zat antara tersebut merupakan reaksi asam-basa. Tentukan
pereaksi yang bertindak sebagai asam. (2 poin)
d. Zat antara di atas kemudian bereaksi dengan imidazol membentuk ion imidazolium
dan [CH3COSO2] . Reaksi ini juga merupakan reaksi asam-basa. Tentukan pereaksi
yang bertindak sebagai basa. (2 poin)
e. Ada dua macam air yang terkandung dalam bahan makanan yaitu air bebas dan air
yang berikatan dengan senyawa pada makanan, misalnya protein. Tentukan jenis
ikatan yang terbentuk antara molekul air dengan protein. (2 poin)

Metode Karl Fischer dapat digunakan untuk menentukan kadar air di dalam madu. Untuk
keperluan ini 1,00 g sampel madu ditambahkan ke dalam pereaksi Fischer dan dititrasi
dengan larutan I2 dalam metanol.

vi. Jika pada titrasi sampel madu tersebut terukur muatan sebesar 1822,49 C mengalir
melalui elektroda Pt, tentukan persen massa air dalam sampel madu tersebut.
(Muatan 1 mol elektron adalah 96485 C). (10 poin)

Soal 7. Antibiotik Alami (21 poin)

Beberapa puluh tahun lalu, sering kali orang mengabaikan luka sehingga banyak
menimbulkan kematian akibat darahnya keracunan atau mengalami infeksi disebabkan
oleh luka yang dideritanya. Sekarang, permasalahan tersebut sudah dapat diatasi dengan
ditemukannya suatu “antibiotik alami” oleh G.Dogmack pada tahun 1934, yaitu senyawa
X. Kini, senyawa X tersebut dapat disintesis menggunakan bahan awal nitrobenzena,
C6H5NO2 .

Berikut ini diberikan skema sintesis senyawa X:

HNO3, H2SO4 Fe
o HCl
Nitrobenzena (A) 100 C B (aq) I

Fe
HCl
(aq)
1). NaNO2
H2SO4 SOCl2 NH3 2). I
C D E II X
o
200 C
Berdasarkan skema di atas, dengan bahan awal A (nitrobenzena), tentukan struktur
molekul dari semua senyawa (B, I, C, D, E, II dan X) dalam tahapan reaksi sintesis
tersebut (setiap struktur senyawa bernilai masing-masing 3 poin).

OSN2012 Halaman 5
Soal 8. Asam Levulinat sebagai Sumber Biofuel (24 poin)

Asam levulinat (1, C5H8O3) atau nama lainnya adalah asam 4-oksopentanoat merupakan
suatu asam -keto karboksilat yang memiliki banyak aplikasi, diantaranya sebagai
prekursor potensial untuk pembuatan polimer sejenis nilon, karet sintesis dan plastik.
Asam levulinat juga merupakan senyawa antara yang bermanfaat untuk sintesis obat-
obatan/farmasi dan sebagai prekursor komoditas bahan kimia di inddustri, seperti
metiltetrahidrofuran (2), valerolakton dan etil levulinat (3). Senyawa 2 dan 3 merupakan
salah satu senyawa yang potensial sebagai gasolin oksgenat dan aditif biodisel, yaitu
sumber bahan bakar terbarukan (biofuel) masa kini dan masa depan. Asam levulinat
dapat diperoleh dari konversi biomassa selulosa secara hidrolisis menjadi glukosa yang
kemudian berubah menjadi hidroksimetilfurfural dan akhirnya menjadi asam levulinat
dengan melepaskan asam format. Konversi selulosa menjadi asam levulinat dapat dilihat
pada skema berikut.

Metiltetrahidrofuran (2) sebagai salah satu alternatif bahan bakar terbarukan dapat
diperoleh dari asam levulinat (1) berdasarkan skema reaksi berikut:

OSN2012 Halaman 6
a. Gambarkan struktur asam levulinat (1). (2 poin)
b. Berdasarkan skema pembuatan senyawa 2 dari senyawa 1 di atas, gambarkan
struktur senyawa A, B, C dan senyawa 2. (12 poin)
c. Gambarkan mekanisme reaksi dehidrasi senyawa asam levulinat 1 dalam kondisi
H+/H2O menjadi senyawa angelicalactone berdasarkan skema di atas. (4 poin)
d. Etil levulinat (3) sebagai bahan baku biofuel lainnya dapat diperoleh dari asam
levulinat (1). Tuliskan reagen dan kondisi yang digunakan untuk menghasilkan 2 dari
1. (2 poin)
e. Gambarkan mekanisme reaksi pembentukan senyawa etil levulinat (3) dari asam
levulinat (1) menggunakan reagen dan kondisi pada (d). (4 poin)

SEMOGA BERHASIL

OSN2012 Halaman 7
NAMA :
No. Peserta :
ASAL SEKOLAH:

OLIMPIADE SAINS NASIONAL 2012


Jakarta
2- 7 September 2012

Bidang Kimia
UjianTeori

LEMBAR JAWABAN
Waktu: 210 menit

Kementerian Pendidikan Nasional dan Kebudayaan


Direktorat Jenderal
Managemen Pendidikan Dasar dan Menengah
Direktorat Pembinaan Sekolah Menengah Atas
2012

OSN2012 Halaman 1
NAMA :
No. Peserta :
ASAL SEKOLAH:

Soal 1. Alloy (31 poin)


No Uraian Jawaban Poin Nilai
a.
2 Al +6 HCl + 12 H2O  + 2 [Al(H2O)6]Cl3 + 3 H2 (3 poin) 6

Zn + 2 HCl  ZnCl2 + H2 (1 poin)

residu: Si dan Cu (2 poin)

b.
2 Al + 2 NaOH + 10 H2O  2 Na[Al(OH)4(H2O)2] + 3 H2 (3poin) 9

Zn + 2 NaOH + 2 H2O  Na2[Zn(OH)4] + H2 (3 poin)

Si + 2 NaOH + H2O  Na2SiO3 + 2 H2 (2 poin)

residu: Cu (1 poin)

(Persamaan reaksi dengan ion dan tanpa kompleks yang mengikat air
dalam a) dan b) dianggap benar).

c.

=+=
16
Dengan H 1000mg sampel, 899 mL H2 na mol H2
na = (102,25 kPa)(899.10-3)L/(8,3145)(294) mol =0,03760 mol= 37,60 mmol
(2 poin)

Dengan OH- 500 mg sampel 552 mL H2 nb mol H2


nb = (102,25.kPa)(552.10-3)L/8,3145 (294 ) mol =0,02309 mol = 23,09 mmol
(2 poin)

1000 mg sampel = 2nb = 2x 23,09 mmol = 46,18 mmol


silicon:
1000 mg sampel ; n(Si) = (46,18 - 37,60)/2 = 4,29 mmol
m(Si) = n(Si).M(Si) = 4,29 mmol.28 mg/mmol = 120,12mg
%Si = (120,12/1000)x100% = 12,01%
(3 poin)

Tembaga:
m(Si) + m(Cu) = 170 mg m(Cu) =170 -120,12= 49,88 mg
% Cu = (49,88/1000)x100% = 4,99%
(3 poin)

OSN2012 Halaman 2
NAMA :
No. Peserta :
ASAL SEKOLAH:

Aluminium:
m(Al) + m(Zn) = 1000 mg – 170 mg = 830 g
x mg Al memberikan 3/2 . x mg/M(Al) = 3/2 . x/26,98 mmol H2
(830-x) mg Zn memberikan (830-x) mg/M(Zn) = (830-x)/65,40 mmol H2
3/2 . x/26,98 + (830-x)/65,40 = 37,60 x = 617,99
Al = 617,99 mg = (617,99 /1000) x 100% = 61,799 61,80 %
(4 poin)

Zn:
m(Zn) = (830-617,99) mg
m(Zn) = 212,01 mg 21,20%
(2 poin)

Catatan: Penulisan persamaan reaksi yang tidak setara tidak mendapatkan


nilai.

Total 31

OSN2012 Halaman 3
NAMA :
No. Peserta :
ASAL SEKOLAH:

Soal 2. Senyawa Oksida (20 poin)


No Uraian Jawaban Poin Nilai
a.(i) Dengan NaOH(aq):
Oksida yang cenderung bereaksi:
2
SnO2
Persamaan reaksi:

2NaOH + SnO2 → Na2SnO3 + H2O 3

a.(ii) Dengan HCl(aq):


Oksida yang cenderung bereaksi:
PbO
2
Persamaan reaksi:
PbO + 2HCl → PbCl2 + H2O 3

b.(i) Koordinat titik HfO2:


Koordinat Titik HfO2 adalah (2,39; 2,45) 4
b.(ii) Persamaan reaksi:
Reaksi HfO2 dengan asam klorida: HfO2 + 4HCl → HfCl4 + 2H2O 3

b.(iii) Persamaan reaksi:


Reaksi HfCl4 + 2Mg → Hf + 2MgCl2 3
20
Total

OSN2012 Halaman 4
NAMA :
No. Peserta :
ASAL SEKOLAH:

Soal 3. Reaksi Redoks (24 poin)


No Uraian Jawaban Poin Nilai
a.

H2S(g)  S(s) + 2H+ + 2 e- 2

b.
12H+ + 2NO3- + 10e-  N2(g) + 6H2O (2 poin) 8

2H+ + NO3 - + e-  NO2(g) + H2O (2 poin)

4H+ + NO3 - + 3e-  NO(g) + 2H2O (2 poin)

10H+ + NO3 - + 8e-  NH4+ + 3H2O (2 poin)

c.
5H2S + 2H+ + 2NO3 - 5S(s) + N2(g) + 6H2O (2 poin) 8
H2S + 2H+ + 2NO3- S(s) + 2NO2(g) + 2H2O (2 poin)

3H2S + 6H+ + 2NO3 - 3S(s) + 2NO(g) + 4H2O (2 poin)

4H2S + 2H+ + NO3- 4S(s) + NH4++ 3H2O (2 poin)

d.
struktur H2S bengkok dengan 2 pasang elektron disekitar S (2 poin) 6

struktur NO linier, ikatan rangkap dengan 1 e tunggal pada N (2 poin)

struktur NO2 bengkok dengan 1 e tunggal pada N (2 poin)

Total 24

OSN2012 Halaman 5
NAMA :
No. Peserta :
ASAL SEKOLAH:

Soal 4. Pembuatan Minuman Anggur (17 poin)


No Uraian Jawaban Poin Nilai
c) Perhitungan

HOOC(C2H4O2)COOH + H2O (ℓ) HOOC(C2H4O2)COO– + H3O+ 7

Awal 1,75.10-3 – 0 0
(M)
Perubahan –x – +x +x
(M)
Kestmbngn 1,75.10-3– x – x x
(M)

- + x
Ka = 4,60 ×10-5 =
HOOC C 4
H4 O2 COO HO
3 =
2

HOOC C H O COOH 1,75.10-3 - x


4 4 2
-3
( 1,75.10 - x) ( 4,6.10-5 )= x 2
1,75.10-3 - x ≈ 1,75.10-3
( 1,75.10-3 ) ( 4,6.10-5 )= x 2
x 2 = 8,05.10-8
x = 2,84.10-4
Jadi : konsentrasi ion tartarat adalah 2,84x10-4 M > 1,5.10-5 M

(5 poin)

Sebelum didinginkan, maka konsentrsi asam tartrat dalam minuman


anggur harus diturunkan terlebih dahulu (2 poin)

b.
Berdasarkan uraian jawaban (a), konsentrasi ion tartarat dalam larutan 3
4
adalah 2,84x10 M, sehingga pH larutan adalah:
pH = - log 2,84.10-4 = 3,55

OSN2012 Halaman 6
NAMA :
No. Peserta :
ASAL SEKOLAH:
c.
Konsentrasi maksimum asam tartarat 7

HOOC(C2H4O2)COOH + H2O (ℓ) HOOC(C2H4O2)COO– + H3O+

Awal 1,75.10-3 – 0 0
(M)
Perubahan –x – +x +x
(M)
Kestmbngn 1,75.10-3– x – x x
(M)

HO
-5 HOOC C 4 H4O2 COO 3 1,5 ×10 1,5 ×10

-
+ = -4 -4
Ka = 4,60 ×10 = HOOC C HO COOH a - 1,5x10 -4

4 4 2

( a - 1,5x10-4 ) ( 4,6x10-5 )= 2,25x10-8


4,6x10-5 a - 6,9x10-9 = 2,25x10-8
2,25x10-8 + 0,69x10-8 = 2,94x10-8
a=
4,6x10-5 4,6x10-5
a = 6,39x10-4
konsentrasi maksimum ion tartarat supaya tidak mengendap
dalam proses pendinginan adalah 6,39x10-4 M

(5 poin)

Berdasarkan perhitungan pada (b) konsentrasi awal ion tartarat dalam sampel
anggur lebih besar daripada batas maksimum agar tidak mengendap
(1,75x10 3 M > 6,39x10 4 M), sehingga analis tidak perlu mendinginkan anggur
tersebut. (2 poin)

Total 17

OSN2012 Halaman 7
NAMA :
No. Peserta :
ASAL SEKOLAH:

Soal 5. Gas Nitril Klorida (21 poin)


No Uraian Jawaban Poin Nilai
a. 6
Masing-masing struktur resonansi 2 poin

2
b. hibridisasi orbital atom N: sp 2
c. NO2Cl(g)NO2(g) + Cl(g) 2
d. Laju= r = k[NO2Cl] (reaksi orde 1) 2

e. Ea untuk Langkah 1 lebih besar dibandingkan terhadap Ea untuk langkah 2. 3


Besarnya nilai Ea ditunjukkan oleh lambatnya laju langkah reaksi 1 seiring
dengan hanya beberapa molekul saa yang mempunyai cukup energi pada
temperatur tertentu T untuk mengatasi rintangan energi yang lebih besar
dibandingkan untuk mengatasi rintangan energi pengaktifan yang lebih kecil
pada langkah reaksi 2.

f. Masing-masing spesi yang benar bernilai 2 poin 6

Total 21

OSN2012 Halaman 8
NAMA :
No. Peserta :
ASAL SEKOLAH:

Soal 6. Kadar Air dalam Makanan (24 poin)


No Uraian Jawaban Poin Nilai
a.
sp2 untuk N dengan ikatan rangkap dan dan sp3 untuk N yang mengikat H. 4

b.
4

c.
SO2 2
d.
imidazol 2
e.
ikatan hidrogen 2

f.
[CH3COSO2] + H2O[CH3COSO3] + 2H+ + 2e 10
I2 + 2e 2I-
[CH3COSO2] + I2 + H2O 2I- + [CH3COSO3] + 2H+ (4 poin)

1 mol air menghasilkan 2 mol elektron (1 Poin)

Mol air = 0,5 mol e = 0,5 x (1822,49 C)/(96485 C mol-1) = 9,4444 mmol
(2 poin)

Massa air = mol air x Mr air = 9,4444 x 18 mg = 0,169999 g (2 poin)

Persen massa air = (0,169999 g/1 g)x100% = 16,99% ~ 17% (1 poin)

Total 24

OSN2012 Halaman 9
NAMA :
No. Peserta :
ASAL SEKOLAH:

Soal 7. Antibiotik Alami (21 poin)


Senyawa Struktur Poin Nilai
3

OSN2012 Halaman 10
NAMA :
No. Peserta :
ASAL SEKOLAH:
3

II

Total 21

OSN2012 Halaman 11
NAMA :
No. Peserta :
ASAL SEKOLAH:

Soal 8. Asam Levulinat sebagai Sumber Biofuel (24 poin)


No Uraian Jawaban Poin Nilai
a. 2

b. 12
Masing-masing struktur bernilai 3 poin

c. 4
Setiap tahapan reaksi bernilai 1 poin

OSN2012 Halaman 12
NAMA :
No. Peserta :
ASAL SEKOLAH:

d. 2
Reagen: etanol (1 poin); kondisi: asam (H+/H2O), panas (1 poin)

e. 4
Setiap tahapan reaksi bernilai 1 poin

Total 24

SEMOGA BERHASIL

OSN2012 Halaman 13
Diunduh dari http://urip.wordpress.com

W
SOAL UJIAN
SELEKSI CALON PESERTA OLItllIPIADE SAINS NASIONAL2Ol4
TINGKAT PROVINSI

BIDANG KIMIA
Waktu : 180 menit

KEMENTERIAN PENDIDIKAN DAN KEBUDAYAAN


DIREKTORAT JENDERAL PENDIDIKAN MENENGAH
DIREKTORAT PEMBINAAN SEKOLAH MENENGAH ATAS
TAHUN 2014

Diunduh dari http://urip.wordpress.com facebook@urip.kalteng


Diunduh dari http://urip.wordpress.com
ffi
ffi
@
Petunjuk:

1. lsilah Biodata anda dengan lengkap (di lembar Jawaban)


Tulis dengan huruf cetak dan jangan disingkat !

2. Soal Teori ini terdiri dari dua bagian:


A. 30 soalpilihan Ganda @ 3 poin = 90 poin
jawabanbenar = 3poin
jawaban salah = -l poin

tidakmenjawab = 0poin

B. 7 Nomor so?l essay = 141 poin


TOTAL Poin = 231 poin

3. Tidak ada ralat soal


4. Waktu yang disediakan:180 menit
jawaban
5. Semua harus ditulis di lembar jawaban yang tersedia

Jawaban
6. soal essay harus dikerjakan dalam kotak yang tersedia fiawaban tidak boleh
tersebar)

7. Diberikan Tabel Periodik Unsur, formula dan tetapan yang diperlukan


8. Diperkenankan menggunakan kalkulator.
9. Tidak diperbolehkan membawa Hand Phone (HP) atau peralatan Komunikasi lainnya
10. Anda dapat mulai bekerja bila sudah ada tanda murai dari pengawas.
11. Anda harus segera berhenti bekerja bila ada tanda berhenti dari Pengawas.

12. Letakkan jawaban anda di meja sebelah kanan dan segera meninggalkan ruangan.
13. Anda dapat membawa pulang soal ujian t!

osP-2014

Diunduh dari http://urip.wordpress.com facebook@urip.kalteng


Diunduh dari http://urip.wordpress.com

Tetapan dan rumus berguna

Tetapan (bilangan)
Avogadro

Tetapan gas universal, R


0sP-2014

Tekanan gas

1 molgas (STP)

Persamaan gas ldeal


Tekanan Osmosa pada
larutan
Tetaoan Kesetimbanqan
pada
air (K*) 25o"
Konstanta kesetimbangan
dan tekanan oarsial oas
Temperatur dan
konstanta kesetimbangan
Hubungan tetapan
kesetimbangan dan
enerqi Gibbs
Energi Gibbs pada
temperatur konstan
Keria maksimum, w
lsotherm
reaksi klmia
Potensial sel
dan enerqi
Gibbs
Konstanta
Faradav
Muatan elektron
A) dan
(C
Coulomb
Reaksi orde pertama:
A*B

Reaksi orde kedua: A-B

Tetapan laju dan


temperatur
ktr
on
N e = 6.022'1023 1,
60
partikel.mol- 22
x
10-
R = 8,314 J.K-1.mol-l= 1e
C
8,314 x10'erg.Mol-'.K' A = C/det
oLA)
= 1,987 cal.mol-l.K-1 =
0,082054 L.atm.mol-1.K-1 dt : *rar
Al. : i,4l^ e-&'
1Pa=1N/m2=1kg/
(m.sz)
1 atm. = 760 mmHg =
ratr:=-"]i{i =6'1rl
760 torr 1
= 101325Pa = l
1,01325 bar

I bar = '10sPa
22,4 L *
PV = nRT t
tr= M RT o
1
K* = ,0x10-14 -
(o
= K6(RT)^n
rlt
V
lnK: -LHo
p
\
i
T)
+ko w
nsta
nta -
zk,t E^ L
AGo = -RT ln K
LG LH
= -ZA,S -') 1,

*
w = AnRT
AG = AG" + Rf'lnQ
AGo - - nFEo
F
'"\or): E
=
9
6
5
0
0
C
/
m
o
l
e
l
e tIl

Diunduh dari http://urip.wordpress.com facebook@urip.kalteng


Diunduh dari http://urip.wordpress.com

r-(9o o.qr=
s< N*E ;ZR!-i F .
r.<

FJ
\a< oaoE
!-{ \O
E
m<
Fl lfi F. I*HC
t€
{<
Rfi
- ui
!-f $
E
ro< .n 8s
A-
tr
I
J AS
o
E
^
:) EQ
L
E

dT5A
J \Ct-= Or!

o {a
a

-
J 2t
I
oo lI
rg
fi, & E6
-u .o
L
Frl
:i
o
cl - I;rr S
-o,
-o \
(E
F oIq9
I\,9J

o\"H
n

EA
[3
N< 6l
. oI
!fao
5fl
€Fr
a
. Ho

q
* >,
.:i
c
H< - l*i o
,
F
e
d

osP-2014
r.5i \o Ec \o(J! 6Us
€fi8 rn E5
o,<d
fl tso =t e(rs c\P=6.$
mQF
\.o^ -L c'7.S
\ozj i.trx-
c

O\ P;
d

O\tI * cg3 rrtr<Y O\Aio

co
P: xFs lv
Diunduh dari http://urip.wordpress.com
facebook@urip.kalteng
Diunduh dari http://urip.wordpress.com

Diunduh dari http://urip.wordpress.com


facebook@urip.kalteng
Diunduh dari http://urip.wordpress.com

Diunduh dari http://urip.wordpress.com


facebook@urip.kalteng
Diunduh dari http://urip.wordpress.com

Diunduh dari http://urip.wordpress.com


facebook@urip.kalteng
Diunduh dari http://urip.wordpress.com

Diunduh dari http://urip.wordpress.com


facebook@urip.kalteng
Diunduh dari http://urip.wordpress.com

Diunduh dari http://urip.wordpress.com


facebook@urip.kalteng
Diunduh dari http://urip.wordpress.com

Diunduh dari http://urip.wordpress.com


facebook@urip.kalteng
Diunduh dari http://urip.wordpress.com

Diunduh dari http://urip.wordpress.com


facebook@urip.kalteng
Diunduh dari http://urip.wordpress.com

Diunduh dari http://urip.wordpress.com


facebook@urip.kalteng
Diunduh dari http://urip.wordpress.com

Diunduh dari http://urip.wordpress.com


facebook@urip.kalteng
Diunduh dari http://urip.wordpress.com

Diunduh dari http://urip.wordpress.com


facebook@urip.kalteng
Diunduh dari http://urip.wordpress.com

Diunduh dari http://urip.wordpress.com


facebook@urip.kalteng
Diunduh dari http://urip.wordpress.com
SOAL UJIAN
SELEKSI CALON PESERTA OLIMPIADE SAINS NASIONAL 2015
TINGKAT PROVINSI

Waktu : 180 menit

KEMENTERIAN PENDIDIKAN DAN KEBUDAYAAN


DIREKTORAT JENDERAL PENDIDIKAN MENENGAH
DIREKTORAT PEMBINAAN SEKOLAH MENENGAH ATAS
TAHUN 2015

1
OSP-2015
Petunjuk :

11 Isilah Biodata anda dengan lengkap (di lembar Jawaban)


Tulis dengan huruf cetak dan jangan disingkat!

12 Soal Teori ini terdiri dari dua bagian:


A. 30 soal pilihan Ganda @ 3 poin = 90 poin
jawaban benar = 3 poin
jawaban salah = -1 poin
tidak menjawab = 0 poin
C. 6 soal essay= 110 poin

TOTAL Poin = 200 poin

14. Tidak ada ralat soal

15. Waktu yang disediakan : 180 menit

16. Semua jawaban harus ditulis di lembar jawaban yang tersedia

17. Jawaban soal essay harus dikerjakan dalam kotak yang tersedia (jawaban tidak boleh tersebar)

18. Diberikan Tabel Periodik Unsur, Rumus, dan Tetapan yang diperlukan

19. Diperkenankan menggunakan kalkulator

20. Tidak diperbolehkan membawa Hand Phone (HP) atau peralatan Komunikasi lainnya

21. Anda dapat mulai bekerja bila sudah ada tanda mulai dari Pengawas

22. Anda harus segera berhenti bekerja bila ada tanda berhenti dari Pengawas

23. Letakkan jawaban anda di meja sebelah kanan dan segera meninggalkan ruangan

24. Anda dapat membawa pulang soal ujian!!

2
OSP-2015
3
OSP-2015
4
OSP-2015
5
OSP-2015
2 Pilihan Berganda: pilihlah jawaban yang paling tepat

A. Bila Cu(CN)2 dipanaskan, dihasilkan C 2N2 (sianogen) dan CuCN. Massa Cu(CN) 2
yang dibutuhkan untuk membuat C 2N2 sebanyak 5,00 g adalah
20,2 g
22,2 g
24,2 g
26,4 g
28,6 g

B. Bila persen hasil reaksi:



3NO2(g) + H2O (l) 2HNO3(aq) + NO(g)
Adalah 75,0%, dan dalam reaksi tersebut dikonsumsi sebanyak 45,0 g gas NO 2, maka
massa (dalam satuan gram) asam nitrat, HNO 3(aq) yang dihasilkan adalah
K 22,5 g
L 30,8 g
M 41,1 g
N 54,8 g
O 69,3 g

6. Suatu pil sakit kepala mengandung 200 mg ibuprofen (C 13H18O2) diminum dengan 0,5 L air
oleh siswanya yang perutnya kosong. Bila semua pil tersebut larut, maka konsentrasi larutan
(dalam satuan molal) yang terbentuk dalam perut siswa tersebut adalah
2,3 x 10-3 m
4,1 x 10-3 m
9,7 x 10-4 m
1,9 x 10-3 m
1,7 x 10-2 m

7. Pada tekanan 50 kPa dan 127 oC, sebanyak 100 cm3 gas pada mempunyai massa 0,120
g. Massa molekul relatif gas tersebut adalah
1,2
25
80
120
160

8. Diketahui terdapat larutan zat dalam air sebagai berikut:


KCl, CH3CH2COOH, CH3CH2CH3, CH3CH2CH2OH, dan CH3C(O)CH3
Urutan yang paling tepat untuk kelarutan zat-zat tersebut di dalam air adalah
KCl < CH3CH2COOH < CH3CH2CH3 < CH3CH2CH2OH < CH3C(O)CH3
KCl < CH3CH2CH2OH < CH3CH2CH3 < CH3CH2COOH < CH3C(O)CH3
CH3CH2CH3 < KCl < CH3C(O)CH3 < CH3CH2CH2OH < CH3CH2COOH
CH3CH2COOH < CH3CH2CH2OH < CH3C(O)CH3 < CH3CH2CH3 < KCl
CH3CH2CH3 < CH3C(O)CH3 < CH3CH2CH2OH < CH3CH2COOH < KCl

6
OSP-2015
o
8. Suatu zat padat mempunyai titik leleh yang tajam dan jelas di atas 100 C. Zat padat tersebut
tidak dapat menghantarkan listrik bahkan dalam keadaan lelehan. Zat padat tersebut larut
dalam pelarut hidrokarbon. Struktur yang paling tepat mengenai zat padat tersebut adalah
Kristal atom
Kristal ion
Kristal molekul raksasa
Kristal molekul
Logam

9. Suatu sampel dari senyawa X, bila dipanaskan dengan larutan natrium hidroksida akan
menghasilkan gas A. Bila X dipanaskan dalam asam sulfat pekat, akan dihasilkan gas B. Bila gas
dan B direaksikan, maka akan dihasilkan kembali senyawa X. Berdasarkan informasi tersebut
maka senyawa X adalah
F. CH3CO2C2H5
G. NH2CH2CO2CH3
H. NH4CI
I. NH4I
J. (NH4)2SO4

11. Pernyataan paling tepat yang dapat menjelaskan bahwa endapan magnesium hidroksida
dapat larut dalam larutan NH4Cl(aq), tetapi tidak larut dalam larutan NaCl(aq) adalah
A Dalam air, larutan NH4Cl menghasilkan NH4OH, dan ion OH- yang terbentuk
kemudian memberikan efek ion sejenis
B Ion NH4+ dalam larutan NH4Cl akan menurunkan nilai hasil kali kelarutan Mg(OH) 2
C Larutan garam NH4Cl kurang berdisosiasi sempurna dibandingkan larutan NaCl
D Ion Na+ dan ion Mg2+ adalah isoelektron (mempunyai jumlah elektron sama)
E Ion NH4+ dalam air akan menghasilkan sejumlah H3O+

12. Alanin. H2NCH(CH3)CO2H adalah suatu asam amino dengan nilai Ka = 4,5 x 10 -3 dan nilai Kb
= 7,4 x 10-5. Di dalam air, spesi yang mempunyai konsentrasi paling tinggi pada pH 7 adalah
A H2NCH(CH3)CO2H
B +H3NCH(CH3)CO2H
C H2NCH(CH3)CO2-
D +H3NCH(CH3)CO2-
E Semua jawaban, A, B, C dan D benar

13. Pada molekul berikut ini.

Jumlah atom karbon yang mempunyai hibridisasi sp 2 adalah


19. 0
20. 1
21. 2
22. 3

7
OSP-2015
E. 4

L. Perhatikan reaksi pembentukan glukosa (C 6H12O6) berikut ini:

CO2(g) + 2C2H5OH(l) + energi panas ↔ C6H12O6(aq)


Di antara pernyataan berikut yang paling tepat mengenai persen hasil C 6H12O6 adalah
F. Persen hasil C6H12O6 bertambah besar jika tekanan parsial CO2 diturunkan
G. Persen hasil C6H12O6 naik dua kali lipat jika tekanan parsial CO 2 diduakalikan
H. Persen hasil C6H12O6 bertambah besar jika suhu dinaikkan
I. Persen hasil C6H12O6 berkurang jika suhu diturunkan
J. Persen hasil C6H12O6 berkurang jika bila tekanan total sistem reaksi dinaikkan

13. Di dalam reaksi kimia perubahan senyawa X menjadi senyawa Z, melalui mekanisme
reaksinya ditemukan bahwa tahap reaksinya berlangsung melalui pembentukan senyawa Y,
yang dapat diisolasi. Tahap yang dilalui adalah:
X --> Y , ∆H = positif
Y --> Z , ∆H = negatif
Berdasarkan informasi tersebut, profil reaksi yang sesuai dengan data tersebut adalah

13. Perhatikan reaksi gas pencemar NO2 dan ozon berikut ini:

2NO2(g) + O3(g) N2O5(g) + O2(g)
Reaksi tersebut diamati lajunya dan diperoleh data berikut ini:
Percobaan NO2(g), M O3(g), M Laju awal, Ms-1
1 0,0015 0,0025 4,8 x 10-8
2 0,0022 0,0025 7,2 x 10-8
3 0,0022 0,0050 1,4 x 10-7
Dari percobaan tersebut, penyataan paling tepat mengenai hukum laju reaksi (r) adalah
24. r = k[NO2]2[O3]
25. r = k[NO2][O3]2

8
OSP-2015
G. r = k[NO2][O3]
H. r = k[NO2]
I. r = k[O3]

15. Reaksi berikut ini, 3ClO- (aq) ClO3-(aq) + 2Cl-(aq) telah disusulkan berlangsung melalui
mekanisme berikut ini:

ClO-(aq) + ClO-(aq) ClO2-(aq) + Cl-(aq) (lambat)

ClO2-(aq) -
ClO3-(aq) -
+ ClO (aq) + Cl (aq) (cepat)
Hukum laju yang konsisten dengan mekanisme tersebut adalah:
G. Laju = k[ClO-]2
H. Laju = k[ClO-]
I. Laju = k[ClO-][ClO-]
J. Laju = k[ClO-][Cl-]
K. Hukum laju harus ditentukan secara eksperimen, bukan dari stoikiometri

17. Reaksi kesetimbangan berikut terjadi dalam campuran asam nitrat pekat dan asam sulfat
pekat:
+ - +
HNO3(aq) + 2H2SO4(aq) ↔ NO2 (aq) + 2HSO4 (aq) + H3O (aq)
Pernyataan yang paling tepat mengenai reaksi kesetimbangan ini adalah
Penambahan H2O akan mengurangi konsentrasi NO2+
HNO3 dan NO2+ adalah pasangan asam-basa konjugasi
Asam nitrat bertindak sebagai suatu oksidator
Asam sulfat bertindak sebagai dehidratator
Asam sulfat bertindak sebagai suatu basa

18. Tetapan kesetimbangan reaksi berikut ini masing-masing adalah K 1, K2, dan K3

HNO2(aq) + H2O(l) ↔ NO2-(aq) + H3O+(aq) K1


2 H2O(l) ↔ H3O+(aq) + OH-(aq) K2
NH3(aq) + H2O(l) ↔ NH4+(aq) + OH-(aq) K3
Tetapan kesetimbangan untuk reaksi di bawah ini adalah
HNO2(aq) + NH3(aq) ↔ NO2-(aq) + NH4+(aq)
F. K1 – K2 + K3
G. K1K3
H. K1K3/K2
I. K1K2K3
J. K2/(K1K3)

-
18. Setengah reaksi yang terjadi di anoda pada reaksi setara di bawah ini 3MnO 4
+ 2+ 3+
(aq) + 24H (aq) + 5Fe(s) ↔ 3Mn (aq) + 5Fe (aq) + 12H2O(l)
Adalah

16. 2MnO4-(aq) + 12H+(aq) + 6e- 2Mn2+(aq) + 3H2O(l)
- + -
17. MnO4 (aq) + 8H (aq) + 5e Mn2+(aq) + 4H2O(l)
 3+ -
18. Fe(s) Fe (aq) + 3e
19. 2+
Fe (aq)Fe (aq) + e
3+ -


20. Fe(s) Fe2+(aq) + 2e-

9
OSP-2015
18. Perhatikan sel volta berikut ini

Cu2+(aq) + 2e- --> Cu(s) o


E = 0,340 V
Potensial sel volta ini, Esel, adalah
+0,0296
-0,0370 V
+0,0592 V
-0,399 V
0V

22. Bentuk geometri, bilangan oksidasi, bilangan koordinasi tembaga, untuk ion
kompleks. [Cu(NH3)4(OH2)2]2+ adalah
Tetrahedral ; +2 ; 6
Square planar ; -2 ; 4
Oktahedral ; +2 ; 6
Linier ; +3 ; 2
Trigonal Planar ; +1 ; 4

23. Mengenai garam kompleks [Co(NH3)5Cl]Cl2, pernyataan yang tidak tepat adalah
Larut dalam air
Dapat menghantarkan listrik
Larutan 1 mol [Co(NH3)5Cl]Cl2 menghasilkan 1 mol kation dan 3 mol anion
Dalam air, kation kompleks yang terbentuk adalah [Co(NH 3)5Cl]+2
Mengandung ligan NH3 DAN Cl-

24. Berikut ini adalah asam-asam karboksilat


J. CHF2CH2CH2CO2H
36. CH3CF2CH2CO2H
III. CH3CH2CF2CO2H
IV. CH3CH2CH2CO2H
Dari keempat asam karboksilat tersebut, urutan yang paling tepat berdasarkan kenaikan
keasaman, mulai dari yang paling asam hingga yang paling lemah asamnya, adalah
11. I > II > III > IV
12. I > IV > III > II
13. III > II > I > IV
14. III > IV > I > II
15. IV > I > II > III

10
OSP-2015
22. Di antara pernyataan mengenai senyawa berikut yang sesuai dengan aturan Huckle adalah

Naftalen Pirol Sikloheptatriena Piridin Stirena

6. Naftalen bukan senyawa monosiklik, oleh karena itu bukan suatu senyawa aromatik
7. Pirol bukan senyawa hidrokarbon, dan bukan termasuk senyawa aromatik
8. Sikloheptatriena bukan senyawa konjugasi sempurna, yang bukan senyawa aromatik
9. Piridin merupakan basa lemah, dan juga bukan senyawa aromatik
10. Stirena mempunyai 8 π elektron, dan juga bukan senyawa aromatik

HBr berlebih
C CH

23. Dalam reaksi adisi berikut ini, yang merupakan produk utama
adalah:
Br

A.
CH3

Br

Br Br
D.
B. C

CH2 CHBr
Br
CHBr2

C. E. CH
CH2

CH2Br

24. Senyawa 2-bromobutana jika direaksikan dengan metanol, seperti pada


persamaan reaksi berikut
H2
C CH3 CH3OH
CH3 CH

Br
Maka produk utama yang dihasilkan adalah
H
2 H
A. C CH3 D. C2 CH2
H3C CH H3C C
H
Br
H
2
C CH3
B. E. C CH3
H3C CH
H3C C
OCH3
H2 OCH3
C CH3
C. H3C C
OCH3

11
OSP-2015
25. Urutan yang paling tepat berdasarkan kenaikan kereaktifan senyawa alkohol di bawah ini
terhadap reaksi dehidrasi dalam suasana asam adalah:
HO CH3
H3C OH
H 3C CH OH
H3C C H3C CH
CH CH3
C C

H
C CH3 2 H2
H
CH3 2

I II III
I < II < III
I < III < II
II < III < I
III < I < II
III < II < I

27. Senyawa yang terbentuk dari reaksi kesetimbangan antara siklopentanon dengan HCN berikut
O

HCN

Adalah
H CN
O H
CN
A. C. E.
CN

OH
NC
CN CN
D.
B.

27. Perhatikan rangkaian reaksi berikut ini:

O O
H
2 1. NaOCH2CH3 etanol KOH, H2O H30+
C C C
H 3C C O CH3 panas panas
H2 2 CH2Cl
Senyawa yang merupakan produk dari rangkaian reaksi di atas adalah :

12
OSP-2015
O O
A. D. H2 O CH3

C
C C CH3
C C O C C C
H2 H2 H2 H2
O H2 O

B. C
C C CH3 E. H
2
H2 C CH3
O C C
H2
O O

C. C CH3

C C
H2 H2

28. Persamaan reaksi berikut ini adalah perubahan dari suatu alkena menjadi alkohol
H2 H2
H2 pereaksi ??
C CH2 C C
3HC C 3HC C OH
H H2
Pereaksi yang dipakai untuk reaksi perubahan tersebut adalah
KOH
BH3/THF kemudian H2O2, NaOH
Hg(O2CCH3)2/ H2O lalu NaBH4
H2O , H2SO4
H2O , OH-

30. Produk dari reaksi Wittig di bawah ini adalah


O

H3C
3(C6H5)P CHCH C H
2 6 5 + H

A. C6H5
D. OH

OH P(C6H5)3
H
CH
B. E. 6 5

H
C.
O
P O (C6H5)3

30. Produk utama dari reaksi di bawah ini adalah


13
OSP-2015
NH2

CuCN
NaNO2 , H2SO4
H2O , 0 celcius

Br
6. p-cyano aniline
7. p-cyano nitro benzene
8. p-bromo cyano benzene
9. 2-nitro-4-bromo cyanobenzene
10. 2-cyano-4-bromo aniline

14
OSP-2015
B. Essay

9 Mineral dan Senyawa Mangan (20 poin)

Pyrolusite adalah suatu mineral mangan dioksida yang berwarna kehitaman atau coklat
yang merupakan sumber utama bijih mangan

Gambar 1 Padatan mineral Pyrolusite


Carl Scheele di tahun 1774 melakukan percobaan dengan menambahkan sejumlah asam sulfat ke
dalam mineral pyrolusite, ternyata dia memperoleh suatu gas A yang berupa suatu unsur. Pada
temperatur ruang, gas A tersebut tidak berwarna, serta tidak mempunyai bau dan rasa. Pada
percobaan berikutnya, ke dalam mineral tersebut dia menambahkan larutan asam klorida, dan
menghasilkan gas B, yang juga berupa unsur berwarna kuning-kehijauan dengan bau yang kuat dan
khas, serta gas tersebut dikenal luas banyak dihasilkan dalam kebanyakan zat pemutih rumah tangga.

2. Tuliskan persamaan reaksi untuk eksperimen Scheele tersebut, yaitu reaksi antara mineral
pyrolusite dengan asam sulfat dan dengan asam klorida, serta tuliskan nama gas A dan gas B
(4 poin)

b. Jelaskan jenis reaksi apakah yang terjadi (3 poin)

Ke dalam mineral MnO2 ditambahkan campuran larutan BaCl2 dan larutan H2SO4
c. Tuliskan reaksi yang terjadi dalam campuran tersebut (3 poin)

Pyrolusite mempunyai struktur tetragonal ( Gambar 2)

Dalam sel unitnya (sel satuannya) , a = b = 4,4 Å dan c = 2,9 Å


d. Hitunglah rapat massa pyrolusite (dalam g/cm 3) (5 poin)

Mangan dioksida yang terkandung dalam mineral pyrolusite dapat digunakan sebagai sumber untuk
pembuatan KMnO4 berdasarkan reaksi:

2MnO2 + 2KOH + O2 2KMnO4 + H2
15
OSP-2015
Untuk membuat KMnO4 ke dalam 1 kg bubuk pyrolusite ditambahkan sejumlah larutan pekat KOH
berlebih dan kemudian ke dalamnya dialirkan gas O2 yang juga berlebih. Setelah reaksi berakhir, dan
kemudian produk KMnO4 diisolasi, ternyata diperoleh sebanyak 1,185 kg KMnO4 murni. Bila dianggap
dalam proses tersebut hanya MnO2 dalam pyrolusite yang bereaksi, maka:
e. Hitunglah persen berat MnO2 dalam pyrolusite (3 poin)

8 Hitunglah berapa liter volume gas H2 yang dapat dihasilkan reaksi tersebut (diketahui volume
gas pada STP = 22,4 L/mol) (2 poin)

3. Medali Emas Hadiah Nobel (14 poin)

Di tahun 1940 ketika Nazi Jerman menginvasi Denmark, ahli kimia Hungaria George de Hevesy,
melarutkan medali emas dari pemegang Hadiah Nobel Max von Laue dan James Franck dalam suatu
larutan asam tertentu untuk mencegah penyitaan oleh penguasa Nazi Jerman selama pendudukan
Denmark. Dia menempatkan larutan kuning kebiruan tersebut di rak laboratoriumnya di Niels Bohr
Institute

Setelah perang selesai, de Hevesy kembali ke laboratorium, dan ternyata larutan tersebut masih
berada di raknya. Untuk mendapatkan kembali, emas yang terdapat dalam larutan asam tersebut
diendapkannya. Emas yang diperoleh kembali tersebut, diserahkan ke Royal Swedish Academy of
Sciences

Nobel Foundation kemudian membuat-ulang medali Nobel dengan menggunakan emas semula yang
asli, dan pada tahun 1952 diserahkan kembali kepada Max von Laue dan James Franck.

a. Jelaskan dalam pelarut apakah George de Hevesy melarutkan medali ini? Tuliskan komposisi
yang tepat untuk pelarut tersebut (3 poin)

b. Tuliskan persamaan reaksi untuk proses pelarutan emas dengan pelarut asam yang digunakan
(4 poin)

Emas sebagai salah satu logam paling mulia melarut hanya dalam pelarut ini karena energi
pelarutannya yang sangat kuat
o 3+
c. Berikan alasan secara kualitatif, mengapa pelarut asam ini yang dipilih. (Diketahui: E Au/Au
= 1,5 V) (3 poin)

Emas dalam larutan kuning kebiruan tersebut diendapkan (direduksi) dengan menggunakan larutan
Na2S2O5 segar sebagai sumber ion SO32-
d. Tuliskan reaksi pengendapan emas tersebut. (4 poin)

(g) Vanadium dan Senyawanya (20 poin)

Logam Vanadium mengkristal dengan sel satuan kubus berpusat badan (bcc).
Diketahui rapat massa vanadium = 6,11 g/cm 3

16
OSP-2015
a. Gambarkan satu sel satuan vanadium kemudian jelaskan posisi atom-atom vanadium dalam
sel satuan tersebut,
dan hitung jumlah atom Vanadium dalam satu sel satuan (6 poin)

b. Hitung jari-jari atom Vanadium (dalam pm) (4 poin)

Natrium vanadat, Na3VO4 adalah salah satu senyawa yang mengandung ion vanadium.
c. Tuliskan konfigurasi elektron spesi vanadium pada senyawa natrium vanadat dan tentukan
bilangan oksidasi spesi vanadium tersebut. (3 poin)

Ion vanadat dapat direduksi dengan penambahan logam Zn dalam suasana basa menjadi ion V 3+

d. Tuliskan persamaan reaksi yang setara untuk proses tersebut (3 poin)

e. Tuliskan orbital atom manakah pada vanadium yang menerima elektron pada reaksi reduksi
tersebut (2 poin)

7. Jika sebanyak 100 mL larutan natrium vanadat 0,2 M direaksikan dengan 6,54 g Zn, tentukan
konsentrasi V3+(aq) dalam larutan setelah reaksi (dalam Molar) (2 poin)

e. Senyawa Pembentuk Tulang dan Gigi (20 poin)


Senyawa hidroksi apatit adalah senyawa pembentuk tulang dan gigi yang memiliki rumus kimia:
Ca5(PO4)3OH. Dalam air, senyawa tersebut terionisasi menjadi 3 jenis ion (1 jenis kation dan 2
jenis anion)

a. Tuliskan persamaan reaksi kesetimbangan dalam proses pelarutan hidroksi apatit dalam air
dan tuliskan rumusan Ksp untuk pelarutan senyawa hidroksi apatit (3 poin)

b. Jika diketahui Ksp untuk hidroksi apatit pada T = 25 oC adalah 6,8 x 10-37, tentukan
solubilitas molar (kelarutan molar) dari senyawa hidroksi apatit tersebut. (4 poin)

- Sebuah sampel gigi dengan massa 0,100 gram (diasumsikan seluruhnya terdiri dari hidroksi
apatit) dimasukkan dalam 1,000 L air murni dan dibiarkan hingga mencapai keadaan
setimbang. Tentukan massa sampel gigi yang tidak larut. (5 poin)

d. Jelaskan mengapa jika mulut kita keasamannya meningkat, akan menyebabkan gigi
berlubang? (3 poin)

Untuk mencegah lubang pada gigi, pasta gigi mengandung senyawa yang dapat melepaskan ion F - dan
membentuk senyawa Ca5(PO4)3F pada gigi.

Reaksi pertukaran F- dengan OH- adalah sebagai berikut:


Ca5(PO4)3OH + F- ↔ Ca5(PO4)3F + OH- K = ?
e. Tentukan nilai tetapan kesetimbangan, K, jika diketahui Ksp untuk Ca5(PO4)3F adalah 2,0 x 10-61
(5 poin)

17
OSP-2015
5. Aspirin dan Minyak Winter (19 poin)

Senyawa A (asam o-hidroksi benzoat, atau asam salisilat) merupakan prekursor (zat
intermediet) untuk membuat obat analgesik (Aspirin) dan minyak winter (untuk salep)

COOH COOH CO2CH3

I II

OCOCH3 OH OCOCH
Aspirin
3

Minyak Winter

a. Tentukan pereaksi dan kondisi pada reaksi I dan II (4 poin)

b. Tentukan struktur produk melalui hasil reaksi jika senyawa A direaksikan dengan:
i.) Larutan natrium karbonat, Na2CO3 (2 poin)
ii.) Larutan NaOH (2 poin)
iii.) Asam nitrat encer (2 poin)

Bila sebuah tablet Aspirin dihaluskan, ditambahkan air (dipanaskan sampai larut), kemudian dititrasi
dengan 0,1 mol L-1 NaOH, maka dibutuhkan sebanyak 13,9 mL larutan alkali untuk menetralkan
aspirin.

c. Hitung berapa gram Aspirin yang ada dalam satu tablet Aspirin (4 poin)
d. Aspirin yang mudah larut biasanya sebagai garam kalsium aspirin
i.) Tuliskan reagen apa yang dipakai untuk mengubah Aspirin menjadi garam kalsium,
dan tuliskan reaksinya. (3 poin)
ii.) Jelaskan mengapa Aspirin kurang larut dalam air (2 poin)

g. Polimer PET (Poli Etilen Tereftalat) dan Turunannya

Senyawa poli etilen tereftalat (PET) banyak diaplikasikan untuk serat sintesis, seperti Dacron, film tipis
seperti Mylar dan sebagai bahan pembuat botol minuman bersoda.

O O
H
2

* O C C O C CH2 *

Poli Etilen Tereftalat (PET)


n

18
OSP-2015
a. PET dibuat dari monomer etilen glikol (1,2-etanadiol) dan monomer lainnya.
Gambarkan
struktur etilen glikol dan struktur monomer lainnya tersebut. (4 poin)

b. Tuliskan nama senyawa monomer penyusun PET selain etilen glikol pada soal (a)
di atas (2 poin)

iv. Apabila senyawa monomer PET selain etilen glikol pada jawaban soal (b) di atas
direaksikan dengan 1,2-etanadiamina (etilen diamina), maka akan terbentuk polimer
lain. Gambarkan
struktur polimer tersebut. (merujuk pada cara penggambaran PET di atas)! (3 poin)

e. Apabila etilen glikol direaksikan dengan asam 1,4-butanadioat, maka akan


terbentuk polimer lain. Gambarkan struktur polimer tersebut. (merujuk pada cara
penggambaran PET di atas)
(3
poin)

e. Etilen glikol dapat dioksidasi lebih lanjut menjadi senyawa turunan aldehidnya dan
kemudian mengalami oksidasi lebih lanjut menjadi turunan asam karboksilatnya.
Gambarkan struktur aldehid dan asam karboksilat hasil oksidasi total etilen glikol.
Tuliskan nama senyawa asam karboksilat hasil oksidasi total etilen glikol tersebut (5
poin)

SEMOGA BERHASIL

19
HAK CIPTA
DILINDUNGI UNDANG UNDANG

OLIMPIADE SAINS NASIONAL 2015


SELEKSI KABUPATEN / KOTA

Kimia
UjianTeori

Waktu: 120 menit

KEMENTERIAN PENDIDIKAN DAN KEBUDAYAAN


DIREKTORAT JENDERAL PENDIDIKAN MENENGAH
DIREKTORAT PEMBINAAN SEKOLAH MENENGAH ATAS
2015
Petunjuk

B. Isilah Biodata anda dengan lengkap di lembar yang tersedia

C. Soal Teori ini terdiri dari dua bagian :

A. 25 soal pilihan Ganda = 50 poin

B. 5 Nomor soal essay = 103 poin

TOTAL = 153 poin

5. Waktu yang disediakan: 120 menit.

6. Semua jawaban harus ditulis di lembar jawaban yang tersedia

7. Diperkenankan menggunakan kalkulator.

8. Diberikan Tabel periodik Unsur.

9. Anda dapat mulai bekerja bila sudah ada tanda mulai dari pengawas.

10. Anda harus segera berhenti bekerja bila ada tanda berhenti dari Pengawas.

11. Letakkan jawaban anda di meja sebelah kanan dan segera meninggalkan ruangan.

12. Anda dapat membawa pulang soal ujian !!

OSK 2015 ii
This file was downloaded from
http://ivanjoannes.wordpress.com
LEMBAR JAWABAN

Bagian A

Beri Tanda Silang (X) pada Jawaban Yang Anda Pilih

No Jawaban No Jawaban

1 A B C D E 16 A B C D E

2 A B C D E 17 A B C D E

3 A B C D E 18 A B C D E

4 A B C D E 19 A B C D E

5 A B C D E 20 A B C D E

6 A B C D E 21 A B C D E

7 A B C D E 22 A B C D E

8 A B C D E 23 A B C D E

9 A B C D E 24 A B C D E

10 A B C D E 25 A B C D E

11 A B C D E

12 A B C D E

13 A B C D E

14 A B C D E

15 A B C D E

OSK 2015 iii


This file was downloaded from
http://ivanjoannes.wordpress.com
OSK 2015 iv
This file was downloaded from
http://ivanjoannes.wordpress.com
Tetapan dan Rumus

Bilangan Avogadro NA = 6.022∙1023 partikel.mol–1


R = 0,08205 L·atm/mol·K
= 8,3145 L·kPa/mol·K
Tetapan gas universal, R = 8,3145 x107 erg/mol·K
= 8,3145 J/mol·K
= 1,987 kal/mol·K
= 62,364 L·torr/mol·K
1 atm = 101,32 kPa
1 atm=760 mmHg =760 torr
= 101325 Pa = 1,01325 bar
Tekanan gas
1 torr = 133,322 Pa
1 bar = 105 Pa
2 2
1 Pa = 1 N/m = 1 kg/(m.s )
Volume gas ideal (S,T,P) 22,4 liter/mol = 22,4 dm3/mol
1 kal = 4,182 J
Energi
1 J = 1 L·kPa

Persamaan gas Ideal PV= nRT

Tekanan Osmosis pada larutan  = M RT

Tetapan Kesetimbangan air (Kw) pada 25oC Kw= 1,0x10-14


Tetapan kesetimbangan dan tekanan parsial Kp = Kc(RT)∆n
gas
Temperatur dan Tetapan kesetimbangan −∆ 1

ln = ()+

Tetapan Faraday F = 96500 C/mol elektron

Muatan elektron 1,6022 x 10-19 C

Ampere (A) dan Coulomb (C) A =C/det


Reaksi orde pertama: A B


Reaksi orde kedua: A B

OSK 2015 v
This file was downloaded from
http://ivanjoannes.wordpress.com
Pilih jawaban yang paling tepat ( 25 soal @ 2 poin)

Hasil analisis senyawa berupa gas menunjukkan kandungan (% massa) 33,0% Si dan
67,0% F. Pada temperatur 35oC, sebanyak 0,210 L senyawa tersebut memberikan tekanan
1,70 atm. Jika massa 0,210 L senyawa tersebut adalah 2,40 g, maka rumus molekulnya
adalah :
SiF4
SiF3
Si2F8
Si2F6
Si3F9

2. Dari persamaan reaksi berikut ini, yang merupakan reaksi oksidasi-reduksi adalah :
2HCl(aq) + Mg(s)  MgCl2(aq) + H2(g)
Na2O(s) + H2O(l)  2NaOH(aq)
CO2(g) + H2O(l)  H2CO3(aq)
CaO(s) + SO3(g) CaSO4(s)
NH3(g) + HCl(g)  NH4Cl(s)

Konsentrasi (dalam satuan molalitas) senyawa para-diklorobenzena (C6H4Cl2) dalam


suatu larutan yang dibuat dengan cara melarutkan 2,65 g C6H4Cl2 dalam 50 mL benzena
(densitas = 0,879 g/mL) adalah :
0,018 m
0,041 m
0,180 m
0,410 m
1,810 m

C. Titanium(IV)oksida secara luas digunakan dalam industri cat sebagai pigmen putih.
Senyawa ini larut dalam asam sulfat pekat panas sesuai persamaan reaksi berikut,
TiO2 (s) + H2SO4 (aq)  (TiO)2+ (aq) + SO42- (aq) + H2O (l).
Pernyataan yang tepat untuk menyatakan jenis reaksi tersebut adalah :
A. Asam-basa
B. pembentukan ion kompleks
C. dehidrasi
D. pertukaran
E. redoks.

5. Di antara pernyataan berikut yang benar adalah :


18
A. O dan 19F mempunyai jumlah neutron sama.
14
B. C dan 14N adalah isotop karena nomor massanya sama .
18 -2
C. O mempunyai jumlah elektron yang sama dengan 20Ne.
D. A dan B benar.
E. A dan C benar

OSK 2015 1
This file was downloaded from
http://ivanjoannes.wordpress.com
6. Dari serangkaian diagram berikut ini, diagram yang terbaik untuk menjelaskan jari-jari
relatif dari atom dan ion natrium serta atom klor dan ion klorida adalah :

Jwab: B

7 Energi ionisasi kedua kalsium adalah 1150 kJ.mol-1. Di antara persamaan reaksi
termokimia berikut yang menggambarkan energi ionisasi kedua kalsium adalah :

A. Ca(g) Ca2+(g) + 2e- H° = + 1150 kJ/mol.

B. Ca+(g) Ca2+(g) + e- H° = + 1150 kJ/mol.

+
C. Ca (g) 2+
Ca (g) + e -
H° = - 1150 kJ/mol.

D. Ca(s) 2+
Ca (g) + 2e -
H° = + 1150 kJ/mol.

E. Ca+(s) Ca2+(g) + e- H° = - 1150 kJ/mol
8. Diagram berikut ini menggambarkan suatu percobaan untuk pembuatan dan
pengumpulan gas SO2. Ternyata setelah dilakukan percobaan, percobaan dengan
sistem tersebut gagal (tidak dapat mengumpulkan gas SO2).

Di antara modifikasi berikut yang akan membuat eksperimen ini berhasil untuk
memperoleh gas SO2 adalah :
1. Menghilangkan labu P seluruhnya.
2. Menghilangkan labu Q seluruhnya
3. Memakai asam sulfat encer pengganti asam hidroklorida.
4. Memakai air dalam labu P pengganti akua kalium hidroksida.
5. Pengumpulan dengan cara pemindahan ke atas.

OSK 2015 2
This file was downloaded from
http://ivanjoannes.wordpress.com
9. Anda mempunyai zat berikut ini:
Kristal garam NaCl
Lelehan garam NaCl
Larutan garam NaCl

Yang dapat menghantarkan listrik adalah :


i dan ii
i dan iii
ii dan iii
i, ii, dan iii
hanya iii

10 Yang merupakan diagram orbital hibrida dari boron dalam BF3 adalah :

Jawab:E

11. Dalam molekul 1-butena, atom karbon yang diberi label 1 mempunyai hibridisasi

10. sp2
11. sp3
12. sp4
13. sp
14. sp3d

OSK 2015 3
This file was downloaded from
http://ivanjoannes.wordpress.com
Di antara pasangan molekul berikut yang mempunyai geometri sama adalah :
AlCl3 dan BCl3
AlCl3 dan PCl3
BF3 dan NH3
BeCl2 dan H2O
CO2 dan SO2.

Di antara molekul atau ion berikut : SO 2, CO2, NO2+, ClO2–, molekul atau ion yang
isoelektronik adalah :
SO2, NO2+, dan ClO2–
SO2 dan CO2
SO2 dan NO2+
CO2 dan ClO2–
CO2 dan NO2+

Satu dari struktur resonansi ion OCN– yang digambarkan di bawah ini

Muatan formal untuk setiap atom dalam struktur resonansi di atas adalah :

atom O = –1, atom C atom = –1, dan atom N = +1


atom O = 0, atom C = 0, dan atom N = –1
atom O = –1, C atom = 0, dan atom N = 0
atom O = 0, C atom = 0, dan atom N = 0
atom O = +1, atom C = 0, dan atom N = –2

O. Persamaan reaksi kimia yang menunjukkan hubungan Kp = Kc adalah :


MgCO (s) + 2 HCl (g) ⇌MgCl (s) + CO (g) + H O (l)
3 2 2 2
C (s) + O2 (g) ⇌ CO2 (g)
CH4 (g) + 2O2 (g) ⇌ CO2 (g) + 2H2O (l)
Zn (s) + 2 HCl (aq) ⇌ H2 (g) + ZnCl2 (aq)
2N2 (g) + 5O2 (g) ⇌ 2N2O5 (g)

Dalam suatu wadah tertutup yang suhunya 25 oC, sejumlah ammonium karbamat
(N2H6CO2) menyublim dan terdisosiasi menjadi ammoniak (NH3) serta karbondioksida
(CO2) sesuai persamaan reaksi berikut:

N2H6CO2 (s) 2 NH3 (g) + CO2 (g).
Setelah didiamkan beberapa lama, terjadi kesetimbangan dengan tekanan total gas
sebesar 0,116 atm. Nilai Kp untuk reaksi tersebut adalah :
4,20 x 10-3
2,99 x 10-3
C. 4, 64 x 10-4
D. 3,40 x 10-4
E. 2,31 x 10-4

OSK 2015 4
This file was downloaded from
http://ivanjoannes.wordpress.com
17. Larutan garam-garam di bawah ini masing-masing konsentrasinya adalah 0,1 M.
Larutan yang memiliki nilai pH paling tinggi adalah :

A. Larutan NH4Cl (Kb NH4OH = 2,0 x 10-5)


B. Larutan (NH4)2SO4 (Kb NH4OH = 2,0 x 10-5)
C. Larutan NaClO (Ka HClO = 3,4 x 10-8)
D. Larutan NaCN (Ka HCN = 4,0 x 10-10)
E. Larutan Na2S (Ka H2S= 1,3 x 10-20)

18. Dalam fotografi, padatan AgBr yang tersisa dilarutkan dalam larutan Na2S2O3. Ion Ag+
bereaksi dengan ion S2O32 membentuk senyawa kompleks [Ag(S2O3)2]3, dengan
persamaan reaksi sebagai berikut :
13
+ 
AgBr (s) ⇌ Ag (aq) + Br (aq) Ksp = 5,4 x 10
+ 2 3
Ag (aq) + 2 S O (aq) ⇌ [Ag(S O ) ] (aq)
2 3 2 3 2 Kf = 2,0 x 1013
Jumlah padatan AgBr yang dapat larut dalam 125 mL larutan Na2S2O3 1,20 M adalah :
26. 7,14 g
27. 12,22 g
28. 14,08 g
29. 16,72 g
E. 40,65 g

19. Percobaan yang melibatkan reaksi oksidasi NO menjadi NO2 berlangsung sesuai
persamaan reaksi berikut :
2NO(g) + O2(g) 2NO2(g)
Data yang diperoleh dari percobaan tersebut adalah sebagai berikut :
Percobaan [O2], M [NO], M Laju NO2 (M/det)
1 0,001 0,001 7,10
2 0,004 0,001 28,40
3 0,004 0,003 255,6
4 0,002 0,002 X

Nilai X dalam tabel adalah :


3. 3,65
4. 14,20
5. 28,40
6. 56,80
7. 85,20

OSK 2015 5
This file was downloaded from
http://ivanjoannes.wordpress.com
20. Di dalam reaksi yang digambarkan oleh persamaan reaksi berikut ini :

C6H6 + Cl2 C6H5Cl + HCl.
Jenis reaksi yang terjadi pada benzena adalah :
8. Addisi elektrofilik
9. Substitusi elektrofilik
10. Substitusi radikal bebas
11. Addisi nukleofilik
E. Substitusi nukleofilik

22. Aldehida dan keton dihasilkan secara industri oleh katalitik oksidasi alkena, seperti
etanal dibuat di industri petrokimia dari etena seperti persamaan reaksi berikut :

H2C=CH2 + ½ O2 ---katalis CH3CHO
Proses ini juga dipakai di industri untuk membuat 2-butena. Yang menggambarkan
struktur senyawa yang dapat dihasilkan 2-butena adalah :
CH3CH2CHO
CH3CH2CH2CHO
CH3COCH2CH3
CH3COCH3
(CH3)2CHCHO

22. Di antara senyawa berikut yang dapat dibuat dari reaksi antara bromoetana dengan
kalium sianida dan kemudian produk yang terbentuk direduksi lebih lanjut, adalah :
CH3CH3
CH3CH2NH2
CH3CH2CH3
CH3CH2CH2NH2
CH3CH2CH2CH2NH2

23. Produk yang akan terbentuk jika propenaldehida dioksidasi dengan KMnO 4 pada suhu
ruang adalah :
CH2 (OH)CH (OH)COOH
CH2 (OH)CH (OH)COH
CH2 =C(OH)COOH
CH2 =CHCOOH .
CH3CO + CO2

OSK 2015 6
This file was downloaded from
http://ivanjoannes.wordpress.com
24. Produk yang akan dihasilkan jika 3-metil-1-pentena direaksikan dengan Cl2 dengan
adanya sinar UV adalah :
5-kloro-3-metil-1-pentena
4-kloro-3-metil-1-pentena
3-kloro-3-metil-1-pentena
1,2-dikloro-3-metilpentana
1-kloro-3-metilpentana

25. Senyawa ester P yang berbau buah-buahan mempunyai struktur seperti berikut:

Senyawa yang dihasilkan jika P dihidrolisis dengan asam hidroklorida adalah


J. CH3COCl dan (CH3)2CHCH2CH2OH
K. CH3CHO dan (CH3)2CHCH2CH2OH
L. CH3CO2H dan (CH3)2CHCH2CHO
M. CH3CO2H dan (CH3)2CHCH2CH2OH
N. CH3CH2CO2H dan (CH3)2CHCH2CHO

OSK 2015 7
This file was downloaded from
http://ivanjoannes.wordpress.com
B. Soal Esai

Soal 1. (23 poin)

Suatu senyawa organik hanya mengandung karbon, hidrogen dan sulfur. Sampel senyawa
organik tersebut dengan massa 1,045 g dibakar dalam oksigen menghasilkan gas CO2,
H2O dan SO2. Gas-gas tersebut dialirkan ke dalam 500 mL larutan KMnO4 0,0200 M yang
diasamkan, sehingga menyebabkan gas SO2 teroksidasi menjadi spesi SO42-. Hanya
sebagian dari KMnO4 tersebut yang tereduksi menjadi Mn2+.
Selanjutnya, sebanyak 50 mL larutan SnCl2 0,0300 M ditambahkan ke dalam 50 mL larutan
sebelumnya yang masih mengandung KMnO4 yang belum tereduksi. SnCl2 yang digunakan
berlebih sehingga dapat mereduksi semua KMnO4 yang masih tersisa menjadi Mn2+.
Kelebihan Sn2+ yang masih tersisa dalam larutan kemudian dioksidasi menjadi Sn 4+
dengan cara dititrasi oleh 27,28 mL larutan KMnO 4 0,0100 M untuk mencapai titik akhir
titrasi.

Kandungan gas CO2 dan H2O yang dihasilkan dari proses pembakaran sampel senyawa
organik tersebut setelah dianalisis adalah 1,660 g CO2 dan 0,4590 g H2O.

a. Tuliskan persamaan reaksi yang setara untuk reaksi oksidasi gas SO2 oleh KMnO4.
(2 poin)
b. Tuliskan persamaan reaksi yang setara untuk reaksi dalam titrasi Sn2+ oleh KMnO4.
(2poin)
c. Hitung mol Sn2+ yang bereaksi dalam tahap mereduksi semua sisa KMnO4 menjadi
Mn2+. (3poin)
d. Hitung mol KMnO4 yang bereaksi dalam tahap oksidasi gas SO2 menjadi SO3.
(4poin)
e. Hitung massa SO2 yang dihasilkan dari reaksi pembakaran sampel senyawa organik
tersebut. (3poin)
f. Hitung persentase sulfur (%S) dalam sampel senyawa organik tersebut. (3poin)
g. Tentukan rumus empiris senyawa organik tersebut. (3poin)
Dalam analisis spektometri massa diketahui sampel senyawa organik tersebut memiliki
massa molekul 247,98. Tentukan rumus molekul senyawa tersebut.

Jawab:

a. SO2 + 2H2O → SO42– + 4H+ + 2e– (x 5)

MnO4– + 8H+ + 5e– → Mn2+ + 4H2O (x 2)

5SO2 + 2MnO4– + 2H2O → 2Mn2+ + 5SO42– + 4H+ (2 poin)

b. Sn2+ → Sn4+ + 2e– (x 5)

MnO4 – + 8H+ + 5e– → Mn2+ + 4H2O (x 2)

5Sn2+ + 2MnO4– + 16H+ → 5Sn4+ + 2Mn2+ + 8H2O (2 poin)

OSK 2015 8
This file was downloaded from
http://ivanjoannes.wordpress.com
c. mol Sn2+ yang yang bereaksi dalam tahap mereduksi semua sisa KMnO4 menjadi Mn2+:
(3 poin)

 0, 0100mol MnO  5mol Sn 2+ 
mol Sn 2+  0, 02728L MnO  
 
  6,82 x104 mol Sn 2+

 1L  2mol MnO 
d. mol KMnO4 yang bereaksi dalam tahap oksidasi gas SO2 menjadi SO3:

mol Sn2+ awal yang ditambahkan ke dalam larutan KMnO4 = 50 mL x 0,0300 M = 1,5 x
10 mol.

Mol Sn2+ yang bereaksi dengan KMnO4 sisa = mol Sn2+ awal – mol Sn2+ jawaban (c) =
3  4
1,5 x 10 mol – 6,82 x 10 mol = 8,18 x 10 mol
4 4
Sehingga mol KMnO4 sisa = 2/5 x 8,18 x 10 mol = 3,27 x 10 mol KMnO4
4 3
Mol KMnO4 sisa dalam larutan awal = 10 x 3,27 x 10 mol = 3,27 x 10 mol KMnO4
2
Mol KMnO4 total dalam larutan awal = 500 mL x 0,0200 M = 1 x 10 mol KMnO4
2 3 3
Mol KMnO4 yang bereaksi dengan SO2 = 1 x 10 mol - 3,27 x 10 = 6,73 x 10 mol
KMnO4 (4 poin)

5. Berdasarkan persamaan reaksi: mol SO2 yang bereaksi dengan KMnO4 = 5/2 x 6,73 x
2
10-3 mol = 1,68 x 10 mol SO2
2
Massa SO2 = 1,68 x 10 mol x 64 g/mol = 1,0752 g SO2 (3 poin)
2
f. Massa Sulfur = ArS/MrSO2 x massa SO2 = 32/64 x 1,68 x 10 mol x 64 g/mol = 0,5376 g
Persentase sulfur dalam sampel = 0,5376 g/1,045 g x100% = 51,44% S. (3 poin)

7. Dari data kandungan CO2 dan H2O bisa ditentukan massa C dan massa H.

massa C = ArC/MrCO2 x massa CO2 = 12/44 x 1,660 g = 0,4527 g C

massa H = 2xArH/MrH2O x massa H2O = 2/18 x 0,4590 g = 0,0510 g H

massa S (dari jawaban f) = 0,5377 g

mol C = 0,4527g : 12g/mol =0,0376 mol

mol H = 0,0510g : 1g/mol = 0,0510 mol

mol S = 0,5376g : 32g/mol = 0,0168 mol

Dibagi dengan mol terkecil, maka rasio C:H:S = 2,23:3,03:1

Jika semua dikalikan 4 = 8,92 : 12,12 : 4 ~ 9 : 12 : 4

Maka rumus empirisnya menjadi C9H12S4 (3 poin)

h. (C9H12S4)n = 247,98 → 248n = 247,98 → n = 1

OSK 2015 9
This file was downloaded from
http://ivanjoannes.wordpress.com
maka rumus molekul senyawa organik tersebut adalah (C9H12S4)1 = C9H12S4 (3 poin)

Soal 2. ( 26 poin)

Kalium superoksida, barium peroksida dan titanium dioksida adalah oksida yang memiliki
rumus empiris yang sama, yaitu setiap atom logam bergabung dengan 2 atom oksigen.

11. Tuliskan rumus oksigen yang ada pada titanium dioksida, gambarkan struktur
Lewisnya dan tentukan bilangan oksidasi titan pada senyawa tersebut. (4 poin)
Jawab:
oksida, O2,(1 poin)

Lewis, O2- ada 8 elektron mengelilingi satu atom O, (2 poin)

bilangan oksidasi Ti = 4 (1 poin)

2. Tuliskan rumus oksigen yang ada pada barium peroksida, gambarkan struktur
Lewisnya dan jelaskan ikatan yang terbentuk di antara kedua atom oksigen.
(5 poin)
Jawab :
Peroksida, O22; (1 poin)

ada 14 elektron disekitar 2 atom oksigen, (3 poin)

ikatan tunggal kovalen O-O (1 poin)

c. Tuliskan rumus oksigen yang terdapat pada kalium superoksida, hitung jumlah elektron
terluar yang terdapat pada oksigen dalam kalium superoksida dan gambarkan struktur
Lewis serta resonansinya. (6 poin)
Jawab:

super oksida, O2 ; (1 poin)

ada 13 elektron disekitar 2 atom oksigen, (4 poin)

ada 2 struktur resonansi (1 poin)

d. Dari ketiga senyawa tersebut, oksida mana yang dapat larut dalam air? Tuliskan
persamaan reaksinya dan jelaskan pH larutan yang dihasilkan. (6 poin)
Jawab:
Larut: Kalium superoksida dan barium peroksida, (1 poin)

2KO2(s) + 2H2O(l) →2K+ + H2O2 + O2+ 2OH  (2 poin)

BaO2(s) + 2H2O(l) →Ba2+ + H2O2+ 2OH  (2 poin)
Keduanya menghasilkan larutan basa pH> 7 (1 poin)

OSK 2015 10
This file was downloaded from
http://ivanjoannes.wordpress.com
5. Jika 7,39 kg Titanium dioksida bereaksi dengan karbon berlebih dan gas klorin
membentuk 14,24 kg titanium tetraklorida. Tuliskan persamaan reaksinya dan
hitung persen rendemen hasil sintesis tersebut. (5 poin)
Jawab:
TiO2(s) + C(s) + 2Cl2(g) →TiCl4(l) + CO(g). (2 poin)

Rendemen = ⌈ 14,24 / 7,39 ⌉ 100% = 81,09 % (3 poin)

189,88 79,88

Soal 3 (12 poin)

Suatu padatan A dipanaskan dalam wadah tertutup. Pada suhu T, padatan A tersebut
terurai membentuk gas B dan gas C. Penguraian A tersebut mencapai keadaan
kesetimbangan pada saat perbandingan molar B : molar C = 1 : 2 , menurut persamaan :

A(s) B(g) + 2C(g).
a. (i) Tuliskan persamaan tetapan kesetimbangan, Kp dalam tekanan gas parsial.
(2 poin)
Jelaskan dengan singkat mengapa persamaan ini tidak melibatkan A. (3 poin) b.
Tekanan kesetimbangan untuk sistem di atas pada suhu T adalah 3x103 kPa.
(i) Hitung tekanan parsial masing-masing komponen. (4 poin)
(ii) Tentukan nilai Kp lengkap dengan satuannya. (3 poin)
Jawab:

1. (i) Kp = pB x pC2.
(ii) A adalah dalam bentuk padatan. Padatan tidak ikut dalam pernyataan
kesetimbangan, tekanan uapnya jenuh adalah konstan pada suhu tersebut.
2. (i) tekanan kesetimbangan = pB + pC.
Tetapi, pC = 2pB.

Maka: (3x103)x103 = pB + 2 pB = 3 pB
pB = 106 Pa (2 poin)
6
pC = 2x10 Pa (2 poin)

(ii) Kp = pB x pC2 = 106 x (2x106)2 = 4x1016 Pa3. (3 poin)

Soal 4 (20 poin)

Tersedia 5 buah botol berlabel A, B, C, D dan E masing-masing berisi garam tunggal


berupa padatan berwarna putih. Semua padatan tersebut mengandung unsur-unsur
golongan utama (non-transisi). Semua zat tersebut dapat larut dalam air menghasilkan
larutan jernih tidak berwarna.
Dari percobaan yang dilakukan, diperoleh data berikut ini:

OSK 2015 11
This file was downloaded from
http://ivanjoannes.wordpress.com
Larutan E dapat mengubah warna lakmus biru menjadi merah, larutan D dapat
mengubah warna lakmus merah menjadi biru.

Larutan A bereaksi dengan larutan E membentuk endapan putih yang sangat halus.

Larutan A bereaksi dengan larutan B membentuk endapan berwarna putih yang


larut dalam air panas.

Larutan B bereaksi dengan larutan C membentuk endapan kuning.

Larutan B bereaksi dengan larutan D membentuk endapan berwarna hitam.

Larutan C dicampur dengan larutan D dan larutan E tidak menghasilkan endapan

Padatan E bereaksi dengan basa kuat membentuk gas yang berbau khas dan
mengubah lakmus merah basah menjadi biru.

Test nyala untuk larutan C dan D menunjukkan warna kuning

Dari data tersebut, perkirakan senyawa kimia yang mungkin dari senyawa A, B, C,
D dan E. Buktikan dengan persamaan reaksi selengkapnya. (20 poin)

JAWAB:

11. Larutan E: lakmus biru menjadi merah (bersifat asam), hidrolisis


 
kation lemah Anion kuat (tidak terhidrolisis) kation E = (NH4+)
 
12. Larutan A + larutan E endapan putih halus BaSO4, bukan AgCl Ag+ = logam

transisi. Kation A = Ba2+, anion E = SO42-

13. Larutan A + larutan B endapan putih larut dalam air panas PbCl 2.
Anion A = Cl- dan Kation B = Pb2+.

14. Larutan B + Larutan C endapan kuning, kemungkinan PbI2, dan bukan AgI (Ag = logam

transisi). kation B = Pb , Anion C= I-
2+

 
15. Larutan B + larutan D endapan hitam kemungkinan garam sulfida, tetapi
bukan kation logam transisi. Kation B = Pb2+, anion D = S2-
 
16.Larutan C + Larutan D + larutan E tidak ada endapan, tetap berupa larutan
kation Gol IA , Na+ (nyala kuning) atau dan NH4+ bukan K+
-   +
17. Padat E + OH gas bersifat basa (kemungkinan NH3) kation E = NH4
 
18. Test nyala C dan D nyala kuning (Na) Kation C dan D = Na+
10 = BaCl2  Ba2+(aq) + SO42-(aq) BaSO4(s) putih halus

11 = Pb(NO3)2  Cl-(aq) + Pb2+(aq)  PbCl2 (s) larut dalam air panas

C = NaI  I- (aq) + Pb2+(aq) PbI2(s), kuning

D = Na2S  S2-(aq) + Pb2+(aq)  PbS(s), hitam

E = (NH4)2SO4  larutan bersifat asam,


NH4+(aq) + OH-(aq)  H2O + NH3(g) lakmus merah menjadi biru

OSK 2015 12
This file was downloaded from
http://ivanjoannes.wordpress.com
Soal 5. (22 poin)

Cyanidin (Cy) adalah pigmen organik alamiah yang terdapat dalam buah blackberry,
redberry, anggur, ceri, dan lain lain. Cyanidin mempunyai warna ungu kemerah-merahan,
dan dapat berubah warna seiring dengan perubahan pH. Dalam larutan, pada pH rendah,
cyanidin (Cy) berada sebagai CyH+, yang berwarna merah, sedangkan pada pH tinggi
berada sebagai Cy yang berwarna ungu.

CyH+ Cy + H+
Merah ungu

a. Tuliskan persamaan untuk tetapan dissosiasi asam, Ka dari CyH+. (2 poin)


b. Dalam suatu buffer pH = 5,00, rasio dari bentuk merah ke bentuk ungu adalah 1 :
5. Hitunglah nilai Ka. (4 poin)
c. Hitunglah rasio bentuk merah terhadap bentuk ungu dalam buffer jus buah pada
pH = 3,00, dan perkirakanlah warnanya. (5 poin)

Jus buah sering diawetkan dengan penambahan sedikit sulfur dioksida yang mengkibatkan
terjadinya reaksi, dan menghasilkan senyawa tambahan yang tidak berwarna. Untuk
perubahan warna merah menjadi tidak berwarna digambarkan dengan reaksi berikut:

CyH+ + SO2 + H2O CySO2H2+ H+ (1)
Merah tak-berwarna

Bila sulfur dioksida secukupnya ditambahkan ke buffer jus buah pada pH = 3,00
menghasilkan konsentrasi kesetimbangan, [SO2], 1,0 x 10-2 M, dan intensitas warna merah
berkurang sepersepuluh dari nilai intensitas awalnya.

d. Tuliskan persamaan tetapan kesetimbangan reaksi (1) dan pakailah data yang
tersedia untuk menghitung nilainya. (11 poin)

JAWAB :

[Cy][H+]
a.K = +
(2 poin)
[CyH ]

+
b. H = − log H = 5,00 → [H+] = 10−5
(2 poin)

[Cy][H+] 5

[CyH+] = 1 =5

+
 =5x[ ] = 5 10−5
(2 poin)

c. pH = 3,00  [H+] = 10-3 mol/L

OSK 2015 13
This file was downloaded from
http://ivanjoannes.wordpress.com
+ karena: K =
[Cy][H ] [CyH+]

−3
[Cy][10 ]

maka: 5 10−5 = (2 poin)


[CyH+]
[Cy] −2
[CyH +] = 5 10
+
[ ] = 20 [ ]

Atau

= 20
(3 poin)

Rasio dari bentuk merah terhadap bentuk ungu adalah 20 : 1. Maka, juice
buah dibuffer pada pH = 3,00 berwarna merah.
[ ][ +] +
d. = 22 [ ]{ 2]

Misal a = konsentrasi awal CyH+


[CySO H ][H+]
= 2 2 [CyH+][SO2]

Maka: Pada kesetimbangan [CyH+] = 1/10 a [CySO2H2] = 9/10 a


9
−3
[ ][10 ]
10

 = = 0,9
1

[
][10x10−2 ]
10

K = 0,9 (4 poin)

 
SEMOGA BERHASIL

OSK 2015 14
This file was downloaded from
http://ivanjoannes.wordpress.com

Anda mungkin juga menyukai